Download as pdf or txt
Download as pdf or txt
You are on page 1of 818

PREFACE

My previous books “Handbook-Diseases of Ear, Nose and Throat” and


“Concept Book-Diseases of Ear, Nose and Throat” by Fahim Awan
helped thousands of students clear their concepts and professional
examinations.
“CONCEPT BOOK-DISEASES OF EAR, NOSE AND THROAT
BY FAHIM AWAN” is the result of unwavering enthusiasm of my
students, junior and senior colleagues to produce more reading stuff for
them.
The book is easily readable, understandable, palatable and
comprehensive. There are diagrams, illustrations and tables to clear the
concepts. The book is perfectly tailored to encompass the need of
undergraduate students. In this book knowledge is displayed in unique
ascending pattern. In this way book will be equally beneficial for the
average, good and excellent students.
In every topic, important points are highlighted in bold letters to
show their importance and for rapid revision. At the end of every topic,
relevant MCQs and scenarios with their answers are given. After reading
the topic, reader must be able to answer MCQs and scenarios. During
reading of the text, some points may not be perceived well but as the
reader goes through MCQs and scenarios of topic, important points
would float on his mind and will be permanently enshrined in his
memory.
MCQs given in this book are best choice question (BCQ) type according
to international standards. To eliminate any chances of clues or guesses
in MCQs, options are given in alphabetical and numeric in ascending
pattern. At the end, instruments and OSPEs are given to help the students
to prepare UHS and other examinations.
This book is strategically designed for MBBS students but it will
also help MCPS, IMM, MS, Part II students and even junior consultants
in better understanding of diseases and their management.
I hope the book will not only help in crystal clearing the concept of
disease process pertaining to ear, nose and throat but also the best
management of patients.
CONTENTS
PART A EVALUATION OF ENT PATIENT -----------------------------1
Chapter 1 History taking and prerequisites of examination--------------1
o History taking in ENT 1
o Prerequisites of examination of an ENT patient 4
PART B EAR -----------------------------------------------------------------------------------4
Chapter 2 Symptomatology and examination of ear-------------4
Chapter 3 Diseases of external ear-----------------------------------18
Congenital anomalies of ear--------------------------------------------------------------18
Injuries-------------------------------------------------------------------------------20

Hematoma auris----------------------------------------------------------------------------------------20

Keloid -----------------------------------------------------------------------------------------------------23

o Inflammations of external ear (otitis externa)----------------- -24

Boil (furuncle) of ear……………………………………………………………………24

Diffuse infective otitis externa………………………………………………………………………28

Otomycosis…………………………………………………………………………………30

Seborrhoic otitis externa…………………………………………………………………… 34

Otitis externa hemorrhagica……………………………………………………………35

Herpes zoster oticus…………………………………………………………………………37

Malignant otitis externa………………………………………………………………….40

Perichondritis of pinna…………………………………………………………………….44

Miscellaneous diseases of external ear…………………………………….46

Foreign body of ear……………………………………………………………………….46


Wax…………………………………………………………………………………….49

Keratosis obturans……………………………………………………………………………………………...53

Meatal atresia and stenosis ……………………………………………………………………55

Osteoma…………………………………………………………………………………………….55

Exostosis……………………………………………………………………………………56

Chapter 4 Diseases of middle ear cleft……………………………………………………………………….58


o Injuries 58
Fracture of temporal bone………………………………………………………………..58
Otitic barotrauma…………………………………………………………………………61
Traumatic perforation of tympanic membrane…………………………………64
Blast injuries…………………………………………………………………………………………….68

o Otitis media 70
Acute otitis media/Acute suppurative otitis media………………………70
Definitions of certain otological procedures…………………………………76
Myringotomy…………………………………………………………………………………………………77
Myringoplasty……………………………………………………………………………………78
Cortical mastoidectomy……………………………………………………………………..79
Radical mastoidectomy… … … … … … … … … … … … … … … … … … … … … . 81
Modified radical mastoidectomy………………………………………………………………81
Meatoplasty………………………………………………………………………….…..81
Chronic suppurative otitis media (CSOM)………………………………………85
Complications of suppurative otitis media……………………………………………90
Tuberculous otitis media………………………………………………………………….108
Otitis media with effusion (OME)……………………………………………………………110
Eustachian tube disorders………………………………………………………………………116
o Facial paralysis……………………………………………119
Bell’s palsy………………………………………………………………………………………122
Neoplasm of middle ear cleft…………………………………………………….127
Glomus tumor……………………………………………………………………………………..127
Squamous cell carcinoma………………………………………………………………………………134
Chapter 5 Diseases of otic capsule……………………………………………..136

Otosclerosis…………………………………………………………………………………………….136
Stapedectomy…………………………………………………………………………………….139
Chapter 6 Diseases of inner ear……………………………………………146
o Inflammatory condition of inner ear……………………...146
Vestibular neuronitis…………………………………………………………………..146
Viral Labyrinthitis………………………………………………………………………148
o Miscellaneous diseases of inner ear……………………...149
Vertigo………………………………………………………………………………………….…………..149
Meniere’s disease………………………………………………………………………………………………..152
Benign paroxysmal positional vertigo (BPPV) ……………………………………………161
Tinnitus ………………………………………………………………………………………………..166
Ototoxicity……………………………………………………………………………………….…………169
Chapter 7 Miscellaneous diseases of ear…………………………………………………….172
Hearing loss……………………………………………………………………………………………….172
Otalgia………………………………………………………………………………………………..177
Vestibular schwannoma…………………………………………………………………179
Chapter 8 Audiology………………………………………………………….186
Audiometric tests………………………………………………………………………………...186
Hearing loss in children……………………………………………………………………..196
Hearing aid…………………………………………………………………………………….…201
Cochlear implant…………………………………………………………………………………204
PART C NOSE AND PARANASAL SINUSES…………………209
Chapter 9 Symptomology and examination of nose……………………………..209
Chapter 10 Diseases of nose and paranasal sinuses……………………………...219
o Congenital anomalies………………………………………………………….219
Meningoencephalocele………………………………………………………………….………….219
Choanal atresia…… …… …… …… ……… …… …… …… ……… …… … …… .…. 222
Trauma of nose and paranasal sinuses…………………………………………226
Fracture of nose…………………………………………………………………………………..226
Deflected nasal septum…………………………………………………………………232
Septoplasty/SMR………………………………………………………………………….234
Cerebrospinal fluid (CSF) rhinorrhea……………………………………………….240
o Inflammations of nose and paranasal sinuses……………………...245
Boil (furuncle) of nose…………………………………………………………………………..…...245
Rhinosinusitis……………………………………………………………………….……… ……….. 249
Common cold……………………………………………………………………………………….251
Allergic rhinitis……………………………………………………………………………………………………………255
Vasomotor rhinitis………………………………………………………………………………………261
Atrophic rhinitis…………………………………………………………………………….264
Rhinitis sicca……………………………………………………………………………………………268
Rhinolith……………………………………………………………………………….……….270
o Complications of rhinosinusitis………………………………………………272
Local complications………………………………………………………………………….272
Orbital complications…………………………………………………………………….272
Intracranial complications………………………………………………………………………274
Cavernous sinus thrombosis……………………………………………………….275
Toxic shock syndrome……………………………………………………………………276
Antral lavag……………………………………………………………………………………………281
Inferior meatal antrostomy……………………………………………………………………….282
Caldwell-Luc operation…………………………………………………………………….283
Ehmoidectomies…………………………………………………………………………………….……..287
o Granulomatous diseases of nose and paranasal sinuses……292
Wegener’s granuloma………………………………………………………………………….292
Stewart’s granuloma…………………………………………………………………………….296
Sarcoidosis…………………………………………………………………………………………………299
Tuberculosis……………………………………………………………………………………………….302
Lupus vulgaris…………………………………………………………………………………………………304
Leprosy…………………………………………………………………………………………………..305
Mucormycosis………………………………………………………………………………………….308
Rhinoscleroma………………………………………………………………………………………….311
Rhinosporidiosis……………………………………………………………………………………….313
o Tumors of nose, paranasal sinuses and nasopharynx……………...313
Hemangioma……………………………………………………………………………………………………..314
Transitional cell papilloma……………………………………………………………………316
Malignant tumors of nose and paranasal sinuses…………………………… 319
Nasopharyngeal Angiofibroma…………………………………………………………….322
Nasopharyngeal carcinoma (NPC)………………………………………………………………327

o Miscellaneous diseases of nose and paranasal sinuses…………..332


Nasal Polyps……………………………………………………………………………………….…..332
Antrochoanal polyp………………………………………………………………………….……337
Epistaxis ………………………………………………………………………………………………………………..341
Heriditary hemorrhagic telangectasia (HHT)…………………………………………348
Septal hematoma ……………………………………………………………………………………..351
Septal abscess………………………………………………………………………………………….355
Septal perforation……………………………………………………………………357
Maggots/Myiasis…………………………………………………………………………….………360
Foreign body of nose……………………………………………………………………………..363
Rhinophyma…………………………………………………………………………………….…………366
Cystic fibrosis…………………………………………………………………………….…………367
Mucocele………………………………………………………………………………………………………………370

PART D ORAL CAVITY AND PHARYNX………………………………………………374


Chapter 11 SYMPTOMATOLOGYand examination of throat ………….374
Chapter 12 Diseases of oral cavity and pharynx …………………………….378
o Cleft palate…………………………………………………………………………..378
o Ulcers of oral cavity……………………………………………………………..380
o Inflammations of mouth and pharynx……………..……………………386
Pharyngitis………………………..………………………………………………………………386
Adenoiditis…………….…………………………………………………………………….389
Adenoidectomy.………………………………………………………………………………………………………394
Tonsillitis…………………………………………………………………………………………………………….396
Tonsillectomy……………………………………………………………………………………………….402
Diptheria…………………………………………………………………………………………………….407
Peritonsillar abscess (PTA)……………………………………………………………………………….411
Parapharyngeal abscess………………………………………………………………… 415
Retropharyngeal abscess………………………………………………………………. 419
Tumors of oral cavity, oropharynx and hypopharynx…………………..424
Tumors of oral cavity………………………………………………………………………….424
Tumors of oropharynx ……………………………………………………………………………429
Tumors of hypopharynx………………………………………………….…………….432
Miscellaneous diseases of oral cavity and pharynx…………………….438
Ludwig’s angina…………………………………………….………………………………. 438
Vincent angina……………………………………………………………….……………..442
Ranula…………………………………………………………………….………………………..444
Submucous fibrosis………………………………………………………………….…….446
Globus pharyngeus……………………………………………………………….………..450
Parapharyngeal space……………………………………………………….………..452
Parapharyngeal space tumors………………………………………………………………454
Sleep apnea syndrome…………………………………………………………………….457
Pharyngeal pouch…………………………………………………………………………..462

o Diseases of salivary glands 466


Mumps/Viral parotitis…………………………………………………………………….466
Acute suppurative parotitis……………………………………………………………468
Chronic recurrent sialadenitis…………………………………………………….…470
Sialectasis…………………………………………………………………………………………….471
Sjogren’s syndrome………………………………………………………………………471
Sialolithiasis……………………………………………………………………………………………………474
Salivary gland tumors………………………………………………………….………………… 479
Xerostomia……………………………………………………….…………………………………….. 483
Ptyalism……………………………………………….…………………………………………………… 485

PART E LARYNX…………………………………………………………………………………………………………………….487
Chapter 13 Diseases of larynx………………………..………………….487
o Congenital anomalies of larynx 487
Laryngomalacia……………………………………………………………….…………487
Laryngeal web……………………………………………………………………….……491
Inflammations of larynx………………………………………………………………493
Acute
laryngitis………………………………………………………………………………………….493
Acute epiglottitis…………………………………………………………………………..495
Laryngotracheobronchitis (LTB)………………………………………………………………500
o Granulomatous diseases of larynx………………………………………..503
Tuberculosis of larynx………………………………………………………………………………….503
Syphilis of larynx…………………………………………………………………………………505
Tumors of larynx……………………………………………………………………………506
Papilloma of larynx……………………………………………………………………………………506
Carcinoma of larynx……………………………………………………………………………506
Laryngectomy…………………………………………………………………………………….509

o Upper airway obstruction………………………………………………515


Endotreacheal inubation……………………………………………………………………………………………………………………………………………………….516
Minitracheostomy……………………………………………………………………… 518
Tracheostomy………………………………………………………………………………………………518
o Miscellaneous diseases of larynx and tracheobronchial tree……………526
Foreign body of larynx………………………………………………………………………………………526
Foreign body of tracheobronchial tree…………………………………………528
Vocal nodules…………………………………………………………………………………..531
Vocal fold oedema and laryngeal polyp…………………………………………535
Stridor……………………………………………………………………………………………………..538
Stertor ………………………………………………………………………………………………….542
Vocal cord paralysis………………………………………………………………………………..544
Recurrent respiratory papillomatosis…………………………………………………………551
Dysphonia plica ventricularis …………………………………………………………………555
Puberphonia…………………………………………………………………………………………….558
Functional aphonia… … … … … … … … … … … … … … … … … … … … … … … . 561
Laryngocele…………………………………………………………………………………………564

PART F ESOPHAGUS… … … … … … … … … … … … … … … … … … … … … … … … … … … … … . . 5 6 7
Chapter 14 Diseases of esophagus………………………………………………….……....567
Plummer-Vinson syndrome…………………………………………………………………,….. 567
Foreign body of pharynx and esophagus……………………………………… …571
Esophagoscopy……………………………………………………………………………………… ….573
Achalasia cardia ……………………………………………………………………………………..576
Hiatus hernia……………………………………………………………………………………………. 579
Corrosive stricture of esophagus………………………………………………,……………………583
Perforation/Rupture of esophagus……………………………………………………585
Gastroesophageal reflux disease (GERD)………………………………………….589
PART G NECK………………………………………………….……………..592
Chapter 15 Classification of neck masses………................592
Thyroid tumors …………………………………………………………………………………………...594
Thyroglossal cyst………………………………………………………………………….598
Branchial cyst……………………………………………………………………………………………….…….602
PART H GENERAL TOPICS…………………………………………………………………….……………………..604
Chapter 16……………………………………………………………………….…………………604
AIDS …………………………………………………………………………………………………………..……. .604
Lymphoma……………………………………………………………………………………………….607
Level/Region of lymph node involvement……………………………………..614
Treatment of neck nodes ……………………………………………………………..616
Neck dissection…………………………….………………………………………………….618
Occult primary…………………………………………………………………………………….……621
Radiotherapy……………………………………………………………………………….….……..622
Chemotherapy …………………………………………………………………………………….…..626
Cryosurgery………………………………………………………………………………..……. 629
Laser…………………………………………………………………………………………….………………630
Functional Endoscopic Sinus Surgery (FESS)………………………………………………………632
Differences between CT and MRI …………………………………………………………………634
Syndromes in ENT……………………………………………………………………………………………634
PART I MISCELLANEOUS MCQS………………………………………………………………639
Ear…………………………………………………………………………….639
o Nose………………………………………………………………………………………642
o Throat……………………………………………………………………………………645

PART J OSPE……………………………………………………...647
o Ear………………………………………………………………………………………...647
o Nose……………………………………………………………………………………….652
o Throat……………………………………………………………………………………657

PART K INSTRUMENTS …………………………………………………………………………………………………………………………………………………………6. 63


Ear……………………………………………………………………………………………………………………………………….664
Nose…………………………………………………………………………………………………………………………………….671
Throat………………………………………………………………………………………………………………………………….679
Index……………………………………………………………………………………………………………………………………688
PART A
EVALUATION OF ENT PATIENT
Chapter 1
EVALUATION, HISTORY TAKING AND
PREREQUISITES OF EXAMINATION
OF ENT PATIENT
EVALUATION
Evaluation of a patient means;
1. Complete history
2. Detailed examination
3. Relevant investigations
Aims and objectives of evaluation of a patient:
Minimum aims and objectives of evaluation of any patient no matter whether the patient is
in outdoor, indoor or emergency are as follows;
1. To make a provisional diagnosis
2. To make a differential diagnosis
3. To make a definitive diagnosis
4. To find out cause of disease
5. To find out extent of disease
6. To find out complications of disease
7. To find out associated diseases i.e., comorbidities
8. To establish a treatment plan
9. To determine the prognosis
HISTORY TAKING IN ENT
The pattern of history taking is the same as that for other disciplines of medicine. It is as
follows;
2
1. Patient profile:
a. Name
b. Age
c. Gender
d. Marital status
e. Occupation
f. Cell number
g. Phone number
h. e-mail address
i. Postal address

2. Presenting complaints:
These are mentioned in chronological order, in patient’s language and maximum
informations are given in minimum words.

3. History of present illness:


The inquiry under the heading is done as follows;
Detailed history of presenting complaint such as
a. Bilateral or unilateral (if unilateral then right or left)
b. Duration (acute, subacute or chronic)
c. Mode of onset
d. Progress (increasing, decreasing or decreasing)
e. Intensity (mild, moderate or severe)
f. Persistent/intermittent
g. Initiating/aggravating factors
h. Relieving factors
i. In case of discharge
• Amount (scanty or copious)
• Color
• Bloodstained or not
• Consistency (thin or thick)
• Smell
History of associated symptoms:
The symptoms which are generally, present in a patient in that particular part with which the
patient presents are known as associated symptoms e.g., if the patient presents with nasal
obstruction then the history of associated symptoms would be about nasal discharge, sneezing,
3
epistaxis and facial pain etc. The history of present illness and of associated symptoms is
together known as local history.

4. Regional history:
Then the history is taken of the remaining systems/organs which are routinely dealt by that
particular department and are likely to be the cause/effect of the same disease process or may
harbor some other disease. For example, in this particular case the patient presented with
nasal obstruction, after taking history of nasal symptoms (i.e., local history) history of throat
and ear (regional history) is taken.

5. Systemic history:
After local and regional history, detailed history is taken of the rest of the systems (like GIT,
CNS, CVS etc.) of the body. This is known as systemic history.
Past history, family history, social and occupational history is the same as that for other
disciplines of medicine.

PART B

EAR
Chapter 2
SYMPTOMATOLOGY AND EXAMINATION
OF EAR:
SYMPTOMATOLOGY OF EAR
Every system and organ in the body has got its own set of symptoms and signs. More than
90% of patients presenting to ENT department, present with one or more of the following
complaints;
1. Hearing loss
2. Earache
3. Ear discharge
4. Itching
5. Tinnitus
6. Vertigo
PREREQUISITES OF EXAMINATION OF AN
ENT PATIENT
1. Greeting: The clinician must say greeting to the patient with a smiling face.
2. Introduction: The clinician must introduce himself to the patient.
3. Consent: The clinician must inform the patient about details of examination and must
ensure that no harm would be incurred during examination.
4. Exposure: Adequate exposure of the patient is essential for examination, which
includes exposure of at least down up to clavicle anteriorly, acromion laterally, and the
vertebra prominence posteriorly. Rarely, more extensive exposure may be required in
special conditions. Any turban, cap, shawl, muffler, scarf, hearing aid, glasses, contact
lenses, hair clips, should be taken off gently with consent.
5. Positioning: Examination of ear, nose and throat is performed with the clinician sitting
in confrontation with the patient, sitting side to side with no legs crossing. Only for
examination of the right ear, whole of the patient is turned to the left side rather than
simply turning the head. The clinician sits at the same horizontal level as that of the
patient. So, the clinician should be sitting in confrontation with the
patient.

To examine the ears of a young child, it is best to sit the child


sideways on the parent's lap, with one of the parent's hands holding
both the child's hands and the other holding the child's head with
one ear against the parent's shoulder while you examine the other
one.
To view the throat of a refractory toddler, insert a tongue depressor
into the gap between clamped teeth and cheek, the teeth may
briefly open allowing you to insert the tongue depressor.

6. Illumination: Examination of an ENT patient is


performed with a head light or head mirror. With the
introduction of use of head light, head mirror is
becoming obsolete.

Headlight
Head mirror Bull’s eye lamp

EXAMINATION OF EAR:
It is important to remember that examination starts as the
patient enters the clinic. After completing all the
prerequisites, examination of ear using head light/head
mirror is done as follows:
1. Inspection:
As a general rule. examination is started with inspection.
Inspection is done of the:
a. Preauricular region
b. Pinna
c. Postauricular region
Examiner has to look for:
5
a. Any congenital anomaly like preauricular sinus/pit, anotia (absence of pinna),
microtia (small pinna), macrotia (large pinna), meatal atresia/stenosis.
b. Anatomical landmarks like helix, antihelix, tragus, antitragus, crus of helix, lobule,
triangular fossa, cymba conchae etc.
c. Symmetry in comparison to opposite side. There may be obliteration of retroauricular
groove or protuberance/erection of pinna in cases of mastoiditis or mastoid abscess.
d. Any other positive finding like scar of postaural or endaural incision of previous
surgery, mastoid fistula, ulcer, swelling, hemangioma, lymphangioma etc.
2. Palpation:
On palpation, clinician has to look for;
a. Local temperature
b. Tenderness
Palpation is done of the;
a. Preauricular region; It may be tender in cases of infected preauricular cyst/sinus.
b. Pinna; It may be tender in boil ear or perichondritis of pinna.
c. Postauricular region especially mastoid antrum and mastoid tip. It may be tender in
acute mastoiditis.
d. Temporomandibular joint; It may be tender in temporomandibular arthralgia.
3. Pull the pinna upwards, backwards and laterally
to straighten the external auditory canal. At the same time
retract the tragus anteriorly and perform inspection of the
external auditory canal and tympanic membrane. On
inspection look for;
a. Any congenital anomaly like meatal atresia,
meatal stenosis.
b. Anatomical landmarks of cartilaginous and bony
meatus like roof, floor, anterior and posterior wall.
c. Symmetry in comparison to opposite side.
d. Any other positive findings like wax, boil, foreign body, debris,
fungus, discharge, edema, erythema, granulation tissue, polyp or
swelling etc.
e. Last but not least, assess the size of external auditory canal so that
the clinician must be able insert largest possible aural speculum
without causing discomfort to the patient.
4. Examination with aural speculum/otoscope:

Largest possible aural speculum is used for examination Aural speculum of


of external auditory canals and tympanic membranes. different sizes
For examination of left ear, speculum is held in the left
hand and vice versa. Aural speculum/otoscope is pushed in with rotatory
movement under direct vision. It should not be passed beyond the junction of
the cartilaginous with the bony meatus. It may induce pain
Aural examination
6
and reduce the field of vision because bony meatus is unable to be expand with speculum.
Clinician has to inspect the;
A. External auditory canal
B. Tympanic membrane
Clinician has to look for;
a. Any congenital anomaly
Otoscope
b. Anatomical landmarks
c. Symmetry
d. Any other positive findings like wax, boil, foreign body, debris, fungus, discharge,
edema, erythema, granulation tissue, polyp, osteoma or exostosis etc.
A. External auditory canal: Lateral 1/3rd cartilaginous and medial 2/3rd bony are
identified. Similarly roof, floor, anterior and posterior walls are examined.
B. Tympanic membrane:
a. Anatomical structures of the tympanic membrane
like pars flaccida, pars tensa, anterior and posterior
malleolar folds, lateral process of malleus, handle of
malleus, umbo and cone of light are identified.
b. Color of tympanic membrane is noted. It is red and
congested in acute otitis media. White plaques are
seen in tympanosclerosis.
c. Position of tympanic membrane is noted. It is
retracted in tubal catarrh and in initial stages of otitis
media with effusion. It is bulging in acute otitis media
and late stages of otitis media with effusion.
d. Mobility of tympanic membrane is assessed by
Anatomy of tympanic membrane
pneumatic bulb of otoscope, Siegle’s pneumatic speculum
or by asking the patient to perform Valsalva maneuver.
Siegle’s pneumatic speculum allows magnification through an obliquely
set lens. Mobility of the tympanic membrane can be determined by
alternate compression and release of bulb of otoscope/Siegle’s
pneumatic speculum.
Siegle’s pneumatic
Valsalva maneuver: It is the production of a high nasopharyngeal pressurespeculm
by blowing out against closed lips and nose. It normally results in an increase in middle ear
pressure with the tympanic membrane bulging outwards. It is important to
have the otoscope in place before the patient starts blowing as otherwise
trauma to external auditory meatus may occur with head movement.
If there is perforation of tympanic membrane, assess;
a. Site of perforation
Valsalva maneuver
7
• Central (in which all the margins of the perforation are formed by tympanic
membrane); It indicates acute suppurative otitis media or safe variety of chronic
suppurative otitis media.
• Marginal (in which any of the margin of the perforation is formed by the bone);
It indicates unsafe variety of chronic suppurative otitis media.
• Attic; It also indicates unsafe variety of chronic suppurative otitis media.
b. Size of perforation
• Small (limited to one quadrant of the
tympanic membrane)
• Medium (involving two quadrants of
the tympanic membrane)
• Large (involving three quadrants of
the tympanic membrane) or
• Subtotal (involving all four
quadrants of the tympanic
membrane)
c. Shape of perforation
• Rounded
• Oval
• Kidney shape
d. Margins of perforation
• Smooth
• Irregular Types of perforation
Condition of rest of the tympanic membrane is noted. Condition of
the middle ear visible through the perforation is also noted.
C. In case of ear discharge, firstly, it is mopped or discharge is sucked out and then the
respective ear is observed for;
a. Amount: It is copious in cases of acute suppurative otitis media and safe variety of
chronic suppurative otitis media and scanty in cases of unsafe variety of chronic
suppurative otitis media.
b. Color: It is whitish in cases of acute suppurative otitis media and yellowish/greenish
in cases of chronic suppurative otitis media.
c. Consistency: It is thin in cases of acute suppurative otitis media and thick in cases of
chronic suppurative otitis media.
d. Smell: It is less in cases of acute suppurative otitis media and safe variety of chronic
suppurative otitis media. It is foul-smelling in cases of unsafe variety of chronic
suppurative otitis media.
(Remember 3F i.e., Fork test, Facial nerve, Fistula test).
5. Tuning fork tests: For assessment of hearing, tuning fork of 256 Hz, 512 Hz and 1024 Hz
may be used. The most frequently used tuning fork is 512 Hz because;
a. Its frequency lies in the speech
frequencies i.e., (500 Hz, 1000 Hz and
2000 Hz).
b. Tone of tuning fork of higher frequency
tends to decay quickly.

Tuning forks of different


frequencies
8
c. Tuning forks of lower frequency tend to enhance perception of vibration sensation.
The tuning fork is set into vibrations by striking gently at the junction of distal one
third with proximal two third of prongs of tuning fork. Tuning fork is striked on some
resilient surface like rubber pad, patella or back of elbow. Tone of tuning fork is
familiarized to the patient.
A. Rinne test: A vibrating tuning fork is applied over the mastoid. When the patient stops
listening, it is moved to the lateral aspect of the external meatus at a
distance of 02 cm from the concha.
If air conduction is better than bone conduction, then the test is
known as Rinne positive. Rinne is positive in normal persons or in
patients with sensorineural hearing loss.
If bone conduction is better than air conduction, then the test is
known as Rinne negative. Rinne is negative in patients with conductive
hearing loss. Negative Rinne using tuning fork of 512 Hz, usually Checking air conduction
indicates a hearing loss of more than 25 dB. in Rinne test

False negative Rinne test: If the patient has profound hearing loss or no hearing in the test
ear, the bone conduction stimulus may be perceived by the contralateral ear (non-test ear).
This mistaken impression of function in a non-functional ear is called as false negative Rinne.
In such cases diagnosis is confirmed by Weber test.
B. Weber test: This is very sensitive test in conductive hearing loss. The vibrating tuning
fork is placed in the midline over the vertex, forehead, dorsum of nose or
upper teeth. Patient is asked for any hearing of the tone in the ear.
In a patient with unilateral conductive hearing loss, Weber is
lateralized towards the affected ear.
In a patient with unilateral sensorineural hearing loss, it is referred to
the good ear.
Checking lateralization
The Weber test is of great value in recognizing the false negative in Weber test
Rinne, as it will be referred to the good ear. A lateralized weber
in a conductive hearing loss may indicate a hearing loss
of only 10-15 dB.
C. Absolute Bone Conduction (ABC)/Modified
Schwabach test: This compares bone conduction of
the patient with the bone conduction of a normal hearing
person (usually the clinician). The vibrating tuning fork
is placed on the patient’s mastoid with the meatus
ABC test
blocked and when the patient no longer hears it, the tuning fork is placed on the normal
hearing person’s mastoid again with the meatus blocked. If the clinician hears the tone, the
patient’s bone conduction is said to be reduced.
Interpreting the results of tuning fork tests
9
Test Normal Conductive hearing loss Sensorineural hearing loss

Rinne AC > BC (Rinne +ve) BC > AC (Rinne -ve) AC > BC

Weber Not lateralized Lateralized to poor ear Lateralized to better ear

ABC Same as clinician Increased Decreased

6. Facial nerve:
It is a nerve of the second arch. It has got all four physiological components i.e., general
motor, general sensory, special sensory, and secretomotor.
a. General motor component: There
are three orifices on the face i.e.,
nose, eyes and mouth. Facial nerve
is dilator and closure of all three
orifices. It is checked by asking the
patient to frown the forehead, close
the eyes forcibly, show the teeth and
blow out the cheeks etc. In upper
motor neuron lesion, upper half of
the face is spared because it has got
bilateral cortical presentation. In
lower motor neuron lesion, both the
upper and lower half of the face are
affected.
b. General sensory component is
checked with a special probe by
touching the posterosuperior part of
the external meatus near tympanic
membrane. Sensations on this part
are absent in vestibular schwannoma
which is known as Hitselberger
sign.
c. Special sensory component is checked by
testing the four basic tastes i.e., sweet, sour, bitter
and salt on ipsilateral 66% of anterior two third Anatomy of facial nerve nuclei
of tongue.
d. Secretomotor component innervate lacrimal glands and glands of floor of mouth.
It is checked by looking for wetting of special strips of papers placed in the lower
fornix of eye (Schirmer’s test). It can also be checked by filling rate of special cups
placed in the ducts of submandibular salivary gland.
In an advancing lesion of facial nerve, it is more important to check the sensory component of
facial nerve but it is more convenient to check the motor component.
7. Nystagmus:
Involuntary conjugate rhythmic movements of the eyeball are known as nystagmus.
Nystagmus is checked by placing a small object in front of the patient at a distance of 45 cm
from the eyes and patient is asked to focus on that particular object. The object is moved 30
degrees on either side. The clinician continuously watches the medial or lateral limbus
(corneoscleral junction) of any eye of the patient. Any rhythmic movement of the eyeballs is
noted. Nystagmus is present in certain diseases like labyrinthitis, vestibular neuronitis etc.
8. Fistula test:
If nystagmus is negative, then fistula test is performed.
Clinician sits in confrontation with the patient. Patient is asked to focus his eyes on the
nasion of the clinician. Suddenly patient’s tragus of ear is pushed medially to raise
10
intratympanic pressure and nystagmus is looked for. At the same time patient is asked about
any vertigo. In cases of chronic suppurative otitis media a positive fistula indicates the
presence of a third window into perilymphatic space usually due to cholesteatoma.
A positive fistula test may also indicate presence of a surgically created window in the
horizontal semicircular canal e.g., fenestration operation or may indicate an abnormal
opening in the oval window e.g., post stapedectomy fistula.
Hennebert’s sign: It is a false positive fistula test. It is seen in congenital syphilis and some
cases of Meniere’s disease with intact tympanic membrane.
9. Cerebellar tests:
These should be performed especially if the patient is having vertigo
or suspected to have cerebellar disease (like cerebellar abscess due to
suppurative otitis media). Rapidly repeated movements and finger
nose test are very important for detecting adiadochokinesia, asthenia,
atonia, asynergia, past pointing and intention tremors.
10. Romberg test:
Patient is asked to stand erect looking forwards with feet together. If
the patient is stable, he/ she is asked to close the eyes. Patient may be
asked to engage fingers of both hands with each other and stretch them
apart. Finger nose test
With labyrinthine lesion, the patient will sway often to the side of
lesion. The swaying is accentuated by closing the eyes.
Whereas in cerebellar lesion there is symmetrical swaying that is less affected by eye
closure.
11. Extended/Tandem Romberg test:
It is a further refinement of Romberg’s test. In this test the patient
is asked to stand erect with heel to toe position.
12. Unterberger test:
Patient is asked to stand with outstretched hands and march (just
like an army person) on the spot with the eyes closed.
The patient will rotate towards the side of the paralytic
labyrinthine lesion. In the presence of irritative lesion, balanced
disturbance is so significant that patient cannot perform the test.
13. Gait:
The patient is asked to walk on the straight line between the two
points and then quickly turn to return on the same line. Patient with
labyrinthine lesions deviates to the side of the lesion, whereas
marked imbalance on turning or drunken gait indicates the
cerebellar lesion. Unterberger test
14. Lymph nodes:
Draining lymph nodes are examined in detail.
15. Redrape the patient and pay thanks.
11
16. Examination Under Microscope (EUM):
It is carried out in doubtful cases or where further detailed
examination of ear is required.
17. Caloric test: Synonym: Vestibular caloric stimulation test
It is a test of vestibulo-ocular reflex that involves irrigating cold or
warm water or air into the external auditory canal turn by turn to
produce conventional currents in the endolymph. It is used for
diagnosis of asymmetric function in the peripheral vestibular
system.
It is only a test of lateral semicircular canals and does not assess
vertical canal function.
The patient is placed supine on a couch with head elevated to an Examination under microscope
angle of 30 degrees to the horizontal. This brings the lateral
semicircular canal into the vertical plane. Both ears are checked for
wax or presence of perforation. In cases of perforation the
test is performed by cold or warm air.
Each ear is irrigated by warm water at 44 degrees
Celsius and cold water at 30 degrees Celsius i.e., 07 degrees
Celsius above and below the normal body temperature
for 40 seconds.
A douche-can is taken to which is attached a rubber tube
at the end of which is applied an ear piece of such a bore
that it delivers 300 ml of fluid in 40 seconds. The tube is
filled with a pinched cork. Warm water is used first and
Caloric test
tympanic membrane is checked for a hyperemic blush which indicates
adequate irrigation. The patient is asked to look at a fixed spot in the ceiling
and light is thrown to his eyes to observe the eye movements and nystagmus.
A normal caloric reaction is seen between 90 and 140 seconds after the onset of
irrigation. The same procedure is then done on opposite side after 05 minutes of rest. Then
this procedure is done with cold water in one ear and time is noted and repeated for the other
ear.
Cold water produces a nystagmus away from the stimulated ear and warm water towards
the stimulated ear.
SELF ASSESSMENT
Q. 1. Which statement is correct about external acoustic meatus?
a. Has sebaceous glands which produce medial 1/3 is bony
cerumen (ear wax) d. It follows a straight course
b. Its lateral 1/3 is cartilaginous and e. Only cartilaginous part is covered
medial 2/3 is bony with skin
c. Its lateral 2/3 is cartilaginous and
Q. 2. Due to bend in external auditory canal, on otoscopy pinna is pulled
a. backward downward laterally d. medially downward posteriorly

b. downward laterally e. upward backward laterally

c. laterally upward
Q. 3. Focal length of head mirror is usually
12

a. 15 cm d. 60 cm

b. 20 cm e. 90 cm
c. 30 cm
Q. 4. Tuning fork most frequently used for assessment of hearing is
a. 128 Hz d. 1024 Hz

b. 256 Hz e. 2048 Hz

c. 512 Hz
Q. 5. Rinne test is negative in

a. conductive hearing loss d. all of the above

b. mixed hearing loss e. none of the above

c. sensorineural hearing loss


Q. 6. Rinne test is positive in

a. ceruminosis d. presbycusis

b. chronic suppurative otitis media e. serous otitis media


c. otosclerosis
Q. 7. The conductive hearing loss which can be detected by Rinne test using 512 Hz
tuning fork is
a. 05 dB d. 20 dB

b. 10 dB e. 25 dB

c. 15 dB
Q. 8. False negative Rinne test indicates

a. conductive hearing loss d. moderate hearing loss

b. malingering e. profound hearing loss


c. mild hearing loss
Q. 9. A 38 years old gentleman complains of decreased hearing in the right ear for the last
02 years. On testing with a 512 Hz tuning fork, the Rinne test (without masking) is
negative on the right ear and positive on the left ear. With the Weber test the tone is
perceived louder in the left ear. The patient most likely has
a. left conductive hearing loss c. left sensorineural hearing loss
b. left mixed hearing loss
d. right conductive hearing loss e. right profound sensorineural hearing
loss
Q. 10. Weber test is lateralized towards side of
a. conductive hearing loss d. all of the above

b. mixed hearing loss e. none of the above


c. senorineural hearing loss
Q. 11. The minimum conductive hearing loss that can be detected by Weber test using
13 Part B Ear

512 Hz tuning fork is

a. 5-10 dB d. 20-25 dB

b. 10-15 dB e. 25-30 dB
c. 15-20 dB
Q. 12. Absolute bone conduction test is also known as
a. modified Schwabach test d. Stenger test

b. Rinne test e. Weber test

c. Schwabach test
Q. 13. With tuning fork of 512 Hz, Weber test is lateralized to the right ear. It de-
notes
a. conductive hearing loss in the right but dead right ear

and normal left ear


d. sensorineural hearing loss in right
b. conductive hearing loss in the left ear and normal left ear

with normal right ear


e. all of the above

c. sensorineural hearing loss in the left


Q. 14. A 38 years old man complains of decreased hearing in the right ear for the last 02
years. On testing with a 512 Hz tuning fork, the Rinne test is negative on the right ear and
positive on the left ear. With the Weber test the tone is perceived louder in the right ear.
The patient most likely has
a. left conductive hearing loss c. right conductive hearing loss

b. left sensorineural hearing loss d. right sensorineural hearing loss

e. none of the above


Q. 15. In left middle ear pathology, Weber test will be
a. centralized d. lateralized to right side

b. lateralized to both sides e. normal


c. lateralized to left side
Q. 16. False negative Rinne test is confirmed by
a. short increment sensitivity index d. all of the above

b. Weber test e. none of the above


c. Tone decay test
Q. 17. Facial nerve has got following components

a. general motor d. special sensory

b. general sensory e. all of the above


c. secretomotor
Q. 18. All are true about facial nerve except
a. carries secretomotor fibers to nasal glands
14
b. controls secretions of lacrimal gland d. carries taste from posterior part of
tongue
c. carries taste from anterior 2/3 of e. controls secretions of submandibular
tongue gland
Q. 19. Nystagmus is checked by placing a small object in front of eyes at a distance of
a. 30 cm d. 75 cm

b. 45 cm e. 90 cm

c. 60 cm
Q. 20. To check nystagmus the object is moved

a. 15 degrees on either side d. 60 degrees on either side

b. 30 degrees on either side e. 75 degrees on either side

c. 45 degrees on either side


Q. 21. A positive fistula test indicates

a. attic cholesteatoma d. erosion of labyrinth

b. congenital cholesteatoma e. raised endolymphatic pressure

c. dead labyrinth
------------------------------- ANSWERS--------------------------
------------------------- ---------------------------

Q.1. b 2. e 3. c 4. c 5. a 6. d 7. e 8. e 9. e 10. a
11. b 12. a 13. a 14. c 15. c 16. b 17. e 18. d 19. b 20. b
21. d
--------------------------------------------------------------------------------------------------------------------------
-
15

Chapter 3
DISEASES OF EXTERNAL EAR
CONGENITAL ANOMALIES OF EAR
1. Preauricular pit/cyst:
These develop from abnormalities of fusion of hillocks/tubercles during auricular
development. These are usually bilateral. These are usually skin lined depressions
which are found on or just anterior to anterior crus of helix. These
occasionally contain keratin debris. If infected, there may be pain and
swelling (abscess) along with purulent discharge.
If there is no symptom, no treatment is required. If preauricular
pit/cyst is infected, antibiotics are given. In recurrent cases, complete Infected
excision of pit/cyst is required. preauricular cyst

2. Preauricular sinus/fistula:
It is deeper than preauricular pit. It may extend medially to end blindly
when it is known as preauricular sinus. It may open into external
meatus, parotid gland or in the region of neck when it is known as
preauricular fistula. If one opening is present in the neck and other is
present in the external meatus, it is known as collaural fistula.
Clinical features:
1. Recurrent pain and swelling are usual presenting symptoms.

2. Discharge just in front of one or both pinnae.


Preauricular sinus
On examination; there is;
1. Opening just in front of one or both pinnae.
2. Hyperemia, swelling, discharge and tenderness is present if infected.
Investigations:
Sinogram/fistulogram is carried out to know the extent of the tract.
Treatment:
If there is no symptom, no treatment is required.
Complete surgical excision of the tract of preauricular sinus/fistula.
1. Anotia: Absence of pinna is known as anotia.
2. Microtia: Underdevelopment of pinna is known as microtia. It may
be associated with atresia of external auditory canal.

Microtia
16
3. Macrotia: Excessively large pinna is known as macrotia.
4. Melotia: Misplacement of pinna is known as melotia.
5. Cryptotia: Upper 1/3rd of pinna is embedded under the skin of scalp.
6. Accessory auricle (Preauricular tags or appendages): Extra tubercles may be
present in a line drawn from tragus to angle of mouth.
SELF ASSESSMENT Anotia
Q. 1. The most common symptoms of preauricular sinus are
a. hearing loss and earache d. vertigo and tinnitus

b. recurrent pain and swelling e. all of the above


c. tinnitus and earache
Q. 2. A 10 years old child presents with a preauricular sinus. There is no discharge or
inflammation. Similar sinus was also observed in his mother who got treated at the age
of 30 years. What will be your line of management?
a. Education and observation of the c. Order an axial CT
patient d. Order MRI to exclude a cyst
b. Order a sinogram after injection of e. Order X-ray temporal bones
radiopaque dye
Q. 3. All of the following statements are true except
a. anotia is absence of pinna d. melotia is misplacement of pinna

b. cryptotia is excessively large pinna e. microtia is underdevelopment of pinna

c. macrotia is excessively large pinna


Scenario: A 20 years old male presents to you with the complaint that whenever he instills
ear drops in his left ear, the drops come out of opening in the neck in front of sternomastoid
and behind the angle of mandible.
1. What is the most probable clinical diagnosis?
2. How does it originate?
3. How will you manage this patient?
--------------------------------------------------------ANSWERS-----------------------------------------------------
Q.1. b 2. a 3. b
Scenario: 1. Collaural fistula 2. Originates from abnormalities of fusion of hillocks/tubercle
3. Complete history, examination, fistulogram, complete excision of fistulous tract.
--------------------------------------------------------------------------------------------------------------------------
-
17
INJURIES
HEMATOMA AURIS
Synonyms: Pinna hematoma, Cauliflower ear, Wrestler’s ear, Turban ear, Boxer’s ear,
Pugilistic ear
Definition:
Collection of blood or serum between the perichondrium and cartilage of pinna
is known as hematoma auris.
Etiopathology:
It usually occurs as a result of trauma to the pinna. Trauma may be because of Hematoma
auris
assault, road traffic accidents, sports or personal injuries. Trauma leads to rupture
of vessels in the perichondrium and hence, collection of blood or serum. Occasionally the
condition may occur spontaneously or as a result of insect bite.
Cartilage of pinna is elastic in nature and has no blood supply of its own. The cartilage
gets its nutrients and oxygen from overlying perichondrium.
If hematoma persists for 48-72 hours, the underlying cartilage undergoes avascular
necrosis leading to deformity of pinna. If the hematoma gets infected, then it may change
into perichondritis.

Clinical features:
1. Usually there is a history of trauma.
2. There is severe pain.
On examination;
1. The pinna appears swollen and blue.
2. Outline of conchal fold is lost.
3. The pinna is usually tender.

Complications:
If the treatment is not given well in time, there may be;
1. Perichondritis
2. Organization of clot with permanent thickening of the auricle known
as cauliflower ear, wrestler’s ear, turban ear, boxer’s ear or pugilistic
ear. Cauliflower ear
Investigations:
It is diagnosed on clinical grounds.
18
Treatment:
1. Wide bore needle aspiration is usually sufficient when collection is small.
2. Incision and drainage under full aseptic measures is done under local anesthesia when
collection is large. Wound is kept open by placing a drain.
3. Silicone or other moulded splints which conform to the shape of pinna are applied.
4. Compression dressing with head bandage is applied for 48 hours to avoid recollection.
Alternatively, dental rolls or shirt buttons tied with through and through sutures may be
applied.
5. Systemic broad-spectrum antibiotics are given if presentation is delayed or there is risk of
infection.
SELF ASSESSMENT
Q. 1. Hematoma auris is also known as

a. cauliflower ear d. all of the above

b. pinna hematoma e. none of the above

c. wrestler’s ear
Q. 2. Hematoma auris is also known as

a. boxer’s ear d. all of the above

b. pugilistic ear e. none of the above


c. turban ear
Q. 3. Hematoma auris is a collection of blood or serum in the
a. subcutaneous layer d. subperichondrial layer

b. subepithelial layer e. none of the above


c. submucosal layer
Q. 4. First line treatment of choice in small hematoma auris is
a. analgesics d. compression dressing

b. antibiotics e. wide bore needle aspiration


c. anti inflammatory
Q. 5. A boxer was hit on his right ear by opponent 02 days back and developed a 04 cm
soft fluctuant swelling of the auricle, the treatment includes all except
a. compression dressing d. systemic antibiotics

b. incision and drainage e. wide bore needle aspiration


c. plastic reconstruction of pinna
Scenario: A 20 years old female presents to you with a soft fluctuant swelling of the auricle
immediately after a road traffic accident.
1. What is the most probable clinical diagnosis?
19
2. How will you manage this patient?
3. What possible complications can occur if left untreated?
--------------------------------------------------------ANSWERS-----------------------------------------------------
Q.1. d 2. d 3. d 4. e 5. c
Scenario: 1. Hematoma auris 2. Wide bore needle aspiration, incision and drainage,
moulded splints, compression dressings 3. Perichondritis, organization of clot leading to
cauliflower ear.
--------------------------------------------------------------------------------------------------------------------------
-

KELOID
A sharply elevated, irregularly shaped, progressively enlarging scar resulting from formation
of excessive amount of collagen in the dermis during connective tissue repair.
Etiopathology:
There is a genetic susceptibility for keloid formation. Black races are more
often affected than the white races. It often follows trauma such as piercing
pinna for ornaments or a surgical incision. It usually presents as a
pedunculated lesion.
Treatment:
Treatment is surgical excision followed by repeated injections of A case of keloid
triamcinolone into the surgical site or immediate postoperative radiotherapy
with 300 rads.
SELF ASSESSMENT
Q. 1. Keloid formation of the auricle has following feature

a. always give radiotherapy if recurs change

b. caused by ear piercing trauma d. rarely seen at lobule or helix

c. depot steroid results in malignant e. surgical removal cures completely


Scenario: A 30 years old female belonging to black race, presented to ENT OPD with
progressively increasing swelling on her right ear lobule which followed 02 years after
piercing ear lobule for ornament. Swelling is non tender and firm.
1. What is the most probable clinical diagnosis?
2. What is your differential diagnosis?
3. How will you treat this case?
-----------------------------------------------------ANSWERS--------------------------------------------------------
Q.1.b
Scenario: 1. Keloid 2. Dermoid cyst, hematoma 3. Surgical excision followed by repeated
injections of triamcinolone into the surgical site or immediate postoperative radiotherapy
with rads.
-----------------------------------------------------------------------------------------------------------------
----------------

20
INFLAMMATIONS OF EXTERNAL EAR
OTITIS EXTERNA
Definition:
It is defined as condition of skin of the external meatus that is
characterized by edema, erythema and discomfort/pain.
Etiopathology:
It includes all the inflammatory conditions of the pinna, external
meatus and outer surface of tympanic membrane.
Causes of otitis externa may be divided into; Pathophysiology of otits externa
1. Infective (bacterial, fungal, viral)
2. Reactive (eczema, seborrheic dermatitis, neurodermatitis)
Classification of otitis externa:
Otitis externa is classified into;
1. Localized otitis externa: (boil ear)
2. Diffuse otitis externa: It is further subclassified into;
a. Diffuse infective otitis externa
b. Otomycosis
c. Seborrheic otitis externa
d. Otitis externa hemorrhagica
e. Herpes zoster oticus
f. Malignant otitis externa
g. Perichondritis of pinna
BOIL (FURUNCLE) OF EAR A case of
otitis externa
Synonym: Localized acute otitis externa
Definition:
A boil is defined as Staphylococcus aureus infection of hair follicle
(pilosebaceous gland).

Etiopathology:
As hair follicles are limited only to the cartilaginous portion of external meatus, boil ear
occurs only in this portion. In severe cases, inflammation may spread to postauricular sulcus
causing forward displacement of the pinna. Eventually the furuncle may A case of boil ear
discharge and the condition resolve quickly.
Clinical features:
1. Severe pain in the meatus which is aggravated by chewing movement is the usual
presenting symptom. Boil of external meatus is very painful because skin is tightly
adherent to underlying tissue through rete pegs.
2. Hearing loss may occur if the meatus is completely occluded by swelling.
On examination;
1. Boil is visible in the cartilaginous portion of meatus.
2. Severe tenderness on movement of the pinna. Because of edema of the external
auditory meatus, it may not be possible to pass speculum or otoscope beyond the
swelling. If the canal is completely occluded, tuning fork tests may show mild conductive
hearing loss. Periauricular (anterior, posterior and inferior) lymph nodes may also be
enlarged and tender. Boil of posterior meatal wall may cause edema over the mastoid.
Treatment:
1. Swab for culture and sensitivity is taken.
2. Systemic antibiotics such as penicillin or flucloxacillin are given, especially if there is
marked edema, multiple boils or associated lymphadenopathy.
3. Analgesics are given to relieve severe pain.
4. Local heat is provided with hot water bottle, electric pad or a short-wave diathermy. A
wick soaked in 10% Ichthammol glycerine/magnesium sulphate is placed over the boil in
the external meatus. It has got a hygroscopic action. It reduces edema and pain.
5. In recurrent furunculosis it is very important to exclude diabetes mellitus and other
causes of immunosuppression.
6. Incision should be avoided unless boil is very protuberant and pointing. If incision is to
be done at all, it is done under full aseptic measures.
7. Staphylococci should be eliminated from the external meatus, nasal vestibule and nail
beds by twice daily application of cream containing neomycin or gentamicin.
Differences between boil ear and mastoiditis

S.No. Features Boil ear Mastoiditis


No previous history of otitis Previous history of otitis media
1. History
media is present
Fever and
2. Absent or minimal Present and marked
toxicity
Pushed directly forwards Pushed forwards and
3. Pinna
downwards
22
Swelling of meatus is confined
4. Swelling Swelling of the whole meatus
to cartilaginous part only
Tenderness on moving the pinna Tenderness on pressure over
5. Tenderness but no tenderness on pressure the mastoid but not on moving
over the mastoid the pinna
6. Discharge Discharge is purulent Discharge is mucopurulent
Tympanic membrane shows
7. Otoscopy Tympanic membrane is normal
signs of middle ear infection
Hearing is normal unless the
Tuning fork Tuning fork tests show
8. tests meatus is completely blocked by conductive hearing loss
furuncle
9. Postauricular Postauricular groove is normal Postauricular groove is
groove obliterated
X-ray mastoid shows haziness
10. X-ray X-ray mastoid is normal and destruction of intercellular
septa

Differential diagnosis:
1. Acute mastoiditis
2. Diffuse otitis externa
3. Herpes zoster oticus
SELF ASSESSMENT
Q. 1. Boil (furuncle) of ear is also called

a. diffuse otitis externa d. both a & b

b. localized otitis externa e. both b & c


c. seborrhoic otitis externa
Q. 2. Boil of external meatus is very painful because
a. it is associated with perichondritis d. skin is tightly adherent to underlying
b. it is extremely rich in blood supply tissues through rete pegs.
c. it is extremely rich in sebaceous e. all of the above
glands.
Q. 3. Boil ear is caused by
a. E. coli d. Staphylococcus aureus

b. Pneumococci e. Streptococcus pyogenes

c. Pseudomonas
Q. 4. Boil ear is infection of
23

a. ceruminous glands d. pilosebaceous glands

b. goblet cells e. sweat glands

c. mucous glands
Q. 5. Differential diagnosis of boil ear is

a. acute mastoiditis d. all of the above

b. diffuse otitis externa e. none of the above

c. herpes zoster oticus


Q. 6. Treatment of boil ear includes all of the following except

a. analgesics d. local heat

b. antibiotics e. 10% ichthammol glycerine

c. incision & drainage


Scenario A: A thirty years old female presented with pain right ear for last three days which
has increased in intensity gradually. The pain is aggravated on chewing movements. There is
no history of ear discharge. On examination movements of pinna are painful. Meatus is
occluded by swelling and there is some forward displacement of pinna.
1. What is the most probable clinical diagnosis?
2. What is the differential diagnosis?
3. How will you manage this patient?
B: A 30 years old male complains of severe pain in left ear for last 03 days. On examination
his pinna is protruding outwards and tender to pull. Otoscopy could not be performed due to
excruciating pain:
1. What is the most probable clinical diagnosis?
2. Which organism is responsible for this condition?
3. How will you treat this patient?
---------------------------------------------------------ANSWERS----------------------------------------------------
-
Q.1. b 2. d 3. d 4. d 5. d 6. c
Scenario A: 1. Boil right E.A.C. 2. Acute mastoiditis, diffuse otitis externa, otomycosis
3. After complete history and examination, swab for culture and sensitivity, antibiotics,
analgesics, local heat.
B: 1. Localized acute otitis externa (boil ear) 2. Staphylococcus aureus 3. Antibiotics,
analgesics, local heat.
--------------------------------------------------------------------------------------------------------------------------

DIFFUSE INFECTIVE OTITIS EXTERNA


Synonyms: Swimmer’s ear, Tropical ear, Singapore ear
It occurs frequently in hot and humid climate e.g., swimmer’s ear,
tropical ear or Singapore ear. The most important aggravating factor is
local trauma by scratching of ear with hair grip/key or vigorous
trauma by dirty towel. A few cases are secondary to chronic suppurative
otitis media.
There are two stages of diffuse otitis externa; A case of diffuse
A. Acute stage: otitis externa

Clinical features:
1. Pain in and around the ear which is aggravated by chewing movement is the main
symptom in acute stage.
2. Swelling of the surrounding tissues and outward displacement of the pinna may be
present.
On examination;
1. Meatal skin is red, swollen and tender.
2. Tympanic membrane is often dull and congested.
Treatment:
1. Swab is taken for culture and sensitivity.
2. Aural toilet of meatus is done under operating microscope.
3. One cm wide ribbon gauze soaked in antibiotic (neomycin or gentamicin) is placed in
the external meatus and changed daily. The wick is moistened with drops containing
similar antibiotic preparations.
4. Systemic antibiotics are required for complicated diffuse otitis externa.
5. Non steroidal anti-inflammatory drugs are used as analgesic.
6. Any rubbing or scratching of ear should be avoided. When any skin disease exists in
external meatus, opinion of dermatologist must be taken.
B. Chronic stage:
Clinical features:
1. Irritation and discharge are main symptoms in chronic stage.
2. Hearing loss may develop as a result of accumulation of debris in the meatus.
3. Thickening of meatal skin. There is no tenderness. There may be small granulations on
the surface of tympanic membrane.
Treatment:
1. Same as that for acute
stage. In addition,
2. One cm wide ribbon gauze soaked with steroid is placed in the external meatus to
reduce the inflammatory swelling and irritation.
3. Underlying condition like chronic suppurative otitis media and fungal infections should
25
be addressed on their own merits.
SELF ASSESSMENT
Q. 1. Diffuse infective otitis externa is also known as

a. singapore ear d. all of the above

b. swimmer’s ear e. none of the above


c. tropical ear
Q. 2. All are the symptoms of acute diffuse infective otitis externa except
a. ear discharge d. pain in and around the ear

b. hearing loss e. tinnitus


c. irritation
Q. 3. Symptoms of chronic diffuse infective otitis externa are
a. ear discharge and tinnitus d. pain in and around the ear

b. hearing loss and tinnitus e. tinnitus and vertigo

c. irritation and ear discharge


Scenario: A young lady developed severe pain in her right ear three days ago after an
attempt to relieve itching in the ear by her hair grip. She also feels difficulty in opening and
closing the mouth.
1. What is the most probable clinical diagnosis?
2. What precaution will you suggest to this patient to prevent recurrence?
3. How will you treat this patient?
-------------------------------------------------------- ANSWERS-----------------------------------------------------
Q.1. d 2. e 3. c
Scenario: 1. Diffuse otitis externa 2. Rubbing or scratching should be avoided
3. Swab for culture and sensitivity, aural toilet, local and systemic antibiotics, analgesics.
--------------------------------------------------------------------------------------------------------------------------

OTOMYCOSIS
Definition:
The term otomycosis is used to describe a superficial, diffuse fungal infection of ear canal.
Etiopathology:
Warm humid environment supports fungal growth and human ear canal is ideal for them. It
is prevalent in tropical and sub-tropical climates. Otomycosis may develop as a primary
infection. It may also develop as a result of prolonged treatment with antibiotics and steroid
preparation.
The fungi commonly found are;
26
1. Candida albicans
2. Aspergillus niger (which produce blackish
colony).
3. Aspergillus flavus (which produce yellowish
colony).
4. Aspergillus albus (which produce whitish
colony).
Clinical features:
1. Severe itching and mild watery discharge
are usual presenting symptoms.
2. Hearing loss and fullness in the ear canal
occur if there is complete blockade of external
meatus. While pain usually indicates bacterial
infection. Otoscopic finding of otomycosis
On examination;
1. Fluffy material which looks like a wet blotting paper or cotton wool is usually visible in
the external meatus. There may be black specks.
2. Mycelia may be seen as a mass of fine filaments. Meatal skin may be red and swollen.
Investigations:
Diagnosis is made on clinical grounds. In doubtful cases material from the ear canal
should be taken for microscopic examination. PAS (Periodic Acid Schiff) stain is used which
reveals branching hyphae, budding cells or both.
Treatment:
1. Removal of debris: All the debris and discharge should be removed from ear canal by
suction or instruments like ring ended probe, blunt hook or aural dressing forceps. The
ear canal is cleaned thoroughly.
2. Topical antifungal agents: Nystatin or imidazole (such as clotrimazole or econazole
nitrate) can be used topically in the form of solution (03 drops 06 hourly) or cream with
the help of a cotton bud 12 hourly.
Another method is to insert a wick soaked in imidazole prepration for a few days and this
should be changed at frequent intervals.
In chronic cases, the fungi may penetrate into deeper layers of skin and may be difficult
to eradicate with above-mentioned treatment. So, in these cases, 02% salicylic acid in
rectified spirit is used 3 drops every 6 hours. Salicylic acid is keratolytic and exposes the
fungi from deeper layers to exterior, whereas rectified spirit is fungicidal. Antifungal
treatment should be continued for 2-3 weeks even after apparent recovery to avoid
recurrence.
3. Avoidance of water: Patient is asked to avoid entry of water into the ear. This may be
achieved by blocking the ears with ear plugs meant for swimming or with cotton ball
soaked in oil, liquid paraffin or petroleum jelly (e.g., vaseline) before taking bath. Patient
is asked to thoroughly mop ears after bath.
4. Avoid poking and scratching of the external meatus: As this may cause abrasion of skin
and penetration of the fungi into the deeper layers from where it is very difficult to
27
eradicate the fungi.
If there is associated otitis externa then the treatment is same as that for diffuse otitis
externa.
SELF ASSESSMENT
Q. 1. Otomycosis is usually caused by

a. actinomycetes d. rhinosporidiosis

b. candida albicans e. sporotrichosis


c. mucormycosis
Q. 2. Otomycosis is usually caused by
a. actinomycetes d. rhinosporidiosis

b. aspergillus e. sporotrichosis
c. mucormycosis
Q. 3. The most common symptoms of otomycosis are
a. bleeding and earache d. severe itching and ear discharge

b. earache and vertigo e. tinnitus and earache


c. hearing loss and tinnitus
Q. 4. All of the following are used in otomycosis except
a. clotrimazole d. soda glycerine ear drops

b. econazole e. 02% salicylic acid in rectified spirit


c. nystatin
Q. 5. For how long treatment of otomycosis should be continued even after apparent
recovery?
a. 2-3 days d. 3-4 months

b. 2-3 weeks e. 4-6 months


c. 2-3 months
Q. 6. Treatment of otomycosis includes all of the following except
a. avoidance of water into ear d. removal of debris

b. avoidance of poking ear e. topical antifungal agents


c. frequent irrigation of ear
Q. 7. A 31 years old female presents with 06 days’ history of irritation and blockage of
right ear. On examination there is whitish debris in right external meatus, removal of
which shows macerated ear walls with intact tympanic membrane. Her treatment is
a. aural toilet and topical antibiotic ear
drops d. aural toilet and oral antifungal drugs
b. aural toilet and topical antifungal ear
drops e. topical steroid ear drops

c. aural toilet only


28
Scenario: A swimmer presents with itching in both ears with scanty thin ear discharge for
last few weeks. Otoscopy shows whitish fluffy material in both external ear canals.
a. What is the most likely clinical diagnosis?
b. How will you treat this patient?
c. What measures can be done to prevent its recurrence?
ANSWERS---------------------------------------------------
-------------------------------------------------------- ---
Q.1. b 2. b 3. d 4. d 5. b
6. c 7. b
Scenario: 1. Otomycosis 2. Removal of debris, topical antifungal agents 3. Antifungal
treatment should be continued for 02-03 weeks even after apparent recovery to avoid
recurrence, avoid entry of water in ear, avoid scratching of ear.
--------------------------------------------------------------------------------------------------------------------------
SEBORRHEIC OTITIS EXTERNA
Etiopathology:
Seborrheic dermatitis, commonly referred to as scurf or dandruff, is characterized by scaling
or erythema of the external auditory canal. Similar findings are usually present on the scalp,
postauricular sulcus and adjacent areas of the face. The responsible organism is pityrosporum
species of fungi.

Clinical features:
These are same as that for otomycosis.
Treatment:
It is same as that for otomycosis. In addition, the patient is advised to
control the scalp condition by regular shampooing at least twice
weekly with anti-dandruff shampoos or creams containing antifungal
agents like miconazole, ketoconazole or itraconazole.
SELF ASSESSMENT A case of seborrheic
Q. 1. The responsible organism of seborrheic dermatitis is otitis externa

a. Pityrosporum species of bacteria


b. Pityrosporum species of fungi
c. Pityrosporum species of protozoa
d. Pityrosporum species of virus
e. Spirochetes

--------------------------------------------------------ANSWERS-----------------------------------------------------
29
Q.1. b
--------------------------------------------------------------------------------------------------------------------------

OTITIS EXTERNA HEMORRHAGICA


Synonyms: Bullous myringitis, Myringits bullousa hemorrhagica
Etiopathology:
Exact etiology is unknown. It is thought to be a result of viral
infection particularly influenza virus.
The condition is characterized by formation of hemorrhagic
blebs on the tympanic membrane and deep meatus. The blebs
appear purple due to hemorrhagic effusion in the vesicles.

Clinical features:
1. Very severe pain in the ear is the usual presenting symptom.

A case of otitis
externa
hemorrhagica

2. There may also be complaint of serosanguineous discharge from ear as a result of


rupture of the blebs.
In this condition, pain is not relieved with the onset of ear discharge. Whereas in acute
otitis media pain is relieved by onset of ear discharge. If uncomplicated, the middle ear is not
involved and hearing remains almost normal.
On examination;
1. Hemorrhagic blebs are visible on the tympanic membrane and deep meatus.
2. Tuning fork tests may show mild conductive hearing loss.
Treatment:
1. Analgesics are given to relieve pain.
2. Ear is kept dry and clean.
3. Blebs should not be incised as this is of no value in relieving the pain and may only
introduce secondary infection.
Antibiotics have no role on the natural course of the disease however, may be given to
prevent secondary infection.
Differential diagnosis:
1. Acute otitis media
2. Glomus tumor
3. Diffuse otitis externa

SELF ASSESSMENT
30
Q. 1. Otitis externa hemorrhagica is also known as
a. bullous myringitis d. Both a & b

b. malignant otitis externa e. Both a & c


c. myringitis bullousa hemorrhagica
Q. 2. All are true about otitis externa hemorrhagica except
a. blebs should not be incised d. pain relieved by onset of ear dis-
b. caused by virus charge
c. hemorrhagic blebs on tympanic mem- e. severe pain in ear
brane
Q. 3. An adult presents with acute pain in the ear and blood-filled vesicles on the tym-
panic membrane and deep part of the meatus, the diagnosis is
a. diffuse otitis externa d. myringitis bullosa hemorrhagica

b. herpes zoster oticus e. otomycosis

c. malignant otitis externa


Scenario: A 25 years old patient presents with right sided very severe earache for last 02
days. There is also complain of right ear discharge for last one day. But earache did not
improve on onset of discharge. On examination there is serosanguineous discharge from the
ear but tympanic membrane is intact.
1. What is the most likely clinical diagnosis?
2. What is the differential diagnosis?
3. What is the treatment of this patient?
--------------------------------------------------------ANSWERS-----------------------------------------------------
Q.1. e 2. d 3. d
Scenario: 1. Otitis externa hemorrhagica 2. Acute otitis media, glomus tumor, diffuse otitis
externa 3. Analgesics, keep ear dry and clean, antibiotics for secondary infection.
HERPES ZOSTER OTICUS
Synonyms: Herpes zoster cephalicus, Ramsay Hunt syndrome, Shingles
Definition:
It is defined as herpetic vesicular rash on external meatus or pinna with facial paralysis.
Etiopathology:
It is caused by varicella zoster infection. These dormant viral particles reside in the
geniculate ganglion of the facial nerve or spiral and vestibular ganglion of the
vestibulocochlear nerve. The virus may become reactivated and cause herpes zoster oticus.
Clinical features:
1. It is characterized by a viral prodrome.
31
2. There is severe neuralgic earache. Pain may
precede vesicles for several days. This is called as
preherpetic neuralgia. In some cases, pain may
subside when eruptions develop but may persist
for days, weeks or even months afterwards when it
is called as postherpetic neuralgia.
3. There are vesicles on the pinna. The distribution of
vesicles depends on which particular sensory
afferent fibers are involved in viral eruption.
4. Facial paralysis occurs and peaks in 10-14 days. Recovery of Ramsay hunt syndrome
facial nerve occurs in 60% of patients.
5. Anesthesia of face may also occur due to involvement of trigeminal nerve.
6. There may also be tinnitus, vertigo and hearing loss due to involvement of
vestibulocochlear nerve.
Investigations:
Diagnosis is made on clinical grounds.
Treatment:
1. Steroid antibiotic creams are applied on vesicular eruptions.
2. Acyclovir 800 mg, after every five hours is started within 72 hours of onset of vesicles.
3. Strong analgesics are given to control pain. Narcotic analgesics may be required to
relieve pain. Topical application of 05% xylocaine gel may be helpful.
4. Management of facial paralysis is the same as that for Bell’s palsy.
Prevention:
Herpes zoster is reactivation of chicken pox virus. Prevention of chicken pox by
immunization has almost eliminated Herpes zoster.
Differential diagnosis:
1. Bell’s palsy
2. Acute otitis media
3. Diffuse otitis externa
4. Otitis externa hemorrhagica
Difference between herpes zoster oticus and Bell’s palsy
Sr.No Features Herpes zoster oticus Bell’s palsy
1. Cause Varicella zoster virus Unknown
2. Family history Negative Positive
3. Symptoms Facial paralysis peaks in 10- Facial paralysis peaks in 5-10
14 days days
32
4. Recovery 60% 80%
5. Recurrence Rare May occur
6. Treatment Acyclovir and steroids are Symptomatic treatment and
mainstay of treatment. physiotherapy.

SELF ASSESSMENT
Q. 1. Herpes zoster oticus is also called

a. herpes zoster cephalicus d. all of the above

b. Ramsay Hunt syndrome e. none of the above


c. Shingles
Q. 2. In Ramsay Hunt syndrome, herpes zoster involves which of the following ganglia?
a. gasserian ganglia d. spiral ganglia

b. geniculate ganglia e. stellate ganglia


c. scarpa’s ganglia
Q. 3. All are true about herpes zoster oticus except
a. caused by varicella zoster virus d. severe neuralgic pain in ear
b. diagnosis is on clinical grounds e. vesicles formation on the pinna
c. good prognosis for facial paralysis
Q. 4. Features of Ramsay Hunt syndrome are
a. facial paralysis, otalgia, foul-smelling d. facial paralysis with long standing
discharge bloodstained ear discharge
b. facial paralysis, otalgia and vesicles on e. sudden facial palsy with acute upper
pinna. respiratory infection
c. facial paralysis with CSF otorrhoea and
sensorineural hearing loss
Q. 5. A young man who recently developed complain of severe pain in the ear, vesicles in
the concha and ipsilateral facial palsy, the likely diagnosis is
a. aero-otitis media d. malignant otitis externa

b. acute otitis media e. myringitis bullosa hemorrhagica

c. herpes zoster oticus


Scenario: An adult patient develops excruciating pain on left side of face and vesicles
formation on auricle with ipsilateral facial weakness for last three days.
1. What is the most probable clinical diagnosis?
2. What are three differential diagnosis of the disease?
3. How will you treat such a case?
33
-----------------------------------------
--------------- ANSWERS-----------------------------------------------------

Q.1. e 2. b 3. c 4. b 5. c
Scenario: 1. Herpes zoster oticus 2. Bells palsy, acute otitis media, myringitis bullosa
3.Steroid antibiotic cream, antiviral and analgesics.
--------------------------------------------------------------------------------------------------------------------------
MALIGNANT OTITIS EXTERNA
Synonyms: Granulomatous otitis externa, Invasive
otitis externa, Necrotizing otitis externa, Skull base
cellulitis/osteitis

Etiopathology:
The term malignant is a misnomer.
It is a severe rapidly progressing infection starting
in the external auditory meatus and rapidly involves the
temporal bone and adjacent soft tissue.
It usually occurs after the age of 50 years in diabetics. It may Pathophysiology of malignant
also occur in other patients with immunosuppression like leukemia, otitis externa
lymphoma, uremia, cytotoxic drugs, steroids and malnutrition.
The most common causative organism is Pseudomonas aeruginosa. It
tends to spread along vascular channels. It produces a number of endotoxins
and enzymes like elastase and collagenase which digest vessel wall. The
infection starts as cellulitis of external auditory meatus and rapidly spreads to
adjacent soft tissues either via tympanomastoid sutures or via the clefts of
Santorini.
Facial palsy may occur due to involvement of stylomastoid foramen.
Ninth, tenth and eleventh cranial nerves are paralyzed due to involvement of
jugular foramen. Death may result from spread of infection to sigmoid sinus
and meninges.
Clinical features:
1. Initially there are features of otitis externa. A case of malignant otitis externa
2. Very severe excruciating earache is the most common
symptom.
On examination;
1. Granulations in the deeper meatus especially along the floor at
its junction of bone and cartilage.
2. Cranial nerve palsies. Facial nerve is the most common nerve paralyzed due to
involvement of stylomastoid foramen. Ninth, tenth and eleventh cranial nerves may
also be paralyzed due to involvement of jugular foramen.
Investigations:
1. Ear swab is taken for culture and sensitivity.
2. ESR is significantly raised.
34
3. Fasting blood sugar usually shows diabetes mellitus.
4. Renal and liver function tests must be done as patient are put on antibiotics for long
duration.
5. Biopsy of granulation tissue must be taken to rule out malignancy.
6. High-definition CT/MRI of temporal bones is useful to find out extent of disease.
7. Isotope bone scan; Gallium-67 scan is useful for diagnosis and follow up of the patients.
Treatment:
1. Control diabetes or other cause of immunosuppression.
2. Antibiotics to cover Psuedomonas aeruginosa such as aminoglycosides (e.g., gentamycin
40-80 mg 8 hourly), third generation cephalosporins (e.g., ceftriaxone 1-2 g 24 hourly) or
quinolones (e.g., ciprofloxacin 750 mg 24 hourly). There is rapid relief of pain following
commencement of intravenous antibiotic therapy.
3. Strong analgesics are given to relieve pain.
4. Bone sequestrum should be removed. Incision and drainage of subperiosteal abscess
should be carried out.
Recurrence of pain and development of new cranial nerve palsies and elevation of ESR
indicate reactivation of the disease. Long term treatment is usually required.
Differential diagnosis:
1. Diffuse otitis externa
2. Malignancy of ear
3. Herpes zoster oticus

SELF ASSESSMENT
Q. 1. Malignant otitis externa is also known as
a. granulomatous otitis externa d. skull base osteitis

b. invasive otitis externa e. all of the above


c. necrotizing otitis externa
Q. 2. Which of the following is not a feature of malignant otitis externa?
a. biopsy shows malignant cells d. patients are usually diabetic
b. caused by Pseudomonas aeruginosa e. patients are usually over the age of 50
c. involves facial nerve at stylomastoid years
foramen
Q. 3. Following scan is useful for diagnosis and follow up of the patients of malignant
otitis externa
a. DEXA scan b. gallium-67 scan
35

c. iodine-131 scan e. technetium-99m scan


d. MDP scan
Q. 4. Treatment of malignant otitis externa includes all of the following except
a. analgesics d. radiotherapy

b. antibiotics e. removal of sequestrum

c. correction of immunosuppression
Q. 5. Antibiotic of choice in malignant otitis externa is

a. aminoglycoside d. penicillin

b. first generation cephalosporin e. tetracyclin

c. macrolide
Scenario A: A 50 years old male complains of otalgia for the last few days which is
increasing in intensity. There is also complaint of facial asymmetry. Patient is a known
diabetic for last many years. Random blood sugar is 300 mg/dl.
1. What is the most probable clinical diagnosis?
2. What investigations will you advise?
3. How will you treat this case?
4. Name other cranial nerves that may also be involved in this condition?
B: A 55 years old female with uncontrolled diabetes mellitus develops very severe intractable
pain in and around right ear for last two weeks. Otoscopy shows granulations along the floor
in the deep meatus. Her ESR is 60 mm in first hour.
1. What is the most probable clinical diagnosis?
2. What is the differential diagnosis?
3. How will you treat this patient?
--------------------------------------------------------ANSWERS-----------------------------------------------------
Q.1. e 2. a 3. b 4. d 5. a
Scenario A: 1. Malignant otitis externa 2. Swab for culture and sensitivity, ESR, biopsy of
granulation tissue 3. Control of diabetes, antibiotics, strong analgesics, treatment of
complications like facial paralysis 4. 9th, 10th and 11th cranial nerves.
B: 1. Malignant otitis externa 2. Diffuse otitis externa, malignancy of ear,
herpez zoster oticus 3. Control of diabetes, antibiotics, analgesics, bone sequestrum
should be removed, incision and drainage of subperiosteal abscess.
--------------------------------------------------------------------------------------------------------------------------
-
PERICHONDRITIS OF PINNA
Definition:
Inflammation of perichondrium of the pinna is known as perichondritis.
36
Etiopathology:
Perichondritis usually occurs as a result of piercing of the pinna
cartilage for ornaments. Other causes of perichondritis are laceration,
prick, surgery or frost bite. It may occur as a result of extension of
infection from boil or diffuse otitis externa. Abscess formation may
occur in later stages. The most common causative organism is
Psuedomonas aeuroginosa.
Clinical features:
1. Severe pain over the pinna is the most common presenting
symptom. A case of perichondritis
2. There may be rise in body temperature.
3. On examination, pinna is red, swollen, tender and markedly deformed.
Investigations:
If there is discharge, it is sent for culture and sensitivity.
Treatment:
1. Antibiotics such as aminoglycosides, third generation cephalosporin or quinolones are
given parenterally.
2. Incision and drainage under full aseptic measures is necessary when abscess has formed
and collection is large.
3. Necrosed part of cartilage is removed. Wound is kept open by placing a drain. Aseptic
dressing is done on daily basis.

Complications of piercing ears:


1. Perichondritis of pinna
2. Hepatitis B
3. Hepatitis C
4. Keloid formation
SELF ASSESSMENT
Q. 1. The most common causative organism in perichondrititis of pinna is
a. E. coli d. Streptococcus pneumoniae

b. Pseudomonas aeroginosa e. Streptococcus viridians

c. Staphylococcus aureus
Q. 2. Antibiotic of choice in perichondritis of pinna is

a. aminoglycoside d. quinolones

b. macrolide e. tetracycline

c. penicillin
Scenario: A 15 years old female presents to you with severe pain, redness and swelling of
right pinna 02 days after piercing the ear. On examination pinna is tender and markedly
deformed.
1. What is the most probable clinical diagnosis?
2. What are other possible complications of piercing ear?
37
3. What is the treatment of this patient?
--------------------------------------------------------ANSWERS-----------------------------------------------------
Q.1. b 2. a
Scenario: 1. Perichondritis of pinna 2. Hepatitis B, hepatitis C, keloid formation
3. Antibiotics, incision and drainage, removal of necrosed cartilage, aseptic dressing on daily
basis.

-------------------------------------------------------------------

MISCELLANEOUS DISEASES OF EXTERNAL EAR


FOREIGN BODY OF EAR
Etiopathology:
Children are very curious about their surroundings and their body
orifices. These have a tendency to place toys and other household
items in their own or their playmate’s ear, nose or mouth.
Foreign bodies may be classified in different ways such as;
1. Living (animate)
2. Nonliving (inanimate)
Non living foreign bodies can be classified into;
a. Inert or irritant
Foreign body of ear
b. Organic or non organic
c. Hydrophobic or hydrophilic
Common foreign bodies include sponge, paper pieces, plastic beads, ball bearings, dry
batteries and seeds like rice, wheat, corn, maize etc. Foreign bodies are frequently impacted
at isthmus. Isthmus is the narrowest part of external meatus and is about 05 mm lateral to
the tympanic membrane. Objects lateral to the isthmus are usually easily removed.
Otitis externa may develop, if foreign body is not removed.
Clinical features:
Insects produce very distressing symptoms especially when multiple bites or stings occur.

Inert foreign bodies may be completely asymptomatic. Patient may present with
irritation, hearing loss, earache, ear discharge or tinnitus.
Dry batteries are the most dangerous foreign bodies in the ear, as the alkaline fluid
seeps out of them and produces tissue necrosis and granuloma formation.
Treatment:
Foreign body of ear should only be removed by a skilled doctor. Attempt at removal of
foreign
38
body ear by unskilled individuals may actually push it into deeper meatus or even middle ear
causing severe damage to tympanic membrane and middle ear structures. It is extremely
difficult to retrieve foreign body from these sites.
Living foreign body is initially killed by instilling a
few drops of oil, spirit, chloroform, ethanol, water or
saline. Then its removal is performed unhurriedly.
Techniques for removal of foreign body include;
a. Suction Living foreign body in
b. Instrumentation ear

c. Syringing/irrigation
d. Combination of the above
Whenever possible, try to get a template of the foreign
body. It helps in making the strategy and choosing
proper instruments for safe removal of foreign body. It
also helps in anticipating the possible complications of
the foreign body and its removal.
Use of operating microscope is very important for removal of foreign body ear.
Suction is the best method of removing foreign body ear.
Spherical or oval foreign bodies are removed with a wax hook. Irregular foreign bodies are
removed with forceps.
In cooperative patient, majority of foreign bodies can be easily removed without any
anesthesia.
In children especially when they are uncooperative or in cases where foreign bodies
which are already attempted usually general anesthesia is required for removal of foreign
body.
Foreign bodies which have gone into the middle ear or deeply impacted foreign bodies
are removed by endaural incision.
Irrigation technique is contraindicated for hydrophilic foreign bodies such as pea, bean
and other vegetable matter.
SELF ASSESSMENT
Q. 1. Most dangerous foreign body of ear is

a. dry battery d. rubber piece

b. paper piece e. sponge

c. plastic piece
Q. 2. Clinical feature of foreign body ear includes
a. earache d. irritation

b. ear discharge e. all of the above

c. hearing loss

39

Q. 3. Technique for removal of foreign body includes

a. instrumentation d. all of the above

b. suction e. none of the above


c. syringing/irrigation
Q. 4. Treatment of choice in living foreign body at home is
a. instillation of oil d. suction

b. irrigation e. syringing

c. removal with instruments


Scenario: A 05 years old child was brought to the ENT OPD, his mother told that he has
put a small dry battery in his right ear a few hours before. Patient is non cooperative and
crying.
1. Why dry batteries are more dangerous?
2. How will you manage this patient?
--------------------------------------------------------ANSWERS-----------------------------------------------------
Q.1. a 2. e 3. d 4. a
Scenario: 1. Dry batteries are the most dangerous foreign bodies in the ear, as the alkaline
fluid seeps out of them and produces tissue necrosis and granuloma formation 2. Removal of
dry battery under general anesthesia.
-------------------------------------------------------------------------------------------------------------------------
WAX
Synonym: Cerumen
Etiopathology:
The skin lining the cartilaginous portion of external auditory meatus
contains two types of glands;
1. Sebaceous glands which produce fatty acids.
2. Modified sweat (ceruminous) glands which produce peptide
secretions. Wax consists of secretions of the above-mentioned
glands, desquamated keratin, lysozymes, immunoglobulins and
hyaluronic acid. A case of wax ear
Wax is yellowish in color. If it persists in the meatus for a long time, it appears dark brown or
blackish in color due to oxidation.
Normally, it is expelled spontaneously from the canal in the form of flakes, aided by the
movements of the jaw. If the wax is pushed medially by manipulation, secretions are
excessive, environment is dusty or there are anatomical variations, the wax gets accumulated
in the ear canal.
Because of its peculiar odor, it acts as a repellant for the insects. Thus,
has got a protective function for the delicate tympanic membrane. Wax also
40
lubricates the meatus and entraps any insect that enters into it. It is acidic in nature and
hence, fungistatic and bacteriostatic.

Clinical features:
1. Earache is the most common symptom. It occurs as a result of expansion of the impacted
wax especially after swimming or taking bath.
2. Hearing loss is only produced when there is complete occlusion of the external meatus.
3. There may also be occasional complaint of tinnitus and giddiness.
4. Reflex cough may occur because of stimulation of auricular branch of vagus nerve.

5. Examination of ear canal with an aural speculum or an otoscope shows yellowish, dark
brown or blackish wax.
Treatment:
Soft wax is removed by;
1. Suction
2. Instrumentation
3. Syringing
Suction or instrumentation should be done by some
skilled person preferably using operating microscope. All
the wax should be removed from ear canal by suction or
instruments like ring ended probe, blunt hook or aural
dressing forceps.
If wax is hard, dry or impacted, patient is advised to
instill wax softener like 05% sodium bicarbonate in
equal part with glycerine (also known as soda glycerine
drops), olive oil or liquid paraffin. The drops are used 4-
6 times a day for a few days before removal.
Ceruminolytic agent like 02% paradichlorobenzine
may also be used. Hydrogen peroxide should not be used Instrumental removal of ear wax
to soften the wax as it can induce severe otitis externa.
Syringing:
Procedure: Syringing should only be done by
experienced staff members. The saline should be at about
body temperature otherwise it may stimulate semicircular
canals leading to vertigo and nystagmus which is known as
caloric effect. The patient sits before the operator with the
ear to be syringed facing him. The nozzle of the syringe is
detached and the syringe is filled with saline. The nozzle is
then tightly reapplied and air pushed out of syringe in
vertical position. Light is thrown on the patient`s ear by
means of head light. The kidney tray is held below the ear. The
Syringing of ears
pinna of the patient is pulled upwards, backwards and outwards with one hand of the operator
to straighten out the meatus. By means of the syringe, the jet of the fluid is directed towards
the posterosuperior wall of the meatus. The force used should be moderate and if the wax or
any other thing in the meatus does not come out with two or three syringefuls, the clinician
should resort to some other means. The meatus should be inspected after each attempt to see
the tympanic membrane and also to make sure that the whole wax has been removed. After
syringing, the meatus should be dried with cotton on carrier.
Indications:
1. Removal of wax
2. Removal of fungus
3. Removal of copious discharge
4. Removal of small foreign body from the external meatus
Contraindications:
1. Severe otitis externa
2. Acute otitis media
3. Fracture of base of skull
4. CSF otorrhoea
Complications of syringing:
1. Tympanic membrane perforation
2. Otitis externa
3. Stimulation of auricular branch of vagus nerve leading to cough reflex/ bradycardia
4. Caloric effect
Differential diagnosis:
1. Otomycosis
2. Diffuse otitis externa
3. Acute otitis media
SELF ASSESSMENT
Q. 1. All are true about wax ear except
d. produced by sebaceous glands and
a. acts as a lubricant modified sweat glands
b. acts as a repellant
c. expelled spontaneously e. should be frequently removed

Q. 2. All are indications of ear syringing except


a. acute otitis media c. removal of fungus
b. copious discharge from external meatus d. removal of small foreign body
e. removal of wax
Q. 3. All are contraindications of ear syringing except
42

a. acute otitis media d. severe otitis externa

b. CSF otorrhoea e. wax


c. fracture of base of skull
Q. 4. All are usual complications of ear syringing except
a. caloric effect d. tympanic membrane perforation

b. otitis media with effusion e. vagal stimulation


c. otitis externa
Q. 5. During removal of ear wax, patient suddenly developed cough, syncope, generalized
sweating and bradycardia. The cause was stimulation of
a. auriculotemporal nerve (5th cranial c. facial nerve
nerve) d. glossopharyngeal nerve
b. auricular branch of vagus nerve. (10th e. greater auricular nerve
cranial nerve)
Scenario A: A 20 years old boy complains of right sided earache and hearing loss just after
swimming.
1. What is the most probable clinical diagnosis?
2. What is the differential diagnosis?
3. What is the treatment of this patient?
B: A 50 years old lady complained of blocked left ear for last 03 days. On otoscopy she had
hard wax in ear canal. When doctors tried to remove this wax she started coughing and
procedure was abandoned due to patient’s non cooperation.
1. Why she had cough during wax removal?
2. How will you remove her wax?
3. List three contraindications of ear syringing.
--------------------------------------------------------ANSWERS-----------------------------------------------------
Q.1. e 2. a 3. e 4. b 5. b
Scenario A: 1. Wax right ear 2. Otomycosis, otitis externa, acute otitis media
3. Suction, instrumentation and syringing.
B: 1. Due to stimulation of auricular branch of vagus nerve 2. By instilling soda glycerine
drops for a few days and then suction of wax 3. Severe otitis externa, CSF otorrhoea, acute
otitis media.
--------------------------------------------------------------------------------------------------------------------------

KERATOSIS OBTURANS (KO)


Synonym: Cholesteatoma of external auditory canal
Definition:
43
It is an accumulation of large plug of desquamated keratin in the deeper part of external
meatus.
Etiopathology:
Exact etiology of keratosis obturans is unknown. It is said to be because of abnormal
desquamation of epithelium in the deeper meatus. It is also said to be due to failure of
migration of cells and so a mass of desquamated epithelium accumulates. There is a common
association between chronic sinusitis, bronchiectasis and keratosis obturans. It resembles
cholesteatoma of middle ear.

Clinical features:
1. Pain and hearing loss are the most common presenting
symptoms.
2. Ear canal is full of hard pearly white mass present in the
form of layers.

Complications:
1. The KO has got pressure effects and causes erosion of bone.
2. It leads to widening of the meatus.
3. There may be granulations on the exposed cartilaginous
meatus and otitis externa. A case of keratosis obturans
4. Facial paralysis may occur occasionally.

Treatment:
1. Removal of the KO plug is similar to wax with the help of different instruments.
Sometimes general anesthesia is required to remove the plug.
2. Any granulations should be treated appropriately.
3. Regular follow up of the patient is also necessary to prevent recurrence.
SELF ASSESSMENT
Q. 1. All are true about keratosis obturans except
a. abnormal desquamation of epithelium d. needs regular follow-up

b. causes hearing loss e. occurs in superficial meatus

c. causes pain
Q. 2. All are true about keratosis obturans except

a. association with bronchiectasis d. non-recurrent

b. association with sinusitis e. pearly white mass in deeper meatus

c. causes widening of meatus


Q. 3. All are true about keratosis obturans except
44

a. may be associated with facial paral otitis externa

Ysis
d. may be associated with otitis externa

b. may be associated with granulations e. may cause erosion of bone

c. may be associated with malignant


Scenario: A 30 years old male patient presents to you with right sided progressive hearing
loss and earache for last one month. There is no other aural complaint. On examination there
is cholesteatoma like mass found filling the deep meatus. The mass has pearly white surface.
1. What is the most probable clinical diagnosis?
2. What are the possible complications of the mass?
3. How will you treat the patient?
--------------------------------------------------------ANSWERS-----------------------------------------------------
Q.1. e 2. d 3. c
Scenario: 1. Keratosis obturans 2. Erosion of bone, widening of meatus, facial paralysis
3. Removal of plug and regular follow up.
--------------------------------------------------------------------------------------------------------------------------

MEATAL ATRESIA AND MEATAL STENOSIS


Etiopathology:
Meatal atresia and stenosis may be classified into;
1. Congenital e.g., Treacher Collins syndrome, Goldenhar
syndrome
2. Acquired
a. Trauma
o Surgical e.g., of external auditory canal or mastoid A case of meatal atresia
and meatal stenosis
o Non-surgical e.g., fracture of tympanic plate, corrosive or
chemical burn

b. Inflammatory e.g., acute and chronic otitis externa


c. Neoplastic e.g., osteoma and exostosis
d. Idiopathic

Investigations:
1. Audiometric tests are done to identify whether cochlear and eighth nerve are functional
or not.
2. CT/MRI is done to assess status of middle and inner ear.

Treatment:
45
1. In cases of congenital meatal atresia, treatment consists of functional (hearing)
rehabilitation and cosmesis. Functional (hearing) rehabilitation may be done with bone
anchored hearing aid (BAHA). Cosmesis may be improved with plastic surgery or
prosthesis.
2. In cases of acquired meatal atresia, treatment is carried out only when there is
marked symptom of hearing loss or when toilet and inspection of post mastoidectomy
cavity is impossible. Meatoplasty is performed with postaural incision. Thickened skin
and scar tissue is removed. Bony meatus is enlarged and meatal wound is covered with a
flap or split skin graft.
OSTEOMA
This is a unilateral, solitary, smooth, pedunculated benign tumor in
the outer part of bony meatus. It is composed of cancellous bone.
Exact cause is unknown. It arises from only one wall of external meatus.
Treatment:
A case of osteoma
If there is no symptom, no treatment is required.
If there is symptom of hearing loss, as much of osteoma as possible should be removed by
fracturing the pedicle. It may be done by transcanal or endaural approach.
EXOSTOSIS
These are bilateral, multiple (usually three), smooth, sessile, hemispherical
elevations in the deeper meatus adjacent to tympanic membrane. These are
composed of compact bone.
There is strong association between cold water bathing and formation of meatal
exostosis.
A case of exostosis
When small, these do not cause any symptom. When large, these
completely block the meatus and there is collection of wax and epithelial debris leading to
conductive hearing loss.
Treatment:
When exostosis is small, no treatment is required.
When large enough to cause symptoms, these should be removed with high-speed drill
through a postaural approach. This is known as canal plasty.
Differences between osteoma and exostosis
S.No Features Osteoma Exostosis
1 Laterality Unilateral Bilateral
2. Number Solitary Multiple
3. Shape Pedunculated Sessile

CONCEPT BOOK OF DISEASES OF EAR, NOSE & THROAT BY FAHIM AWAN


46 Part B Ear

4. Site Superficial part of bony Deeper part of bony meatus


meatus
5. Bone Cancellous Compact
6. Treatment Fracture of osteoma Drilling and canal plasty
SELF ASSESSMENT
Q. 1. All are true about osteoma of external meatus except
a. arises from superficial meatus d. solitary

b. multiple e. unilateral
c. pedunculated
Q. 2. What is true about osteoma of external meatus

a. arises from cancellous bone d. all of the above

b. benign tumor e. none of the above

c. removed by fracturing
Q. 3. All are true of exostosis of external meatus except

a. arises from deep meatus d. sessile

b. bilateral e. solitary
c. multiple
Q. 4. What is true about exostosis of external meatus
a. arises from compact bone d. all of the above

b. removed by drilling e. none of the above

c. seen in cold water swimmers


Scenario A: A 40 years old female presents to you with right sided progressive hearing loss
for last one year. There is no other aural complaint. On examination there is bony
pedunculated mass in the superficial bony meatus.
1. What is the most probable clinical diagnosis?
2. What is the etiopathology of this entity?
3. How will you treat this patient?
B: A 50-year-old cold water swimmer presents to you with bilateral progressive hearing loss
for last one year. There is no other aural complaint. On examination there are three bony
sessile masses in the deep bony meatus.
1. What is the most probable clinical diagnosis?
2. What is the etiopathology of this entity?
3. How will you treat this patient?
--------------------------------------------------------ANSWERS-----------------------------------------------------
Q.1. b 2. d 3. e 4. d
CONCEPT BOOK OF DISEASES OF EAR, NOSE & THROAT BY FAHIM AWAN
47 Part B Ear

Scenario A: 1. Osteoma 2. Unknown 3. Removed by fracturing the pedicle. It may be done by


transcanal or endaural approach.
B: 1. Exostosis 2. Cold water bathing 3. Removed with high speed drills through a postaural
approach. This is known as canal plasty.
------------------------------------------------------------------------------------------------------------------------

Chapter 4
DISEASES OF MIDDLE EAR CLEFT
INJURIES
FRACTURES OF TEMPORAL BONE
Fractures of temporal bone are divided into longitudinal, transverse and mixed fractures
depending on the relationship of fracture line to the long axis of petrous temporal bone.
There may be discoloration of the skin over the mastoid region which is known as Battle
sign.
A. Longitudinal fractures:
These are more common (about 80%). These are relatively less dangerous. Fracture line is
along the long axis of petrous temporal bone. It involves tympanic cavity, tympanic
membrane and bony external meatus.
Later on, cholesteatoma may develop as a result of invasion of middle ear cleft by
squamous epithelium.
Clinical features:
1. Bleeding from ear may occur as a result of laceration of
meatal skin or through a ruptured tympanic membrane. If
fracture does not reach or tear the tympanic membrane, the
blood is retained in the middle ear (hemotympanum) and is
visible as blue or black tympanic membrane.
CT-Longitudinal fracture of
2. Conductive hearing loss usually occurs due to ossicular temporal bone
disruption.
3. CSF otorhinorrhoea is usually common. Facial nerve injury is uncommon and when
occurs, is usually delayed in onset.
B. Transverse fractures:
These are less common (about 20%). These are relatively more dangerous. Fracture line is
at right angle to the long axis of the petrous temporal bone. It involves the labyrinth or
internal auditory meatus.
Clinical features:
48
1. Hemotympanum (blood in the middle ear) is usually present.
2. Sensorineural hearing loss which is usually permanent. It is accompanied by tinnitus.
3. Facial paralysis may occur in about 50% of patients. Onset is usually immediate.
There is complaint of severe rotatory vertigo with nausea and vertigo due to damage to
vestibular apparatus. There is nystagmus with fast component to the opposite side.
Severe general injuries (especially head injury) of the patient may dominate the clinical
picture in cases of transverse fracture of temporal bone.
Investigations:
Bleeding from the ear after head injury indicates the presence
of fracture until proven otherwise. Plain radiographs may not
be sufficient for diagnosis.
1. CT is usually required to diagnose the type and extent of
fracture of temporal bone. CT Scan-Transverse
fracture of temporal bone
2. MRI may be required for confirming the continuity of facial
nerve.
3. In cases of facial paralysis, electrophysiological tests may also be required.
Treatment:
Initial treatment is the same as that for head injury.
1. Multidisciplinary approach is usually required.
2. I.V. antibiotics are given. It is better to leave any blood clot untouched in the external
auditory meatus. The ear should be protected with a sterile dressing.
3. CSF leakage and ruptured tympanic membrane are managed conservatively.
4. Ossicular reconstruction may be required in cases of ossicular dislocation. Fracture of
the bony meatus may need reduction of fracture or excision of the displaced fragments at
a later date.
5. Exploration of the facial nerve is considered in cases of immediate and total facial
paralysis.
6. Vertigo may settle in 2-4 weeks in young patients, whereas older patients may remain
unsteady for a few months. During this period, labyrinthine sedative drugs are given.
SELF ASSESSMENT
Q. 1. In cases of fracture of temporal bone discoloration of the skin over the mastoid
region is known as
a. Battle sign b. Hennebert sign
49

c. Hitselberger sign e. Tripod sign


d. Thumb sign
Q. 2. All are true about longitudinal fracture of temporal bone except
a. constitutes a large percentage of frac- d. usually causes conductive hearing
tures of temporal bone loss
b. more chances of facial paralysis e. usually causes CSF otorhinorrhoea
c. usually causes bleeding from ear
Q. 3. All are true about transverse fracture of temporal bone except
a. constitutes a small percentage of d. usually causes facial paralysis
fractures of temporal bone e. usually causes sensorineural hearing
b. usually causes CSF otorhinorrhoea loss
c. usually causes hemotympanum
Q. 4. An eighteen years old student involved in RTA had a brief episode of loss of
consciousness. On regaining consciousness, he is found to have right sided facial
paralysis, hemotympanum and sensorineural hearing loss on same side. CT scan would
most likely reveal
a. fracture of cribriform plate d. transverse fracture of temporal bone
b. longitudinal fracture of temporal bone e. vertical fracture of temporal bone
c. oblique fracture of temporal bone
Scenario: A 45 years old patient presents to you with right sided facial paralysis following
an RTA 02 days ago. On examination there is hemotympanum.
1. What is the most probable clinical diagnosis?
2. How will you confirm the diagnosis?
3. What are other clinical otological features associated with this injury?
--------------------------------------------------------ANSWERS-----------------------------------------------------
Q.1. a 2. b 3. b 4. d
Scenario: 1. Transverse fracture of right temporal bone 2. CT scan temporal bone
3. Sensorineural hearing loss, nausea, vomiting, vertigo and nystagmus with fast component
to the opposite side.
--------------------------------------------------------------------------------------------------------------------------
OTITIC BAROTRAUMA
Synonyms: Barotraumatic otitis media, Aero-otitis media
Definition:
It is a non-infective condition of middle ear cleft resulting from failure of the eustachian tube
to maintain middle ear pressure equal to that of atmospheric pressure. It is produced by rapid
50
fall in intratympanic pressure.
Etiopathology:
It occurs while flying, especially during descent, diving or blast.
Normally pressure of middle ear is maintained equal to that of
atmospheric pressure by tube. The tube temporarily opens by
contraction of the levator and tensor palati muscles as during
swallowing or yawning. When atmospheric pressure is higher
than that of middle ear by critical level of more than 90 mmHg,
the eustachian tubes become “locked” which cannot be
overcome by muscular action.
Locking may occur at a lower pressure difference when the tubal lining is already
edematous. There is persistence of negative pressure in the middle ear initially causing
retraction of the tympanic membrane. Further reduction in pressure occurs when the
remaining air in the middle ear is absorbed. This is followed by venous engorgement, edema,
ecchymosis and transudation within the middle ear. In severe cases, tympanic membrane may
rupture.
Clinical features:
1. Sensation of fullness in the ear, hearing loss and earache are usual
presenting symptoms. There may be sensation of fluid in the ear.
2. There may be complaint of tinnitus and vertigo. A case of otitic
barotrauma
On examination;
1. Tympanic membrane is initially red which may be retracted.
2. There may be effusion in the middle ear, which may appear as air fluid level or air
bubbles, visible through the tympanic membrane.
3. Hemotympanum may be present and tympanic membrane is dusky blue in color.
4. Tympanic membrane may be perforated in very severe cases.

Investigations:
1. PTA shows conductive hearing loss.
2. Tympanogram shows a negative middle ear pressure (type B or C graph).
Treatment:
1. Avoid travelling in a non-pressurized airplane especially when having an upper
respiratory tract infection.
2. If there is nasal congestion and upper respiratory tract infection, it should be treated with
topical or systemic decongestants, antibiotics and anti-inflammatory drugs.
Decongestants e.g., pseudoephedrine 120 mg should be used before descent.
3. Avoid sleep during descent (as frequency of swallowing movements and hence, opening
of eustachian tube is decreased during sleep).
51
4. Use of sweets and chewing gum is encouraged before descent. Autoinflation of the
eustachian tube is encouraged by Valsalva maneuver. If there is no fluid in the middle
ear, autoinflation is usually sufficient.
5. Middle ear aeration may be restored by catheterization and Politzerization.
6. Myringotomy is sometimes performed if there is fluid in the middle ear.
7. Grommet insertion may be required in resistant cases.
8. If the tubes are “locked”, “unlocking” may be possible only when the flight is taken to
the previous height (i.e., pressure) at which the tubes were locked. This is followed by a
gradual descent to avoid ‘relocking’.
9. When the tympanic membrane is already ruptured, it is treated as a case of traumatic
rupture of tympanic membrane.
SELF ASSESSMENT
Q. 1. Otitic barotrauma is also known as

a. aero-otitis media d. Both a & b

b. barotraumatic otitis media e. Both b & c

c. serous otitis media


Q. 2. Otitic barotrauma may be caused by all except

a. blast d. rapid descent of flight

b. diving e. slow descent of flight

c. rapid ascent of flight


Q. 3. All are usual symptoms of otitic barotrauma except

a. earache d. sensation of fullness in the ear

b. hearing loss e. sensation of fluid in the ear

c. loss of facial sensation


Q. 4. All are signs of otitic barotrauma except

a. air bubbles and air fluid level d. positive Schwartz sign

b. hemotympanum e. red and retracted tympanic mem-

brane
c. perforated tympanic membrane
Q. 5. For locking of healthy eustachian tube, the critical pressure difference between
middle ear and nasopharynx in barotrauma is
a. 30 mmHg d. 110 mmHg

b. 60 mmHg e. 130 mmHg


c. 90 mmHg
Q. 6. All are true about investigations of otitic barotrauma except
a. PTA shows conductive hearing loss b. tympanogram may show negative middle
ear pressure
52

c. tympanogram may show type A curve e. tympanogram may show type C curve
d. tympanogram may show type B curve
Q. 7. All are true about management of otitic barotrauma except
a. to avoid travelling during upper respira- plane
tory tract infection d. should use sweets and chewing gums
b. to avoid sleep during descent e. should perform Valsalva maneuver
c. should travel in non pressurized air-
Q. 8. All are true about management of otitic barotrauma except
a. catheterization and politzerization. d. nasal decongestants

b. gradual descent e. rapid descent

c. myringotomy
Scenario: A 25 years old lady presents to you with bilateral hearing loss and tinnitus after an
air travel. She had URTI preceding the journey. On examination there is hemotympanum.
1. What is the most probable clinical diagnosis?
2. What is the etiopathology of this entity?
3. How will you treat this patient?
------------------------------------------- ANSWERS--------------------
------------- ---------------------------------
Q.1. d 2. c 3. c 4. d 5. c 6. c 7. c 8. e
Scenario: 1. Otitic barotrauma 2. “Locking” of eustachian tube while flying 3. Avoid travel
in non pressurized airplane, topical or systemic decongestants, avoid sleep during descent,
chewing gums use.
--------------------------------------------------------------------------------------------------------------------------
TRAUMATIC PERFORATION OF TYMPANIC MEMBRANE
Etiopathology:
It may occur by;
1. Air pressure
2. Fluids
3. Solid objects
1. Air pressure:
This usually occurs from a rapid change in air pressure. The most
common cause is a blow on ear usually with a slap. Sometimes, a blow on
the ear from a ball or a fall on water can lead to perforation of the
tympanic membrane.
Valsalva maneuver
Blast injury of ear and the barotrauma may also lead to perforation of
the tympanic membrane. Eustachian tube inflation by Valsalva maneuver or a eustachian
tube catheter may also lead to perforation of the tympanic membrane especially if the
tympanic membrane
53 Part B Ear

is already weakened by the previous disease.


Perforation due to air pressure changes commonly occurs in the anteroinferior
quadrant of the tympanic membrane.
2. Fluids:
Syringing is a very common example of perforation of the tympanic membrane. Water
sports or skin diving are other examples of tympanic membrane perforation.
3. Solid objects:
Foreign bodies and especially attempts at their removal may lead to perforation of the
tympanic membrane.
Similarly match sticks and hair pins may lead to perforation of the tympanic membrane.
There may be persistent perforation after extrusion of a middle ear
ventilation tube.
Clinical features:
Earache, conductive hearing loss and aural bleeding or
bloodstained discharge are common presenting symptoms.
1. Secondary infection of middle ear is the most common
complication of traumatic perforation of tympanic
Traumatic perforation of TM
membrane.
2. Ossicular dislocation with conductive hearing loss or inner
ear damage with sensorineural hearing loss and tinnitus may
occur depending on the force of injury.

Investigations:
Pure tone audiometry is required to find out the degree of hearing loss. Conductive hearing
loss of more than 30 dB usually indicates ossicular damage.
Treatment:
Usually, the prognosis in traumatic perforation is excellent.
1. Passive approach:
The most effective management is to do nothing. The ear should be kept dry. No ear
drops should be prescribed. Ear should not be cleaned out, unless contaminated material
is found in the external meatus. Systemic antibiotic plus systemic analgesic are given.
Traumatic perforation usually heals spontaneously within 10 weeks.
2. Active approach:
1. Eversion of the edges of the perforation may be carried out under operating
microscope as an alternative to passive approach in early stages.
2. Myringoplasty i.e., surgical closure of perforation is carried out if there is no
spontaneous healing in three to six months.
54
3. If there is facial paralysis or subluxation of stapes (as indicated by, vertigo,
nystagmus and sensorineural hearing loss) then urgent surgical exploration is
required.
SELF ASSESSMENT
Q. 1. Traumatic perforation of tympanic membrane may be due to

a. air pressure d. all of the above

b. fluids e. none of the above


c. solid objects
Q. 2. Traumatic perforation of tympanic membrane may be because of following except
a. air pressure changes such as slap d. otitis media with effusion

b. blast e. solid objects such as hair pin


c. fluids such as syringing
Q. 3. All of the following are common clinical features of traumatic perforation of
tympanic membrane except
a. aural bleeding d. CSF otorrhoea

b. bloodstained ear discharge e. earache


c. conductive hearing loss
Q. 4. All are usual complications of traumatic perforation of tympanic membrane except
a. conductive hearing loss d. secondary infection of middle ear

b. facial anesthesia e. sensorineural hearing loss


c. ossicular dislocation
Q. 5. Traumatic rupture of tympanic membrane usually heals spontaneously within
a. 05 days d. 06 months

b. 10 days e. 01 year
c. 10 weeks
Q. 6. Passive treatment of dry traumatic rupture of tympanic membrane of 01 days
duration is
a. antibiotic ear drops d. protection of ear against entry of
b. ear wick soaked with antibiotic water
c. myringoplasty e. regular cleaning of ear
Q. 7. The most suitable active treatment in traumatic perforation of tympanic membrane
of 01-day duration is
a. eversion of edges of tympanic mem- b. systemic antibiotics
brane under operating microscope
c. systemic steroids e. topical steroids
d. topical antibiotics
Q. 8. Treatment of choice for dry traumatic rupture of tympanic membrane of 06 months
duration is
55

a. antibiotic ear drops d. myringotomy

b. ear pack soaked with antibiotic e. protection of ear against water


c. myringoplasty
Q. 9. A 27 years old housewife presents in ENT OPD with 06 hours history of blunt
trauma over right ear with a slap, followed by severe earache along with hearing loss.
She is diagnosed as a case of traumatic perforation of right tympanic membrane.
What treatment will you offer her?
a. emergency myringoplasty algesic

b. systemic analgesic plus PTA d. topical antibiotic plus analgesic drops

c. systemic antibiotic plus systemic an- e. topical steroids plus PTA


Scenario: A 23 years old player presented in ENT OPD with 06 hours history of a blow on
the left ear with a football, followed by severe pain, aural bleeding along with hearing loss.
1. What is the most probable clinical diagnosis?
2. How will you manage this patient?
------------------------------- ANSWERS-----------------------
------------------------- ---------------------------
Q.1. d 2. d 3. d 4. b 5. c 6. d 7. a 8. c 9. c
Scenario: 1. Traumatic perforation of tympanic membrane 2. Systemic antibiotic plus
systemic analgesic, ear is kept dry, no ear drops, will wait for 10 weeks for spontaneous
healing, myringoplasty if no healing occurs.
--------------------------------------------------------------------------------------------------------------------------
BLAST INJURIES
Etiopathology:
Explosive material changes suddenly from solid to gaseous form
with a massive increase in volume and pressure. It results in a shock
wave spreading out from the site of explosion.

There is a short-lived positive pressure phase and then a longer and Traumatic
less marked negative pressure phase. Damage occurs both as a result perforation following
of high intensity of noise and the shock wave. The ear facing the blast injury
bomb tends to be more severely damaged. There may be damage to both middle and
inner ears. In most cases sensorineural hearing loss rapidly resolves but high frequency
losses may persist.
Clinical features:
1. There may be hyperemia and even subepithelial bleeding in the tympanic membrane.
2. There may be conductive as well as sensorineural hearing loss.
3. Perforation of tympanic membrane is usually seen in the pars tensa.
Investigations:
Pure tone audiometry is required to find out the type and degree of hearing loss.
56
Conductive hearing loss of more than 30 dB usually indicates ossicular damage.
Treatment:
Most blast injuries of the tympanic membrane heal spontaneously with conservative
measures. Occasionally, myringoplasty or tympanoplasty may be required to close the
perforation or repair ossicular chain respectively.
SELF ASSESSMENT
Q. 1. In blast injury, hearing loss occurs as a result of
a. high intensity noise d. Both a & c

b. low intensity noise e. Both b & c

c. shock wave
Q.2. All are true about blast injury except
a. conductive hearing loss d. sensorineural hearing loss
b. perforation of pars flaccida e. subepithelial bleeding in tympanic
c. perforation of pars tensa membrane
Q. 3. All are true about blast injury except
a. conductive hearing loss of more than 30 c. myringoplasty is required usually to close
dB usually indicates ossicular damage the tympanic membrane perforation
b. myringoplasty may be required occa- d. pure tone audiometry shows type and
sionally to close the tympanic membrane degree of hearing loss
perforation
e. tympanoplasty may be required to
repair ossicular chain
Scenario: A 15 years old boy presents in ENT emergency 15 minutes after a bomb blast.
There is bilateral severe degree of hearing loss and aural bleeding.
1. What is the most probable clinical diagnosis?
2. How will you investigate this patient?
3. How will you manage this patient?
--------------------------------------------------------ANSWERS-----------------------------------------------------
Q.1. d 2. b 3. c
Scenario: 1. Perforation of tympanic membrane after blast injury 2. Pure tone audiometry
3. Wait for spontaneous healing, myringoplasty may be required to close the tympanic
membrane perforation or tympanoplasty may be required to repair ossicular chain defect.
--------------------------------------------------------------------------------------------------------------------------
OTITIS MEDIA
ACUTE OTITIS MEDIA /ACUTE SUPPURATIVE OTITIS MEDIA
Definition:
57
Acute otitis media is defined as acute
inflammation of the mucous membrane
lining the middle ear cleft i.e., eustachian
tube, tympanic cavity, mastoid antrum and
mastoid air cells.
When there is drainage of mucopus from
the middle ear, it is labelled as acute
suppurative otitis media.
Etiopathology:
The disease is commonly seen in children but
adults may also get affected. The disease
affects young children because of following
anatomical factors;
Eustachian tube is shorter, wider and
more horizontal as compared to adults. Its
nasopharyngeal end is nearer to the floor of the nose; therefore, the
secretions get an easy access to middle ear in young children. Acute otits media

Bottle fed babies especially if they are fed in lying down position, tend to develop the
disease from infection going into the middle ear along with milk.
It usually starts after an upper respiratory tract infection. It is usually followed by
pyogenic organisms. It is common in swimmers and divers. It is also seen in patients with
palatal paralysis.
The most common organisms are Streptococcus pneumoniae, Haemophilus influenzae
and Moraxella (Branhamella) catarrhalis.
Infection of the middle ear may occur through;
1. Eustachian tube: It is the chief route by which infection reaches the middle ear. The
cause in such cases is rhinitis, sinusitis, pharyngitis or tonsillitis.
2. Perforated tympanic membrane or through a ventilation
tube (grommet).
3. Blood borne rarely
The type of inflammatory reaction and its progress i.e., whether
suppurative or non suppurative depends on;
1. Virulence of the organism
2. Resistance of the patient
3. Treatment particularly with antibiotics.
Clinical features:
There is usually a preceding history of upper respiratory tract
A case of ASOM
infection.
Usual symptoms of acute otitis media in an infant include pulling at ears, excessive crying,
disturbed nights and feeding difficulty.
There are different stages of the disease;
A. In the initial stages:
1. Sensation of fullness in the ear is the most common presentation.
2. There may be complaint of hearing loss.
58
On examination;
1. There is hyperemia of the tympanic membrane.
2. Tuning fork tests show conductive hearing loss.
B. In the late stages:
1. Severe stabbing earache which usually indicates degree of tension of tympanic
membrane. If perforation of tympanic membrane occurs, there is mucopurulent
bloodstained discharge from the ear and earache is relieved due to decrease in
tension of tympanic membrane.
2. There may be constitutional symptoms in the form of rise in body temperature,
pulse rate and malaise.
On examination;
1. Tympanic membrane may be bulging and dull in appearance.
2. There may be central perforation of tympanic membrane in the posterosuperior
quadrant.
3. Tuning fork tests show conductive hearing loss.
Complications:
1. Otitis media with effusion
2. Chronic suppurative otitis media
3. Intracranial complications

4. Extracranial complications

Investigations:
1. Culture and sensitivity of the discharge from the ear.
2. X-ray mastoid lateral oblique view is done to find out haziness of mastoid air cells, bone
erosion, position of the sigmoid sinus/dural plate and sinodural angle.
3. Tympanometry may be done to confirm the presence of fluid in the middle ear.
4. Pure tone audiometry may be performed but should be deferred until acute stage is
over.
5. CT scan of temporal bones including brain must be carried out if complications of acute
otitis media are suspected.

Treatment:
1. Antibiotics like ampicillin or amoxicillin are given for at least 10 days or till tympanic
membrane regains normal appearance and hearing returns to normal.
2. Analgesics are given to relieve pain.
59
3. Topical/systemic decongestants help in reducing congestion around eustachian tube and
promote drainage and aeration of middle ear.
4. Myringotomy is done if tympanic membrane is not yet ruptured and bulging or if there is
persistent effusion beyond 12 weeks. It is done in the posteroinferior quadrant of
tympanic membrane. It may be done under local or general anesthesia.
5. Treatment of underlying cause such as rhinitis, sinusitis and adenoiditis.
Differential diagnosis:
1. Bullous myringitis
2. Boil ear
3. Diffuse otitis externa
SELF ASSESSMENT
Q. 1. All are components of middle ear cleft except

a. eustachian tube d. maxillary antrum

b. mastoid air cells e. tympanic cavity

c. mastoid antrum
Q. 2. All are usual causative organisms of acute otitis media except

a. Branhamella catarrhalis d. Moraxella catarrhalis

b. E. coli e. Streptococcus pneumoniae

c. Haemophilus influenzae
Q. 3. Infection of the middle ear may occur through

a. blood d. ventilation tube (grommet)

b. eustachian tube e. all of the above


c. perforated tympanic membrane
Q. 4. Acute inflammation of middle ear is more common in children because
a. eustachian tube is more horizontal d. nasopharyngeal end of eustachian
b. eustachian tube is shorter tube is nearer the floor of the nose
c. eustachian tube is wider e. all of the above
Q. 5. Usual symptoms of acute otitis media in an infant include all except
a. disturbed nights d. pain abdomen

b. excessive crying e. pulling at ears


c. feeding difficulty
Q. 6. Acute inflammation of middle ear is less common in adults because of
a. better eustachian tube function d. reduction in number of URTI

b. eustachian tube is longer e. all of the above

c. regression of adenoids
60
Q. 7. Usual clinical features of acute otitis media in an adult include all except
c. paralysis of trigeminal nerve
a. conductive hearing loss
b. mucopurulent bloodstained ear dis- d. sensation of fullness in the ear
charge e. severe earache

Q. 8. The bulging of tympanic membrane in acute otitis media is most prominent in


a. anterior half d. pars flaccida

b. anterosuperior quadrant e. posterior half


c. anteroinferior quadrant
Q. 9. All are usual clinical features of acute
otitis media except
d. tympanic membrane may show marginal
a. tympanic membrane may be bulging perforation
b. tympanic membrane may be red e. tuning fork tests show conductive
c. tympanic membrane may show central hearing loss
perforation
Q. 10. All are usual investigations of acute otitis media except
a. BERA d. tympanometry

b. pure tone audiometry e. X-ray mastoid

c. swab for culture and sensitivity


Q. 11. Differential diagnosis of acute otitis media is

a. boil ear d. all of the above

b. bullous myringitis e. none of the above


c. diffuse otitis externa
Q. 12. All are treatments of acute suppurative otitis media except
a. analgesics d. myringotomy

b. antibiotics e. radical mastoidectomy


c. decongestants
Q. 13. In acute otitis media, myringotomy is done in the following compartment
a. anteroinferior quadrant of tympanic d. posteroinferior quadrant of tympanic
membrane membrane
b. anterosuperior quadrant of tympanic e. posterosuperior quadrant of tympanic
membrane membrane
c. pars flaccida
Q. 14. A 07 years old child developed acute otitis media. He was treated with antibiotics
for 10 days. His pain and fever subsided completely but still had conductive hearing loss.
Your next line of treatment is
a. another course of a different antibiotic c.myringotomy and insert a grommet

b. myringotomy and culture the middle

ear fluid
61

d. parenteral steroids e. to wait and watch for 3 months for fluid


to drain spontaneously
Scenario A: A 16 years old boy presented during winter with nasal obstruction, sneezing,
rhinorrhoea and left otalgia for last 07 days. There is bloodstained mucopurulent discharge
from the left ear. Clinical examination revealed congested nose and after cleaning the
external auditory meatus, central perforation of tympanic membrane is seen.
1. What is the most probable clinical diagnosis?
2. What is the causative agent in this case?
3. How will you manage this patient?
B: A 02 years old child presents in emergency with 06 hours history of pulling at left ear,
crying and history of upper respiratory tract infection. On examination, his left tympanic
membrane is congested and bulging. His temperature is 39 degrees Celsius. WBC count is
14000/cubic millimeter.
1. What is the most likely clinical diagnosis?
2. What are possible complications of the disease?
3. How will you treat this patient?
-------------------------------------- ANSWERS-----------------------------------------
------------------ ------------
Q.1. d 2. b 3. e 4. e 5. d 6. e 7. c 8. e
9. d 10. a 11. d 12. e 13. d 14. e
Scenario A: 1. Acute suppurative otitis media 2. Streptococcus pneumoniae, Haemophilus
influenzae 3. Antibiotics, analgesics, topical/systemic decongestants.
B: 1. Acute otits media 2. Otitis media with effusion, chronic suppurative otitis media,
intracranial complications, extracranial complications 3. Antibiotics, analgesics, systemic
decongestants, myringotomy.
--------------------------------------------------------------------------------------------------------------------------
DEFINITIONS OF CERTAIN OTOLOGICAL PROCEDUERS
Myringotomy:
It is an operation performed to make an incision in the tympanic membrane.
Myringoplasty:
It is an operation performed to repair or reconstruct the tympanic membrane.
Ossiculoplasty:
It is an operation performed to repair or reconstruct the ossicular chain.
Tympanoplasty:
It is an operation performed to remove disease from the middle ear (not mastoid) and to
reconstruct hearing mechanism.
Tympanoplasty with mastoidectomy:
62
It is an operation performed to remove disease from the middle ear and mastoid (both)
and to reconstruct the hearing mechanism.
Cortical mastoidectomy:
It is an operation performed to remove disease from all accessible mastoid air cells while
preserving the posterior external meatus wall.
Radical mastoidectomy:
It is an operation to remove disease from the middle ear and mastoid in which middle ear,
mastoid, attic and antrum are exteriorized to the external meatus by removing the posterior
meatal wall. All the remnants of the tympanic membrane, malleus, incus (not the stapes),
chorda tympani and the mucoperiosteal lining are removed. Eustachian tube is blocked.
Modified radical mastoidectomy:
It is an operation performed to eradicate disease from the mastoid and attic, both of which are
exteriorized to external meatus by removing the posterior meatal and lateral attic walls.
Tympanic membrane remnants, ossicles, chorda tympani and eustachian tube are preserved.
If the disease is limited and there is mild hearing loss, modified radical mastoidectomy is
performed.
MYRINGOTOMY
Indications:
1. Otitis media with effusion which has failed to respond to
medical treatment. It is performed to confirm the diagnosis,
to aspirate fluid, and when necessary to insert a ventilation
tube. Incision is made in the anteroinferior or anterosuperior
quadrant of tympanic membrane.
2. Acute suppurative otitis media if there is bulging of the tym-
panic membrane and persistence of pain and pyrexia despite
adequate medical treatment. Incision is made in the postero-
inferior quadrant of tympanic membrane.
3. For reversal of retraction and retraction pockets.
4. Suspected malignant disease of the nasopharynx. Fluid aspi- Myringotomy
rated can be sent for cytology and may show malignant cells.
Procedure:
Under local or general anesthesia, position of the patient is
made, scrubbing and draping is done. Examination of ear is done
with operating microscope using as large speculum as possible.
A sharp Agneo’s myringotome is taken.
In cases of otitis media with effusion, a radial incision Agneo’s myringotome
is made in the tympanic membrane in the anteroinferior or
anterosuperior quadrant of tympanic membrane. Radial
63
incision separates rather than cuts across the fibers of middle layer of tympanic membrane
which heals more readily and with less scarring.
If a ventilation tube is inserted, a radial incision offers less barrier for epithelial
migration, so extrusion of ventilation tube is delayed.
In cases of acute otitis media, the incision is always made in the posteroinferior
quadrant of tympanic membrane. Posteroinferior incision avoids damage to the ossicular
chain, facial nerve and chorda tympani. Any pus coming out of the incision is sent for
bacteriological evaluation.
Complications of myringotomy:
There may be damage to;
1. Facial nerve
2. Ossicular chain leading to conductive hearing loss.
3. Labyrinth leading to sensorineural hearing loss, vertigo and tinnitus.

4. Damage to jugular bulb (if high lying jugular bulb).

MYRINGOPLASTY
Indications and advantages of myringoplasty:
Closure of tympanic membrane perforation;
1. Restores vibratory area of tympanic membrane.
2. Affords round window protection thus, improving hearing and lessening tinnitus.
3. Lessens susceptibility of middle ear mucosa to infections.
4. Provide sound pressure transformation.
Prerequisites of myringoplasty:
1. Eustachian tube should be functional.
2. Contributing factors such as allergic rhinitis, sinusitis and adenoiditis should be treat-
ed.
3. Ear should be dry for at least last 3-6 months.
4. Air bone gap should not be more than 30 dB.
There are different techniques of myringoplasty;
A. Onlay/overlay technique i.e., graft is placed over the perforation.
B. Underlay technique i.e., graft is placed under the perforation.
C. Sandwich technique i.e., perforation is sandwiched between the grafts.
Advantages and disadvantages of different techniques:
A. Onlay technique:
Advantages:
64
1. It can be performed through transcanal approach
2. Avoids external incision
3. Less time consuming
4. Easier preparation of bed
5. Easier application of graft
Disadvantages:
1. Risk of cholesteatoma pearl
formation
Onlay technique Underlay technique
2. Lateralization of graft
3. Anterior blunting
4. Dermoid inclusion

B. Underlay technique:
Advantages:
1. Opportunity to inspect and test the mobility of ossicular chain
2. Squamous epithelium of meatal skin and drum remnant remain lateral to grafts
3. Any intratympanic adhesion can be divided
4. Can be used where previous onlay operation has failed
Disadvantages:
1. Risk of medial prolapse of graft
2. Retraction of anterior edge
Procedure:
It is performed under local or general anesthesia. It may be performed by a
transmeatal/postaural approach. Position of the patient is made and scrubbing is done. Using
a vertical knife, all of the scar tissue is removed from around the whole circumference of the
perforation. A graft of correct size and shape is harvested. Middle ear cavities are packed
with spongostan. The graft is placed over the spongostan bed. The graft may be placed medial
to the perforation (underlay technique) or lateral to the perforation (overlay technique). The
external meatus is packed with spongostan.
If a postaural incision is used, the above-mentioned procedure is carried out using a
tympanomeatal flap.
CORTICAL MASTOIDECTOMY
Synonyms: Simple mastoidectomy, Schwartz operation

Indications of cortical mastoidectomy:


1. Subperiosteal mastoid abscess
65
2. Sagging of posterior meatal wall
3. Positive reservoir sign i.e., meatus immediately fills with pus after it has been mopped
out.
4. Mastoiditis leading to complications e.g., facial paralysis, labyrinthitis or intracranial
complications.
5. As a part of other procedure e.g.,
a. Endolymphatic sac decompression in Meniere’s disease.
b. As an approach to vestibular schwannoma.
c. Prior to cochlear implant surgery.

Procedure:
After general anesthesia, position is made, scrubbing and draping is done. Incision line is
marked and infiltrated with a solution of xylocaine plus adrenaline in ratio of 1:80,000.
Postaural crescenteric incision is made about 3-5 mm behind the retroauricular groove
starting from the upper end of attachment of pinna, to the side of the skull and ending well
above the tip of the mastoid bone.
In younger patients as mastoid is less developed, the incision is made oblique (rather
than crescenteric) to avoid damage to facial nerve. Incision is made through the skin, fascia
and periosteum. With the periosteum elevated, skin edges and periosteum are retracted
anteriorly and posteriorly. Any bleeding at the time of the incision is stopped with diathermy.
Posterior border of the bony external auditory meatus as well as suprameatal triangle and
spine are identified.
Skin from posterior meatal wall elevated and
anterior surface of posterior meatal wall exposed.
Drilling is started with an electric burr at
MacEwen triangle (also known as suprameatal
triangle). Deep to it lies mastoid antrum lies at a
depth of 1.5 cm in adults. All the diseased cells in
the region of mastoid are thoroughly drilled under
operating microscope. The cavity is saucerized
and the mastoid antrum is entered. Aditus is
enlarged without dislocating the short process of
incus so that the middle ear drains well through
the antrum. During the process of drilling,
continuous irrigation with saline is carried out to avoid any
permanent trauma to the facial nerve and to avoid bone dust MacEwen triangle
and smoke formation. The middle ear is not entered and the
chain of ossicles is not disturbed. In this procedure, the
intention is to drain the antrum and exenterate the infected
cells.
A drain is placed in antrum. The wound is stitched in layers i.e., a layer of periosteum
and deep fascia and a layer of skin and superficial fascia. Crape bandage is applied keeping
the pinna in normal anatomical position. Skin stitches are removed on 7th day. Drain is
removed usually after 48-72 hours. External auditory meatus is inspected and cleaned daily
under the operating microscope. Postoperative period should be covered with appropriate
antibiotics and analgesics.
66
RADICAL MASTOIDECTOMY
Procedure:
Initial procedure is the same as for cortical mastoidectomy.
The bone lining lateral to the aditus i.e., the scutum is drilled away with fine burr.
Posterior bony meatal wall i.e., facial ridge is gradually removed keeping in mind that the
facial nerve lies deep to it. While lowering the facial ridge, a continuous watch is made by an
assistant for any twitching of facial muscles.
The middle ear is widely exposed and the mastoid cavity is directly communicated with
the external auditory meatus at the end of the procedure. Any remains of the tympanic
membrane, the malleus and the incus but not the stapes are removed. Any dead, degenerated
tissue must be removed. If there is any cholesteatoma, it is removed along with its matrix.
Sinus tympani must be cleared of granulations and degenerated mucosa.
The cavity thus, formed is kidney shape and comprises of mastoid antrum, aditus, attic
and middle ear. It is smoothened with a diamond paste burr. All the procedure is done with an
operating microscope and with continuous irrigation with normal saline. Some surgeons
curette the opening of eustachian tube. Meatoplasty is carried out and the mastoid cavity and
external meatus is packed with ribbon gauze soaked in antibiotic. Skin stitching is done in
two layers. Pack is removed after 3-5 days.
MODIFIED RADICAL MASTOIDECTOMY
Procedure:
It is the same as for radical mastoidectomy. Cholesteatoma, diseased tissue and granulations
are removed. Usually, the tympanic membrane remnants and the healthy ossicular chain are
conserved. Meatoplasty is performed and the cavity and external meatus are packed with
ribbon gauze soaked in antibiotics, the pack is removed after 3-5 days.

MEATOPLASTY
It is an operation in which a part of the conchal cartilage is
excised to widen the external auditory meatus. It is usually done
at the end of modified radical or radical mastoidectomies for
periodic inspection and cleansing of mastoid cavity
postoperatively. An operated case of
meatoplasty
Complications of mastoidectomy (cortical, radical
or modified radical mastoidectomy):
There may be damage to;
1. Facial nerve
67
2. Labyrinth leading to sensorineural hearing loss, vertigo and tinnitus
3. Dura and CSF leakage
4. Ossicular chain leading to conductive hearing loss in cortical mastoidectomy
SELF ASSESSMENT
Q. 1. All of the following are indications of myringotomy except
a. acute otitis media d. otitis media with effusion

b. acute suppurative otitis media e. reversal of retraction pocket


c. chronic suppurative otitis media
Q. 2. All of the following are usual complications of myringotomy except
a. damage to facial nerve d. damage to vestibulocochlear nerve

b. damage to labyrinth e. middle ear infection


c. damage to ossicular chain
Q. 3. All are prerequisites of myringoplasty except
a. air bone gap should not be more than 30 dB
b. air bone gap should be more than 60 dB
c. eustachian tube should be functional
d. ear should be dry for 3-6 months
e. underlying factors such as allergic
rhinitis, adenoiditis should be treated
Q. 4. All of the following are advantages of myringoplasty except
a. decreases chances of middle ear tion
infection d. restores vibratory area of tympanic
b. decreases sensorineural hearing loss membrane
c. provides sound pressure transforma- e. round window protection
Q. 5. Following is technique of myringoplasty

a. onlay technique d. all of the above

b. sandwich technique e. none of the above


c. underlay technique
Q. 6. Following is advantage of onlay technique of myringoplasty
a. avoids external incision d. all of the above

b. easier application of graft e. none of the above


c. easier preparation of bed
Q. 7. Following is disadvantage of onlay technique of myringoplasty
a. anterior blunting d. all of the above
b. dermoid inclusion e. none of the above

c. lateralizatin of graft
68
Q. 8. Following is advantage of underlay technique of myringoplasty
a. intratympanic adhesion can be divided to graft
d. all of the above
b. opportunity to inspect middle ear e. none of the above
c. squamous epithelium remains lateral
Q. 9. All of the following are disadvantages of underlay technique of myringoplasty
except
a. intratympanic adhesion can be divided
b. retraction of anterior edge d. Both a and b

c. risk of medial prolapse of graft e. Both b and c


Q. 10. Cortical mastoidectomy is also called

a. radical mastoidectomy d. Both a & b

b. Schwartz operation e. Both b & c


c. simple mastoidectomy
Q. 11. All are indications of cortical mastoidectomy except
a. chronic SOM with intracranial com- c. positive reservoir sign
plications d. sagging of poterosuperior meatal wall
b. cochlear implant surgery e. subperiosteal mastoid abscess
Q. 12. Suprameatal triangle overlies
a. frontal sinus d. maxillary antrum

b. lateral sinus e. sigmoid sinus

c. mastoid antrum
Q. 13. In adult the mastoid antrum lies at a depth of

a. 10 mm d. 25 mm

b. 15 mm e. 30 mm
c. 20 mm
Q. 14. Plate of bone separating the middle cranial fossa from middle ear is called
a. epitympanum (attic) d. tegmen tympani

b. korner’s septum e. tympanic plate

c. sinus plate
Q. 15. All are usual complications of mastoidectomy except

a. CSF leakage d. damage to labyrinth


b. damage to facial nerve e. damage to vestibulocochlear nerve

c. damage to ossicular chain


Q. 16. Landmark used for identification of geniculate ganglion of facial nerve is

a. digastric ridge d. processus cochleariformis

b. lateral semicircular canal e. pyramid


c. oval window
--------------------------
-------------------------- ANSWERS--------------------
---- ---------------------------------

Q.1. c 2. d 3. b 4. b 5. d 6. d 7. d 8. d 9. a 10. e 11. a 12. c

13. b 14. d 15. e 16. d


--------------------------------------------------------------------------------------------------------------------------
CHRONIC SUPPURATIVE OTITIS MEDIA (CSOM)
Definition:
Chronic suppurative otitis media (CSOM) is defined as chronic tympanic membrane
perforation persisting for more than three months.

Etiopathology:
The disease is common in poor socioeconomic group. It is also
common in patients with craniofacial anomalies e.g., Down
syndrome, Turner’s syndrome. The most common isolated
organisms are Pseudomonas aeruginosa and Staphylococcus
aureus. Anaerobes have also been isolated from chronic
suppurative otitis media. Chronic suppurative otitis media is
classified into two types;
1. Tubotympanic disease
2. Atticoantral disease
1. Tubotympanic disease
It is relatively a safe variety and is very rarely associated with
complications. It is characterized by central perforation (the perforation in
which all of its margins are formed by tympanic membrane). The disease Central perforation of
usually manifests in childhood. It is a result of acute otitis media tympanic membrane
and there is a central perforation. The perforation persists and permits repeated infection from
the external ear. The disease remains localized to mucosa usually in anteroinferior part of
middle ear cleft. Chronic infection and healing process proceed side by side and either may
dominate the other. Thus, acute exacerbation of the disease is very common.

Clinical features: Clinically tubotympanic disease is again subclassified into;


a. Inactive disease: Patient may be having no symptom at all or may complain of
hearing loss.
b. Active disease: Patient presents with discharging ear and/or hearing loss.
The source of infection in active chronic suppurative otitis media is usually nose, paranasal
sinuses or pharynx. The middle ear may also get infection from the perforated tympanic
membrane.
It only rarely leads to complications. The usual complications are fixation of the
ossicles by fibrosis, necrosis of the ossicular chain especially the long process of incus
and very rarely may lead to complications like facial paralysis, labyrinthitis and
intracranial complications.

Investigations:
1. Examination under microscope (EUM) is very important to find out type and extent of
disease.
2. Pure tone audiometry shows degree and type of hearing loss. It is carried out in patients
over the age of 04 years. Tympanic membrane perforation rarely causes more than 20 dB
hearing loss. Greater conductive hearing losses are caused by associated ossicular
necrosis. Significant ipsilateral sensorineural hearing loss may indicate cochlear
involvement.
3. High resolution CT provides;
a. Anatomical details
b. State of mastoid air cells
c. State of ossicular chain
d. Identify a cholesteatoma or any possible complication
e. Extension of disease

4. Culture and sensitivity of ear discharge helps in selecting appropriate antibiotics. Ear
swab is taken for microscopy as well.
5. X-ray paranasal sinuses and nasopharynx is done to find out source of infection
especially in bilateral tubotympanic disease.
Treatment:
1. Eliminate the underlying cause: This may include correction of nasal septal deviation,
removal of tonsils or adenoids.
2. Aural toilet: It consists of suction clearance of ears and/or dry mopping.
3. Topical antibiotics like polymyxin or chlormycetin are instilled regularly into the
infected ear.
4. Systemic antibiotics are given in severe and recurrent cases.
5. Patients are asked to avoid entry of water into the ears.
6. Myringoplasty: If there is recurrent discharge or if hearing loss is sufficient enough to
cause disability, closure of the perforation i.e., myringoplasty is done. The ear should be
completely dry at least for last six months before myringoplasty.
2. Atticoantral disease
It is a relatively unsafe or dangerous variety. There is high risk of complications with this
disease. It is associated with an attic or marginal perforation. It involves posterosuperior
part of middle ear cleft i.e., attic, antrum, posterior tympanum and mastoid. It is usually
associated
with cholesteatoma, granulations or osteitis.
Cholesteatoma is a keratinized squamous epithelial lined pocket containing keratinous,
debris. It may produce different enzymes which have got bone eroding properties and is,
therefore, liable to cause serious complications. If untreated, cholesteatoma, may continue to
expand and destroy the surrounding structures.

Clinical features:
It is characterized by persistent, mucopurulent, foul-
smelling discharge which may or may not be stained with
blood. There is also complaint of hearing loss. The degree of
hearing loss depends on the size of the perforation and state of
ossicular chain.
Attic cholesteatoma
Investigations:
These are same as those for tubotympanic disease.
Treatment:
1. Systemic antibiotics like fluroquinolones are given in almost all the cases.
2. Surgery: If the infection is not controlled by these conservative measures, surgical
treatment is carried out. Cases with granulation tissue or polyp may be managed
conservatively with removal of granulation tissue or polyp under the operating
microscope.
In most cases of cholesteatoma, exploration of the mastoid is carried out in the form of
radical or modified radical mastoidectomy followed by reconstruction of the hearing
mechanism i.e., tympanoplasty.
Difference between tubotympanic disease and atticoantral disease
Features Tubotympanic disease Atticoantral disease
Perforation Central Marginal or attic
Recurrent, profuse, mucoid Persistent, scanty, purulent and foul
Discharge and odorless smelling

Granulations Rare Common


Cholesteatoma Rare Common
Complications Rare Common
Aims of treatment in chronic suppurative otitis media:
1. To make the ear safe by eradicating active disease
2. To make the ear dry by preventing recurrence of infection
3. To prevent complications
72
4. To restore hearing mechanism

Differential diagnosis:
1. Tuberculous otitis media
2. Wegner’s granuloma
3. Eosinophilic granuloma
SELF ASSESSMENT
Q. 1. All are usually true regarding tubotympanic disease except
a. central perforation d. profuse ear discharge

b. foul-smelling ear discharge e. safe variety

c. intermittent ear discharge


Q. 2. All are true regarding atticoantral disease except

a. attic or marginal perforation d. usually associated with complications

b. dangerous variety e. usually no hearing loss


c. usually associated with cholesteatoma
Q. 3. Which of the following is true about cholesteatoma?
a. benign tumor d. malignant potential

b. contains cholesterol crystals e. metastasis to lymph node


c. erodes bone
Q. 4. Aim of mastoid surgery in chronic suppurative otitis media which should receive
first priority is
a. improvement in hearing d. preventing recurrence of infection

b. making the ear dry e. rendering the ear safe


c. preservation of hearing
Q. 5. Iatrogenic traumatic facial nerve palsy is most commonly caused during
a. mastoidectomy d. ossiculoplasty

b. myringoplasty e. stapedectomy
c. myringotomy
Q. 6. In radical mastoidectomy it is necessary to preserve
a. function of eustachian tube d. stapes

b. Incus e. tympanic membrane or its remnants


c. posterior canal wall
Scenario A: A 09 years old child presents with recurrent bilateral ear discharge for the last
two years. Blood analysis revealed Hb 11 gm/dl and marked lymphocytosis.
73
Clinical examination of ear showed bilateral profuse mucopurulent discharge and bilateral
central perforation of tympanic membranes. Nasopharyngeal examination revealed soft
tissue mass in nasopharynx. Throat examination was unremarkable.
1. What is your provisional diagnosis?
2. What could be the basic cause in this case?
3. What investigations will you suggest from management point of view?
B: A middle-aged man presented with right sided persistent ear discharge for last three
years along with impaired hearing. He experiences vertigo and tinnitus for last few weeks.
Otoscopic examination revealed posterior marginal perforation. Pure tone audiometry
showed mixed type of hearing loss.
1. What is your provisional diagnosis?
2. What investigations would you suggest?
3. What is your management plan?
-------------------------------------------------------- ANSWERS-----------------------------------------------------
Q.1. b 2. e 3. c 4. e 5. a 6. d
Scenario A: 1. Bilateral chronic suppurative otitis media (tubotympanic disease).
2. Hypertrophic adenoids 3. X-ray nasopharynx lateral view, X-ray paranasal sinuses,
culture and sensitivity of ear discharge, PTA, CT temporal bones.
B: 1. Chronic suppurative otitis media with labyrinthitis 2. Culture and sensitivity of ear
discharge, PTA, CT scan temporal bones 3. Systemic antibiotics, treatment of labyrinthitis,
mastoid exploration and removal of cholesteatoma, followed by reconstruction of the hearing
mechanism i.e., tympanoplasty.
--------------------------------------------------------------------------------------------------------------------------
COMPLICATIONS OF SUPPURATIVE OTITIS MEDIA
In the past complications of infection of middle ear used to occur due to acute suppurative
otitis media but nowadays because of common use of antibiotics, complications occur due to
chronic suppurative otitis media.
Complications of acute and chronic suppurative otitis media are similar.
These may be divided into;
A. Extracranial complications
B. Intracranial complications
General clinical features of complications in acute/chronic suppurative otitis media:
1. Pain: It is usually the first symptom of complications of acute/chronic suppurative otitis
media. It may indicate mastoiditis, extradural abscess or brain abscess.
2. Headache: It usually indicates intracranial complication.
74
3. Drowsiness: It usually indicates intracranial complication.
4. Nausea, vomiting and fever: These may indicate intracranial complication.
5. Facial weakness
6. Vertigo: It may indicate labyrinthitis or cerebellar abscess.
A. Extracranial complications
These may be;
1. Mastoiditis
2. Facial paralysis
3. Labyrinthitis
4. Petrositis
1. Mastoiditis
Etiopathology:
If acute otitis media is left untreated or fails to respond to treatment, the
inflammatory process may persist and spread to mastoid air cells. This
accumulation of pus in the mastoid air cells leads to necrosis of bony walls of
cells producing coalescent mastoiditis.
The disease may remain walled off within the mastoid bone or may spread
1. Laterally to form; A case of
mastoid abscess
a. Subperiosteal abscess
b. Subcutaneous abscess

Presentation of mastoiditis

2. Superiorly and posteriorly to form;


a. Extradural abscess
b. Subdural abscess
c. Brain abscess (temporal lobe abscess and cerebellar abscess)
a. Lateral sinus thrombosis
b. Otitic hydrocephalus
3. Medially;
a. Labyrinthitis
b. Petrositis and Gradenigo’s syndrome
4. Inferiorly to form;
a. Bezold’s abscess (tracking along the sternomastoid muscle) A case of Bezold‘s
abscess

b. Citelli’s abscess (tracking along the posterior belly of digastric muscle)


75
5. Anteriorly;
a. To the facial nerve canal causing facial
paralysis
b. Luc’s abscess (abscess in posterosuperior
canal wall)
c. Zygomatic abscess may develop above
and in front of the pinna
Clinical features:
Acute mastoiditis is usually a disease of
childhood but occasionally it may also affect
adults.
Abscesses due to CSOM
There is usually history of upper respiratory tract
infection a few days to 3-4 weeks previously. There is
persistence of earache, ear discharge and hearing loss. There is rise in body temperature
and general malaise.
On examination there may be protuberance/erection of the pinna. There is tenderness over
MacEwen’s triangle.
There may be discharge and sagging of the posterior wall of the external audiatory canal
(resulting from subperiosteal abscess formation).
There may be perforation of the posterosuperior part of tympanic membrane. There may
be granulations or polyp pouting through the perforation.
Types of mastoiditis:
Clinically there are different types of mastoiditis;
a. Acute mastoiditis is an acute bacterial infection of mastoid air cells often as a result
of an un-resolved middle ear infection (otitis media).
b. Masked mastoiditis: It is defined as a subclinical inflammation of the mucosal
lining and the bony structure of the mastoid air cells, with intact tympanic membrane.
c. Chronic mastoiditis: It is a chronic bacterial infection of mastoid air cells. It is most
commonly associated with chronic suppurative otitis media and
particularly with cholesteatoma formation.
Investigations:
1. CT of temporal bones and brain is done when intracranial
complications are present or suspected (though MRI may be more
helpful in identifying specific intracranial pathology).
2. Ear swab is taken for culture and sensitivity in cases of discharging
ear.
Rest of the investigations are same as those for suppurative otitis
media.

CT scan of Rt. mastoiditis


Treatment:
1. Intensive antibiotics are given as for suppurative otitis media if the patient fails to
respond to medical treatment for 24-48 hours.
2. Myringotomy is done with or without ventilation tube placement if the tympanic
membrane has not yet perforated and is red and bulging. Culture of the aspirate must be
done.
76
3. Incision and drainage are done initially if there is mastoid abscess. It is followed by
cortical mastoidectomy. The aim of the operation is to exenterate the mastoid air cells as
completely as possible.
Differential diagnosis:
1. Boil ear
2. Bullous myringitis
3. Acute suppurative otitis media
4. Diffuse otitis externa
2. Facial paralysis
It will be discussed later in detail.
3. Labyrinthitis
Synonym: Otitis interna
Definition:
It is defined as inflammation of inner ear.
Etiopathology:
Labyrinthitis can occur when bacteria or the
toxins they produce enter the labyrinth. There are
following types of labyrinthitis;
A. Circumscribed labyrinthitis:
It is usually due to cholesteatomatous erosion of
the bony capsule of the labyrinth (usually lateral
semicircular canal or the promontory). It is also
known as labyrinthine fistula.
B. Diffuse labyrinthitis:
It is usually due to the extension of the circumscribed labyrinthitis. It may follow invasion
through the oval or round windows. It involves the peri or endolymphatic spaces. It is further
classified into;
a. Diffuse serous labyrinthitis: In this there is a general nonpurulent inflammation of
labyrinth with a fibrinous or serous exudate.
b. Diffuse purulent labyrinthitis: In this there is infiltration of the peri and
endolymphatic spaces by polymorphs and pus cells. There is destruction of the
vestibular and cochlear structures.
Clinical features:
77
1. Circumscribed labyrinthitis:
In this, there is complaint of vertigo and hearing loss. Once the fistula is established, the main
complain may be episodic vertigo brought on by sudden movements of head, cold water or
air in the ear.
On examination, there is usually evidence of otitis media and the diagnosis is confirmed by
demonstrating a fistula sign.

2. Diffuse serous labyrinthitis:


In this there is complaint of vertigo associated with nausea and vomiting. There is also
complain of hearing loss. There is sensation of objects moving from diseased to the healthy
side.
On examination, there is nystagmus with fast component towards the diseased side. Caloric
test shows canal paresis.
3. Diffuse purulent labyrinthitis:
In this the symptoms are similar to those of serous labyrinthitis but vertigo and vomiting is
more frequent and severe. Nystagmus, although initially is towards the diseased ear but
later changes its direction towards the healthy ear. Patient develops almost total hearing loss.
Caloric test shows canal paralysis.
Treatment:
1. Circumscribed labyrinthitis: In this the treatment is that of the underlying otitis media.
2. Diffuse serous labyrinthitis
a. Strict bed rest is advised and head is immobilized with affected ear above.
b. Parenteral systemic antibiotics is given to control the infection.
c. Labyrinthine sedatives are given for relief of vertigo.
d. Treatment of the underlying cause:
I. In acute otitis media, if the tympanic membrane is bulging, myringotomy is done
and pus is sent for culture and sensitivity.
II. In acute mastoiditis, cortical mastoidectomy is performed.
III. In chronic otitis media or cholesteatoma, modified radical or radical mastoidecto-
my is performed.
3. Diffuse suppurative labyrinthitis: In this the treatment is same as that for diffuse
serous labyrinthitis.
4. Petrositis
Definition:
78
Infection of petrous apex cells is known as petrositis.
The degree of pneumatization of petrous bone is extremely variable but may extend right up
to the petrous apex. In cases of mastoiditis, infection may spread up to the petrous apex cells.
It is an extremely rare condition.

Clinical features:
Clinical features are similar to those of preceding acute/chronic
suppurative otitis media and acute/chronic mastoiditis, which fails to
respond to treatment.
1. There is persistence of earache and temperature.
2. Gradenigo’s syndrome/petrous apex syndrome Triad of Gradenigo’s
syndrome
may be present which means
a. Acute infection of the middle ear cleft
b. Pain in the distribution of fifth cranial nerve
c. Sixth cranial nerve paralysis

Investigations:
Diagnosis depends on clinical picture as described above.
CT/MRI of the temporal bone is required to confirm the diagnosis.
Treatment:
CT showing petrositis
Intensive intravenous antibiotic treatment is given. If the
patient fails to respond to medical treatment in 24 to 48 hours
then;
a. Cortical mastoidectomy is performed in cases of acute otitis media/acute mastoiditis
b. Modified radical/radical mastoidectomy is performed in cases of chronic otitis
media/ chronic mastoiditis.

B. Intracranial complications
These are classified as follows;
1. Extradural abscess
2. Subdural abscess
3. Meningitis
4. Brain abscess
5. Lateral sinus thrombosis
6. Otitic hydrocephalus
79
1. Extradural abscess
It is the most common intracranial complication arising from middle ear infection. But in
most instances, it remains asymptomatic.
Clinical features:
Symptoms are usually vague and non-specific. A few patients may
have headache on the side of affected ear which is relieved by the
onset of ear discharge. There may be complaint of malaise.
Investigations:
Diagrammatic representation
CT/MRI establish the diagnosis and exact site and size of the
of cholesteatoma with
abscess. extradural abscess
2. Subdural abscess (empyema)
There may be complaint of severe headache and drowsiness. There may be focal
neurological symptoms in the form of fits or paralysis. There may be hemianesthesia and/or
paresis/ paralysis of one upper or lower limb.
Investigations:
CT/MRI establish the diagnosis and exact site and size of the abscess.

3. Meningitis (leptomeningitis)
CT showing
Definition: subdural abscess
Pia mater and arachnoid mater are together known as leptomeninges and their
inflammation is known as leptomeningitis.
Clinical features:
Patient presents with rise in body
temperature, chills and rigors. There is
severe headache and neck stiffness. There
is also complaint of vomiting.
On examination patient is anxious and
restless. There may be periods of
drowsiness. Kernig’s sign is positive i.e.,
extension of leg with thigh flexed on
abdomen causes pain. Head retraction is usually present. There
may be opisthotonus in later stages. Kernig’s sign

Brudzinski’s sign is positive i.e., flexion of neck causes flexion of hip


and knee.

Papilledema is positive in later stages.


Investigations:
Examination of cerebrospinal fluid is
diagnostic. There is:

Brudzinski’s sign
80
a. Increase in CSF pressure, white cell count and protein content.
b. Decrease in glucose concentration.
4. Brain abscesss
Otogenic brain abscess is almost always formed in the temporal lobe or the cerebellum.
Temporal lobe abscesses are twice as common as cerebellar abscesses.
Clinical features:
There are general features of headache, fever, malaise and vomiting followed by drowsiness.
On examination pulse rate is decreased. Focal neurological impairments are seen depending
on the exact site and size of abscess. Pupils may be dilated. There may be papilledema.
Death may occur from the effects of raised intracranial pressure, meningitis or
intraventricular rupture of the abscess.
Investigations:
Contrast CT/MRI is the most important investigation in demonstrating the exact site and size
of the brain abscess.
a. Temporal lobe abscess
Clinical features:
1. Nominal aphasia. If abscess involves dominant hemisphere (usually
left) the patient is unable to tell the name of common object such as
pencil, key etc. but can explain its usage.
2. Homonymous quadrantic hemianopia occurs due to involvement of CT showing left
the optic radiations. There is motor paralysis of the upper and lower temporal lobe
limbs on the opposite side of abscess. abscess

Investigations:
Contrast CT/MRI is the most important investigation in demonstrating the exact site and
size of the temporal lobe abscess.

b. Cerebellar abscess
Clinical features:
Signs of cerebellar disease (mnemonic---6As+DISPRN) such as asthenia,
atonia, ataxia, adiadochokinesia, asynergia, abnormal posture, drunken gait,
intention tremors, scanning speech, past pointing, rebound phenomenon and
nystagmus are present on the side of lesion.
Investigations: CT showing
right cerebellar
Contrast CT/MRI is the most important investigation in demonstrating abscess
the exact site and size of the cerebellar abscess.
81
5. Lateral sinus thrombosis
Sigmoid sinus and transverse sinus collectively are known as lateral sinus.
Clinical features:
1. Picket fence (high and swinging) pattern of fever is
characteristic of lateral sinus thrombosis.
2. Rigors with profuse sweating is very common complaint.
3. There is earache, neck pain, mastoid tenderness and stiffness
around the sternomastoid muscle. Patient is drowsy, lethargic
and may be in coma. There is papilledema.

4. Griesinger’s sign is positive i.e., pitting edema over the Angiogram showing
occipital region (due to clotting within mastoid emissary left lateral sinus thrombosis
veins).
5. Crowe-Beck test. Pressure on healthy internal jugular vein produces engorgement of
retinal and supraorbital veins. Release of pressure relieves the engorgement.

Investigations:
1. Angiography (especially digital subtraction venography) is
the definitive investigation for lateral sinus thrombosis. It will
demonstrate the exact size and extent of the thrombus as well as
the anatomical arrangements of the individual’s venous drainage.
Nowadays MR angiogram is becoming very popular.
2. Contrast CT/MRI may show filling defect within the sinus.
There may be cerebral edema, reduced size of ventricles and
canaliculi.
3. Tobey-Ayer test is diagnostic of Magnetic resonance venography
lateral sinus thrombosis. The test involves measurement of the
CSF pressure and observing its changes on compression of
one or both internal jugular veins by fingers on the
neck.
In normal subjects, compression of each internal
jugular vein in turn is followed by a rapid rise of CSF
pressure of 50-100 mmHg above the normal level.
There is an equally rapid fall on release of pressure. It
is normal for there to be a difference in rise on the two
sides but unusual for it to exceed 50 mmHg.
In a typical case of lateral sinus thrombosis,
pressure over the vein draining the occluded sinus
causes either no rise or a very slow rise by 10-20
mmHg. Compression of the normal internal jugular
vein on the other hand produces a rapid pressure rise of two to
three times the normal level. Tobey-Ayer test
Remember Queckenstedt test is useful only for recognizing
spinal CSF obstruction and not for lateral sinus thrombosis.
4. Radioisotope scanning with gallium can show hot spots of sepsis.
82
6. Otitic hydrocephalous
Synonym: Benign intracranial hypertension
Clinical features:
Patient presents with headache, nausea, vomiting, drowsiness
and blurring of vision. Sometimes diplopia may occur because of
compression or stretching of abducent nerve. Left abducent palsy
Investigations:
1. CT/MRI shows normal ventricular size.
2. Lumbar puncture shows two-to-three-fold increase in CSF pressure. There is normal
CSF cell count, protein and glucose content. CSF is bacteriologically sterile.
Treatment of otogenic intracranial complications:
The following are the principles of treatment:
1. Systemic antibiotics:
When the cause is acute ear infection, Haemophilus influenza is the responsible
organism. So, chloramphenicol 100 mg/kg/day is given, but agranulocytosis and
aplastic anemia are the drastic complications of chloramphenicol. So, methicillin or
flucloxacillin may be given instead.
When the cause is chronic ear infection, Pseudomonas aeruginosa is the responsible
organism and gentamycin is given as dose of 4-5 mg/kg/day (but it is nephrotoxic and
ototoxic). So azlocillin and ticarcillin may be given instead.
If anaerobic infection is present, metronidazole is given in a dose of 400-600 mg 08
hourly.
2. Neurosurgical attention by neurosurgeon is given to the complication identified.
3. Treatment of ear disease after the intracranial infection has been controlled.
In acute suppurative otitis media, myringotomy or cortical mastoidectomy is performed.
In chronic suppurative otitis media, radical or modified radical mastoidectomy is
performed.
SELF ASSESSMENT
Q. 1. All of the following are extracranial complications of suppurative otitis media
except
a. facial paralysis d. mastoiditis

b. labyrinthitis e. petrositis
c. lateral sinus thrombosis
Q. 2. If earache, hearing loss and ear discharge are predominant features in chronic
suppurative otitis media, look for
a. brain abscess c. mastoiditis

b. extradural abscess d. OME


83
e. subdural abscess
Q. 3. All of the following are usual possible complications of mastoiditis except
a. Bezold’s abscess d. Luc’s abscess

b. Citelli’s abscess e. Zygomatic abscess

c. Collar stud abscess

Q. 4. If ear discharge and earache persist in suppurative otitis media even after appropri-
ate antibiotics, one must suspect
a. coalescent mastoiditis d. meningitis

b. extradural abscess e. subdural abscess

c. lateral sinus thrombosis


Q. 5. Operation of choice for coalescent mastoiditis is

a. cortical mastoidectomy d. myringoplasty

b. fenestration operation e. radical mastoidectomy


c. modified radical mastoidectomy
Q. 6. Initial treatment of mastoiditis includes all except
a. cortical mastoidectomy d. myringotomy

b. incision & drainage e. tympanoplasty


c. intensive antibiotics
Q. 7. Treatment of choice in postauricular abscess as a complication of otitis media is
a. antibiotics d. incision and drainage, antibiotics and
b. aspiration mastoid exploration
c. incision and drainage (I&D) e. incision and drainage plus antibiotics
Q. 8. Labyrinthitis is also known as

a. lateral sinus thrombosis d. otitis media

b. otitis externa e. petrositis

c. otitis interna
Q. 9. A positive fistula test indicates

a. attic cholesteatoma d. raised endolymphatic pressure

b. dead labyrinth e. safe variety of chronic suppurative


otitis media
c. erosion of labyrinth
Q. 10. General manifestation of intracranial complications of suppurative otitis media is
a. drowsiness d. pain

b. headache e. all of the above

c. nausea, vomiting and fever


84

Q. 11. Gradenigo’s syndrome/petrous apex syndrome is characterized by all except


a. diplopia nerve

b. lateral rectus palsy d. profuse ear discharge

c. pain in the distribution of trigeminal e. seventh cranial nerve palsy


Q. 12. All are intracranial complications of suppurative otitis media except
a. extradural abscess d. meningitis

b. labyrinthitis e. subdural abscess

c. lateral sinus thrombosis


Q. 13. The most common but silent intracranial complication of SOM is

a. brain abscess d. otitic hydrocephalus

b. extradural abscess e. subdural abscess


c. meningitis
Q. 14. In SOM, headache which is relieved by onset of ear discharge, usually indicates
a. brain abscess d. otitic hydrocephalus

b. extradural abscess e. subdural abscess

c. meningitis
Q. 15. In meningitis, all are true except

a. CSF glucose is increased d. Kernig’s sign is positive

b. CSF protein is increased e. severe headache and neck stiffness


c. CSF white cell count is increased
Q. 16. The most common brain abscess following suppurative otitis media is
a. cerebellar abscess
b. frontal abscess
c. occipital abscess
d. parietal abscess
e. temporal abscess

Q. 17. In brain abscess all are true except


a. death occurs due to decreased intra-
cranial pressure
b. focal neurological impairment
c. papilledema
d. pulse rate is decreased
e. pupils are dilated
Q. 18. Features of cerebellar abscess include all except
a. adiadochokinesia d. intention tremor

b. asthenia e. prepointing
c. atonia
Q. 19. Usual clinical features of cerebellar abscess include all except
a. abnormal posture c. ataxia

b. asynergia d. drunken gait

e. normal speech

Q. 20. Venous sinus commonly involved in mastoiditis is


a. cavernous d. superior petrosal

b. inferior petrosal e. superior sagittal


c. sigmoid
Q. 21. In SOM picket fence fever pattern indicates
a. brain abscess d. otitic hydrocephalus

b. extradural abscess e. subdural abscess


c. lateral sinus thrombosis
Q. 22. In chronic suppurative otitis media Griesinger’s sign may be positive in
a. brain abscess d. meningitis

b. extradural abscess e. otitic hydrocephalus


c. lateral sinus thrombosis
Q. 23. In chronic suppurative otitis media, Tobey-Ayer test is diagnostic of
a. brain abscess d. meningitis

b. extradural abscess e. otitic hydrocephalus

c. lateral sinus thrombosis


Q. 24. Queckenstedt test is diagnostic of

a. brain abscess d. otitic hydrocephalus


b. extradural abscess e. spinal CSF obstruction
c. meningitis
Q. 25. Definitive investigation for lateral sinus thrombosis is
a. angiography d. radioisotope gallium scan

b. CSF study e. X-ray mastoid


c. CT scan
Q. 26. A 33 years old male with 08 years history of left chronic suppurative otitis media
presents with 02 days history of fever with rigors and chills and left temporoparietal
headache. The important investigation in this case is
a. CT scan of temporal bone and brain d. X-ray mastoid

b. lumbar puncture e. X-ray skull

c. radionuclide bone scan


Q. 27. Otitic hydrocephalus is also called

a. benign intracranial hypertension d. malignant intracranial hypertension

b. benign intracranial hypotension e. malignant intracranial hypotension

c. Lateral sinus thrombosis


Q. 28. All are true about otitic hydrocephalus except

b. CSF counts, protein and glucose are


a. CSF contains gram positive bacteria normal
86

c. CSF pressure is increased e. patient presents with headache, blur-

ring of vision and diplopia


d. CT/MRI shows normal ventricular size
Scenario A: A 28 years old female was brought to emergency with history of headache, fever,
nausea and vomiting for last 02 days. There is history of long-standing left ear discharge, for
which she was taking medications. On examination, she had fever and attic perforation.

1. What is the most probable clinical diagnosis?


2. What investigations you will advise to reach the diagnosis?
3. What is the treatment plan of this patient?
B: A seven years old child who had chronically discharging left ear developed left otalgia,
retroauricular tender swelling with pinna protruded outwards and forwards for last 05 days.
1. What is the most likely diagnosis?
2. How will you manage this case?
C: A 45 years old diabetic patient complained of earache. Examination showed mastoiditis.
After several courses of antibiotics, patient developed symptoms and signs of ear infection
again and again.
What is the best plan of management?
D: A 09 years old boy is brought to ENT emergency room with complaints of sudden onset of
fever and pain on right side neck for last 02 days. Examination reveals right sided torticollis
and a tender swelling behind the angle of right mandible. He also has of right ear discharge
since childhood.
1. What is the most probable diagnosis?
2. What will be your treatment plan?
3. Define cholesteatoma?
4. What do you understand by masked mastoiditis?
E: A 30 years old patient presents to you with complaint of pain in the left ear, ipsilateral 6th
nerve paralysis and pain in the distribution of left trigeminal nerve for last 05 days.
1. What is the most probable clinical diagnosis?
2. What is the differential diagnosis?
3. Name different approaches to petrous apex?
F: A 25 years old female complains of persistent left ear discharge and ipsilateral headache
for last 06 months. Headache is relieved during periods of ear discharge.
1. What is the most probable clinical diagnosis?
2. How will you confirm your diagnosis?
3. How will you manage the case?
87

G: A 40 years old male has long standing left sided ear discharge. For last 02 days, he
complains of severe headache and vomiting. He cannot bend his neck without pain. White
cell count is 14000 per cubic mm.
1. What is the most likely diagnosis?
2. Which test you will do to confirm diagnosis?
3. How will you treat the patient?
4. Enumerate the possible complications of this primary ear problem?
H: A 32 years old teacher presents in ENT OPD with complaints of inability to close the left
eye and drooling of saliva from left corner of mouth for last 02 days. He also gives history of
pain in the left ear along with fever for last 08 days. On examination, left tympanic
membrane is congested and bulging.
1. What is the most probable clinical diagnosis?
2. Discuss the differential diagnosis?
3. How are you going to manage such a case?
I: A 35 years old man presented in emergency department with the complaints of headache,
nausea, vomiting and high-grade fever for last 01 week. He has history of foul-smelling
scanty discharge from right ear for last 20 years. On examination he has marginal
perforation, nominal aphasia and homonymous quadrantic hemianopa.
1. What is the most probable clinical diagnosis?
2. How will you investigate this patient?
3. How will you manage this patient?
J: A 25 years old male patient with history of chronic suppurative otitis media presents with
headache, drowsiness, blurring of vision, nausea and vomiting for last 02 months. On
examination there is papilledema and lateral rectus palsy. CSF examination is unremarkable
except increased pressure. CT scan is also unremarkable.
1. What is the most probable clinical diagnosis?
2. What are the most major complications of disease apart from death?
3. What are important steps to treat the patient?
------------------------------- ANSWERS--------------------------------
------------------------- ---------------------
Q.1. c 2. c 3. c 4. a 5. a 6. e 7. d 8. c 9. c 10. e
11. e 12. b 13. b 14. b 15. a 16. e 17. a 18. e 19. e 20. c
21. c 22. c 23. c 24. e 25. a 26. a 27. a 28. a
Scenario A: 1. Left chronic suppurative otitis media complicated with meningitis 2. Culture
and sensitivity of discharge, lumber puncture 3. After treatment of meningitis, mastoid
exploration and removal of cholesteatoma.
B: 1. Chronic suppurative otitis media with postauricular abscess 2. Incision and drainage
88
with mastoid exploration after proper workup, culture and sensitivity of pus.
C: Strict diabetic control, culture and sensitivity of discharge, examination under
microscope, PTA, CT scan temporal bones, mastoid exploration, regular follow up
D: 1. Right chronic suppurative otitis media complicated with right Bezold abscess 2.
Parenteral antibiotics, needle aspiration, culture and sensitivity of pus, CT scan neck and
temporal bone, mastoid exploration for cholesteatoma 3. Cholesteatoma is a keratinized
squamous epithelial lined pocket containing keratin debris 4. Subclinical inflammation of
mucosal lining and bony septa of mastoid air cells with intact tympanic membrane.
E: 1. Left petrositis 2. Acute mastoiditis, meningitis, acute otitis media 3. Subtemporal,
presigmoid, retrosigmoid.
F: 1. Chronic suppurative otitis media with extradural abscess 2. CT scan 3. Mastoid
exploration.
G: 1. Chronic suppurative otitis media with meningitis 2. CSF examination 3. Antibiotics,
analgesics, antipyretics 4. Intracranial and intratemporal complications.
H: 1. Acute otitis media with facial palsy 2. Mastoiditis, bell’s palsy, OME 3. Antibiotics,
analgesics, myringotomy, physiotherapy.
I: 1. Temporal lobe abscess 2. CT scan brain 3. Antibiotics, neurosurgical approach for
draining of abscess.
J: 1. Otitic hydrocephalus 2. Blindness 3. Antibiotics, acetazolamide, lumbar drain, mastoid
exploration.
-------------------------------------------------------------------------------------------------------------------------

TUBERCULOUS OTITIS MEDIA


Etiopathology:
It is usually secondary to severe pulmonary tuberculosis. Infection is usually carried up the
eustachian tube from the nasopharynx but rarely may be blood borne.
Clinical features:
1. Painless hearing loss and ear discharge which do not respond to usual antibiotic
treatment are characteristic findings.
2. On examination the tympanic membrane may appear thick and there may be multiple
perforations or exuberant pale granulations. Tuberculous otitis media is
also suspected when conductive hearing loss is disproportionate with the
clinical findings.
Investigations:
1. Ear swab is sent for microscopy with ZN (Ziehl-Neelsen) staining and
Multiple perforations in
culture. Detection of tubercle bacilli is diagnostic. tympanic membrane

2. X-ray chest and sputum for AFB may reveal pulmonary tuberculosis.
89
3. PCR for Mycobacterium tuberculosis of ear discharge is conclusive.
Treatment:
It is as follows:
1. Antituberculous chemotherapy.
2. Aural toilet when discharge is present.
3. Mastoidectomy is required when sequestrum or other complications are present.

SELF ASSESSMENT
Q. 1. All are true about tuberculous otitis media except
a. conductive hearing loss proportionate d. painless hearing loss and ear dis-
with the clinical findings charge
b. exuberant pale granulations e. usually secondary to pulmonary tu-
c. multiple perforations of tympanic berculosis
membrane
Q. 2. All are true about tuberculous otitis media except
a. antibiotics are treatment of choice copy with ZN staining
b. antituberculous drugs are treatment of d. PCR is diagnostic
choice e. X-ray chest must be done to find out
c. ear swab should be sent for micros- primary focus
Scenario: A 46 years old female presents to ENT OPD with the complaint of right sided ear
discharge for last 01 year. On examination there are exuberant pale granulation in the
middle ear. Tuning fork tests show conductive hearing loss disproportionate with the clinical
findings.
1. What is the most probable clinical diagnosis?
2. What investigations will you advise to reach the diagnosis?
3. What is the treatment plan of this patient?
--------------------------------------------------------ANSWERS-----------------------------------------------------
Q.1. a 2. a
Scenario: 1. Tuberculous otitis media 2. Ear swab for Z.N. staining and culture, PCR, X-ray
chest 3. Antituberculous drugs, aural toilet, mastoidectomy.
--------------------------------------------------------------------------------------------------------------------------

OTITIS MEDIA WITH EFFUSION (OME)


Synonyms: Glue ear, Serous otitis media, Secretory otitis media, Catarrhal otitis media,
Mucinous/Mucoid otitis media
90
Definition:
It is defined as an accumulation of nonpurulent fluid in the middle ear. The term otitis
media is basically misnomer as there is no inflammation of middle ear.
Etiopathology:
It occurs commonly in children but occasionally adults may also be affected. It is the most
common cause of acquired hearing loss in childhood. It is usually bilateral but may be
unilateral. Exact etiology is unknown but there are following contributing factors.
1. Eustachian tube dysfunction which may be because of adenoiditis, rhinosinusitis,
tonsillitis, neoplasms of postnasal space (e.g., angiofibroma and nasopharyngeal
carcinoma), palatal paralysis, nasopharyngeal disproportion (e.g., Down syndrome),
postnasal pack, cleft palate etc. Nasopharyngeal carcinoma must be ruled out in unilateral
otitis media with effusion in adults.
2. Unresolved acute otitis media
3. Allergy
4. Barotrauma
5. Radiotherapy
Initially there is transudation of fluid in the middle ear cleft. In some patients, the fluid in the
middle ear may resolve completely. In others, it may persist and lead to certain complications.
Clinical features:
1. Aural symptoms: Parents of the affected child may complain
that the child watches television at louder volume. Teachers
may complain of inattention and educational retardation. There
may be complaint of hearing loss, earache and tinnitus.
2. Speech development problems:
Different presentations of
Due to long standing hearing loss, there may be delayed or otitis
defective speech development. media
with effusion
On examination;

1. Tympanic membrane may be retracted with leash of blood vessels on it. Handle of
malleus may be horizontal with prominent knuckle. There may be restricted mobility of
tympanic membrane.
2. It may be dull, bulging and opaque with loss of cone of light or may appear blue.
3. There may be fluid level and air bubbles.
4. Tuning fork tests show conductive hearing loss.

Complications:
Complications of otitis media with effusion are tympanosclerosis, chalk patches, thin atrophic
91
tympanic membrane, cholesterol granuloma, cholesteatoma, ossicular necrosis and chronic
suppurative otitis media.
Investigations:
1. Tympanometry usually shows type B tympanogram. It is
diagnostic of otitis media with effusion. But tympanogram
may also show decreased compliance or negative pressure.
2. Pure tone audiometry shows an air-bone gap
indicating conductive hearing loss of 30-50 dB.
3. X-ray mastoid shows clouding of mastoid air cells due to
presence of fluid in the middle ear.

Treatment:
Medical treatment is given in initial stages. This includes
topical/systemic decongestants, antihistamines and antibiotics.
Further management includes;
1. Removal of underlying factors: It is directed towards
removal of etiological and predisposing factors such as
adenoids, rhinosinusitis, allergy etc.
2. Aeration of middle ear may be done with repeated
Valsalva maneuver. Patient may be given chewing gum to
encourage repeated swallowing which opens up
the eustachian tube.
3. Myringotomy: If conservative measures fail, myringotomy
i.e., incision of tympanic membrane in anteroinferior
quadrant and evacuation of fluid from middle ear under local Grommet in place
or general anesthesia is performed.
4. Ventilation tube (grommet) insertion: Insertion of ventilation tube
may be considered if thick
fluid is encountered on
myringotomy.
5. Cortical mastoidectomy:
Rarely cortical
mastoidectomy is
indicated in resistant
cases.

Different types of ventilation tubes


SELF ASSESSMENT

Q. 1. Otitis media with effusion is also known as

a. glue ear d. all of the above

b. secretory otitis media e. none of the above

c. serous otitis media


Q. 2. Otitis media with effusion is also known as

a. catarrhal otitis media d. all of the above

b. mucinous/mucoid otitis media e. none of the above


c. mucoid otitis media
Q. 3. In otitis media with effusion, fluid in the middle ear is
a. mucopurulent b. non purulent
92

c. purulent e. serosanguineous
d. sanguineous
Q. 4. The most common cause for bilateral conductive hearing loss in a child is
a. acute otitis media d. otitis media with effusion

b. congenital cholesteatoma e. otosclerosis


c. Meniere’s disease
Q. 5. Which of the following conditions predispose to otitis media with effusion?
a. adenoiditis d. Down syndrome

b. carcinoma nasopharynx e. all of the above


c. cleft palate
Q. 6. Serous otitis media may usually occur in any of the following conditions except
a. carcinoma of larynx c. nasopharyngeal carcinoma
b. inadequately treated acute otitis media d. rhinosinusitis
e. tonsillitis
Q. 7. Serous otitis media may occur in the following condition
a. allergy d. postnasal pack

b. barotrauma e. all of the above


c. palatal paralysis
Q. 8. In serous otitis media, tympanic membrane may usually show any of the following
except
a. air bubbles d. retracted with thin leash of blood vessels
b. dull and opaque with loss of cone of along the handle of malleus
light e. very severe congestion
c. fluid level
Q. 9. Usual clinical features of otitis media with effusion include all except
a. earache -tion

b. ear discharge d. hearing loss

c. educational retardation and inatten- e. tinnitus


Q. 10. A mother brings 09 years old child with history of progressive decrease in hearing
bilaterally for last four months. Child has history of upper respiratory tract infection. On
examination, child’s ear drums are dull and blue colored. What is the most probable
clinical diagnosis?
a. Acute suppurative otitis media d. Otosclerosis

b. Acute otitis media e. Otitis media with effusion


c. Chronic suppurative otitis media
Q. 11. All are usual complications of otitis media with effusion except
a. chalk patches d. otosclerosis

b. cholesteatoma
c. ossicular necrosis e. tympanosclerosis
Q. 12. All are usual complications of serous otitis media except

a. chalk patches d. retraction pockets

b. cholesteatoma e. thickened tympanic membrane

c. ossicular necrosis
Q. 13. For diagnosis investigation of choice in otitis media with effusion is
a. CT temporal bones d. X-ray PNS

b. speech audiometry e. X-ray mastoid


c. tympanometry
Q. 14. A 05 years old girl is suspected of suffering from bilateral otitis media with
effusion. On tympanometry the following curve will confirm the diagnosis.
a. Type A curve d. Type B curve

b. Type Ad curve e. Type C curve


c. Type As curve
Q. 15. An 08 years old boy having bilateral otitis media with effusion for last one year,
not responding to multiple courses of antibiotics/decongestants presents to you. The
best treatment is
a. antibiotics ing cause

b. antihistamines d. decongestants

c. bilateral myringotomy and grommet e. tympanocentesis


insertion plus treatment of any underly-
Q. 16. Treatment of otitis media with effusion include all except

a. decongestants d. removal of underlying cause

b. myringotomy e. ventilation tube insertion


c. radical mastoidectomy
Q. 17. In otitis media with effusion, myringotomy is done in the following compartment.
a. Anteroinferior quadrant of tympanic e. Posterosuperior quadrant of tympanic
membrane membrane
b. Anterosuperior quadrant of tympanic
membrane
c. Attic part of tympanic membrane

d. Posteroinferior quadrant of tympanic


membrane

Q. 18. Grommet is used


a. as a prosthesis after stapedectomy
b. as a ventilation tube for middle ear
c. to close a septal perforation
d. to close a tympanic membrane perfo-
ration
e. to divert tracheal air into esophagus
94
Scenario A: A five years old baby brought to OPD with complaint of watching television at a
loud volume, educational retardation and lack of concentration for last 06 months. On
examination tympanic membrane is dull and bulging.
1. What is the most probable clinical diagnosis?
2. How will you confirm your diagnosis?
3. What is the treatment of this patient?
B: A 03 years old child is very slow in developing speech. On otoscopy, he has bilateral dull
tympanic membranes, horizontal handle of malleus and prominent knuckle of malleus.
1. What is the most probable clinical diagnosis?
2. How will you investigate this patient?
3. How will you treat this patient?
--------------------------------- ANSWERS--------------------
----------------------- ---------------------------------
Q.1. d 2. d 3. b 4. d 5. e 6. a 7. e 8. e 9. b
10. e 11. d 12. e 13. c 14. d 15. c 16. c 17. a 18. b
Scenario A: 1. Otitis media with effusion 2. PTA, type B curve on tympanometry 3. Removal
of underlying cause, decongestants, myringotomy with/without grommet insertion.
B: 1. Otitis media with effusion 2. Tympanometry 3. Removal of underlying cause,
decongestants, myringotomy with/without grommet insertion.
--------------------------------------------------------------------------------------------------------------------------
EUSTACHIAN TUBE DISORDERS
Normally eustachian tube is closed. It opens temporarily during swallowing, yawning and
sneezing through contraction of tensor palati muscle. Air composed of oxygen, CO2 and
nitrogen fills the middle ear and mastoid. When eustachian tube is blocked, oxygen is
absorbed, CO2 and nitrogen also diffuse into the blood. This results in negative pressure in
middle ear and results in retraction of tympanic membrane. Obstruction of eustachian tube
may be mechanical, functional or both.
Tubal dysfunction may be because of following conditions:
1. Adenoids: These may cause tubal dysfunction due to;
a. Mechanical blockage of pharyngeal opening of eustachian tube.
b. Acting as a source of infection.
c. By chemical mediators of inflammation.
2. Tubal blockage: It may be due to trauma to pharyngeal opening of eustachian tube or
nasopharyngeal tumor.
3. Cleft palate: Due to weakness of tensor palati muscle.
95
4. Nasopharyngeal disproportion: Due to abnormal shape of the nasopharynx e.g.,
Down syndrome.
Symptoms of tubal occlusion include earache, hearing loss, tinnitus and disequilibrium or
even vertigo.
On examination; signs of tubal obstruction vary widely. These include retracted tympanic
membrane, congestion along handle of malleus/pars tensa, transudate behind the tympanic
membrane, fluid level and conductive hearing loss.
Patency of eustachian tube can be assessed by;
1. Otoscopy with Siegel’s speculum
2. A. Valsalva maneuver: It is the production of a high nasopharyngeal pressure by
blowing out against closed lips and nose. It normally results in an increase in middle ear
pressure with the tympanic membrane bulging outwards.
B. Toynbee maneuver: pinching the nose and swallowing. Swallowing pulls open the
eustachian tubes while the movement of the tongue, with the nose closed,
compresses air which passes through the tubes to the middle ear.

C. Frenzel’s maneuver: the way out of the nasal cavity is blocked, by pinching the
nostrils or by a nose-clip;

• the larynx is closed, sealing off the lungs;


• the body of the tongue is gently moved upwards and backwards, in order to compress the
air;
• further compression of the air follows from moving the closed glottis upwards (this is
easily noticed in the elevation of the "Adam's Apple").
By performing this technique, the compressed air is forced into eustachian tubes, and thus
into the middle ear. In situations where the ambient pressure rises (typical causes are
decreasing altitude in the case of an airplane or increasing depth in the case of a diver
submerging), the maneuver results in the equalization of the pressure on both sides of
the eardrum.
Compared with the Valsalva maneuver, the Frenzel maneuver carries a significantly lower
risk of over-pressurizing the middle or inner ear; given e.g., a nose-clip, it also allows hands-
free equalisation. The maneuver can be done at any time during the respiratory cycle and it
does not inhibit venous return to the heart. Effort to perform the maneuver is minimal, and it
can be repeated many times quickly.

3. Swallowing or yawning
4. Tympanometry
5. Politzerization
6. Flexible fiberoptic nasopharyngoscopy
Patulous eustachian tube
It occurs due to sudden loss of weight, pregnancy, contraceptive pills, diuretics and elderly
patients. Patient presents with autophony (hears his own voice during talking, eating and
breathing). There is no hearing loss. On examination tympanic membrane moves
synchronous with respiration.
Treatment options of patulous eustachian tube consist of removal of underlying cause,
reassurance, grommet insertion and injection of teflon paste around eustachian cushion.
SELF ASSESSMENT
Q. 1. Tympanic membrane is normal looking in all except

a. eustachian tube blockage d. presbycusis

b. Meniere’s disease e. vestibular neuronitis


c. otosclerosis
Q. 2. A pregnant woman in third trimester complains of hearing her own sounds. Examina-
tion of the ear shows movements of tympanic membrane synchronous with respiration
and especially exaggerated when nostril on the contralateral side is occluded. Your
diagnosis is
a. eustachian tube obstruction d. otosclerosis

b. labyrinthitis e. patulous eustachian tube

c. otitis media with effusion


Scenario: A 04 years old child was brought to ENT OPD. His mother said that her child has
impaired hearing for last 6 months. He also has recurrent attacks of fever, sore throat and
breath from mouth during sleep. On examination tympanic membrane is dull looking.
96
1. What is the most probable clinical diagnosis?
2. What investigations will you carry out?
3. How will you manage this case?
--------------------------------------------------------ANSWERS-----------------------------------------------------
Q.1. a 2. e
Scenario: 1. Eustachian tube obstruction due to enlarged adenoids 2. X-ray soft tissue
nasopharynx lateral view for adenoids, tympanometry
3. Removal of underlying cause i.e., adenoidectomy.
-------------------------------------------------------------------------------------------------------------------------
FACIAL PARALYSIS
Pathologically facial nerve is divided into two parts;
1. Central part.
2. Peripheral part.
It is again divided into:
a. Intratemporal part
b. Extratemporal part
Types of facial paralysis:
Facial paralysis may be divided into central and peripheral
types. Features of facial paralysis

A. Central paralysis:
This is also known as supranuclear type of paralysis. This type of paralysis is rare.
It is caused by CVA (hemorrhage, thrombosis or embolism), tumor or abscess. This
affects only the lower part of the face, whereas the upper half of the face is spared (because
upper half of the face has a bilateral innervation. It is almost always incomplete. Involuntary
emotional movements and the tone of facial muscles is retained.
B. Peripheral paralysis:
This is also known as infranuclear type of paralysis. This type of paralysis is common. This
affects both the upper and lower parts of the face.
Causes of facial paralysis:
A. Intracranial:
1. Trauma e.g., head injury
2. Inflammations e.g., meningitis
3. Tumors e.g., vestibular schwannoma, meningioma
B. Intratemporal:
1. Trauma (surgical and accidental) of middle ear
97
2. Inflammations (acute and chronic) of middle ear e.g., herpes zoster oticus
3. Tumors of middle ear e.g., glomus tumor, carcinoma of middle ear
C. Infratemporal:
1. Trauma (surgical and accidental) of the face.
2. Tumors of parotid
D. Miscellaneous:
Idiopathic, sarcoidosis, polyneuritis, infectious mononucleosis, leukemia
Causes of recurrent facial paralysis:
1. Bell’s palsy
2. Melkersson-Rosenthal syndrome
3. Infectious mononucleosis
4. Osteopetrosis
5. Viral disorders causing facial paralysis
6. Guillain-Barre syndrome (cytomegalovirus, herpes zoster)
Causes of recurrent alternating facial paralysis:
1. Bell’s palsy
2. Melkersson-Rosenthal syndrome
Investigations:
Investigations of facial paralysis depend highly on clinical findings. Generally, following
investigations are carried out as below:
1. ESR to detect chronic illness.
2. Glucose tolerance test to detect diabetes mellitus.
3. Bone marrow examination to detect leukemia or lymphoma.
4. TPI and FTA test to detect syphilis.
5. High resolution CT/MRI may be done to find out the cause and site of facial paralysis.
6. Electrophysiological tests:
These are used to assess the degree of facial nerve dysfunction. These tests help in
determining the prognosis of facial nerve recovery.
These tests include;
a. Minimal nerve excitability test
b. Maximal stimulation test
c. Electroneurography
d. Electromyography
98
7. Topodiagnostic tests:
These are used to determine site of the lesion. In these tests function of different branches
of facial nerve is assessed e.g., taste, salivary flow rate, Schirmer test, stapedial reflex test
etc. These are of only historical interest.
Complications of facial paralysis:
1. Synkinesis:
This is an abnormal synchronization of movements of facial muscles. The patient is unable
to move each part of the face separately.
2. Crocodile tears:
There is increased unilateral lacrimation of the involved side associated with eating. It is
the result of faulty regeneration of parasympathetic fibers which innervate the lacrimal gland
instead of salivary glands.
3. Stapedius tendon contraction:
It occurs due to faulty regeneration and causes fullness or roaring in the ear.
4. Hemifacial spasm:
It is characterized by mild intermittent spasm in the orbicularis oculi muscle.
5. Facial myokemia:
This is continuous fine fibrillary or undulating movement of facial muscles giving appearance
of “a bag of worms”.
6. Blephrospasm:
Involuntary spasmodic closure of eye may occur due to spasm of orbicularis oculi muscle.
Classically it is a bilateral disorder of facial muscles but may occur on one side only.
7. Psychogenic or habit tic:
The movements of facial muscles are repetitive.
Surgical treatment:
Surgical treatment is only carried if there is BAD syndrome.
B--Absence of Bell’s phenomenon (upwards rolling of eyeball on closure of eye)
A--Anesthesia of cornea
D--Dry eyes
BELL’S PALSY
Synonyms: Idiopathic facial paralysis
Definition:
It is defined as acute onset of peripheral (infranuclear) facial paralysis/paresis

99
of unknown origin. It is named after Sir Charles Bell who
first described the condition.
Etiopathology:
Exact cause of Bell’s palsy is unknown. It is said to be due to
viral infection, vascular ischemia or hereditary factors. Bell’s
palsy constitutes about 80% of peripheral facial paralysis. It
affects both genders equally. It affects all age groups.
Positive family history is present in 10% of patients of Bell’s
palsy. It is recurrent in 5-10% of cases.
Clinical features:
1. Facial asymmetry, drooling of saliva and watering of
eyes (epiphora) are the most common presentations.
2. There may be complaint of loss of taste due to
involvement of chorda tympani nerve.
Bell’s palsy
3. There may be complaint of ipsilateral earache, hyperacusis and
tinnitus due to paralysis of stapedius muscle.
On examination;
1. Patient is unable to frown the forehead, unable to close the eyes forcibly, unable to show
teeth on the affected side.
2. Ear is usually unremarkable.
Investigations:
It is a diagnosis by exclusion. All the known causes of the peripheral facial paralysis must be
excluded by history, examination and relevant investigations like CRP, ESR, blood sugar,
viral markers and CT/MRI.
Topodiagnostic tests help in detecting the site of the lesion.
Nerve excitability tests are performed to determine the prognosis.
Treatment: (Remember 5 Ps)
1. Pharmacological treatment; If patient presents within one week, prednisolone 1 mg/kg/
day is the drug of choice. Then gradually taper the dose over a period of 2-3 weeks.
2. Protection of eyes to avoid exposure keratitis. Artificial tears during day, eye ointment
and padding of eyes at night is regularly carried out.
3. Physiotherapy of the facial muscles to stimulate recovery process.
4. Psychological support to avoid depression and to improve compliance in treatment.
5. Pain (if present) should be relieved by analgesics.
6. Decompression of vertical and tympanic segment of facial nerve by postaural route may
improve the paralysis.
Prognosis:
About 80% of patients, completely recover, whereas 20% of the patients get incomplete
recovery. Prognosis is excellent in cases of incomplete paralysis and in cases where recovery
100
of facial nerve function starts within 03 weeks.

Differential diagnosis:
1. Herpez zoster oticus
2. Acute otitis media
3. Tumors of middle ear
SELF ASSESSMENT
Q. 1. Bell’s palsy is also known as

a. herpes zoster oticus d. all of the above

b. idiopathic facial paralysis e. none of the above

c. Melkerson-Rosenthal syndrome
Q. 2. Recurrent alternating facial palsy is most common in

a. Bell’s palsy d. Ramsay Hunt syndrome

b. Gradenigo’s syndrome e. all of the above


c. Guillian-Barre’s syndrome
Q. 3. Recurrent alternating facial palsy may be found in
a. Gradenigo’s syndrome d. Ramsay Hunt syndrome

b. Guillian-Barre’s syndrome e. all of the above


c. Melkerson-Rosenthal syndrome
Q. 4. Which of the following statement represents Bell’s palsy?
a. combined paralysis of facial and nerve paralysis
vestibulocochlear nerve d. idiopathic paralysis of facial nerve
b. facial nerve paralysis with vesicles e. all of the above
c. hemiparesis and contralateral facial
Q. 5. Cause of Bell’s palsy is
a. granulomatous d. neoplastic
b. idiopathic e. traumatic
c. inflammatory
Q. 6. Which statement is correct regarding Bell’s palsy.
a. always a complete paralysis facial paralysis

b. associated with acute otitis media d. good prognosis

c. constitutes a small percentage of e. slow in onset


Q. 7. Hyperacusis in Bell’s palsy is due to the paralysis of
a. levator palati c. stapedius

b. orbicularis oris d. tensor palati


e. tensor tympani
Q. 8. Usual treatment of Bell’s palsy includes all except

a. antibiotics d. psychological support

b. physiotherapy e. steroids
c. protection of eyes
Q. 9. Usually, prognosis of complete facial nerve recovery in Bell’s palsy is
a. 20% d. 80%

b. 40% e. 100%

c. 60%
Q. 10. Bell’s phenomenon is
a. corneal ulceration in Bell’s palsy d. uncontrollable contractions of orbicu-
b. downwards rolling of eyeball on clo- laris oculi muscle
sure of eye e. upward rolling of eyeball on closure of
c. epiphora associated with Bell’s palsy eye
Q. 11. The cranial nerve which travels the longest distance in the bony canal is
a. 3rd cranial nerve d. 9th cranial nerve

b. 5th cranial nerve e. 11th cranial nerve

c.7th cranial nerve


Q. 12. Smallest segment of facial nerve is

a. intracanalicular d. parotid segment before its division

b. labyrinthine e. tympanic
c. mastoid
Q. 13. Bony facial canal has the narrowest diameter in which of the following segments?
a. Labyrinthine d. Tympanic

b. Mastoid e. none of the above

c. Meatal
Scenario: A 40 years old male presents 03 days after spontaneous sudden right sided facial
palsy, with no history of previous ear disease. On examination, he is having complete right
infranuclear facial palsy with normal ear examination.
1. What is the most likely diagnosis?
2. What is the differential diagnosis?
3. What is the treatment of this patient?
4. What are the precautionary measures to avoid complications?
------------------------------------------- ANSWERS--------------------
------------- ---------------------------------

Q.1. b 2. a 3. c 4. d 5. b 6. d 7. c 8. a
9. d 10. e 11. c 12. b 13. a
Scenario: 1. Bell’s palsy 2. Herpez zoster oticus, acute otitis media, tumors of middle ear
3. Pharmacological, protection of eyes, physiotherapy, psychological support, decompression
4. Protection of eyes, physiotherapy and psychological support.
--------------------------------------------------------------------------------------------------------------------------

NEOPLASMS OF MIDDLE EAR CLEFT


Tumors of middle ear may be divided into;
1. Benign tumors
2. Malignant tumors
1. Benign tumors
The most common benign tumor of middle ear is glomus tumor.
GLOMUS TUMOR
Synonyms: Paraganglioma, Chemodectoma
Etiopathology:
The term glomus is misnomer and refers to originally held belief that their origin was
vascular. It is the most common benign tumor of middle ear. There are two types of glomus
tumors in the middle ear;
1. Glomus tympanicum arise from the area of Jacobson’s nerve on the promontory of
middle ear.
2. Glomus jugulare arise from the jugular bulb involving jugular foramen and adjacent
structures.
These are five times more common in females as compared to
males. These are more common in middle age. These are usually
found in people living at high altitude.
These have a well defined thin fibrous capsule and are locally
invasive and destructive of bone and facial nerve.
These tumors arise from paraganglionic cells which are widely
distributed in the autonomic nervous system. The usual sites of
paraganglionic cells are the carotid, ciliary and vagal bodies.
A case of glomus tumor
These may also occur along the aorta and its main branches, along
internal jugular vein and para-adrenal areas. Natural history of their growth is extremely
variable.
Remember rule of 10 %. In 10% of cases these are multicentric. In 10% of cases these
are familial. In 10% of cases these are secretors i.e., secrete catecholamines. In about 10 %
103
of cases malignant change can occur in them.

Clinical features:
1. Pulsatile tinnitus; The first symptom generally, noted is unilateral tinnitus which
synchronizes with the pulse.
2. Hearing loss and aural bleeding; Some patients may present with hearing loss and aural
bleeding.
3. Occasionally it may present with friable aural polyp prone to bleeding or a mass over
the mastoid.
On examination;
1. Otoscopy shows a red mass (‘rising sun’ sign or also known as
‘setting sun’ sign) behind the tympanic membrane.
2. Pulsation sign (also known as Brown sign) is positive i.e., tumor
pulsates vigorously when pressure in the ear canal is increased
with the help of Siegel’s speculum. Reverse phenomenon happens
on release of pressure.
3. Examination under microscope (EUM) shows pulsation of the Rising sun sign
mass which will be soft and often blanch on palpation.
4. Tuning fork tests show conductive hearing loss.
5. On auscultation there may be bruit over the mastoid.
6. Paralysis of cranial nerves especially 7th, 9th, 10th, 11th and 12th may occur
depending on the site and extent of tumor.
Investigations:
A. Radiology:
1. CT: On CT of skull there is absence of normal crest of bone between the carotid canal
and jugular fossa (Phelps sign). It is virtually diagnostic of glomus jugulare tumor.
2. MRI: It shows exact extent of the tumor.
3. Angiography: It is the best method for preoperative detailed assessment of the extent
of tumor and indication of its arterial blood supply. Angiography shows “tumor blush”.
It is also helpful in identifying other glomus tumors of the body
as these may be multicentric.
It may also help in embolization of the feeding vessels to minimize
the peroperative bleeding. Embolization of the feeding vessels is done
with gel foam or lyophilized dural fragments. It is carried out 24-48
hours before surgery. Tumor blush on
B. Endocrine activity: angiography
104
Prior to any surgery it is very important to exclude any vasoactive hormones produced by the
tumor. It is done by estimation of 24 hours urinary catecholamines and Vanillylmandelic
acid (VMA). Normal level of VMA is upto 7 mg/24 hour.
C. Biopsy:
As it is a vascular tumor, biopsy should be avoided. It is occasionally required to exclude
squamous cell carcinoma.
D. Pure tone audiometry:
Pure tone audiometry is important to assess any conductive hearing loss and to ensure that
normal or better hearing exists in the other ear.

Treatment:
No treatment is given especially when the tumor is extremely slow growing, patient’s
symptoms are minimal or if the patient presents in 6th or 7th decade.
1. Surgery:
The objective of surgery is total resection of the tumor without increasing patient’s
neurological deficit. Exact approach depends on exact extent of the tumor. Often help of a
neurosurgeon is required for its excision. Vascularity of the tumor is reduced by
preoperative irradiation or embolization.
It is important to exclude any production of vasoactive hormones by the tumor. If there is
production of vasoactive hormones, patient may have profound hypertension. This profound
hypertension is controlled by intra-arterial monitoring. Drugs used to lower blood pressure
during surgery are phenolamine or phenoxybenzamine. This may significantly reduce
peroperative bleeding.
Initial hypertension often requires both alpha and beta blockade and following
Embolization and surgical resection, the loss of vasoconstrictor tone results in circulatory
collapse, needing correction with massive intravenous infusions and the use of an anti-
gravity suit to the lower part of body to increase venous return.
2. Radiotherapy:
It often reduces the size of the tumor and bleeding generally, stops after radiotherapy.
Usually, 4000-5000 cGy are delivered over a period of 3-4 weeks.
SELF ASSESSMENT
Q. 1. Glomus tumor is also called

a. chemodectoma d. both a & b

b. paraganglioma e. both b & c


c. schwannoma
Q. 2. The most common benign tumor of middle ear is
105

a. adenoma d. papilloma

b. epithelioma e. paraganglioma
c. osteoma
Q. 3. Which disease is predominant in females?
a. Glomus tumor d. Nasopharyngeal carcinoma

b. Inverted papilloma e. Puberphonia

c. Nasopharyngeal angiofibroma
Q. 4. All are true of glomus tumor of the middle ear except

a. may secrete vasoactive hormones d. slow growing

b. multicentricity e. very vascular

c. predominantly affects male


Q. 5. Which of the following is not the site of paraganglioma?

a. Carotid body d. Jugular foramen

b. Ciliary body e. Promontory

c. Geniculate ganglion
Q. 6. Glomus tumor may secrete

a. histamine d. prostaglandins

b. leukotrienes e. VMA

c. prostacyclins
Q. 7. Normal level of VMA in 24-hour urine is

a. 3 mg d. 9 mg

b. 5 mg e. 11 mg

c. 7 mg
Q. 8. All of the following are true about glomus tumor except

a. 10 % cases are familial d. 10 % cases are multicentric

b. 10 % cases are in females e. 10 % cases are non-secretors


c. 10 % cases are malignant
Q. 9. Unilateral pulsatile tinnitus is a typical feature of

a. acoustic neuroma d. palatal myoclonus

b. glomus tumor e. vestibular neuronitis


c. otosclerosis
Q. 10. Usual clinical features of glomus tumor include all except
a. aural bleeding d. pulsatile tinnitus

b. aural polyp e. sensorineural hearing loss

c. conductive hearing loss


Q. 11. Brown’s sign is seen in

a. acoustic neuroma d. Meniere’s disease

b. glomus tumor e. otosclerosis


c. labyrinthitis
Q. 12. All of the following are true in the diagnosis of glomus tumor of the middle ear
except
a. bleeding polyp in ear d. needs to estimate vanillylmandelic acid

in urine
b. bruit over the mastoid

c. cranial nerves never affected e. swelling over the mastoid

Q. 13. The most common cranial nerve involved in glomus tympanicus is


a. 4th cranial nerve d. 7th cranial nerve

b. 5th cranial nerve e. 9th cranial nerve


c. 6th cranial nerve
Q. 14. Hearing loss, dizziness, fullness of ear and pulsatile tinnitus in a 50-year-old
female which has a red mass behind an intact and normal tympanic membrane
points towards
a. aberrant internal carotid artery d. high jugular bulb

b. acute otitis media e. persistent stapedial artery


c. glomus tympanicum
Q. 15. Investigation that should be avoided in glomus tumor is
a. angiography d. MRI

b. biopsy e. PTA

c. CT scan
Q. 16. Phleps sign is seen in

a. glomus jugulare d. neurofibroma

b. glomus tympanicum e. vestibular schwannoma


c. Meniere’s disease
Q. 17. 24-hour urine catecholamine estimation done and found high in a patient. What is
your diagnosis?
a. Acoustic neuroma d. Tumor of inner ear

b. Glomus tumor e. Tumor of thyroid


c. Tumor of cerebellopontine angle
Q. 18. All are true in the diagnosis of glomus tumor of middle ear except
a. bleeding polyps in the ear d. needs to estimate vanillylmandelic acid
b. bruit over the mastoid in urine
c. common in females e. uncommon in people living at high
altitude
107

Q. 19. Which of the following signs is not associated with glomus tumor?
a. Brown’s sign d. Rising sun sign

b. Griesinger’s sign e. Setting sun sign


c. Phelp’s sign
Q. 20. Regarding glomus tumor of the middle ear, which of the following statement is
not correct?
a. Diagnostic biopsy is contraindicated d. More common in men

b. Family history may be positive e. Multicentric in origin


c. Grows very slowly
Q. 21. In glomus tumor, peroperative bleeding can be reduced by all except
a. alpha blockers d. preoperative emobilization

b. antigravity suit e. preoperative radiotherapy


c. beta blockers
Q. 22. Treatment of an early case of glomus tumor confined to middle ear is
a. chemotherapy d. surgery

b. embolization e. surgery followed by chemotherapy

c. radiotherapy
Scenario: A 50 years old female presents with right sided pulsatile tinnitus and progressive
hearing loss for last 06 months. On examination, there is a red mass behind the intact
tympanic membrane which blanches on pressure with pneumatic speculum.
1. What is the most likely diagnosis?
2. How will you investigate this patient?
3. What is the treatment of this patient?
--------------------------------- ANSWERS--------------------------------
----------------------- ---------------------
Q.1. d 2. e 3. a 4. c 5. c 6. e 7. c 8. e
9. b 10. e 11. b 12. c 13. d 14. c 15. b 16. a
17. b 18. e 19. b 20. d 21. b 22. d
Scenario: 1. Glomus tumor 2. Contrast MRI of temporal bone, angiography, estimation of
24-hour urinary catecholamines and Vanillylmandelic acid (VMA) 3. Surgery, radiotherapy.
--------------------------------------------------------------------------------------------------------------------------
-
2. Malignant tumors
The most common malignant tumor of middle ear is squamous cell carcinoma. The other
malignant tumors of the middle ear are adenocarcinoma and rhabdomyosarcoma.
108
SQUAMOUS CELL CARCINOMA
It is the most common primary malignant tumor of middle ear. It may be confused with
chronic suppurative otitis media and, therefore, may be quite advanced at the time of
diagnosis. It mainly affects elderly patients. It usually invades the surrounding structures like
eustachian tube and mastoid. It causes multiple cranial nerve palsies. Facial nerve is the first
to be affected.

Clinical features:
1. Severe earache is the most common
presenting symptom.
2. Usually there is history of
chronically discharging ear.
On examination;
1. There are hemorrhagic granulations
in the ear canal.
2. There may be facial paralysis along
with paralysis of other cranial
nerves.

Investigations:
1. High resolution CT/MRI are very
important to find out exact extension of disease. Endoscopic view of SCC of middle ear
2. Biopsy is essential to make a tissue diagnosis.

Treatment:
The treatment is primarily surgery with radiotherapy.
Radiotherapy may be given in patients who are medically unfit or refuse surgery.
Five years survival rate in cases of carcinoma of middle ear is very low.
Differential diagnosis:
1. Malignant otitis externa
2. Diffuse otitis externa
3. CSOM with complication
SELF ASSESSMENT
Q. 1. The most common primary malignant tumor of middle ear is

a. adenocarcinoma d. squamous cell carcinoma

b. adenoid cystic carcinoma e. transitional cell carcinoma


c. lymphoma
Q. 2. A 02 years old child presents with mastoid swelling, polypoidal mass in the right
ear and right facial paralysis for last one month. The most likely diagnosis is?
a. adenocarcinoma of middle ear d. lymphoma

b. carcinoma of middle ear e. rhabdomyosarcoma


c. glomus tumor.
Q. 3. Usually, malignancy in any of the following regions can involve middle ear except
109
a. external ear d. parotid

b. hypopharynx e. none of the above


c. nasopharynx
Q. 4. All are true about middle ear malignancy except
a. adenocarcinoma is the most common d. causes intolerable earache

b. affects elderly patients e. facial nerve is the first to be affected

c. causes multiple cranial nerve palsies


Scenario: A 60 years old male with the history of right sided chronic ear discharge presented
with severe earache for last 01 month. On examination patient is having hemorrhagic
granulations in his right external ear canal and paralysis of 7th, 9th, 10th and 11th cranial
nerves. Patient is non diabetic. CT shows widespread erosion of temporal bone.
1. What is the most likely diagnosis?
2. What is your differential diagnosis?
3. How will you investigate this patient?
4. What is the treatment of this patient?
--------------------------------------------------------ANSWERS-----------------------------------------------------
Q.1. d 2. e 3. b 4. a
Scenario: 1. Malignancy of middle ear 2. Malignant otitis externa, CSOM with complication,
diffuse otitis externa 3. HRCT, MRI, biopsy 4. Surgery followed by radiotherapy.
--------------------------------------------------------------------------------------------------------------------------
-
110

Chapter 5
DISEASES OF OTIC CAPSULE
OTOSCLEROSIS
Synonym: Otospongiosis
Definition:
It is a hereditary localized disease of otic capsule. In it, normal lamellar bone is removed by
osteoclasts and is replaced by new spongy cancellous bone of greater thickness, cellularity
and vascularity. This phenomenon causes ankylosis of footplate of stapes.
Etiopathology:
It is a progressive bilateral disease but usually one ear is
affected more than the other. It affects 2% of all white
(Caucasian) population.
It generally, starts in teens. Male are affected more frequently
with a male to female ratio of 2:1. Hormonal influences such as
puberty, pregnancy, menstruation and menopause may cause
the disease to advance more rapidly in females. Hence, females
are three times more likely to present with otosclerosis.
The term otospongiosis is used when there is active vascular
focus. The term otosclerosis is used when the disease finally
becomes inactive. Otosclerosis
Exact cause of otosclerosis is unknown but there are many theories;
1. Hereditary theory; It is an autosomal dominant disease with incomplete penetrance.
There is a positive family history in 50% of the patients.
2. Infective theory; Measles virus has been suggested as a causative factor. Vaccination
against measles has significantly decreased the incidence of otosclerosis.
3. Autoimmune/inflammatory theory;
4. Hormonal theory; Sex hormones especially estrogen is thought to be responsible for
otosclerosis.
Types of otosclerosis;
Depending on the site of involvement, there are following types of otosclerosis;
1. Stapedial otosclerosis: It is the most common type of otosclerosis. It causes fixation of
stapes footplate and hence, causes conductive hearing loss.
111
Cochlear otosclerosis: It involves round window and other areas in otic capsule and hence
causes sensorineural hearing loss.
2. Histological otosclerosis: It remains asymptomatic.

Clinical features:
1. Hearing loss:
There is gradually increasing bilateral hearing loss but initially there may be unilateral
problem. Although disease starts in teens but symptoms become severe enough to present
frequently in the 3rd decade. The patient may remark that the hearing is better in the presence
of background noise which is known as paracusis willisii. Often one ear shows a greater loss
than the other. Patient characteristically speaks in a low tone, whereas patient with
sensorineural hearing loss speaks in a sharp tone.
2. Tinnitus:
There is also complaint of unilateral or bilateral roaring or hissing tinnitus.
Rarely there may be episodes of vertigo which are thought to result from the action of
toxic enzymes released by otosclerotic focus.
On examination;
1. External meatus and tympanic membranes are unremarkable.
2. There may be ‘flamingo flush/blush’ also known as
positive Schwartz sign. It is a red blush of tympanic
membrane over the promontory as a result of vascular
bone on the promontory or prominent blood vessels in the
submucous layer of promontory. This usually indicates
rapid progression of the disease.
3. Tuning fork tests show conductive hearing loss.
Absolute bone Schwartz sign

conduction (ABC test) is lengthened in otosclerosis. Sensorineural hearing loss


indicates involvement of cochlea by the otosclerotic process.
Investigations:
1. Pure tone audiometry (PTA):
A. Initial stages show an air bone gap which is
usually greater at lower frequencies.
B. Later stages show bone conduction curve with a
characteristic dip at 2 kHz which is known as
Carhart notch. Carhart notch at 2 kHz

Cookie bite pattern is seen in mixed hearing loss.


112
2. Tympanometry:
‘As’ type of tympanogram is diagnostic of otosclerosis.

a. In the initial stages, tympanogram may be normal or


Tympanogram

may show a decrease in compliance.


b. In the later stages tympanogram usually shows
compliance which may be less than 0.25 mm.
When stapes footplate becomes completely fixed,
compliance becomes almost zero. Middle ear
pressure remains normal throughout the disease
process.
3. Stapedial reflex is absent when there is fixation of stapes.
4. Speech audiometry; It shows normal discrimination score except in
cochlear otosclerosis.
5. High resolution CT shows thickening of the stapes footplate and/or
evidence of cochlear involvement.
Treatment: CT scan showing
1. Conservative: thickening of stapes footplate

a. Sodium fluoride may be given in active stage of


otosclerosis where Schwartz sign is positive. It is given in a dose
of 25-75 mg/ day for at least 02 years.
Contraindications to fluoride therapy are children, peptic ulcer,
gastritis, chronic nephritis, chronic rheumatoid arthritis,
pregnancy, lactation, and those who are allergic to fluoride.
Complications of fluoride therapy are gastritis, peptic ulcers,
allergic reaction, skeletal fluorosis, increased risk of fractures and
arthritis.
b. Hearing aid may be applied in one or both ears to decrease the disability of hearing
loss. Patients may also be given training on lip reading and auditory training.
2. Surgical:
a. Stapedectomy/stapedotomy is the treatment of choice. In this the fixed
otosclerotic stapes is removed and a prosthesis inserted between the incus and oval
window. Prosthesis may be made of stainless steel, teflon or titanium-teflon. Ninety
percent of patients achieve good hearing improvement after stapedectomy.
b. Bone Anchored Hearing Aid (BAHA) may be used as an alternative to address
hearing loss of otosclerosis.
Differential diagnosis:
1. Otitis media with effusion
2. Eustachian tube dysfunction
3. Congenital footplate fixation
4. Healed suppurative otitis media
5. Ossicular dislocation or fixation
STAPEDECTOMY
Indications:
1. Hearing threshold should be 30 dB or worse.
2. Average air bone gap should be at least 15 dB with Rinne negative for 256 and 512 Hz.
3. Speech discrimination score should be 60 % or more.
Indications of stapedectomy other than otosclerosis:
1. Congenital stapes footplate fixation
2. Fibro osseus stapes footplate fixation
3. Degenerative stapes footplate arthritis
4. Fracture of stapes footplate
5. Osteogenesis imperfecta
Contraindications:
1. Otitis externa
2. Only hearing ear
3. Associated Meniere’s disease
4. Tympanic membrane perforation
5. Young children
6. Professional athletes, divers, frequent air
travelers, high altitude construction workers
7. Those who work in noisy surroundings
Procedure:
It may be carried out under local or general
anesthesia. Meatal incision is made and elevation
of tympanomeatal flap is done to expose the
stapes area. This may require removal of
posterosuperior bony overhang of the canal.
Removal of stapes suprastructure and creation of
hole in stapes footplate (stapedotomy) or removal of
part of footplate (stapedectomy) is done. Prosthesis is Prosthesis used in stapedectomy
placed between the incus and oval window.
Repositioning of the tympanomeatal flap is carried out at the end.
Complications of stapedectomy:
1. Tear of tympanic membrane
2. Sensorineural hearing loss
114
3. Dead ear
4. Floating/depressed/submerged footplate
5. Damage to ossicular chain
6. Facial nerve paresis/paralysis
7. Damage to the chorda tympani nerve
SELF ASSESSMENT
Q. 1. Otosclerosis is also known as

a. osteopetrosis d. all of the above

b. osteoprosis e. none of the above

c. otospongiosis
Q. 2. Otosclerosis is

a. autosomal dominant disease d. X-linked disease

b. autosomal recessive disease e. Y-linked disease


c. mitochondrial disease
Q. 3. In the pathogenesis of otosclerosis, the disease process starts in
a. bone of otic capsule which develops from cartilage
b. endosteal layer of otic capsule
c. mucoperiosteum of the promontory
d. mucosa of the promontory
e. periosteal layer of otic capsule
Q. 4. The commonest site of predilection for otosclerotic focus is
a. behind the round window d. promontory

b. footplate of stapes e. round window


c. front of round window
Q. 5. The prevalence of clinical otosclerosis is highest in
a. Africans d. Chinese
b. Americans e. Japanese
c. Caucasians
Q. 6. In cochlear otosclerosis usually
a. family history is positive c. sensorineural hearing loss
b. paracusis Willisii is absent d. all of the above
e. none of the above
Q. 7. Chief symptom of otosclerosis is
a. ear discharge d. hearing loss and fullness of ear

b. hearing loss and tinnitus e. tinnitus and vertigo

c. hearing loss and vertigo


115
Q. 8. All are usually true about hearing loss in otosclerosis except
a. bilateral d. one ear is affected more than the
b. insidious and painless other
c. manifests in childhood e. progressive in nature
Q. 9. Tympanic membrane in otosclerosis is usually
a. atrophic d. slightly congested

b. bluish e. yellow
c. normal
Q. 10. Conductive type of hearing loss is seen in all except
a. chronic suppurative otitis media d. secretory otitis media

b. cochlear otosclerosis e. tympanosclerosis


c. necrosis of incus
Q. 11. Absolute bone conduction (ABC test) is lengthened in
a. acoustic neuroma d. presbycusis

b. Meniere’s disease e. vestibular neuronitis

c. otosclerosis
Q. 12. Positive Schwartz’s sign is

a. bluish hue over the promontory behind an intact tympanic membrane

b. improved hearing in noisy surround- d. reddish hue over the promontory

Ings
e. swelling over the mastoid

c. reddish hue in the hypotympanum


Q. 13. A 25 years old woman suffering from bilateral hearing loss for 06 years which
became profound with pregnancy. On tympanogram, which of the following curve will
be obtained?
a. A type d. B type

b. Ad type e. C type
c. As type
Q. 14. In otosclerosis, when stapes footplate is completely fixed, the compliance is
a. -0.25 mm d. 0.75 mm

b. Zero mm e. 01 mm

c. 0.25 mm
Q. 15. Carhart’s notch is found at

a. 2000 Hz in air conduction curve d. 4000 Hz in bone conduction curve

b. 2000 Hz in bone conduction curve e. 8000 Hz in bone conduction curve

c. 4000 Hz in air conduction curve


116
Q. 16. Cookie bite pattern of graph on pure tone audiometry is seen in
a. conductive hearing loss d. all of the above

b. mixed hearing loss e. none of the above


c. sensorineural hearing loss
Q. 17. Which of the following statements is not true about stapedial otosclerosis?
a. Eustachian tube is patent loss with normal tympanic membrane

b. Rinne test is negative d. Tympanogram is Ad type

c. Slowly progressive conductive hearing e. Tympanogram is As type


Q. 18. Which statement is correct regarding the use of sodium fluoride in otosclerosis?
a. May be given safely in children d. May be given safely in chronic ne-
b. May be given safely during pregnancy phritis
c. May be given safely in rheumatoid e. Used in patients with positive
arthritis Schwartz sign
Q. 19. A 31 years old male patient complains of bilateral progressive hearing loss for
past 05 years. On examination, tympanic membrane is normal and audiogram shows
a bilateral conductive hearing loss. Impedance audiometry shows As type of curve and
absent acoustic reflex. All constitute part of treatment except
a. BAHA d. sodium fluoride

b. gentamicin therapy e. stapedectomy


c. hearing aid
Q. 20. A 30 years old woman with family history of hearing loss from her mother’s side
developed hearing problem during pregnancy. Hearing loss is bilateral, slowly
progressive with bilateral tinnitus that bothers her at night. Pure tone audiometry shows
conductive hearing loss with an apparent bone conduction hearing loss at 2000 Hz. What
is the most likely diagnosis?
a. Acoustic neuroma d. Otosclerosis

b. Meniere’s disease e. Vestibular neuronitis


c. Otitis media with effusion
Q. 21. A 25 years old, 05 months pregnant Caucasian patient presents to you with hearing
loss. She first noticed it during her first pregnancy but it has now deteriorated
considerably. On further inquiry she says that she hears better in noisy surroundings and
also has roaring and hissing noise in her left ear that often fluctuates. On examination you
find normal tympanic membranes however, tuning fork tests show negative Rinne test
bilaterally with a Weber lateralized to left ear. She has incidentally brought her
audiogram that shows a dip at 02 kHz level on left side. The most probable clinical
diagnosis is?
a. Left sided otitis media with effusion d. Left sided osteogenesis
b. Left sided ossicular discontinuity e. Left sided soensorineural hearing loss

c. Left sided otosclerosis (left sided)


117
Scenario A: A 20 years old male presents with gradually increasing bilateral hearing loss for
last 02 years. Clinical examination reveals bilateral normal and intact tympanic membrane.
Tuning fork test revealed conductive type of hearing loss. Pure tone audiometry revealed a
dip at 2000 Hz in bone conduction.
1. What is the most probable clinical diagnosis?
2. Mention tuning fork tests which are helpful in confirmation of the diagnosis?
3. Mention one audiological test which confirms your diagnosis?
4. Enumerate different management plans?
B: A middle age female presents with progressive bilateral hearing loss for last one year. She
is also disturbed with tinnitus in both ears. Her hearing acuity worsened during pregnancy
period. Otoscopic examination revealed bilateral normal tympanic membranes.
1. What is the most probable clinical diagnosis?
2. How will you confirm your diagnosis?
3. How will you manage this case?
C: A 29 years old male presents with progressive bilateral hearing loss for last 10 years.
There is no other otological complaint. On examination, tympanic membranes are in mint
condition. Rinne test is negative bilaterally.
1. What is the most likely diagnosis?
2. How will you investigate the patient?
3. What treatment options will you offer the patient?
D: A 25 years old girl complains of right sided hearing loss for last 08 years. She can hear
better at noisy places. Otoscope revealed normal ear drums.
1. How will you investigate her?
2. What are the treatment options for her?
E: A 20 years old female presents to you with bilateral progressive hearing loss for last 03
years. On examination tympanic membranes are unremarkable. Tuning fork tests show
bilateral Rinne negative. Bilateral tympanogram shows compliance of 0.10 ml with normal
middle ear pressure.
1. What is the most probable clinical diagnosis?
2. What are the treatment options for this condition?
3. What are other conditions in which you may perform same surgical treatment?
------------------------------------ ANSWERS--------------------------
-------------------- ---------------------------
Q.1. c 2. a 3. a 4. b 5. c 6. d 7. b 8. c 9. c
10. b 11. c 12. d 13. c 14. b 15. b 16. b 17. d 18. e
19. b 20. d 21. c
--------------------------------------------------------------------------------------------------------------------------
118
Scenario A: 1. Otosclerosis 2. Rinne, Weber, ABC 3. Tympanometry 4. Sodium fluoride,
hearing aid, stapedectomy, BAHA.
B: 1. Otosclerosis 2. PTA, tympanometry, CT scan 3. Sodium fluoride, hearing aid,
stapedectomy, BAHA.
C: 1. Otosclerosis 2. PTA, tympanometry 3. Sodium fluoride, hearing aid, stapedectomy,
BAHA.
D: 1. PTA, tympanometry 2. Sodium fluoride, hearing aid, stapedectomy, BAHA.
E: 1. Otosclerosis 2. Sodium fluoride, hearing aid, stapedectomy, BAHA 3. Congenital stapes
footplate fixation, fibro-osseus stapes footplate fixation, degenerative stapes footplate
arthritis
--------------------------------------------------------------------------------------------------------------------------
119
Chapter 6
DISEASES OF INNER EAR
INFLAMMATORY CONDITIONS OF INNER EAR
VESTIBULAR NEURONITIS
Synonym: Vestibular neuritis
It is characterized by sudden severe episode of vertigo,
nausea and vomiting without hearing loss and tinnitus.
Etiopathology:
Exact etiology is unknown. It is said to be result of Vestibular neuronitis
inflammation of vestibular ganglion by various factors like
viruses. There is usually a preceding history of upper respiratory tract infection (URTI). It is a
self-limiting disease and settles within 2-3 weeks. Usually, patient recovers spontaneously
within a few days. Younger patient recovers quickly but an older patient may be unsteady for
months after an episode.
The most commonly affected age group is between 30 to 50 years.

Clinical features:
1. Sudden severe episode of vertigo is the most common presentation.
2. There is no hearing loss or tinnitus.
On examination;
1. Tuning fork tests are unremarkable.
2. Nystagmus is positive during the episode with fast component towards the affected side.
There is no sign of neurological involvement.
3. On caloric testing, there is reduced vestibular sensitivity (canal paresis) on the affected
side. Clinical features are those of acute vestibular failure.
Differential diagnosis:
1. Viral labyrinthitis
2. Meniere’s disease
3. Vestibular schwannoma
Treatment:
Treatment is directed towards relief of symptoms. It is the same as that for acute episode of
Meniere’s disease.
120
SELF ASSESSMENT
Q. 1. All are true of vestibular neuronitis except
a. affects adults in 30-50 age group d. devoid of cochlear symptoms

b. causes persistent vertigo e. inflammation of vestibular ganglion

c. caused by Treponema pallidum


Q. 2. Vestibular neuronitis is characterized by all of following except

a. caloric testing shows reduced re- c. no tinnitus

sponse
d. sensorineural hearing loss

b. history of upper respiratory infection e. sudden severe episode of vertigo

Scenario: A 50 years old male presents with sudden, severe vertigo 02 days after the upper
respiratory tract infection. There is no tinnitus or hearing loss. On caloric testing there is
reduced vestibular sensitivity on right side.
1. What is the most probable clinical diagnosis?
2. What is the differential diagnosis?
3. What is your treatment plan?
--------------------------------------------------------ANSWERS-----------------------------------------------------
Q.1. c 2. d
Scenario: 1. Right vestibular neuronitis 2. BPPV, Meniere’s disease, labyrinthitis
3. Symptomatic treatment.
--------------------------------------------------------------------------------------------------------------------------
VIRAL LABYRINTHITIS
Etiopathology:
It usually occurs during the course of
herpes, measles, mumps or as a part of
influenza type illness.
Clinical features:
Vertigo, tinnitus and hearing loss are the
most common presentations. It is
differentiated from vestibular neuronitis
where there is absence of tinnitus and
hearing loss.
Differential diagnosis:
1. Vestibular neuronitis
2. Meniere’s disease
3. Vestibular schwannoma
Treatment:
It is same as that for serous labyrinthitis.
SELF ASSESSMENT
121
Q. 1. Vertigo, sensorineural hearing loss and tinnitus are seen in all except
a. hypothyroidism d. syphilitic labyrinthitis

b. Meniere’s disease e. viral labyrinthitis


c. perilymh fistula
Q. 2. All are true about viral labyrinthitis except
a. managed surgically d. there may be vertigo
b. there may be hearing loss e. usually occurs during course of viral
c. there may be tinnitus infection
--------------------------------------------------------ANSWERS-----------------------------------------------------
Q.1. a 2. a
--------------------------------------------------------------------------------------------------------------------------
MISCELLANEOUS DISEASES OF INNER EAR
VERTIGO
Definition:
It is defined as a subjective sense of imbalance.
Disorientation in space causes vertigo or dizziness and can arise from
disorders of any of the three systems i.e., vestibular, visual or somatosensory
(proprioceptive). Normally, the impulses reaching the brain from the three
systems are equal and opposite.

The sensation of vertigo

If any component on one side is inhibited or stimulated, the information reaching the cortex is
mismatched, resulting in disorientation and vertigo. The vestibular inhibitio n on one side e.g.,
acute vestibular failure, labyrinthectomy, Meniere’s disease, 8th nerve section causes vertigo.
Similarly, stimulation of labyrinth by thermal or rotational stimulus causes vertigo.
Dizziness can also result from the ocular causes, e.g., high errors of refraction or acute
extraocular muscle paralysis with diplopia.
Types of vertigo:
A. Physiological:
It is the vertigo which is caused by stimulation of normal and intact sensory structures. e.g.,
1. Giddiness of heights from visual stimulation.
2. Giddiness after spinning movements from stimulation of semicircular canals.
3. Giddiness from sudden change of floor texture from stimulation of skin and deeper
tissues of the feet (proprioceptors).
B. Pathological:
122
It is the vertigo resulting from any disease which affects anyone of the many functions used
in balance. It may be;
a. Peripheral i.e., due to diseases of the vestibular end organs or vestibular nerve.
b. Central i.e., due to diseases of central connections. It involves vestibulo-ocular,
vestibulospinal and other central nervous system pathways.
Causes of vertigo:
a. Peripheral
1. Benign paroxysmal positional vertigo
2. Labyrinthitis
3. Meniere’s diseases
4. Vestibular neuronitis
5. Vestibulotoxic drugs
6. Vestibular schwannoma
7. Perilymph fistula
8. Syphilitic labyrinthitis
b. Central
1. Cerebellar disease
2. Vertebrobasilar insufficiency
3. Multiple sclerosis
4. Posterior inferior cerebellar artery (PICA) syndrome
5. Tumors of brainstem
Clinical features:
Vertigo is a symptom for which underlying cause must always be identified. A detailed
history of patient with vertigo is extremely important. If diagnosis of cause of vertigo is not
made at the end of history it is unlikely to have diagnosis at the end of examination or even
investigations.
History is best taken chronologically. It is very important to get description of the first
episode, duration, frequency and interval between vertigo, initiating/aggravating and
relieving factors, associated symptoms such as aura, hearing loss, tinnitus, nausea, vomiting,
blurring of vision, diplopia or loss of consciousness.
Detailed examination of ears, cranial nerves and cerebellar system is extremely
important.
Clinician must look for nystagmus and its specific characteristics.
Treatment:
Treatment of any patient with vertigo is carried out on following principles;
123
1. Address the underlying cause:
2. Suppress the vestibular system: In vertigo usually, there is disproportion
between the activities in the two sets of vestibular nuclei. So labyrinthine sedative
drugs such as cinnarizine (cerebrin, stugeron), cyclizine (migril), dimenhydrinate
(dramamine) and prochlorprazine (stabil, stemitil) are given to suppress the offending
labyrinth.
3. Suppress the patient’s emotional reaction: Patient needs strong
reassurance both about the nature of dizziness and the problems caused by it.
4. Wait for compensation: In the early stages there is ‘cerebellar clamping’ of
the activity of the vestibular nuclei. In young healthy patient compensation by the
healthy side occurs in about 04 weeks. Compensation may be accelerated by the
performance of Cawthorne-Cooksey exercises.
5. Eliminate the offending labyrinth: Sometimes labyrinthectomy or vestibular
nerve section may be the treatment of choice in any condition where labyrinthine
function is already very low or absent.
6. Acceptance of the problem: Patient may be advised to accept the problem. A
walking stick or walking frame is advised.
SELF ASSESSMENT
Q. 1. Condition in which loud sounds produce giddiness is called
a. bobbing oscillopsia c. paracusis willisi

b. Hennebert’s sign e. Tullio’s phenomenon


c. otolithic crisis of Tumarkin
Q. 2. Which of the following organs responds to angular acceleration?
a. Cochlear duct d. Semicircular canals

b. Cochlear nerve e. Utricle

c. Saccule
Scenario: A 30 years old weight lifter presents to you with complaint of severe vertigo which
started during weight lifting 2 hours before.
1. What is the most probable clinical diagnosis?
2. How would you confirm the diagnosis?
3. How will you manage this case?
--------------------------------------------------------ANSWERS-----------------------------------------------------
Q.1. e 2. d
Scenario: 1. Perilymph fistula 2. MRI 3. A; Conservative; antibiotic prophylaxis, strict bed
124
rest, elevation of head end of bed, spinal drain. B; Surgical; surgical exploration,
tympanotomy and closure of the site of leakage
--------------------------------------------------------------------------------------------------------------------------
-
MENIERE’S DISEASE
Synonym: Endolymphatic hydrops, Aural glaucoma

Definition: It is defined as recurrent, spontaneous, sudden and severe


episodic vertigo with tinnitus and fluctuating sensorineural hearing
loss.
Etiopathology:
Exact aetiology of Meniere’s disease is unknown. It is probably as a result of Inner ear with
Meniere’s
an insult to the labyrinth with dilatation of the endolymphatic system i.e., scala
disease
media compartment of inner ear. There is an endolymphatic hydrops. This can occur
either from;
1. Increased production of endolymph by stria vascularis.
2. Decreased absorption by the endolymphatic sac.

The initiating factors of endolymphatic hydrops may be idiopathic, congenital, acquired,


traumatic or inflammatory.
Symptoms are thought to arise from rupture of
Reissner’s membrane. This leads to toxic
stimulation of neuroepithelium of inner ear.
Meniere’s disease generally, starts before the age of
50 years. The second ear involvement usually occur
within 02 years in 50% of the cases.
Clinical features:
1. Symptoms in acute phase:
Usually the patient gets a warning (aura) of an acute episode by
discomfort, fullness or pain in the ear, an alteration in the hearing or
roaring tinnitus.
Feeling of vertigo in
There is extremely severe episodic vertigo associated with tinnitus and Meniere’s disease
hearing loss. Duration of vertigo is 20 minutes to not more than 24 hours.
There is associated nausea, vomiting and other symptoms of vagal disturbance such as
sweating, pallor and slowing of pulse. Patient is usually fully oriented and conscious
throughout the episode. Nystagmus is always present during an episode.
There may be single episode or multiple episodes in the form of clusters. It may occur at
any time. It may awaken the patient from sleep.
2. Symptoms in remission phase:
In the early stages of the disease, patient is usually asymptomatic in the remission phase. As
the disease progresses tinnitus and hearing loss become permanently established.
There may be difference in the pitch of the tone in different ears which is known as
diplacusis.

125
Detailed examination of ear along with
examination of CNS and CVS is carried out.
Tuning fork tests show sensorineural hearing loss.
Nystagmus may be present in acute phase.
Fistula test should always be carried out.
Hennebert sign is positive in 50% of the patients (i.e.,
positive fistula test in the presence of intact tympanic PTA in Meniere’s disease
membrane).
Clinical variant of Meniere’s disease:
Lermoyez syndrome is a rare variant of Meniere’s
disease. In this hearing loss and tinnitus occur over a
period of hours, vertigo then occurs often quite
suddenly and with it hearing and tinnitus improves.
Investigations:
1. Pure tone audiometry shows sensorineural
hearing loss in lower frequencies (rising type
of curve) in early stages of the disease. As
the disease progresses, there is sensorineural
hearing loss in the lower as well as higher
frequencies (flat curve).
As the disease progresses further, there is
sensorineural hearing loss which is more in
the higher frequencies (falling type of curve)
in later stages.
2. Speech audiometry shows speech reception threshold very closely matching the pure tone
threshold. Other audiological tests include tests for loudness recruitment, stapedius reflex
threshold and tone decay test.
3. Electrochochleography is diagnostic of Meniere’s disease.
There is increase in the ratio of summating potential to action
potential. Normally ratio of summating potential to action
potential is 30 %.
4. Dehydration test is based on use of glycerol, urea or
furosemide. This test is not only important for the diagnosis of
Meniere’s disease but also in the assessment for the suitability of Electrochocleography
diuretic treatment and in the selection of patients for operations on the
endolymphatic sac.
5. MRI of the internal auditory meati may be required to exclude vestibular schwannoma.
Treatment:
A. General measures:
1. Reassurance; Patient’s anxiety is
alleviated by reassurance. Patient is
explained about the non fatal nature of the
disease.
2. Cessation of smoking and alcohol.
3. Low salt intake.
126
4. Decrease intake of fluid, tea and coffee.

5. Physical and mental relaxation. Change in life style.


6. Avoid activities which need optimum balance: As episode of Meniere’s disease may
manifest at any moment, so professions like driving, diving and flying should be avoided.
Working near rapidly moving machines and at heights should also be avoided.
B. Treatment of acute phase:
In addition to general measures patient is given strict bed rest.
Following drugs are given;
1. Vestibular sedatives;
a. Dimenhydrinate (dramamine), prochlorperazine (stemitil, stabil) chlorpromazine
(largectil) are given orally or parenterally to relieve vertigo and vomiting.
b. Diazepam (valium) 5-10 mg may be given to relieve an acute episode.
2. Vasodilators;
a. Inhalation of carbogen i.e., 05% carbon dioxide with 95% oxygen. It acts as a
vasodilator and increases labyrinthine circulation.
b. Betahistines may be used which act as vasodilator in the inner ear.
3. Thiazide diuretics; If episode is not relieved by vestibular sedatives and beta-
histines then furosemide 40 mg on alternate day may help to reduce the volume of
endolymphatic compartments and relieve the vertigo. These are especially helpful in
recurrent cases.
C. Treatment of chronic phase:
Relief of tension and anxiety state resulting from unpredictable nature of disease is of prime
importance. Strong reassurance is given emphasizing the non fatal nature of the disease.
a. Conservative:
Various vestibular sedatives and vasodilators such as carbogen and betahistines
greatly increase cochlear blood flow. Labyrinthine sedatives and tranquilizers such as
prochlorperazine (stemetil, stabil) are useful to suppress vertigo, nausea and
vomiting.
b. Ototoxic treatment:
Intratympanic gentamicin therapy: It is injected locally into middle ear either by
local injections or through a ventilation tube. Injections are given daily or on
alternate days. Total dose of gentamicin is 30-40 g. It has the advantage of
vestibulotoxic effect. However, there is some risk of cochlear damage. It is used in
cases of unilateral disease.
Some centers prefer streptomycin over gentamicin.
127
c. Surgical treatment:
Surgical treatment is considered when adequate remission is not achieved with
medical treatment for about 06 months. Most surgical procedures are considered
under following main headings:
1. Conservative procedures:
a. Procedure to influence endolymph production e.g., cervical sympathectomy
b. Portmann shunt. A tube is placed between the endolymphatic sac and subarachnoid
space (in the posterior fossa) to drain excess endolymph.
c. Fick and Cody stainless steel track left permanently in the stapes footplate. A
distended saccule lies close to the stapes footplate. The track causes periodic
decompression of the distended saccule.
d. Selective denervation of the vestibular labyrinth (vestibular neurectomy).
e. Selective destruction of vestibular labyrinth (by ultrasound or cryosurgery).
2. Destructive procedures:
a. Total destruction of labyrinth (labyrinthectomy)
b. Total denervation (vestibulocochlear neurectomy)
The exact procedure depends on the level of hearing in each ear and whether the disease is
unilateral or bilateral.
Differential diagnosis:
A large number of clinical conditions enter into differential diagnosis which include
1. Viral labyrinthitis
2. Vestibular neuronitis
3. Vestibular schwannoma
4. Positional vertigo
5. Syphilitic labyrinthitis
SELF ASSESSMENT
Q. 1. Meniere’s disease is also known as

a. aural glaucoma d. Both a & b

b. endolymphatic hydrops e. Both b & c

c. hydrops fetalis
Q. 2. Endolymph is formed in

a. endolymphatic sac d. scala tympani

b. perilymhatic space e. utricle


c. scala media
128 Part B Ear
Q. 3. Boundaries of scala media (cochlear duct) include the following except
a. basilar membrane c. stria vascularis

d. tectorial membrane
e. None of the above
b. Reissner’s membrane
Q. 4. Crista ampularis is situated in

a. cochlea d. utricle

b. saccule e. vestibule

c. semicircular canal
Q. 5. The endolymph is present in the

a. cochlear duct d. all of the above

b. scala tympani e. none of the above

c. scala vestibuli
Q. 6. In Meniere’s disease, duration of episodic vertigo is

a. 5-10 secs d. 02-03 days

b. 1-5 mins e. 03-04 days


c. 20 mins-24 hours
Q. 7. The type of hearing loss in Meniere’s disease is
a. cochlear d. retrocochlear

b. conductive e. none of the above

c. mixed
Q. 8. Hearing loss in Meniere’s disease is

a. conductive hearing loss d. retrocochlear hearing loss

b. fluctuating sensorineural hearing loss e. none of the above


c. mixed hearing loss
Q. 9. Which of the following is not a typical feature of Meniere’s disease?
a. Associated with episodic vertigo d. Associated with pulsatile tinnitus
b. Associated with fluctuating hearing e. Associated with sensorineural hearing
loss loss
c. Associated with nausea and vomiting
Q. 10. All are usual associated symptoms in Meniere’s disease except
a. bradycardia d. tachycardia
b. pallor e. vomiting
c. sweating

Q. 11. Meniere’s disease is characterized by


a. conductive hearing loss and tinnitus
b. sensorineural hearing loss and tinnitus c. vertigo, hearing loss, tinnitus and
facial pain
d. vertigo, tinnitus, hearing loss and
129

headache e. vertigo, tinnitus and hearing loss


Q. 12. All are features of Meniere’s disease except
a. giddiness is provoked in certain head d. sense of pressure or fullness in the ear
positions
b. hearing loss that fluctuates e. sensorineural hearing loss
c. roaring tinnitus
Q. 13. Following sign may be positive in Meniere’s disease
a. Griesinger’s sign d. setting sun sign

b. Hennebert sign e. thumb sign


c. rising sun sign
Q. 14. The order of symptomatology in Lermoyez syndrome is
a. hearing loss preceding tinnitus and and vertigo
vertigo d. vertigo preceding hearing loss
b. hearing loss and tinnitus preceding and tinnitus
vertigo e. None of the above
c. tinnitus preceding hearing loss
Q. 15. All of the following conditions can cause fluctuating hearing loss except
a. chronic SOM d. otosclerosis

b. Cogan’s syndrome e. perilymph fistula

c. Meniere’s disease
Q. 16. Audiogram in early Meniere’s disease shows

a. a falling type of curve d. notch at 2 kHz in bone conduction

b. a flat type of curve e. notch at 4 kHz in air conduction

c. a rising type of curve


Q. 17. Treatment of acute phase of Meniere’s disease includes all except

a. betahistines d. thiazide diuretics

b. inhalation of carbogen e. vestibular sedatives


c. streptomycin/gentamicin therapy
Q. 18. Conservative treatment of chronic phase of Meniere’s disease includes all of the
following except
a. excessive use of water, tea and coffee d. vasodilators
b. streptomycin/gentamicin therapy e. vestibular sedatives
c. thiazide diuretics
Q. 19. A patient of Meniere’s disease has failed to respond to all medical treatment but
still retains a serviceable hearing. All of the following treatments can be considered
except
Part B
130 Ear

a. cervical sympathectomy d. portmann shunt

b. Fick and Codey stainless steel track e. vestibular nerve section

c. labyrinthectomy
Scenario A: A forty years old female presents with episodic vertigo for last 06 months.
Vertigo is intense associated with nausea and vomiting. Vertigo lasts for few hours and
patient complains of decreased hearing during the episode which improves in between.
Patient also complains of tinnitus and fullness of ears.
1. What is the most probable clinical diagnosis?
2. What is the differential diagnosis?
3. How will you prove your diagnosis?
4. How will you manage this patient?
B: A 35 years old patient is suffering from severe episodic vertigo, fluctuating sensorineural
hearing loss and tinnitus for last 06 months. Her otoscopic examination is unremarkable.
1. What is the most likely diagnosis?
2. How will you investigate this case?
3. Describe the treatment of this patient?
C: A 40 years old female presented with history of severe episodic vertigo, right sided
progressive sensorineural hearing loss and tinnitus for last five years. Frequency of episodes
is increasing with the passage of time.
1. What is the most probable clinical diagnosis?
2. What investigations will you order to confirm your diagnosis?
3. What is the treatment of this patient?
D: A 27 years old lady presented with complain of episodic vertigo, tinnitus and reduced
hearing for last 09 months. Her pure tone audiometry showed bilateral sensorineural hearing
loss while tympanogram was normal. Her ESR is 20 mm in first hour.
1. What is the most likely diagnosis?
2. How will you investigate this case?
3. What are different treatment options?
------------------------------- ANSWERS-----------------------------
------------------------- ------------------------
Q.1. d 2. c 3. d 4. c 5. a 6. c 7. a 8. b 9. d
10. d 11. e 12. a 13. b 14. b 15. d 16. c 17. c 18. a 19. c
Scenario A: 1. Meniere’s disease 2. Labyrinthitis, BPPV, vestibular schawanoma
3. Electrochochleography 4. Reassurance, cessation of smoking, low salt, vestibular sedative.
B: 1. Meniere’s disease 2. Electrocochleography 3. Reassurance, cessation of smoking, low
salt, vestibular sedative.
131
C.1. Meniere’s disease 2. Electrocochleography 3. Reassurance, cessation of smoking, low
salt, vestibular sedative.
D.1. Meniere’s disease 2. Electrocochleography 3. Reassurance, cessation of smoking, low
salt, vestibular sedative.
--------------------------------------------------------------------------------------------------------------------------
BENIGN PAROXYSMAL POSITIONAL VERTIGO (BPPV)
Synonym: Top shelf syndrome:

Definition: It is defined as sudden brief episodes of


vertigo associated with nausea and vomiting initiated by
head movements in certain critical positions.

Etiopathology:
It is the most common cause of peripheral vertigo.
Exact cause of BPPV is unknown. Head trauma or ear
surgery may predispose to BPPV. It may follow an attack
of vestibular neuronitis
It is thought to be due to otoconial debris (calcium Pathogenesis of BPPV
carbonate)
from degenerating utricle. The calcium carbonate crystals settle within the most dependent
part of the inner ear i.e., on the cupula of the posterior semicircular canal. This leads to over
excitation of posterior semicircular canal. Males and female are equally affected. It most
often occurs after 40 years.

Clinical features:
Brief episodic vertigo of sudden onset, initiated by head movement is diagnostic of BPPV.
The vertigo occurs especially when rising or rolling over in bed.
Vertigo may also be experienced when looking down or up suddenly. This is also known
as ‘top shelf syndrome’. Vertigo lasts for only seconds but the patient may remain unsteady
for several hours. There are no other aural symptoms. On examination of ear there is no
positive finding.
Dix Hallpike’s maneuver (also known as positional test) usually demonstrates vertigo
and nystagmus. The maneuver is performed as follows;
Patient is asked to sit on a couch. Clinician holds the head of the patient and then turns it
45 degrees to the right and then places the patient in a supine position so that his head hangs
30 degrees below the horizontal. Patient’s eyes are observed for nystygmus. Nystagmus is
present in case of right BPPV. The test is repeated with the head turned to the left and then
again in straight head hanging position. Nystagmus is present in case of left BPPV.
Investigations:
132
Diagnosis is made on clinical grounds. Certain investigations may be required for differential
diagnosis.
Treatment:
A. Avoidance of provoking position.
B. Epley’s maneuver is the best management of
benign paroxysmal positional vertigo of posterior
semicircular canal origin. It is otolithic repositioning
procedure. It involves moving the otoconial debris
back into the utricle from where it is thought to be
reabsorbed. Bone vibrator may be applied at mastoid
during Epley’s maneuver to dislodge the otoconia
from the posterior semicircular canal.
Epley’s maneuver is performed as follows:
1. The patient sits on the examination table, with eyes
open and head turned 45 degrees to the right.
2. The clinician supports the patient’s head as the
patient lies back quickly from a sitting to supine
position, ending with the head hanging 20 degrees off Epley’s maneuver
the end of the examination table.
3. The clinician turns patient’s head 90 degrees to the left side. The patient remains in this
position for 30 seconds.
4. The clinician turns the patient’s head an additional 90 degrees to the left while the patient
rotates his or her body 90 degrees in the same direction. The patient remains in this
position for 30 seconds.
5. The patient sits up on the left side of the examination table.
6. The procedure may be repeated on either side until the patient experiences relief of
symptoms.
A barbeque roll may be used to treat BPPV of lateral
semicircular canal origin.
In very resistant cases plugging of posterior semicircular canal or
gentamicin ablation may be required.
SELF ASSESSMENT
Barbeque roll
Q. 1. Benign paroxysmal positional vertigo is also known as
a. Alport syndrome d. Top shelf syndrome

b. Pendred syndrome e. Usher syndrome

c. Potter syndrome
133

Q. 2. Crista ampullaris is situated in

a. cochlea d. utricle

b. semicircular canal e. vestibule


c. saccule
Q. 3. Benign paroxysmal positional vertigo is due to
a. autoimmune disorder d. transient ischemia of vestibular gan-

glia
b. bacterial infection of vestibular nerve

c. impingement of free otoconia on the e. viral infection of vestibular nerve


cupula of posterior semicircular canal
Q. 4. Otoliths are crystals of
a. calcium bicarbonate d. calcium phosphate

b. calcium carbonate e. calcium sulphate

c. calcium oxalate
Q. 5. Normally otoconia are present in

a. ampulae of semicircular canal d. Reissner’s membrane

b. cristae of semicircular canal e. tectorial membrane

c. maculae of saccule and utricle


Q. 6. Vertigo in BPPV lasts for

a. a few seconds d. a few weeks

b. a few hours e. a few months

c. a few days
Q. 7. In benign paroxysmal positional vertigo
a. cochlear function is abnormal lar canal
b. giddiness is induced when head is d. there is spasm of internal auditory
turned into a critical position artery
c. there is erosion of lateral semicircu- e. there is third window into labyrinth
Q. 8. Vertigo in BPPV is associated with
a. earache d. tinnitus
b. ear discharge e. none of the above
c. hearing loss
Q. 9. All are true about benign paroxysmal positional vertigo except
a. follows an attack of vestibular neuro-nitis c. males are affected more than females
b. head trauma or ear surgery predis- d. more often occurs after 40 years
poses to BPPV e. vertigo lasts for only seconds
Q. 10. Investigation of choice in BPPV is
134 Part B Ear

a. Dix Halpike maneuver d. all of the above

b. Epley’s maneuver e. none of the above


c. Heimlich maneuver
Q. 11. In benign paroxysmal positional vertigo which of the following test is used for
diagnosis?
a. Caloric test d. Rotation test

b. Dix Hallpike maneuver e. All of the above

c. Electronystagmography
Q. 12. Treatment of BPPV is

a. Dix Halpike maneuver d. all of the above

b. Epley’s maneuver e. none of the above


c. Heimlich maneuver
Q. 13. To treat lateral semicircular canal BPPV following maneuver is used
a. Barbeque roll d. Heimlich maneuver

b. Dix Halpike maneuver e. all of the above

c. Epley’s maneuver
Scenario: A 49 years old lady presents with 06 weeks history of multiple episodes of objective
rotatory vertigo with nausea on head movements or rolling over in bed which are of brief
duration, (few seconds). There is no history of tinnitus, hearing loss, otalgia and ear
discharge.
1. What is the most likely diagnosis?
2. How will you investigate this case?
3. What is the treatment option?
----------------------------------------- ANSWERS--------------------------------
--------------- ---------------------
Q.1. d 2. b 3. c 4. b 5. c 6. a 7. b
8. e 9. c 10. a 11. b 12. b 13. a
Scenario: 1. BPPV 2. Clinical diagnosis 3. Epley’s maneuver
--------------------------------------------------------------------------------------------------------------------------
TINNITUS
It is defined as a sensation of unorganized sounds e.g., whistling, hissing, ringing of bells,
buzzing of insects etc. in the ear or head, whereas sensation of organized sounds (e.g., songs,
music, sermon or verses) is known as hallucination. Tinnitus is a symptom. Always look for
the cause of the symptom. Usually it is subjective (perceived by the patient only) but can be
objective (perceived by the clinician as well).
A. Causes of subjective tinnitus:
1. Otitis media with effusion
2. Acute and chronic otitis media
3. Patulous eustachian tube
4. Meniere’s disease
5. Otosclerosis
6. Senile cochleopathy
7. Noise trauma
8. Ototoxic drugs
9. Vestibular schwannoma
B. Causes of objective tinnitus:
1. Vascular tumors of middle ear such as glomus tumor
2. Aneurysm of carotid artery
3. Palatal myoclonus
C. Causes of nonotologic tinnitus:
1. Anemia
2. Hypertension
3. Arteriosclerosis
4. Hypoglycemia
Investigations:
A detailed history and thorough clinical examination are essential for diagnosis.
Pure tone audiometry will confirm the presence and type of hearing loss.
Anemia, diabetes and hypo/hyperthyroidism must be ruled out. FTA and TPI test to rule out
syphilis.
Tympanometry, speech audiometry, CT, MRI and angiogram may be required for the purpose
of diagnosis.
Treatment:
Tinnitus is a symptom. Its cause should be identified.
Treatment includes removal of the underlying cause. When no cause is detected or if
tinnitus persists even after removal of the underlying cause, the treatment is as below;
1. Reassurance: Patient is informed that this is not a dangerous condition.
2. Sedatives and anxiolytics to allay patient’s anxiety are given for short time.
3. Masking of tinnitus: Tinnitus may be masked with sound of a fan or clock, verses or
music. It may also be masked with a tinnitus masker which is a small electronic device
136
that generates noise. Tinnitus maskers are used when tinnitus is present in sleep hours.
The device may mask the patient’s tinnitus and help the patient to go to sleep. Intensity of
tinnitus masker is kept within 20 dB of hearing threshold.

SELF ASSESSMENT
Q. 1. Tinnitus is a sensation of
a. music in ear d. unorganized sounds in ear

b. sermon in ear e. verses in ear

c. songs in ear
Q. 2. All are non otological causes of tinnitus except

a. anemia d. hypoglycemia

b. arteriosclerosis e. patulous eustachian tube


c. hypertension
Q. 3. All are causes of objective tinnitus except
a. aneurysm of carotid artery d. palatal myoclonus

b. glomus tumor e. vascular tumors of middle ear

c. Meniere’s disease
Q. 4. Patient of tinnitus may be benefited by
a. reassurance

b. removal of underlying cause d. tinnitus maskers

c. sedatives and anxiolytics e. All of the above


Q. 5. Intensity of tinnitus masker is kept within
a. 5 dB of hearing threshold d. 20 dB of hearing threshold

b. 10 dB of hearing threshold e. 30 dB of hearing threshold

c. 15 dB of hearing threshold
-------------------------------------------------------- ANSWERS-----------------------------------------------------
Q.1. d 2. e 3. c 4. e 5. d
--------------------------------------------------------------------------------------------------------------------------
OTOTOXICITY
Definition:
Damage to the cochlea and/or vestibular part of the inner ear by different drugs and chemicals
is known as ototoxicity.

Etiopathology:
Common ototoxic drugs are;
137
1. Aminoglycosides e.g., neomycin and tobramycin are mainly cochleotoxic, whereas strep-
tomycin and gentamycin are mainly vestibulotoxic.
2. Diuretics e.g., furosemide, ethacrynic acid.
3. Anti protozoal agents e.g., quinine, chloroquine.
4. Cytotoxic agents e.g., nitrogen mustard, cisplatin and salicylates.
Ototoxic effects are produced by;
1. Parenteral administration usually
2. Oral administration less commonly
3. Topical administration to ear, joint cavities, wounds and burns rarely.

When administered systemically, these reach the labyrinthine fluids via the blood stream.
Topical antibiotic ototoxic drops probably reach the inner ear by permeating the round
window membrane into perilymph and hence, through the regional membrane to the
endolymph.

Clinical features:
Tinnitus is usually the first symptom. It is a warning of possible hearing loss in any patient.
Hearing loss is usually sensorineural in type and affects higher frequencies more than
the lower frequencies. Symptoms are more severe in elderly patients with renal or hepatic
failure.
Vertigo may occur in patients receiving vestibulotoxic drugs. Ototoxic drugs may cross
the placental barrier and affect the fetus in pregnant women.
Treatment:
Treatment in ototoxicity is only preventive.
1. Avoid or discontinue ototoxic drugs whenever indicated.
2. Monitor treatment with regular estimation of serum levels of drugs and/or creatinine.
Monitor hearing regularly with different types of audiometry in patients receiving
ototoxic drugs.
3. Hearing aid may be helpful when there is hearing loss.
4. Tinnitus masker is used when there is disturbing tinnitus.
5. Disequilibrium is treated with reassurance and physiotherapy.
SELF ASSESSMENT
Q. 1. The following is well known ototoxic drug

a. aminoglycosides c. cytotoxic drugs

b. antiprotozoal d. diuretics
138
e. all of the above
Q. 2. All of the following are well known ototoxic drugs except
a. aspirin d. penicillin

b. erythromycin e. streptomycin
c. gentamicin
Q. 3. Ototoxic drug where sensorineural hearing loss can be reversed by stopping the
drug is
a. amikacin d. neomycin

b. furosemide e. all of the above


c. gentamicin
Q. 4. Treatment of ototoxicity includes all except
a. to avoid ototoxic drugs d. monitor treatment with estimation of
b. discontinue ototoxic drugs serum levels of drugs
b.
c. monitor hearing regularly with audi- e. use antidote with ototoxic drugs
ometry
Scenario: A 25 years old girl taking antituberculous therapy and streptomycin injection for
MDR-tuberculosis. After one month patient presents with complaint of hearing loss, tinnitus
and mild vertigo.
1. What is the most probable clinical diagnosis?
2. How will you investigate this patient?
3. How will you manage this case?
---------------------------------------------------
----- ANSWERS-----------------------------------------------------
Q.1. e 2. d 3. b 4. e
Scenario: 1. Ototoxicity due to streptomycin 2. P.T.A, assessment of renal and liver functions
and serum drug levels. 3. Avoid or discontinue drug, monitor treatment, hearing aid, tinnitus
masker.
--------------------------------------------------------------------------------------------------------------------------
-
139

Chapter 7
MISCELLANEOUS DISEASES OF EAR
HEARING LOSS
Definition:
Any decrease in hearing capability below normal is known as hearing loss. Hearing loss is
wrongly called deafness. The term deafness is used when there is no useful hearing.
Degrees of hearing loss:
Degree of hearing loss is categorized on the basis of pure tone audiogram at speech
frequencies i.e., 500, 1000 and 2000 Hz.
According to WHO hearing loss is categorized as follows;
1. Mild hearing loss 26 - 40 dB
2. Moderate hearing loss 41 - 55 dB
3. Moderately severe hearing loss 56 - 70 dB
4. Severe hearing loss 71 - 90 dB
5. Profound hearing loss More than 90 dB
Classification:
Hearing loss is classified in different ways. Mostly it is classified as follows;
A. Conductive hearing loss
B. Sensorineural hearing loss
A. Conductive hearing loss:
It is that type of hearing loss in which there is some obstruction to the conduction of
sound from the atmosphere to the stapes footplate. So, the causes may lie either in the
external or middle ear.
Conditions of external ear which may cause conductive hearing loss are;
1. Impacted wax
2. Fungus
3. Foreign body
4. Otitis externa
5. Neoplasms

140
Conditions of middle ear which may cause conductive hearing loss are;
1. Tympanic membrane perforation
2. Acute otitis media
3. Chronic otitis media
4. Serous otitis media
5. Eustachian tube dysfunction
6. Otosclerosis
7. Tympanosclerosis
8. Neoplasm
B. Sensorineural hearing loss:
It is that type of hearing loss in which there is problem in the reception of sound.
So, the causes of sensorineural hearing loss may lie anywhere between the membranous
cochlea to the hearing center in the superior temporal gyrus.
The conditions which may cause sensorineural hearing loss are;
1. Presbycusis
2. Meniere’s disease
3. Ototoxic drugs
4. Noise trauma
5. Labyrinthitis
6. Vestibular schwannoma
7. Congenital
Presbycusis manifests usually at 60 years of age. It causes high frequency hearing loss and
tinnitus. There is difficulty to hear in noisy surroundings.
Noise induced hearing loss depends on frequency and intensity of noise. Maximum safe
limit of industrial noise for 08 hours a day for 05 days in a week is 90 dB. It manifests with
tinnitus. Hearing loss usually starts at 4000 Hz.
Sudden sensorineural hearing loss is defined as sensorineural hearing loss of 30 dB or more
over at least 03 consecutive frequencies within a period of 03 days.

Management of hearing loss:


A. Conductive hearing loss:
Most cases of conductive hearing loss are managed by removal of the underlying cause by
medical or surgical means. For example:
1. Removal of wax, keratosis obturans, osteoma, exostosis.
141
2. Removal of fluid from the middle ear by myringotomy with or without grommet
insertion.
3. Removal of cholesteatoma.
4. Repair/reconstruction of tympanic membrane defect and ossicular chain.
5. In stapes footplate fixation, hearing aid may be applied or stapedectomy may be
performed.
Hearing aid is a last resort where medical and surgical treatment has failed or patient has
refused other treatment options.
B. Sensorineural hearing loss:
In cases of sensorineural hearing loss, early diagnosis is very important as measures can
usually be taken to reverse the condition, to stop its progress and to start an early
rehabilitation. In cases of syphilis, high doses of penicillin and steroids are given.
In serous labyrinthitis, attention is paid to treat middle ear infection to reverse the hearing
loss.
Sensorineural hearing loss due to ototoxic drugs must be managed by stopping further
intake of drugs. Hearing loss due to hypothyroidism may be reversed by replacement
treatment.
Noise induced hearing loss may be prevented or reversed by removing the person from
noisy surroundings. Rehabilitation of hearing loss is carried out with hearing aid and other
devices.
SELF ASSESSMENT
Q. 1. A person feels socially handicapped when his hearing loss reaches
a. 10 dB d. 40 dB

b. 20 db e. 50 dB

c. 30 dB
Q. 2. Which of the following is speech
frequency?

a. 500 Hz d. all of the above

b. 1000 Hz e. none of the above


c. 2000 Hz
Q. 3. A hearing loss is called profound when average hearing loss of three speech fre-
quencies exceeds
a. 30 dB d. 90 dB

b. 50 dB e. 110 dB
c. 70 dB
Q. 4. All are causes of conductive hearing loss except
142

a. fungus ear d. presbycusis

b. otitis media with effusion e. wax

c. otosclerosis
Q. 5. Conductive type of hearing loss is seen in all except

a. Meniere’s disease d. suppurative otitis media

b. otosclerosis e. tympanosclerosis
c. secretory otitis media

Q. 6. All are causes of sensorineural hearing loss except


a. labyrinthitis d. tympanosclerosis
b. Meniere’s disease e. vestibular schwannoma
c. ototoxic drugs
Q. 7. All are true about presbycusis except
a. causes high frequency hearing loss c. causes tinnitus
b. causes difficulty to hear in noisy sur- d. manifests usually at 40 years
roundings e. none of the above
Q. 8. What is the maximum safe limit of industrial noise for 8 hours a day for 5 days in a
week?
a. 70 dB d. 100 dB

b. 80 dB e. 110 dB
c. 90 dB
Q. 9. Noise induced hearing loss begins at which of the following frequencies?
a. 1000 Hz d. 6000 Hz

b. 2000 Hz e. 8000 Hz

c. 4000 Hz
Q. 10. Which of the following is not true about noise induced hearing loss?

d. Results from damage to central auditory


a. Depends on frequency of noise pathways

e. Starts at 4000 Hz.


b. Depends on intensity of noise
c. Manifest with tinnitus
Q. 11. Acoustic trauma causes hearing loss due to damage to
a. outer hair cells d. all of the above

b. organ of corti e. none of the above

c. Reissner’s membrane
Scenario: A 65 years old male presented to ENT OPD with impaired hearing for last 01 year.
There is no abnormality in EAC and middle ear. On pure tone audiometry there is
sensorineural hearing loss more for higher frequencies.
1. What is the most probable clinical diagnosis?
143
2. How will you manage this patient?

ANSWERS-----------
---------------------------- ------------------------------
----------------- --------------------
Q.1. c 2. d 3. d 4. d 5. a 6. d 7. d 8. c 9. c 10. d 11. d
Scenario: 1. Presbycusis 2. Rehabilitation with hearing aid.
--------------------------------------------------------------------------------------------------------------------------
-
OTALGIA
Conditions of outer, middle and inner ear may be associated with pain e.g.,
A. Pinna:
1. Trauma (tear, laceration)
2. Hematoma auris
3. Frost bite
4. Sun burn
5. Chondrodermatitis nodularis chronica helices
6. Infected basal or squamous cell carcinoma
B. External meatus:
1. Impacted wax
2. Otitis externa
3. Boil
4. Herpes zoster oticus
5. Keratosis obturans
6. Malignant otitis externa
C. Middle ear:
1. Acute otitis media
2. Otitis media with effusion
3. Otitic barotrauma
4. Traumatic perforation
D. Mastoid:
1. Acute mastoiditis
2. Complications of cholesteatoma
3. Zygomatic mastoiditis
4. Wegener’s granuloma
144
E. Inner ear:
1. Noise (in noise-sensitive people, noise may be perceived as pain).
2. Tinnitus (it may be described as throbbing pain in some patients).
F. Referred pain:
Ear is supplied by 5th, 7th, 9th and 10th cranial nerves. It is also supplied by second and
third branches of the cervical plexus. Earache may develop due to any painful condition
in the region supplied by the above-mentioned nerves. e.g.,
1. Throat (tonsillitis, peritonsillar abscess, parapharyngeal abscess or retropharyngeal
abscess)
2. Parotid (parotitis, sialectasis, mumps)
3. Thyroid (sub-acute thyroiditis, Hashimoto’s thyroiditis)
4. Larynx (granulomatous lesions of larynx e.g., tuberculosis)
5. Teeth (carious teeth, eruption of teeth, periodontal and dental abscess especially of
lower molars)
G. Functional (psychogenic):
It is a very common symptom in children who tend to avoid school. A history of
recurrent episode of earache severe enough to cause several days of absence from school
is usually obtained. Tympanic membrane and hearing are usually normal. Obviously, all
other causes of earache must be excluded before making a diagnosis of psychogenic pain.
SELF ASSESSMENT
Q. 1. Unilateral pain in the ear in the absence of ear disease can be due to all except
a. allergic rhinitis d. temporomandibular joint dysfunction

b. cancer of the pyriform fossa e. ulcer of tongue

c. peritonsillar abscess
-------------------------------------------------------- ANSWERS-----------------------------------------------------
Q.1. a
--------------------------------------------------------------------------------------------------------------------------
VESTIBULAR SCHWANNOMA
Synonym: Acoustic neuroma/Neurinoma/Neurilemmoma
Etiopathology:
The term acoustic neuroma is basically a misnomer.
The tumor most commonly arises from schwan cells of
superior vestibular nerve in the internal auditory
meatus and extends towards cerebellopontine angle.

Diagram of vestibular schwannoma


145
It accounts for 80% of all cerebellopontine angle tumors. It causes progressive but slow
destruction of vestibular, cochlear and facial nerve.
Histologically there are two characteristic patterns;
1. Antoni A or fascicular type
2. Antoni B or reticular type
Clinical features:
In sporadic type patient presents in 5th and 6th decade. Whereas in familial type with
neurofibromatosis type 2 (NF2), patient presents with bilateral vestibular schwannoma by the
age of 20. Its clinicopathological picture may be considered in five stages;
1. Otological stage: (when tumor is less than 2 cm in size). Unilateral tinnitus and
sensorineural hearing loss are the most common presentations. There may be facial
nerve paralysis.
2. Trigeminal stage: (when the tumor is more than 2 cm in size). There is loss of facial
sensations due to involvement of trigeminal nerve. In addition, there are features of
otological stage.
3. Stage of brainstem and cerebellar compression:
There is asthenia, ataxia and involvement of lower cranial nerves.
4. Stage of rising intracranial pressure:
There is headache, vomiting, diplopia, papilledema and decrease in vision.
5. Terminal stage:
There is disturbance of vital brain stem centers and tonsillar herniation.
On examination there may be cutaneous lesions suggestive of von Recklinghausen’s disease/
multiple neurofibromatosis. Tuning fork tests show ipsilateral sensorineural hearing loss.
There may be loss of sensation on the posterosuperior
part of deeper external meatus. This part is supplied by the
sensory component of facial nerve. This loss of sensation is
known as Hitselberger sign. Even when the tumor is
intracanalicular, Hitselberger sign may be positive. Cranial
nerves examination may show paresis/paralysis of the 5th,
6th, 7th, 8th, 9th, 10th, 11th and 12th cranial nerves
depending on the size of the tumor.
Corneal reflex is lost when there is involvement of
either facial nerve (due to motor component) or trigeminal
nerve (due to sensory component). Examination of eyes may
show nystagmus, papilledema, venous congestion and loss of
venous pulsations depending on the size of the tumor.
Investigations:
CT showing vestibular schwannoma
146
A. Radiological
MRI with contrast of internal auditory meatus is used to find out exact site and size of
the tumor and its relation to surrounding structures. It is the most important investigation
in vestibular schwannoma.
B. Audiological
1. Pure tone audiometry:
There is sensorineural hearing loss usually in higher frequencies.
2. Speech audiometry:
May show discrimination score worse than one would expect from consideration of
pure tone threshold.
3. Loudness recruitment:
There is no recruitment phenomenon as the lesion is retrocochlear.
4. Tone decay:
There is rapid tone decay.
5. Stapedial reflex:
Stapedial reflex threshold is elevated and there is increase in stapedial reflex decay.
6. Brainstem electric response audiometry:
This is the most reliable audiological screening test for vestibular schwannoma.
There is absolute latency of wave V. There is interaural latency difference of wave V
by more than 0.2 ms.
7. Caloric test:
May show significant canal paresis in more than 90% of patients.
Treatment:
It depends on;
1. Level of hearing in the affected ear
2. Vestibular function in the affected ear
3. Level of hearing in non-affected ear
4. Vestibular function in the non-affected ear
Surgery is the treatment of choice. Exact surgical approach depends on;
1. Whether the disease is unilateral or bilateral
2. Tumor size
3. Hearing level in both ears
147
4. Age of the patient
Surgery may be carried out through translabyrinthine approach, posterior cranial fossa
approach or middle cranial fossa approach.
Radiotherapy is done to arrest the growth of tumor. The most common radiotherapy for
vestibular schwannoma is stereotactic radiotherapy with gamma ‘knife’ or cyber ‘knife’.
For a patient with bilateral profound sensorineural hearing loss following surgery of
bilateral vestibular schwannoma, the most appropriate treatment is brainstem implant.
Differential diagnosis:
1. Meningioma
2. Congenital cholesteatoma
3. Arachnoid cyst
4. Meniere’s disease
5. Glioma
6. Lipoma
SELF ASSESSMENT
Q. 1. Vestibular schwannoma is also called

a. acoustic neuroma d. all of the above

b. neurinoma e. none of the above


c. neurilemmoma
Q. 2. The most common site of origin of vestibular schwannoma is
a. cochlear nerve d. singular nerve

b. facial nerve e. superior vestibular nerve

c. inferior vestibular nerve


Q. 3. Most frequent tumor of cerebellopontine angle is

a. arachnoid cyst d. neurofibroma

b. meningioma e. vestibular schwannoma

c. metastasis
Q. 4. Vestibular schwannoma arises from

a. Hurthle cells d. parafollicular cells

b. Langhans cells e. schwann cells


c. Mikulicz cells
Q. 5 The most common presentation of vestibular schwannoma is
a. unilateral earache c. unilateral ear discharge

b. unilateral ear bleeding d. unilateral tinnitus


148
e. unsteadiness
Q. 6. All are common presentations of acoustic neuroma except
a. facial paralysis d. unilateral tinnitus

b. unilateral progressive hearing loss e. vertigo

c. unilateral sensorineural hearing loss


Q. 7. Hypoesthesia of posterior meatal wall is seen in

a. carcinoma of the middle ear d. vestibular neuronitis

b. glomus tympanicum e. vestibular schwannoma

c. lateral sinus thrombosis


Q. 8. The following sign is positive in vestibular schwannoma

a. battle sign d. Kernig sign

b. Gresinger sign e. tripod sign

c. Hitselberger sign
Q. 9. Afferent pathway involved in corneal reflex is

a. cranial nerve Vth d. cranial nerve Xth

b. cranial nerve VIIIth e. cranial nerve XIth

c. cranial nerve IXth


Q. 10. Efferent pathway involved in corneal reflex is

a. cranial nerve Vth d. cranial nerve Xth

b. cranial nerve VIIth e. cranial nerve XIth


c. cranial nerve IXth
Q. 11. In vestibular schwannoma corneal reflex is lost, when there is involvement of
a. cochlear nerve d. superior vestibular nerve

b. facial nerve e. vestibulocochlear nerve


c. inferior vestibular nerve
Q. 12. In vestibular schwannoma corneal reflex is lost, when there is involvement of
a. cochlear nerve d. vestibulocochlear nerve

b. trigeminal nerve e. vestibular nerve


c. vagus nerve
Q. 13. Acoustic neuroma initially presents with
a. flat sensorineural hearing loss loss
b. high frequency sensorineural hearing d. sudden sensorineural hearing loss
loss e. all of the above
c. low frequency sensorineural hearing
Q. 14. Hearing loss of acoustic neuroma
is:
149 Part B Ear

a. cochlear d. vestibular

b. conductive e. vestibulocochlear
c. retrocochlear
Q. 15. The most important audiological investigation in vestibular schwannoma is
a. BERA d. speech audiometry

b. PTA e. tympanometry
c. SISI
Q. 16. For diagnosis of vestibular schwannoma, the most important investigation is
a. caloric test d. recruitment test

b. MRI e. speech audiometry


c. PTA
Q. 17. The mainstay of treatment in vestibular schwannoma is
a. antibiotics d. electrocoagulation

b. chemotherapy e. surgery
c. cryotherapy
Q. 18. Which of the following would be the most appropriate treatment for rehabilita-
tion of a patient with bilateral profound sensorineural hearing loss following surgery for
bilateral acoustic schwannoma?
a. BAHA c. Bilateral cochlear implants

b. Bilateral high-powered digital hearing d. Brainstem implant

Aid
e. Unilateral cochlear implant
Scenario: A 40 years old male presented to ENT department with right sided tinnitus and
sensorineural hearing loss. On examination corneal reflex is absent and Hitselberger’s sign
is positive.
1. What is your clinical diagnosis?
2. How will you investigate?
3. How will you treat?
--------------------------------- ANSWERS-----------------------------
----------------------- ------------------------
Q.1. d 2. e 3. e 4. e 5. d 6. e 7. e 8. c 9. a
10. b 11. b 12. b 13. b 14. c 15. a 16. b 17. e 18. d
Scenario: 1. Vestibular schwannoma 2. MRI with contrast, PTA 3. Surgery, radiotherapy
--------------------------------------------------------------------------------------------------------------------------
150

Chapter 8
AUDIOLOGY
AUDIOMETRIC TESTS
A. Pure tone audiometry:
This is a subjective test. This is the most commonly used
method of measuring hearing acuity. A pure tone
audiometer is an instrument which delivers tones of
variable frequency and intensity to the ear by earphones.
The frequency usually tested are at octave steps, i.e.,125,
250, 500, 1000, 2000, 4000 and 8000 Hz. Occasionally
half-octave steps e.g., 1500, 3000, 6000 Hz are used. The
intensity can be decreased or increased for each frequency
and can vary from 10 dB to 120 dB.
Most audiometers used today are calibrated to the
international (ISO) standard level. Both the air conduction Pure tone audiometry in progress
and bone conduction can be measured and are best done
in the same manner. The best frequency to start with is 1000 Hz. A series of short signals or
tone pips are put in at an intensity above the patient’s suspected threshold, and the patient is
instructed to signal every time he hears a sound. The intensity is reduced in 10 dB steps until
no sound is heard. The signal is then increased in 5 dB steps until half of the tone of the pips
are consistently heard. This is the patient’s threshold for that frequency. The threshold for the
remaining of frequency is then measured.
The bone conduction is measured in similar fashion by putting a receiver onto mastoid
bone. The sound emitted by this is transmitted by the bones of the skull to the cochlea, thus,
by passing the external and middle ears and giving a measure of inner ear function. The
results are charted as audiogram. In audiometry it is important to eliminate the possibility that
the test sound is being heard in the opposite ear. The audiometer provides a masking noise
which may be played into the opposite ear. Masking must be applied to the better ear when
testing the deafer ear if the difference in threshold is found to be 40 dB or more. When testing
the bone conduction threshold, the other ear should always be masked because of the ease
which bone conduction sound is transmitted through the bones of skull.

Uses of pure tone audiogram:


1. It is a measure of threshold for hearing by air and bone conduction and thus, the degree
and type of hearing loss.
2. Audiogram is absolutely essential for prescription of hearing aid.
151
3. It helps to find degree of handicap for medicolegal purposes.
4. It helps to predict speech reception threshold
5. A record can be kept for comparison.
B. Speech audiometry:
It is a subjective test. In this test, the patient’s ability to hear and understand speech is
measured. Following two parameters are studied in speech audiometry;
1. Speech reception threshold
2. Speech discrimination score
1. Speech reception threshold (SRT):
It is the minimum intensity at which 50% of the words are repeated correctly by the
patient. A set of spondee words (two syllable words with equal stress on each syllable, e.g.,
baseball, sunlight, daydream, etc.) is delivered to each ear through the head phone of an
audiometer. The word lists are delivered in the form of recorded tapes or monitored voice.
Their intensity varied in 5 dB steps till half of them are correctly heard. Normally, SRT is
within 10 dB of the average of pure tone threshold of three speech frequencies (500, 1000 and
2000 Hz). An SRT better than pure tone average by more than 10 dB suggests a functional
hearing loss.
2. Speech discrimination score:
Also called speech recognition or word recognition score.
It is a measure of patient’s ability to understand speech. Here, a list of phonetically
balanced (PB) words (single syllable words, e.g., pin, sin, day, bus, etc.) is delivered to the
patient’s each ear separately at 30-40 dB above his SRT. The percentage of words correctly
heard by the patient is recorded. In normal persons and those with conductive hearing loss a
high score of 90-100% can be obtained.
C. Bekesy audiometry:
It is a self-recording audiometry where various pure tone frequencies automatically move
from low to high while the patient controls the intensity through a button. Two tracings, one
with continuous and the other with pulsed tone are obtained. The tracings help to differentiate
a cochlear from a retrocochlear and an organic from a functional hearing loss.
D. Tympanometry:
It is also known as impedance/admittance audiometry.
This is an objective test. The equipment consists of a probe which
snugly fits into the external meatus and has three channels;
1. To deliver a tone of 220 Hz.
2. To pick up the reflected sound through a microphone.
Tympanometer
152
3. To bring about changes in air pressure in the ear canal from positive to normal and
then negative.
It is based on a simple principle i.e., when a sound strikes tympanic
membrane, some of the sound energy is absorbed while the rest is
reflected back. A stiffer tympanic membrane would reflect more of
sound energy than a compliant one. By changing the pressures in a
sealed external meatus and then measuring the reflected sound
energy, it is possible to find the compliance or stiffness of the
tympano-ossicular system and thus, find the healthy or diseased
status of the middle ear.
Because the passage of sound can be said to be impeded by
the drum and ossicles, the technique is also known as impedance Tympanometry
audiometry. Normal values of compliance range between 0.25 to
1.6 ml.
Characteristics patterns are produced by fluid in the ear, by diminished middle ear
pressure due to eustachian tube obstruction and by ossicular discontinuity.
Types of tympanograms:
Type A: Normal tympanogram
Type AS: Compliance is low at or near ambient air
pressure. It is seen in fixation of ossicles,
e.g., otosclerosis or malleus fixation.
Type AD: High compliance at or near ambient
pressure. It is seen in ossicular
discontinuity or thin and lax tympanic
membrane.
Tympanogram showing
Type B: A flat or dome shaped graph. No change in
different types of curves
compliance with pressure changes. It is seen
in middle ear fluid or thick tympanic membrane.
Type C: Maximum compliance occur with negative pressure in excess of 100 mm of water.
It is seen in retracted tympanic membrane and may show some fluid in middle ear.
Uses of tympanometry:
Tympanometer is used to;
1. Measure compliance of tympanic membrane and ossicular chain.
2. Test function of eustachian tube.
A negative or a positive pressure (-200 or +200 mm of water) is created in the middle ear and
the person is asked to swallow 5 times in 20 seconds. The ability to equilibrate the pressure
indicates normal tubal function. The test can also be used to find the patency of the grommet
placed in tympanic membrane in cases of serous otitis media.
153
3. Acoustic reflex:
It is based on the fact that a loud sound, 70-100 dB above the threshold of hearing of a
particular ear, causes bilateral contraction of the stapedial muscle which can be detected by
tympanometry. Tone can be delivered to one ear and the reflex picked from the same or the
contralateral ear. The reflex arc involved is:
a. Ipsilateral acoustic reflex: Eighth cranial nerve→ ventral cochlear nucleus→
seventh cranial nerve nucleus→ ipsilateral stapedius muscle.
b. Contralateral acoustic reflex: Eighth cranial nerve→ventral cochlear nucleus→
contralateral superior olivary nucleus→ contralateral seventh cranial nerve nucleus→
contralateral stapedius muscle.
The test is useful in several ways;
I. To test the hearing in infants and young children.
II. To find the malingerers: A person who feigns total hearing loss and does not give
any response on pure tone audiometry but shows a positive stapedial reflex in a
malingerer.
III. To detect cochlear pathology: Presence of stapedial reflex at lower intensities, e.g.,
40-60 dB than the usual 70 dB indicates recruitment and thus, a cochlear type of
hearing loss.
IV. To detect lesion of 8th nerve: If a sustained tone of 500 or 1000 Hz, delivered l0 dB
above acoustic reflex threshold, for a period of 10 seconds, brings the reflex
amplitude to 50%, it shows abnormal adaptation and is indicative of eighth nerve
lesion (stapedial reflex decay).
V. To detect lesion of 7th nerve: Absence of stapedial reflex when hearing is normal,
indicates lesion of the facial nerve, proximal to the nerve to stapedius. The reflex can
also be used to find prognosis of facial paralysis as the appearance of reflex, after it
was absent, indicates return of function and a favorable prognosis.
VI. To detect lesion of brainstem: If ipsilateral reflex is present but the contralateral
reflex is absent, lesion is in the area of crossed pathways in the brainstem.
4. Physical volume of ear canal:
Tympanometry can also measure the physical volume of air between the probe tip and
tympanic membrane. Normally it is up to 1.0 ml in children and 2 ml in adults. Any increase
in volume by > 2 ml in children and > 2.5 ml in adults, indicates perforation of the tympanic
membrane (because middle ear volume is added up to the volume of external ear canal). This
has also been used to find patency of the ventilation tube.
E. Special tests of hearing:
1. Recruitment:
It is a phenomenon of abnormal growth of loudness. The ear which does not hear low
intensity
154
sound begins to hear greater intensity sounds as loud or even louder than normal hearing ear.
Thus, a loud sound which is tolerable in normal ear may grow to abnormal levels of loudness
in the recruiting ear and thus, becomes intolerable. The patients with recruitment are poor
candidates for hearing aids. Recruitment is typically seen in lesions of the cochlea (e.g.,
Meniere’s disease, presbycusis) and thus, helps to differentiate a cochlear from a
retrocochlear sensorineural hearing loss.
Alternate binaural loudness balance test is used to detect recruitment in unilateral
cases. A tone, say of 1000 Hz is played alternately to the normal and the affected ear and the
intensity in the affected ear is adjusted to match the loudness in normal ear. The test is started
at 20 dB above the threshold of deaf ear and then repeated at every 20 dB rise until the
loudness is matched or the limits of audiometer reached. In conductive and neural hearing
loss, the initial difference is maintained throughout while in cochlear lesions, partial,
complete or over-recruitment may be seen.
2. Short Increment Sensitivity Index (SISI test):
Patients with cochlear lesions distinguish smaller changes in intensity of pure tone better than
normal persons and those with conductive or retrocochlear pathology. SISI test is thus, used
to differentiate a cochlear from a retrocochlear lesion.
In this test, a continuous tone is presented 20 dB above the threshold and sustained for
about 02 minutes. Every 05 seconds, the tone is increased by 01 dB and 20 such blips are
presented. Patient indicates the blips heard. In conductive hearing loss, SISI score is seldom
more than 15%; it is 70-100% in cochlear hearing loss; and 0-20% in nerve hearing loss.

3. Threshold tone decay test:


It is a measure of nerve fatigue and is used to detect retrocochlear lesions. Normally, a person
can hear a tone continuously for 60 seconds. In nerve fatigue, he stops hearing earlier. The
threshold tone decay test is simple and is performed in the following manner:
A tone of 4000 Hz is presented at 05 dB above the patient’s threshold of hearing,
continuously for a period of 60 seconds. If patient stops hearing earlier, intensity is increased
by another 05 dB. The procedure is continued till patient can hear the tone continuously for
60 seconds, or no level exists above the threshold where tone is audible for full 60 seconds.
The result is expressed as number of dB of decay. A decay of more than 25 dB is diagnostic
of a retrocochlear lesion.
4. Evoked response audiometry:
It is an objective test which measures electrical activity in the auditory pathways in response
to auditory stimuli. It requires special equipment with an averaging computer. There are
several components of evoked electric response but only two have gained clinical acceptance.
These are;
A. Electrocochleography (ECoG):
It measures electrical potential arising in the cochlea and cochlear nerve in response to
155
auditory stimuli within first 5 milliseconds. The response is in the form of three phenomena:
a. Cochlear microphonics
b. Summating potentials
c. Action potential of 8th nerve.
The recording electrode is usually a thin needle passed through the tympanic membrane onto
the promontory. In adults, it can be done under local anesthesia but in children and anxious
persons, sedation or general anesthesia is required. Sedation does not interfere in these
responses. ECoG is useful to;
1. Find threshold of hearing in young infants and children to within 5-10 dB.
2. Differentiate lesions of cochlea from those of cochlear nerve. Normally the ratio
between the summating potential to the action potential is less than 30%. An increase
in this ratio is indicative of Meniere’s disease.
B. Auditory Brainstem Response (ABR):
Also called BERA (Brainstem Evoked Response Audiometry),
BAER or BAEP (Brainstem Auditory Evoked Response or
Potential). It is to elicit brainstem responses to auditory stimulation
by clicks or tone bursts. It is a noninvasive technique to find the
integrity of central auditory pathways through the eighth cranial
nerve, pons and mid-brain. In this method, electrical potentials are
BERA in progress
generated in response to several click stimuli or tone-bursts and
picked up from the vertex by surface electrodes. It measures hearing sensitivity in the range
of 1000-4000 Hz. In a normal person, 7 waves are produced in first 10 milliseconds. The
first, third and fifth waves are most stable and are used in measurements. The waves are
studied for absolute latency, inter-wave latency (usually between wave I and V) and the
amplitude.
The exact anatomic site of generations for various waves is disputed but the latest studies
indicate the following sites;
Wave I-------Distal part of eighth cranial nerve.
Wave II----- Proximal part of eighth cranial nerve near the brainstem.
Wave III---- Cochlear nucleus.
Wave IV---- Superior olivary complex.
Wave V----- Lateral lemniscus.
Wave VI and VII-----Inferior colliculus.
Remember the mnemonic EEC SLI (Eight, Eight, Cochlear nucleus, Superior olivary
complex, Lateral lemniscus, Inferior colliculus).
Uses of ABR:
1. As a screening procedure for infants.
156
2. To determine the threshold of hearing in infants, children and adults who do not
cooperate.
3. To detect malingering.
4. To diagnose retrocochlear pathology particularly vestibular schwannoma.
5. To diagnose brainstem pathology e.g., multiple sclerosis or pontine tumors.
6. To monitor 8th cranial nerve intraoperatively in surgery of acoustic neuroma to preserve
the function of cochlear nerve.
5. OtoAcoustic Emissions (OAEs):
These are low intensity sounds produced by outer hair cells of a normal cochlea and can be
elicited by a very sensitive microphone placed in the external meatus and analyzed by a
computer. Sound produced by outer hair cells travels in a reverse direction:
Outer hair cells → basilar membrane → perilymph → oval window → ossicles → tympanic
membrane → ear canal.
OAEs are present when outer hair cells are healthy and are absent when these are
damaged. So otoacoustic emission is helpful to test the function of cochlea. These do not
disappear in eighth cranial nerve pathology as cochlear hair cells are normal.
Types of OAEs:
Broadly OAEs are of two types;
A. Spontaneous OAEs:
These are present in normal hearing persons where hearing loss does not exceed 30 dB.
These may be absent in 50% of normal persons.
B. Evoked OAEs:
These are elicited by a sound stimulus. These are further divided into two types depending on
the sound stimulus used to elicit them.
a. Transient Evoked OAEs (TEOAEs). A series of click stimuli are presented at 80-
85 dB SPL and response recorded.
b. Distortion Product OAEs (DPOAEs). Two tones are simultaneously presented to
the cochlea to produce distortion products. These have been used to test hearing in
the range of 1000-8000 Hz.
Uses:
1. OAEs are used as a screening test of hearing in neonates and to test hearing in
uncooperative or mentally retarded individuals after sedation. Sedation does not interfere
with OAEs.
2. These help to distinguish cochlear from retrocochlear hearing loss. OAEs are absent in
cochlear lesions e.g., ototoxic sensorineural hearing loss. These detect ototoxic effects
earlier than pure-tone audiometry.
157
3. OAEs are also useful to diagnose retrocochlear pathology, especially auditory
neuropathy. Auditory neuropathy is a neurologic disorder of 8th cranial nerve.
Audiometric tests, e.g., SNHL for pure tones, impaired speech discrimination score,
absent or abnormal auditory brainstem response, show a retrocochlear type of lesion but
OAEs are normal.

OAEs are absent in 50% of normal individuals, lesions of cochlea, middle ear disorders (as
sound travelling in reverse direction cannot be picked up) and when hearing loss exceeds 30
dB.
6. Central auditory tests:
These tests are designed to find defects in the central auditory pathways and the temporal
cortex. Several tests with test signal delivered to one ear (monotic) or both ears (dichotic)
have been used, but currently the “Staggered spondaic words” test is widely used. Central
auditory tests are not used routinely.
SELF ASSESSMENT
Q. 1. Decreased bone conduction in an audiogram indicates

a. damage to cochlea d. otosclerosis

b. ossicular dislocation e. tympanic membrane perforation

c. ossicular fixation
Q. 2. Positive recruitment is indicative of

a. central hearing loss d. non-organic hearing loss

b. cochlear hearing loss e. retrocochlear hearing loss

c. conductive hearing loss


Q. 3. Which of the following statements is incorrect?
a. Cochlear duct is filled with endolymph score
b. Cochlear duct is filled with perilymph d. Scala tympani is filled with perilymph
c. Phonetically balanced (PB) words used e. Spondee words are used to measure
to measure speech discrimination speech reception threshold (SRT)
Q. 4. Caloric test determines function of
a. lateral semicircular canal d. superior semicircular canal

b. posterior semicircular canal e. utricle

c. sacule
Q. 5. Speech frequencies include

a. 125, 250, 500 Hz d. 1000, 2000, 3000 Hz


b. 250, 500, 1000 Hz e. 1000, 2000, 4000 Hz
c. 500, 1000, 2000 Hz
Q. 6. The following audiometry is grouped under evoked response audiometry (ERA).
a. Brainstem auditory evoked potential b. Brainstem auditory evoked response
158
c. Brainstem evoked response audiometry d. Transtympanic electrocochleography
(BERA) e. All of the above
Q. 7. Afferent pathway involved in stapedial reflex is
a. cranial nerve VIIth d. cranial nerve XIth

b. cranial nerve VIIIth e. cranial nerve Xth


c. cranial nerve IXth
Q. 8. Efferent pathway involved in stapedial reflex is
a. cranial nerve VIIth d. cranial nerve XIth
b. cranial nerve VIIIth e. cranial nerve Xth
c. cranial nerve IXth
Q. 9. Stapedial reflex is absent in all except
a. lesion of cranial nerve VII at geniculate yrinthine segment
ganglion d. lesion of cranial nerve VII in the
b. lesion of cranial nerve VII at tympanic segment
stylomastoid foramen e. profound deafness
c. lesion of cranial nerve VII in the lab-
Q. 10. The stapedius muscles contract bilaterally at an intensity of sound stimulus above
hearing threshold
a. 30 dB d. 90 dB

b. 50 dB e. 110 dB
c. 70 dB
Q. 11. Tests of hearing, where subjectivity is eliminated, are
a. beksey audiometry d. speech audiometry

b. electric response audiometry e. none of the above


c. pure tone audiometry
Q. 12. Which is the investigation of choice in assessing hearing loss in infants?
a. Behavioral audiometry c. Free field audiometry

b. Brainstem evoked response audiome- d. Impedance audiometry

try (BERA) e. Pure tone audiometry


Q. 13. Otoacoustic emissions are produced by

a. auditory nerve d. outer hair cells

b. basilar membrane e. tectorial membrane

c. inner hair cells


------------------------------------------- ANSWERS--------------------------------------------
------------- ---------
Q.1. a 2. b 3. b 4. a 5. c 6. e 7. b 8. a
9. b 10. c 11. b 12. b 13. d
159
--------------------------------------------------------------------------------------------------------------------------
HEARING LOSS IN CHILDREN
There are several causes of hearing loss in children. These may be divided into;
1. Congenital
2. Acquired
Congenital hearing loss:
It is again divided into;
a. Conductive hearing loss
b. Sensorineural hearing loss
Causes of conductive hearing loss in children:
These may be unilateral or bilateral
I Meatal atresia
II Ossicular abnormalities e.g., Treacher Collins syndrome
Causes of sensorineural hearing loss in children:
A. Prenatal causes:
a. Hereditary causes:
1. Waardenburgs syndrome
2. Alports syndrome
3. Ushers syndrome
b. Non hereditary causes:
1. These may arise from maternal infections (TORCH)
2. Drugs
3. Radiations
4. Other factors e.g.,
• nutritional deficiencies
• diabetes
• toxemia
• thyroid insufficiency
B. Perinatal causes:
a. Anoxia e.g.,
• placenta previa
• prolonged labour
• prolapsed cord
160
b. Prematurity (if weight < 1500 g)
c. Forceps delivery
d. Neonatal jaundice (if bilirubin > 20 mg/dl)
e. Meningitis
f. Ototoxic drugs
C. Postnatal causes:
a. Genetic: But manifest later in the childhood or adulthood
b. Non genetic:

• Viral infections e.g., measles, mumps, varicella, influenza, meningitis,


encephalitis
• Otitis media with effusion
• Ototoxic drugs
• Trauma e.g., fracture of temporal bone, middle ear surgery, and perilymph
fistula
• Noise induced
Assessment of hearing in infants and children:
1. Screening procedures:
a. Arousal test
b. Auditory response cradle
2. Behaviour observation audiometry:
a. Moro’s reflex
b. Cochleovestibular reflex
c. Cessation reflex
3. Distraction techniques:
4. Conditioning techniques:
a. Visual reinforcement audiometry
b. Play audiometry
c. Speech audiometry
5. Objective audiometry:
a. Impedance audiometry
b. Heart rate audiometry
c. Electrocochleography
d. Auditory brainstem response
161
e. Otoacoustic emissions
Management:
Period from birth to 6th months of life is extremely important for development of speech and
language. So, it is absolutely essential to have early identification and assessment of hearing
loss and to start early rehabilitation. It is found that children managed before 6 months had
higher support of vocabulary and better expressive/comprehensive language.

1. Parental guidance:
2. Development of speech and
language: a. Receptive skills:
I. Visual e.g.,
• written language
• lip reading
• sign language
II. Auditory e.g.,
• hearing aid
• cochlear implant
III. Tactile
b. Expressive skills:
I. Oral speech
II. Written speech
III. Manual sign language or finger spelling
3. Education of deaf:
4. Vocational guidance:
SELF ASSESSMENT
Q. 1. Maternal infections which can affect development of cochlea include all except
a. Cytomegalovirus d. Syphilis

b. Herpes e. Toxoplasmosis
c. Malaria
Q. 2. Factor which can identify infant with high risk of hearing loss includes
a. bilirubin more than 20 mg/dl d. malformation of pinna

b. birth weight less than 1500 g e. all of the above


c. hyperbilirubinemia
Q. 3. Inner ear malformation in fetus can occur when mother during pregnancy is ex-
posed to
a. cytomegalovirus d. thalidomide

b. german measles e. all of the above


c. radiation
Q. 4. A child aged 03 years presented with severe sensorineural hearing loss, he
was prescribed hearing aids but showed no improvement. What is the next line
of manage-ment?
a. BAHA d. Fenestration

b. Cochlear implant e. Stapedectomy


c. Conservative
Q. 5. Congenital hearing loss with rhinitis pigmentosa is found in
a. Alports syndrome d. Usher’s syndrome

b. Cogan’s syndrome e. Waardenberg’s syndrome

c. Potter’s syndrome
Scenario A: Mother of a 06 years old child complains that her child has hearing
difficulty in school for last 06 months. He does not cooperate for otoscopic examination
or free field hearing assessment.
1. List four common causes of hearing loss at this age?
2. Name two audiological tests for hearing assessment?

Scenario B: Mother bought her 2 and half years old boy to ENT OPD with complaints of
not responding to sounds and he has not started speaking. On examination both tympanic
membranes, nose and throat is normal.
1. How will you investigate this child?
2. Write down management of this patient?

--------------------------------------------------------ANSWERS----------------------------------------------
-------
Q.1. c 2. e 3. e 4. b 5. d

Scenario A: 1. Otitis media with effusion, eustachian tube dysfunction, wax, acute SOM
2. PTA, tympanometry, BERA.
Scenario B: 1. Conditioning techniques: a.Visual reinforcement audiometry, b. Play
audiometry, c.Speech audiometry. Objective audiometry: a. Impedance audiometry b.
Heart rate audiometry c. Electrocochleography d. Auditory brainstem response
e.Otoacoustic emission 2. If patient has severe sensineural hearing loss then hearing aid
is advised to the patient and if patient suffers from profound sensineural hearing loss
then trial of hearing aid for six months followed by cochlear implant is given.
--------------------------------------------------------------------------------------------------------------------------
HEARING AID
A hearing aid is an electronic device which amplifies sound. Following types of hearing aids
are available.
A. Conventional hearing aids:
It consists of;
a. Microphone which picks up sounds and converts it into
electrical impulses.
b. Amplifier which amplifies electrical impulses.
c. Receiver which converts electrical impulses back to sounds.
BTE hearing aid
Types of conventional hearing aids:
a. Bone conduction hearing aid:
It has a bone vibrator instead of a receiver which fits on the mastoid and directly
163
stimulates the cochlea. It is used in patients with meatal atresia, otitis externa or
discharging ear.
b. Air conduction hearing aid:
In this, the amplified sound is transmitted via the ear canal to the tympanic
membrane. It is the most commonly used hearing aid.
Air conducting hearing aids can be of the following types.
1. Body worn type
2. Behind the Ear (BTE) type
3. Spectacle type
4. In the Ear (ITE) type
5. Canal types;
I. In the Canal (ITC)
II. Completely in the Canal (CIC)
B. Bone Anchored Hearing Aid (BAHA):
In this surgically implanted abutment transmits
sounds by direct conduction through the bone to the
cochlea (by passing the external meatus and middle
ear). BAHA has the following components;
Processor Abutment Fixture
a. Titanium fixture. It is surgically included in
the skull bone. It bonds with the surrounding tissue in a process called osseointegration.
b. Titanium abutment. It is exposed outside the skull.
c. Sound processor. It is attached to the abutment once the osseointegration is
complete (which may take 3 to 6 months).
C. Implantable hearing aid:
This is a new variety of hearing aids. This is implanted into the middle ear and works as a
direct drive system.
It delivers mechanical vibrations directly to the ossicular chain (rather than delivering
acoustic energy into the external meatus as with conventional hearing aids). Implantable
hearing aids are available in the following forms;
a. Piezoelectric devices
b. Electromagnetic devices
D. Contralateral Routing of Signals (CROS):
This type of hearing aid is used when one ear is completely/severely deaf. The microphone is
fitted on the side of the deaf ear and the sound thus, picked up and converged to the receiver
placed in the better ear. It helps in sound localization. This type of hearing aid is obsolete
now.
164
Indications of hearing aid:
Any person whose hearing problem cannot be corrected by medical or surgical means is a
candidate for hearing aid.
1. Sensorineural hearing loss: Hearing aid may not suit all such persons because of
distortion of sound and especially in those with recruitment.
2. Deaf children must be fitted with hearing aid as early as possible for adequate
development of speech.
3. Conductive hearing loss: Majority of such persons may be helped by surgery.
Hearing aid is prescribed when surgery is refused, not feasible or has failed.
In case of bilateral hearing loss, hearing aid is fitted in the better ear.
Fitting a hearing aid:
While fitting a hearing aid consideration is given to;
1. Of hearing loss.
2. Type of hearing loss.
3. Type of frequencies affected.
4. Presence of recruitment.
5. Age of the patient.
6. Condition of outer and middle ear.
SELF ASSESSMENT
Q. 1. Hearing aid consists of following components except
a. amplifier d. all of the above

b. microphone e. none of the above

c. receiver
Q. 2. Following is a type of hearing aid.

a. BAHA d. Implantable hearing aid

b. CROS e. All of the above


c. Conventional hearing aid
Q. 3. Air conduction hearing aid can be of the following type.
a. Body worn type d. In the Ear (ITE) type

b. Behind the ear (BTE) type e. All of the above

c. Canal type
Q. 4. Bone anchored hearing aid (BAHA) consists of

a. sound processor d. all of the above


b. titanium abutment e. none of the above

c. titanium fixture
165
Scenario: A one year old baby was brought to ENT OPD with the complaint of hearing loss
and bilateral meatal atresia. On investigation he has normal hearing structures such as
ossicles and cochlea.
1. What kind of hearing aid will be applied to this baby?
2. Name other different types of hearing aids.
----------------------------------------------------
---- ANSWERS-----------------------------------------------------

Q.1. d 2. e 3. e 4. d
Scenario: 1. Bone conduction hearing aid 2. BTE, ITC, CIC, BAHA
--------------------------------------------------------------------------------------------------------------------------
-
COCHLEAR IMPLANT
It is an electronic device capable of producing useful
hearing to persons who have severe to profound
sensorineural hearing loss and cannot benefit from
hearing aid. It works by producing meaningful electrical
stimulation of the auditory nerve where degeneration of
the hair cells in the cochlea has progressed to a point
such that amplification provided by hearing aids is no
longer effective. It has got two parts;
1. External part consisting of;
a. Microphone, which picks up the sound.

b. Speech processor, which analyses the signals from microphone.


Cochlear implant
c. Transmitter coil, which transmits the signals to implanted coil without need for any
wires passing through skin. The external part may be behind the ear or body worn but
the former is preferred.
2. Internal part consisting of;
a. Receiver coil, which is implanted into the bone of skull behind ear. It receives the
signal and relays it to the electrode.
b. The electrode, which is usually placed within the cochlea inside the scala tympani.

Pathophysiology:
Cochlear implant functions best in profoundly deaf ear which has a
large number of surviving peripheral neural elements capable of
receiving and transmitting electrical impulses to central nuclei.
Successful cochlear implants have been reported after all types of
acquired hearing loss affecting the cochlea.
A child with
Otological selection of patients for cochlear implant: cochlear
implant
166
Potential cochlear implant subjects are classified into following groups indicating increasing
difficulty in auditory rehabilitation.
1. Acquired post lingually deaf adults
2. Acquired post lingually deaf children
3. Acquired pre lingually deaf children
4. Congenitally deaf children
5. Acquired pre lingually deaf adults
6. Congenitally deaf adults
Diseases in which there is profound or total loss of hearing are;
A. Congenital:
B. Acquired:
1. Trauma
2. Labyrinthitis
3. Meningitis
4. Otoscelerosis
5. Meniere’s disease
6. Ototoxicity
Evaluation of patient prior to cochlear implant:
1. Otological examination:
Tympanic membrane must be intact and there must be no active infection. Any chronic
infection of nose or throat must be eradicated prior to surgery. Patient must be medically fit
enough to undergo a 4-hour surgical procedure.
2. Audiological investigation:
Pure tone audiometry must show profound hearing loss (greater than 90 dB hearing loss at
any audiometric frequency). Speech score must be zero. A trial with a suitable hearing aid
must be given before cochlear implant. Electrocochleography is done to ensure that no useful
cochlear function exists. All patients who have a promontory stimulation test should hear
using an implant.
3. Radiological investigation:
CT must show patency of cochlear duct on atleast three consecutive films taken at 02 mm
intervals.
4. Preoperative counselling:
Thorough psychological evaluation of patient must be carried out. Patient and clinician must
167
have realistic approach. Patient must accept that he/she may not be able to understand and
speak at all through the device.

Following facts must be explained to the patient;


1. An implant does not provide recognizable speech sounds.
2. An implant will not work better than a hearing aid in noisy environment.
3. Cochlear implant is not less noticeable than a hearing aid.
4. Several visits are required after the operation to adjust the outputs to obtain the best
speech processor setting or programs.
Cochlear implant surgery:
Standard approach is to embed the body of implant in skull behind the mastoid (after
cortical mastoidectomy) and to trail the electrodes via the mastoid through a posterior
tympanotomy to the round window. A cruciate incision is made in the round window,
electrode is inserted smoothly and at 8 to 9 mm depth till the tip of electrode contacts the
spiral ligament. The round window niche is filled with soft tissue to prevent a perilymph leak
postoperatively.
Complications of surgery:
1. Hematoma
2. CSF leak
3. Facial palsy
4. Tinnitus
5. Persistent perilymph leak
Auditory brainstem implant (ABI)
The implant is designed to stimulate the cochlear nuclear complex in the brainstem. It is
placed in the lateral recess of fourth ventricle. The ABI is used when bilaterally eighth nerve
has been damaged in surgery of say vestibular schwannoma.
SELF ASSESSMENT
Q. 1. A cochlear implant can be used in all of the following conditions causing bilateral
severe to profound hearing loss except
a. meningitis d. ototoxic drugs

b. mumps e. section of cranial nerve VIII

c. otosclerosis

Q. 2. The neural element most likely to be stimulated by cochlear electrode is


a. auditory nerve b. inner hair cells
c. outer hair cells e. tectorial membrane
d. spiral ganglion
Q. 3. A cochlear implant has the following components except
a. amplifier d. speech processor

b. microphone e. transmitter coil


c. receiver coil
Q. 4. A 03 years old child presented with severe sensorineural hearing loss, he was
prescribed conventional hearing aid but showed no improvement. What is the next line
of management?
a. Auditory brainstem implant d. Conservative treatment

b. BAHA e. Stapedectomy

c. Cochlear implant
Q. 5. In cochlear implant the electrode is placed in

a. scala media d. all of the above

b. scala tympani e. none of the above


c. scala vestibuli
Q. 6. In Auditory Brainstem Implant (ABI), the implant is placed in
a. lateral recess of fourth ventricle. d. scala vestibuli

b. scala media e. none of the above

c. scala tympani
Scenario: A 03 years old child presented in ENT unit referred from pediatric department
where she was admitted for treatment of bacterial meningitis. On inquiry, it was found that
patient had undergone an ear surgery for profound sensorineural hearing loss one month
ago. Related to this operation answer the following questions:
1. What type of surgery patient has undergone?
2. What are the other complications related to this surgery?
---------------------------------------------------
----- ANSWERS-----------------------------------------------------
Q.1. e 2. a 3. a 4. c 5. b 6. a
Scenario: 1. Cochlear implant surgery 2. Meningitis, CSF leak, persistent perilymph leak,
hematoma, facial palsy, tinnitus.
--------------------------------------------------------------------------------------------------------------------------
-
169

PART C
NOSE AND PARANASAL
SINUSES Chapter 9
SYMPTOMATOLOGY AND EXAMINATION
OF NOSE AND PARANASAL SINUSES

SYMPTOMS OF NOSE AND PARANASAL SINUSES


Every system and organ in the body has got its own set of symptoms and signs. More
than 90% of patients presenting in ENT department, present with one or more of the
following symptoms;
1. Nasal obstruction
2. Nasal discharge
3. Sneezing
4. Epistaxis
5. Facial pain and headache
6. Snoring
7. Change in sensation of smell
8. Change in quality of speech
a. Rhinolalia clausa (hyponasal/denasal/anasal voice)
b. Rhinolalia aperta (hypernasal voice/nasal twang)
9. Change in nasal shape
10. Miscellaneous
Now we will discuss all these symptoms in detail.
1. Nasal obstruction: This is the most common symptom of nasal diseases. When a
patient complains of nasal obstruction, the following information should be obtained
from patient;
a. Unilateral or bilateral obstruction: Diseases like deflected nasal septum, foreign
body, unilateral polyp and neoplasm etc, often produce unilateral nasal obstruction.
Whereas
170
adenoids, allergic and infective rhinosinusitis generally, produce bilateral nasal obstruction.
b. Duration: It may indicate whether the disease is acute, subacute or chronic.
c. Mode of onset: Allergic disorders produce obstruction which is quick to come on
and quick to go away i.e., it may be a matter of minutes, whereas infective causes
give rise to obstruction which may become complete over a period of months.
Many patients complain of alternating obstruction at times one nasal cavity is blocked the
other is patent and vice versa. This happens particularly when patient goes to bed and when
he lies on one side, the upper nasal cavity opens up and the lower nasal cavity is obstructed.
On changing side, the patent and obstructed sides of the nose also change over. This is a
natural phenomenon due to the nasopulmonary reflex. The lower lung being compressed does
not function as freely as the upper lung and due to reflex action. The nasal cavity of the side
of the compressed lung closes off and when the patient changes the side, the same
phenomenon takes place on the side of the other lung. In severe nasal obstruction the patient
breathes through the mouth (mouth breathing). If this is prolonged over a period of time,
dental and facial deformities and other complications occur.
d. Progress of symptom:
e. Severity: i.e., whether the nose is completely or partially blocked.
f. Persistent or intermittent: Causes which produce persistent obstruction, e.g., nasal
polyps, foreign body, neoplasms etc. The causes which produce intermittent swelling
of the nasal mucous membrane give rise to intermittent obstruction, e.g., allergic and
infective rhinitis and sinusitis.
g. Initiating or aggravating factors:
h. Relieving factors:
2. Nasal discharge (Rhinorrhoea): This is also quite a frequent complaint. Following
characteristics of nasal discharge should be noted.
a. Amount: In allergic rhinitis amount is copious, whereas in infective rhinitis it is
scanty.
b. Color: In allergic disorders, the discharge is almost colorless like water, whereas in
infective disease it is pale or deep yellow, depending upon the severity of the disease.
In epistaxis, the discharge is frankly sanguineous and in diseases like foreign body
and tumors, the discharge is often purulent, mixed up with blood.
c. Consistency: In allergic disorders, the discharge is thin and watery and the tissue
paper or handkerchief on which it is received does not stiffen because the discharge
contains very little mucus. In infective disorders, the discharge is thick and mucoid or
muco-purulent and the handkerchief on which it is received on drying up.
d. Smell: The discharge is odorless in allergic rhinitis, whereas it may have bad smell in
infective diseases.
171 Part C Nose

3. Sneezing: Excessive sneezing on exposure to certain agents such as dust, paint and
perfume is characteristic of allergic rhinitis. v
4. Epistaxis: Frank blood from nose is a symptom of various diseases which will be
discussed later in topic of epistaxis. In many diseases, the blood is mixed with nasal
discharge.
5. Facial pain and headache: Pain in diseases of nose and paranasal sinuses often
occurs in rhinosinusitis. When the ostium of the sinus gets blocked and it fails to
discharge its contents into the nasal cavity. This leads to pain which may be referred to
the vertex or the occiput. Pain from frontal sinusitis has a very typical periodicity. When
the patient gets up in the morning, he has no pain. On assuming erect posture, pain starts
in the forehead which goes on getting worse and worse till it reaches its height about
midday when it starts diminishing, and disappears by sunset. Deflected nasal septum may
press on the middle turbinate to produce anterior ethmoidal nerve syndrome.
6. Snoring:
7. Change in sensation of smell:
a. Anosmia: It is absent sensation of smell. It may occur in atrophic rhinitis, or
sometimes in diseases which do not allow the odoriferous particles to ascend up into
the olfactory region of the nose, e.g., nasal polypi and hypertrophic rhinitis etc. It
may also occur in fracture of the anterior cranial fossa due to tearing off of the
olfactory nerve fibers.
b. Hyposmia: It is decrease sensation of smell. Causes of hyposmia are usually the
same as those for anosmia.
c. Hyperosmia: It is increased sensation of smell. It may be seen in cases of allergic
rhinitis.
d. Parosmia: It is perverted sensation of smell.
e. Cacosmia: It is bad sensation of smell.
f. Phantosmia: It is smelling of non-existent odor.
8. Change in quality of speech: When a person is speaking, there is a beautiful balance
between the air which passes through the oral cavity and which passes through the nasal
cavity. The balance may be disturbed in certain diseases.
a. Rahinolalia clausa: It means decreased nasal character of voice. It is a symptom of
nasal obstruction. The speech becomes toneless and dull. It is found in diseases like
adenoiditis, nasal polypi, neoplasms of nose and paranasal sinuses etc.
b. Rhinolalia aperta: It means increased nasal character of voice. It is also known as
nasal twang. It occurs when during speech there is excessive passage of expired air
through the nose e.g., in cleft palate, paralysis of the palate, etc.
Pharynx is an important organ for giving resonance to the speech. Thus, in
cases of extreme hypertrophy of the tonsils or a tumor in the pharynx, the speech will
appear to be very full. In normal speech production, the palate shuts off the
nasopharynx
CONCEPT BOOK OF DISEASES OF EAR, NOSE & THROAT BY FAHIM AWAN
172 Part C Nose
from the oropharynx during the pronunciation of all the letters of the alphabet except those
which have got nasal twang, i.e., ‘M’, ‘N’ and ‘NG’. During pronunciation of the letters, the
nasopharynx is not completely shut off from the oropharynx; part of the air escapes through
the nose. In paralysis of the palate or in cases of perforation in the palate, much more air
passes out through the nose than is required for proper speech production. The result is that
the patient’s speech contains an excess of nasal twang. This condition is called rhinolalia
aperta. Conversely, if there is blockage in the throat, the nasopharynx of the nose, too little air
may escape through the nose. In this case the speech will contain an abnormally low amount
of nasal twang and it will appear dull or toneless and uninteresting. This condition is called
rhinolalia clausa.
9. Change in nasal shape: Several diseases of the nose produce changes in the shape
and external appearance of the nose. Congenital defects are generally, very obvious.
Fractures of the nasal bones may produce external swelling and deformity, the degree of
which will be determined by the severity of the force used. Lupus may show apple-jelly
nodules on the skin and in later stages there may be destruction of part of the nose. Nasal
polypi may broaden out the nose and deflected septum also often twists the bridge and
tilts the tip. Malignant tumors often deform the nose. A boil in the vestibule shows by
swelling and redness of the tip. Excessive nasal discharge due to rhinosinusitis or
adenoids in children produces fissuring of the external nares.
10. Miscellaneous: Sometimes nasal diseases may cause bulging of eyeballs (proptosis) or
telecanthus (increased distance between two medial canthi).
Nasal diseases may produce other symptoms for example, general disturbances of
temperature and pulse. These may also produce symptoms due to complications e.g.,
otitis media, laryngitis, bronchitis etc.
EXAMINATION OF NOSE AND PARANASAL SINUSES
After completing all the prerequisites of examination, the nose and paranasal sinuses are
examined as follows;
1. Inspection:
Several diseases of the nose and paranasal sinuses can be diagnosed just by looking at it.
Inspection is done of;
a. External nasal pyramid
b. Ethmoid sinus region
c. Maxillary sinus region
d. Frontal sinus region
Clinician has to look for any congenital anomaly, anatomical landmarks, symmetry and any
other positive finding (scar, swelling, ulceration, deformity, hemangioma, lymphangioma
etc.).

CONCEPT BOOK OF DISEASES OF EAR, NOSE & THROAT BY FAHIM AWAN


173 Part C Nose

2. Patency test:
Patency of the nasal cavities is checked with a wisp of cotton, cold spatula or indirect
laryngoscopy mirror.
3. Sense of smell:
Sense of smell is tested separately in each nasal cavity using soap, perfume etc. Patient is
asked to close the eyes during testing. Sense of smell is tested separately in both nasal
cavities using soap, perfume etc.
4. Palpation:
Palpation is done of;
a. Frontal sinuses
b. Supraorbital margins
c. Ethmoids
d. Nasal pyramid
e. Maxillary sinuses
5. Tilt up the tip of nose:
To examine the interior of the vestibule in adults and the vestibule and the nasal fossae in
children, tilting the tip of nose upwards is often sufficient to provide an adequate view. This
is done with the thumb of the left hand and by stabilizing the head with the rest of the
fingers. However, in case of adults, anterior rhinoscopy with the help of a speculum to see the
nasal fossae, is essential.
6. Anterior rhinoscopy:
It is carried out with the help of a Thudicum nasal speculum.
Again, inspection is done for;

Thudicum nasal speculum

a. Any congenital anomaly


b. Anatomical landmarks e.g., lateral wall, septum, roof,
floor etc.
c. Symmetry
d. Any other positive finding e.g., discharge, DNS, polyps,
tumor granuloma etc.
Vibrissae in the vestibule of nose cause considerable obstruction to
the interior of nose. To do away with this obstruction, a Thudicum
nasal
speculum is a simple and good instrument to use. It dilates the nasal
cavities and retracts the vestibule to have a better view of the nasal
cavities. It consists of two flat blades carried on an inverted ‘U’ shaped
metallic loop. Besides removing the obstruction caused by the vibrissae,
the elasticity of the ‘U’ shaped loop also helps in the examination of the Anterior rhinoscopy
CONCEPT BOOK OF DISEASES OF EAR, NOSE & THROAT BY FAHIM AWAN
174 Part C Nose

interior of the nose by dilating the anterior nares.


The speculum with various sizes of blades is available to suit the age of the patient. It is
held in left hand in such a manner that the blades point towards the patient, the index finger is
introduced into the concavity of the loop from the patient’s side towards that of the clinician,
the thumb steadies the speculum from above and the middle and the ring fingers flank on
either side of the bars of the ‘U’. The blades are brought together by means of the middle and
the ring fingers and are introduced into the vestibule of the nose, first of all along the long
axis of the anterior nares and when inside the vestibule, the speculum is brought up in the
vertical position. The pressure from the bars of the speculum is released only sufficiently to
allow a good view of the interior of the nose. Too much dilatation of the anterior nares is to
be avoided for fear of causing pain or discomfort to the patient.
Each nasal fossa is examined in turn. The medial wall, the lateral wall, the roof and the
floor of the nose are examined one after the other in a systematic manner. The position of the
patient’s head has to be changed slightly in various directions to have a good view of the
walls of the nasal fossae, because in no single view are all these walls seen in sufficient
detail. The medial wall or the septum of the nose is seen for its position i.e., whether it is
central or deflected markedly to any side. To decide whether there is any gross deflection, it
is best to compare the sizes of the two nasal fossae. Any spurs on the septum are also noted.
In many cases of nasal diseases, the relationship of the deflection to the middle turbinate
and of the spurs to the inferior turbinate is important. Pressure on the middle turbinate often
gives rise to outward displacement of the turbinate and congestion in the middle meatus.
These may lead to neuralgic headache and sinusitis. If a spur is impinging on the inferior
turbinate there may be allergic symptoms. The septum is further examined for any varicosity
over the Little’s area in cases of epistaxis. By tilting the head a bit downwards to the floor of
the nose is seen particularly for any collection of discharge and by tilting the head slightly
upwards the roof is examined for any polypi and tumor.
In the lateral wall, the inferior turbinate is seen. When hypertrophied, it is often mistaken
for a nasal polyp. The inferior turbinate should be examined for its size, color and surface. It
is hypertrophied, pale blue and smooth in allergic disorders. It is hypertrophied, deep pink
and smooth in chronic catarrhal rhinitis, and irregular on the surface in chronic hypertrophic
rhinitis. In atrophic rhinitis, it is shriveled up. The inferior meatus is seen for any collection of
discharge.
After examination of all the walls, nasal cavity of each side should be examined. There
may be discharge in the nasal cavity. Its amount, color and exact site should be noted.
Discharge of allergic rhinitis is profuse, thin and watery. Whereas discharge of infective
rhinitis is moderate in amount, thick, mucopurulent or purulent and is found in the middle
meatus, between the inferior turbinate and the septum or lying at the floor of the nose. The
nasal cavities are also seen for any foreign body, polypi or tumor. The foreign body is often
covered with purulent discharge. The polypi look pale, smooth and wet like bunch of grapes.
The neoplasm is granular on the surface.

CONCEPT BOOK OF DISEASES OF EAR, NOSE & THROAT BY FAHIM AWAN


175 Part C Nose

If nasal mucosa is congested, it may be necessary to spray some decongestant, for


example, xylometazoline or oxymetazoline in the nose for detailed examination.
The blunt nasal probe is invaluable in distinguishing various swellings inside the nose,
for example, nasal polypi, septal spurs, foreign bodies, tumor and often even a blob of mucus
which may very much resemble a mucous polyp.
Probe test:
If there is any space occupying lesion other than angiofibroma, probe test is preformed to
find out;
a. Sensitivity to touch
b. Bleeding on touch or not
c. Consistency
d. Site of attachment (to lateral wall, septum, roof or floor)
7. Examination of upper part of oral cavity:
As the floor of nose and the maxillary sinus is formed by the upper part of the oral cavity
and diseases of these regions may extend to the later part or vice versa. So, examination of
upper part of oral cavity is included in examination of nose and paranasal sinuses.
Inspection is done of lips, gums, buccal mucosa, teeth, hard palate, soft palate and
movements of the soft palate.
8. Posterior rhinoscopy:
Reflective surface of posterior rhinoscopic (doubly angulated) mirror is warmed on
mirror warmer/spirit lamp. Clinician initially checks mirror on the back of his hand to see that
it is not too hot. Then touches the patient’s hand with it to ensure that it will not cause any
harm. Anterior two third of the tongue is depressed using a
tongue depressor. Warm mirror is gently passed on the
superior surface of the tongue depressor, to one side of the
uvula so as not to touch the posterior pharyngeal wall
(otherwise gag reflex may occur) and inspect for:
a. Any congenital anomaly e.g., choanal atresia,
Posterior rhinoscopy mirror
meningocele, encephalocele etc.
b. Anatomical landmarks e.g., choana, posterior ends of turbinates, posterior portion of
septum, pharyngeal opening of eustachian tube, fossa of Rosenmuller, roof of
nasopharynx etc.
c. Symmetry
d. Any other positive finding e.g., polyp, enlarged turbinate, adenoids, tumor etc.
The posterior rhinoscopic mirror is withdrawn and passed on the other side of the uvula and
inspected for the same findings as mentioned above.

CONCEPT BOOK OF DISEASES OF EAR, NOSE & THROAT BY FAHIM AWAN


176 Part C Nose

9. Examination of lymph nodes:


10. Redrape the patient and pay
thanks.
11. Flexible nasoendoscopic or 0/30-degree
12. endoscopic examination:
In doubtful cases or cases where detailed
examination of nasal cavity and nasopharynx Endoscopic examination of nose
is required, flexible nasoendoscope or zero/ thirty-degree nasoendoscope is used.
SELF ASSESSMENT
Q. 1 The most common symptom of nose and paranasal sinuses is
a. headache d. nasal obstruction

b. nasal discharge e. sneezing

c. nasal hemorrhage
Q. 2. Rhinolalia clausa is also known as

a. anasal voice d. all of the above

b. denasal voice e. none of the above

c. hyponasal voice
Q. 3. Rhinolalia aperta is also known as

a. denasal voice d. both a & b

b. hypernasal voice e. both b & c


c. nasal twang
Q. 4. Rhinolalia clausa is associated with all of the following except
a. adenoiditis d. nasal polyps

b. allergic rhinitis e palatal paralysis

c. antrochoanal polyp
Q. 5. Hyperosmia is a feature of

a. allergic rhinitis d. vasomotor rhinitis

b. hypertrophic rhinitis e. none of the above

c. rhinitis medicamentosa
Q. 6. Parosmia is
a. diminution in sense of smell d. smell when none is present around
b. distorted sense of smell e. unpleasant smell due to infection of
c. increased sensation of smell nose and paranasal sinuses
Q. 7. Smelling of non-existent is known as
a. cacosmia b. dysosmia

CONCEPT BOOK OF DISEASES OF EAR, NOSE & THROAT BY FAHIM AWAN


177 Part C Nose

c. hyperosmia e. phantosmia
d. parosmia
Q. 8. The length of nasal cavity in adult is
a. 04 cm d. 13 cm

b. 07 cm e. 16 cm
c. 10 cm
Q. 9. A patient with complete anosmia still responds to inhalation of
a. ammonia d. mint

b. coffee e. vanilla
c. garlic
Q. 10. In a normal person all of the following structures are seen on anterior rhinoscopy
except
a. cartilaginous septum d. middle turbinate

b. floor of nose e. superior turbinate

c. inferior turbinate
Q. 11. Patency of nose can be checked with all of the following except

a. cold spatula d. all of the above

b. IDL mirror e. none of the above

c. whisp of cotton wool


--------------------------------- ANSWERS--------------------
----------------------- ---------------------------------

Q.1. d 2. d 3. e 4. e 5. a 6. b 7. e 8. b 9. a 10. e 11. d


--------------------------------------------------------------------------------------------------------------------------
CONCEPT BOOK OF DISEASES OF EAR, NOSE & THROAT BY FAHIM AWAN
178 Part C Nose

Chapter 10
DISEASES OF NOSE AND
PARANASAL SINUSES
CONGENITAL ANOMALIES
MENINGOENCEPHALOCELE
Definition:
It is herniation of glial tissue and meninges through a bony defect in skull.
Clinical features:
A meningoencephalocele may present as;
1. Basal type: It presents as a pedunculated intranasal swelling.
2. Frontoethmoidal type: It presents as a subcutaneous soft, cystic
pulsatile swelling over the root of nose.
Examination during crying or straining tends to increase the size and
tension of the swelling. The swelling increases in size on coughing.
This is known as Frustenberg test. Frontoethmoidal
meningoencephalocele
Investigations:
CT is done to find out exact site and size of the cranial defect.
Treatment:
It consists of;

Intranasal meningoencephalocele
1. Excision of the stalk from the brain.
2. Repairing the bony defect with bone/cartilage graft through which herniation has taken
place.
3. Seal the defect with temporalis fascia and fat.
All this is carried out endoscopically by rhinologist or neurosurgeon. Sometimes craniotomy
may be required.
SELF ASSESSMENT
Q. 1. Meningoencephalocele
is

a. herniation of brain tissue


only d. all of the above
b. herniation of brain tissue
along with meninges e. none of the above
c. herniation of dura only
179
Q. 2. Which of the following is true about meningoencephalocele?
a. Increases in size on coughing d. All of the above

b. Increases in size on crying e. None of the above

c. Increases in size on straining


Q. 3. In meningoencephalocele increase in size of swelling on coughing is called
a. Dick test d. Shick test

b. Frustenberg test e. Tobey Ayer test

c. Queckenstedt test
Q. 4. Investigation of choice in meningoencephalocele is

a. angiography d. X-ray PNS

b. CSF analysis e. X-ray soft tissue nasopharynx

c. CT scan
Q. 5. Which of the following statement is not correct?
a. A solitary polyp in an infant is best d. Multiple nasal polyps in a middle-aged
removed by avulsion man may hide malignant tumor under-neath
b. A solitary polyp in an infant may suggest
meningocele e. Vasomotor rhinitis can lead to nasal
c. Bilateral nasal polyp in a child may polypi
suggest cystic fibrosis
Q. 6. All are treatments of meningoencephalocele except
a. endoscopy d. repair of defect

b. excision of stalk e. sealing of defect

c. evulsion of stalk
Scenario: A 01-year-old child presented with glistening mass in right nasal cavity. Its size
increased on coughing and crying (i.e., Frustenberg test wa s positive).
1. What is your clinical diagnosis?
2. What investigations will you carry out?
3. How will you treat this patient?
------------------------------------------------------
-- ANSWERS-----------------------------------------------------
Q.1. b 2. d 3. b 4. c 5. a 6. c
Scenario: 1. Meningoencephalocele 2. CT scan 3. Excision of stalk and repair of bony defect.
--------------------------------------------------------------------------------------------------------------------------
180 Part C Nose

CHOANAL ATRESIA
Definition:
Choana is a Greek word which means a funnel. So, the term choana is used for posterior
aperture of nose.
Etiopathology:
It is due to failure of the buccopharyngeal membrane to undergo involution during
intrauterine life. Choanal atresia may be unilateral or bilateral. It may be
bony in 90% of cases or membranous in 10% of cases.
Clinical features:
Unilateral choanal atresia is more common and may remain
undiagnosed until adult life. There may be complaint of unilateral
mucoid nasal discharge since birth. Sometimes there may be feeding
difficulty especially if healthy side of nose is occluded by contact with the
ipsilateralbreast.
Bilateral choanal atresia presents as a respiratory emergency Left sided choanal atresia
and if diagnosis is not made early, newborn may die. Bilateral choanal
atresia is apparent at birth, as the infants are obligate nasal breathers
(infants cannot breathe through mouth). Nasal
obstruction in older children and adults results in reflex
opening of the mouth. This reflex is somehow absent in
infants. The neonate with bilateral choanal atresia
becomes cyanosed during quiet periods but attains
normal color during crying. It is in contrast to
laryngomalacia.
On patency test, there will be no misting of mirror
or cold spatula placed beneath the nasal cavities. A dye
such as methylene blue instilled into nose will not appear in the oropharynx.
A soft red rubber catheter passed through each side of Diiference between normal and choanal atresia
nose will not come into oropharynx. Flexible
nasopharyngoscopy is very helpful in diagnosis of unilateral or bilateral choanal
atresia.
Choanal atresia is linked with CHARGE association in 60% of cases.
C →Colobomatous blindness
H →Heart disease
A →Atresia of choana
R →Retarded growth
G →Genital abnormality
E →Ear abnormality
Investigations:
1. Choanogram confirms the diagnosis of choanal atresia. Which
CT showing bilateral
means if we take a radiograph after instilling a contrast medium into
choanal atresia
nasal cavities, contrast medium will not pass into oropharynx.
2. Computerized tomography demonstrates whether the atresia is membranous or bony
and it also demonstrates exact thickness of the atresia.
CONCEPT BOOK OF DISEASES OF EAR, NOSE & THROAT BY FAHIM AWAN
181 Part C Nose

Treatment:
In unilateral choanal atresia, there is no emergency and surgery
are performed as an elective procedure.
In bilateral choanal atresia, there is always a respiratory
emergency and management is done in two stages;
1. Primary or emergency treatment
2. Secondary or definitive treatment

1. Primary or emergency treatment: McGroven’s technique


A feeding nipple with a large hole provides a good oral airway
(McGrovern’s technique). A neonatal Water’s airway is inserted and
taped with the face. Orogastric tube is inserted for feeding purpose.
Airway may also be restored temporarily with a laryngeal mask,
endotracheal tube or tracheostomy.
2. Secondary or definitive treatment:
This should be carried out as early as possible. Under general anesthesia,
choana are opened surgically through transnasal or transpalatal approach.
Choana may be opened with Tilley Lichtwitz trocar and cannula, diamond
paste burr or CO2 laser. Opening is maintained with silastic tube for 3-6 Surgical treatment
weeks.
SELF ASSESSMENT
Q. 1. Unilateral choanal atresia presents as

a. epistaxis d. unilateral bloodstained nasal dis-

b. respiratory emergency charge

c. stridor e. unilateral mucoid nasal discharge

Q. 2. Pathognomonic feature of neonate with unilateral choanal atresia is


a. feeding difficulty in ipsilateral breast of
mother
b. feeding difficulty in contralateral
breast of mother
c. hoarse cry
d. respiratory difficulty
e. stridor

Q. 3. Bilateral choanal atresia typically presents as


a. epistaxis
b. respiratory emergency
c. stridor
d. unilateral bloodstained nasal discharge
e. unilateral mucoid nasal discharge

Q. 4. In 60% of cases, choanal atresia is linked with


a. CHARGE syndrome c. Usher’s syndrome

b. Costen syndrome d. Villaret’s syndrome

CONCEPT BOOK OF DISEASES OF EAR, NOSE & THROAT BY FAHIM AWAN


182 Part C Nose
e. Waardenberg syndrome
Q. 5. A newborn with cyanosis and respiratory difficulty improved by insertion of an oral
airway. The most probable clinical diagnosis is
a. bilateral choanal atresia d. congenital subglottic stenosis

b. congenital abductor paralysis e. laryngomalacia

c. congenital laryngeal web


Q. 6. Investigation of choice for choanal atresia is

a. choanogram d. X-ray PNS

b. fistulogram e. X-ray nasopharynx


c. sinogram
Q. 7. Emergency treatment of bilateral choanal atresia is
a. Eply’s technique d. mantle technique

b. inverted Y technique e. McGrovern’s technique


c. Lautenslager technique
Q. 8. Treatment of bilateral choanal atresia includes all except
a. endotracheal intubation d. wait and see

b. laryngeal mask e. water’s airway

c. tracheostomy
Scenario: A newborn baby presents with respiratory emergency apparent at birth. He
becomes cyanosed in quiet periods and attains normal color during crying.
1. What is the most probable clinical diagnosis?
2. What is CHARGE syndrome?
3. What investigations will you carry out?
4. How will you manage this patient?
----------------------------------------- ANSWERS-----------------------------------------
--------------- ------------
Q.1. e 2. a 3. b 4. a 5. a 6. a 7. e 8. d
Scenario: 1. Bilateral choanal atresia 2. Colobomatous blindnesss, Heart disease, Atresia of
choana, Retarded growth, Genital abnormalities, Ear abnormalities 3. Choanogram, CT
4. McGovern’s technique, surgery.
--------------------------------------------------------------------------------------------------------------------------
-
CONCEPT BOOK OF DISEASES OF EAR, NOSE & THROAT BY FAHIM AWAN
183 Part C Nose and Paranasal Sinuses

TRAUMA TO NOSE AND PARANASAL SINUSES


FRACTURE OF NOSE
Etiopathology:
Nasal bones are thin, prominent and are present on exposed part of body. Hence, these are the
most common bones to get fractured. Causes are trauma to the body and therefore, causes of
fracture of nose are;
1. Road traffic accident
2. Assault
3. Personal accident
4. Sports injury
Classification:
Different classes indicate force and direction of trauma to the
nose. Fracture of nose is classified as follows;
1. Type I fracture:
Frontal trauma to nose causes this fracture. In this class, there is
a vertical fracture of the nasal septum (known as Chevallet’s
fracture). The thin distal portion of the nasal bone is fractured. Types/classes of fracture
2. Type II fracture:
Lateral trauma to nose causes this fracture. There is lateral deviation of nose. In this class,
there is a C-shaped fracture of the perpendicular plate of the ethmoids and septal cartilage
(known as Jarjavay’s fracture). The frontal process of the maxilla may be fractured.
3. Type III fracture:
High energy trauma to nose causes this fracture. In this class, the fracture extends to involve
the ethmoidal complex. There is telescoping of the ethmoidal complex and marked
depression of the nasal bridge at the level of the nasion. There is telecanthus. The lacrimal
sac and nasolacrimal duct may also be damaged.
Perpendicular plate of the ethmoid is also rotated thus, pulling the septal
cartilage backwards. Nasal tip is also rotated upwards. It gives the patient a
‘pig-like’ appearance with forwards facing nostrils. There is saddle
deformity of nose.

Clinical features:
1. Patient may present with external nasal deformity. There may be pain
Fracture nose
and epistaxis.
with external
nasal deformity
2. There may be nasal obstruction due to blood clot, septal hematoma,
septal abscess, septal fracture or septal dislocation.
3. Rarely there may be complaint of watery nasal discharge (CSF rhinorrhoea).
CONCEPT BOOK OF DISEASES OF EAR, NOSE & THROAT BY FAHIM AWAN
184 Part C Nose and Paranasal Sinuses

On examination;
1. Inspection shows edema, periorbital ecchymosis (black eye, panda facies, raccoon’s
eyes). There may be deformity of nasal shape and/or telecanthus.
2. Palpation shows tenderness, crepitus and step deformity.
3. Anterior rhinoscopy may show blood clot, septal hematoma, septal abscess, septal
fracture or septal dislocation.

Complications:
1. External nasal deformity
2. Septal hematoma/abscess
3. Epistaxis
4. CSF rhinorrhoea
Investigations:
Diagnosis is made on clinical grounds.
No X-ray is required in simple limited fractures of the nasal bones. There may be fracture
and it may not be visible on X-ray or vascular line may be mistakenly interpreted as fracture
line.
X-ray is required only for medicolegal purposes or to rule out more extensive injuries
to the facial skeleton.
Treatment:
After management of A (airway), B (breathing) and C (circulation), attention is given towards
fracture of nasal bones. There is very limited window of opportunity to manipulate the
fracture to improve cosmatic defect. Best time for reduction of fracture of nasal bones is
either before the appearance of edema or after it has subsided. However, it depends on the
stage at which the patient presents after the trauma.
1. Early/pre-edema stage (within 4 to 6 hours):
If the patient presents early before the swelling appears (within 4-6 hours), fracture is reduced
immediately by digital manipulation, with or without local anesthesia. This digital
manipulation is usually effective in achieving disimpaction and realignment.
2. Intermediate/edema stage (6 hours to 6 days):
If the patient presents late after the development of edema, then
anatomical landmarks are lost. The patient is prescribed anti-
inflammatory, analgesics and asked to revisit after 7-10 days.
3. Late stage/post edema (7 to 14 days):
Usually by this time the swelling has subsided and reduction of
fracture
of nasal bones can be done under local or general anesthesia. Walsham’s Forceps for reduc-
tion of fracture nasal
bones
185 Part C Nose and Paranasal Sinuses

forceps is used for mobilizing the


bone fragments of the lateral
wall. Asch’s forceps is used for
septal fractures and septal
dislocations. After realignment,
external nasal splintage is done
with plaster of Paris.
4. Very late stage (after 3
weeks):
An adult over the age of 18
years, presenting 03 weeks after
the trauma is advised to have
rhinoplasty at least six months after the trauma. Correction and elevation
It is carried out under general anesthesia.
A child presenting 03 weeks after the trauma is advised to have rhinoplasty after the
age of 18 years. It is carried out under general anesthesia.
Fracture or dislocation of the nasal septum is dealt at the time of treatment of fracture of nasal
bones.
It must be remembered that if a patient presents with septal hematoma/abscess at any
stage, it should be drained on urgent basis.
Criteria for operative treatment in cases of trauma nose:
There are only following;
1. If the patient presents with deformity of nose after the trauma, then reduction of fracture
must be undertaken within three weeks. If there is no deformity of nose after trauma, (no
matter how severe the trauma may be, or there may be wide fracture lines or loss of
alignment on X-ray), no reduction of fracture of nasal bones is required.
2. If the patient presents with septal hematoma/abscess, then the patient needs urgent inci-
sion and drainage.
3. Epistaxis sometimes may be severe enough to require anterior nasal packing.
4. CSF rhinorrhoea may occur rarely. It is usually managed conservatively.
SELF ASSESSMENT
Q. 1. The most common fracture in injury to face is

a. ethmoid bone d. maxillary bone

b. frontal bone e. nasal bones


c. mandible
Q. 2. The vertical fracture of nasal septum resutling from the frontal blow is known as
a. Chevallet fracture d. all of the above
b. Guerine fracture e. none of the above
c. Jarjaway fracture
Q. 3. The C-shaped fracture of the nose resutling from the lateral blow is known as
a. Chevallet fracture c. Jarjaway fracture

b. Guerine fracture d. all of the above

186 Part C Nose and Paranasal Sinuses


e. none of the above
Q. 4. Type III fracture of nasal bone includes all except
a. floating palate d. telecanthus

b. pig like appearance e. telescoping of ethmoidal complex

c. saddle deformity
Q. 5. Limited fracture of nasal bones may show all except

a. black eye d. proptosis

b. panda facies e. raccoon’s eyes

c. periorbital ecchymosis
Q. 6. In fracture nasal bone, on palpation there may be

a. crepitus d. all of the above

b. step deformity e. none of the above


c. tenderness
Q. 7. All are true of nasal bone fracture except
a. may not be associated with nasal c. often associated with septal injuries

deformity
d. often reduced by closed methods

b. mostly involves lower part of nasal e. radiology is diagnostic


bones
Q. 8. Fracture of nasal bones should ideally be reduced within

a. 04 to 06 hours d. a & b

b. 06 hours to 06 days e. a & c


c. 07 to 14 days
Q. 9. Fracture nasal bones 06 months after trauma in an adult should be treated by
a. digital manipulation d. all of the above

b. manipulation under anesthesia e. none of the above


c. rhinoplasty
Q. 10. The most important consideration in management of maxillofacial injury is
a. arrest of bleeding d. management of circulation

b. management of airway e. management of maxillofacial fracture

c. management of associated head injury


Q. 11. Fracture of the nasal bones is corrected by

a. Asch’s forceps d. Kerrison’s forceps

b. Hajak’s forceps e. Walsham’s forceps

c. Joseph’s forceps
Q. 12. Fracture of the nasal septum is corrected by

a. Asch’s forceps b. Hajak’s forceps


187 Part C Nose and Paranasal Sinuses

c. Joseph’s forceps e. Walsham’s forceps


d. Kerrison’s punch forceps
Q. 13. A 20 years old female presents to you one month after the trauma. There is com-
plaint of external deformity of nose. The best management is
a. anti-inflammatory drugs d. rhinoplasty after 06 months

b. digital manipulation there and then e. urgent rhinoplasty


c. reduction with Walsham forceps
Q. 14. A 12 years old boy presents to you one month after the trauma. There is complaint
of external deformity of nose. The best management is
a. anti-inflammatory drugs forceps

b. digital manipulation there and then d. rhinoplasty after 06 months

c. reduction with Walsham and Asch e. rhinoplasty after 18 years of age


Scenario A: An 18 years old boy presents with trauma nose 07 days back. There is no
complaint of external deformity of nose. On examination, tenderness and crepitus of external
nasal pyramid is present. X-ray nasal bones shows wide fracture line.
1. What is the most probable clinical diagnosis?
2. Is there any importance of X-ray from management point of view?
3. How will you manage this patient?
B: A 10 years old boy fell from stairs and suffered from severe nasal trauma 08 hours ago.
1. Enlist the first aid management of this child?
2. Enlist three usual complications of nasal trauma?
--------------------------------------------------------ANSWERS-----------------------------------------------------
Q.1. e 2. a 3. c 4. a 5. d 6. d 7. e
8. e 9. c 10. b 11. e 12. a 13. d 14. e
Scenario A: 1. Fracture nasal bone 2. Medicolegal purposes or to rule out more extensive
injuries to the facial skeleton 3. Analgesics, POP dressing.
B: 1. a. Maintain i.v line b. Stop bleeding c. Incision and drainage if septal hematoma
2. External nasal deformity, septal hematoma, septal abscess.
--------------------------------------------------------------------------------------------------------------------------
-

DEFLECTED NASAL SEPTUM (DNS)


Synonyms: Deformed nasal septum, Deviated nasal septum
Etiopathology:
1. Trauma: The most common
cause of deflected nasal
septum (DNS) is trauma to
the nose which may or may
not be known to the patient.

DNS with contralateral inferior


turbinate hypertrophy
188
2. Developmental errors: Another cause of DNS is abnormal posture during intrauterine
life.
3. Space occupying lesions: Nasal polypi and tumors of nose may push the septum leading
to deviated nasal septum.

Classification:
Deformity of nasal septum can be classified into;
1. Deviation: Deviation may be C shape or S shape in the
vertical or horizontal direction.
2. Dislocation: Lower border of septal cartilage may be
dislocated from the maxillary crest into one or the other
nasal cavity. Caudal border of septum may be dislocated
from columella.

Different types of DNS

3. Spur: This is sharp angulated projection usually at the junction of vomer with the
septal cartilage.
Clinical features:
1. Nasal obstruction: This is usually on the side of deviation but occasionally may be
on the contralateral side because of compensatory hypertrophy of the
inferior turbinate which is known as paradoxical nasal obstruction.
Nasal valve is at narrowest part of nasal cavity and is present at
level of upper/lower border of upper lateral cartilage. Even minimal
septal deviation in the region of nasal valve area causes the greatest
obstruction. Moderate
deviations, anterior or posterior to the region of valve may not be too much Left sided DNS
symptomatic.
2. Anosmia/hyposmia: This usually occurs on the side of obstruction due to lack of
entry of inspired air into the olfactory region (upper one third of nasal cavity).
3. Mucosal changes: Crusting may occur on the side of deviation due to concentration
on small areas of nasal mucosa of inspiratory air currents. Removal of these crusts may
produce ulceration and bleeding.
4. External deformity: Patient may complain of deformity of nasal pyramid in severe
deviation of nasal septum.
5. Sluder’s neuralgia: Also known as anterior ethmoidal nerve syndrome. There is
very severe pain on the external nasal pyramid due to compression of the anterior ethmoidal
nerve.
Other symptoms like nasal discharge or headache are usually due to complications of
DNS.
On examination;
189 Part C Nose and Paranasal Sinuses

1. Deviation of nasal septum towards lateral nasal wall.


2. Hypertrophy of inferior turbinate on contralateral side in long standing cases.
3. Cottle test is positive if deviation is in the region of valve.
Cottle test: Pulling of ipsilateral cheek outwards and upwards will relieve nasal
obstruction if the obstruction is in the valve area.
Investigations:
Diagnosis of DNS is on clinical grounds. Cottle test
X-ray PNS may be required to find out any complication of DNS or
associated diseases like sinusitis, polyps, tumors etc.

Treatment:
For treatment purposes, Cottle has classified septal deviations into three
types:
1. Simple deviation: It is a mild degree of deviation of nasal septum. It does not cause
nasal obstruction.
2. Obstruction: It is a moderate degree of deviation of nasal septum. Patient presents
with nasal obstruction but the obstruction is relieved by topical or systemic
decongestants.
3. Impaction: There is severe degree of deviation of nasal septum which touches the
lateral nasal wall. Even after administration of decongestants, patient’s symptoms are not
relieved which indicates skeletal septal abnormality. These are the patients who are real
candidates for septal surgery.
Treatment is in the form of septal surgery which may be;
1. Septoplasty
2. S.M.R. (Sub Mucoperichondrial Resection)
1. Septoplasty:
It is a conservative approach in which deviated part of the septum is mobilized and
repositioned.
Advantage: There are less chances of complications.
Disadvantage: There are more chances of residual disease.
2. SMR:
It is a radical approach in which deviated part of septum is removed.
Advantage: There are less chances of residual disease.
Disadvantage: There are more chances of complications.
Criteria for septoplasty or S.M.R.:
1. Patients under the age of 18 years are dealt with septoplasty while patients over the
age of 18 years are dealt with SMR.
190
2. A vertical line is drawn between nasal process of frontal bone above and nasal
process of maxillary bone below.
If deviation is anterior to this line then septoplasty is performed. If deviation is posterior to
this line then S.M.R is performed.
SEPTOPLASTY/SMR
Indications of septal surgery:
1. Gross DNS
2. As part of other procedures like rhinoplasty
3. Source of grafting material for rhinoplasty and tympanoplasty
4. Closure of septal perforation
5. To obtain surgical access e.g.,
a. Nasal polyps behind DNS
b. Hypophysectomy
Contraindications:
1. Acute upper or lower respiratory tract infection
2. Systemic diseases like hypertension, diabetes, hemostatic diathesis
3. Pregnancy and menstruation
Procedure: After local or general
anesthesia, reverse Trendelenburg position is
made. Throat (epilaryngeal) pack is placed.
Two percent xylocaine with adrenaline
(1:80,000) is injected in the
submucoperichondrial plane of nasal septum.
Incision is made at the caudal border of
septum which is known as Freer incision.
Incision may be made 4 to 5 mm behind
caudal border of septum which is known as
Killian incision.
Mucoperichondrial flaps on both sides of
septum are elevated. Deviated part of septum
is mobilized, straightened and repositioned in
midline which is known as septoplasty.
Deviated part of septum can also be removed
which is known as S.M.R. Flaps replaced
back to their original position. Incision is closed with Incision of septoplasty
absorbable sutures.
Splints in both nasal cavities are placed and transfixed. Anterior nasal packing is done in
both nasal cavities to obliterate potential space and bolster applied. Any blood or secretions in
the oropharynx sucked out with the help of pharyngeal sucker. Throat pack (also known as
epilaryngeal pack) is removed after securing hemostasis. Nasal packs are removed after 24
hours.
Complications:
1. Nasal hemorrhage
2. Septal hematoma
3. Septal abscess
191 Part C Nose and Paranasal Sinuses

4. Septal perforation
5. Saddle nose
6. Columellar retraction
7. Columellar shortening
8. Synechiae formation
SELF ASSESSMENT
Q. 1. Nasal septum is formed of the following.
a. perpendicular plate of ethmoid d. all of the above

b. septal cartilage e. none of the above

c. vomer
Q. 2. Septal cartilage is also known as

a. alar cartilage d. quadrilateral cartilage

b. conchal cartilage e. upper lateral cartilage

c. lower lateral cartilage


Q. 3. The most common cause of DNS is

a. congenital d. trauma

b. developmental e. tumor
c. granulomatous
Q. 4. Following is included in classification of nasal septal deformity
a. deviation d. all of the above

b. dislocation e. none of the above

c. spur
Q. 5. Cottle test includes pulling of cheek

a. inwards & downwards d. outwards & forwards

b. inwards & upwards e. outwards & upwards

c. outwards & downwards


Q. 6. Following is indication of septal surgery

a. as part of septorhinoplasty d. nasal polyps behind DNS


b. closure of septal perforation e. all of the above

c. gross DNS
Q. 7. Contraindications of septal surgery include
a. acute upper & lower respiratory tract and diabetes
infection d. all of the above
b. pregnancy & menstruation e. none of the above
c. systemic diseases like hypertension
192
Q. 8. For treatment purposes nasal septum is divided into anterior and posterior parts by
a vertical line drawn between
a. frontal bone to maxillary bone d. nasal bone to maxillary bone

b. frontal bone to zygomatic bone e. zygomatic bone to maxillary bone


c. nasal bone to frontal bone
Q. 9. For treatment purposes following is included in Cottle classification of septal
deviation.
a. Impaction d. All of the above
b. Obstruction e. None of the above
c. Simple deviation
Q. 10. All are true about septoplasty except
a. conservative approach line drawn from frontal bone to maxil-
b. less chances of complications lary bone
c. more chances of residual disease e. performed mostly over age of 18
d. performed in deviation anterior to a years
Q. 11. All are true about SMR except
a. less chances of residual disease d. performed mostly under age of 18
b. more chances of complications years
c. performed in deviation posterior to a
line drawn from frontal bone to e. radical approach
maxillary bone
Q. 12. Complication of septal surgery is
a. hemorrhage d. septal perforation

b. septal abscess e. all of the above

c. septal hematoma
Q. 13. Complication of septal surgery is

a. columellar retraction d. synechiae formation

b. columellar shortening e. all of the above

c. saddle nose
Q. 14. Saddle deformity of nose results from damage to

a. caudal strut of septal cartilage d. vomer

b. dorsal strut of septal cartilage e. none of above

c. perpendicular plate of ethmoid


Q. 15. Region of valve is at
a. lower border of lower lateral carti- c. lower border of upper lateral carti-
lage lage
b. lower border of septal cartilage d. lower border of alar cartilage

CONCEPT BOOK OF DISEASES OF EAR, NOSE & THROAT BY FAHIM AWAN


193 Part C Nose and Paranasal Sinuses

e. upper border of septal cartilage


Q. 16. Which local anesthetic also produces vasoconstrictor effect on nasal mucosa?
a. cocaine d. procaine

b. lignocaine e. xylocaine

c. prilocaine
Scenario A: A 15 years old boy complains of left sided persistent complete nasal obstruction
for last 02 years and on examination has left sided deflected nasal septum.
1. How will you decide whether boy should have SMR or septoplasty?
2. Enumerate four indications of SMR.
B: A 40 years old man present with nasal obstruction, postnasal drip and occasional frontal
and facial pain for last one year. On examination there is marked deviation of nasal septum
towards right side. The left inferior turbinate is also enlarged.
1. What is your diagnosis?
2. What will be your treatment of choice?
3. Name five main complications of septal surgery?
C: A 13 years old girl complains of persistent nasal obstruction for last one year.
Examination reveals that septum is deviated to right side with some hypertrophy of left
inferior turbinate. Her Hb is 10 gm/dl.
1. What is the best surgical treatment for her?
2. What is the difference between SMR and septoplasty?
3. Enumerate four contraindications of surgery.
----------------------------------------- ANSWERS------------------------------------------
--------------- -----------
Q.1. d 2. d 3. d 4. d 5. e 6. e 7. d 8. a
9. d 10. e 11. d 12. e 13. e 14. b 15. c 16. a
Scenario A: 1. Age, caudal deformity 2. Nasal obstruction, recurrent epistaxis due to spur,
post nasal discharge causing sore throat, approach to pituitary gland.
B: 1. Sinusitis 2. SMR and left inferior turbinectomy 3. Septal hematoma, adhesion, supratip
depression, CSF rhinorrhoea, caudal retraction.
C: 1. Septoplasty 2. In septoplasty minimum cartilage removal, less complications and in
SMR vice versa 3. Acute upper or lower respiratory tract infection, systemic diseases like
hypertension, diabetes, hemostatic diathesis, pregnancy and menstruation
--------------------------------------------------------------------------------------------------------------------------
-
CEREBROSPINAL FLUID (CSF) RHINORRHOEA
Definition:
It is defined as leakage of CSF into the nose.
194
Etiopathology:
Leakage of CSF from the subarachnoid space into the nose is due to defect in the dura, bone

and mucosa. There are following causes of CSF rhinorrhoea;


1. Trauma:
a. Non surgical:
Two percent to 09% of the head injuries are associated with CSF rhinorrhoea.
b. Surgical:
• Any surgery on sphenoid, ethmoid or frontal sinus
• Hypophysectomy
• Functional endoscopic sinus surgery (FESS)
2. Tumors: Tumors of pituitary gland, vestibular schwannoma, nasopharyngeal tumors
and frontoethmoid tumors.
3. Congenital defects: Congenital defects in the skull base with or without associated
meningocele and encephalocele.
4. Idiopathic: CSF leakage may occur spontaneously without any known cause.
Almost all the fistulae arise in the anterior cranial fossa. In fractures of temporal bone, CSF
may escape from the middle ear through the eustachian tube into the nose. This is known as
CSF otorhinorrhoea.
Clinical features:
1. Clear watery nasal discharge which is initiated or increased by
bending forwards is the most common presentation. It is also
increased by activities which increase intracranial pressure such as
Valsalva maneuver, coughing and defecation etc.
2. There may be sweet/salty taste in mouth. CSF rhinorrhea
3. There may be Continuous headache due to low intracranial
pressure secondary to CSF leakage.
4. Halo sign/Double target sign: If CSF is mixed with blood “halo
sign” or “double target sign” may be positive. Which means if the
secretions are taken on a piece of gauze or filter paper, it shows a
central red spot of blood and a peripheral lighter ring of CSF.
Flexible nasopharyngoscope/30-degree endoscope may help to
localize the site of leak.

Investigations:
1. Estimation of β2 transferrin: β2 transferrin is a protein which is present Halo sign
in CSF. Its small amounts are also present in perilymph and aqueous humor. Its
estimation in nasal secretions is diagnostic of CSF rhinorrhoea.
195
2. Estimation of β trace protein: It is highly specific for CSF. But facilities for its
estimation are available only in a few countries.
Nasal secretions do not contain glucose, whereas CSF does have glucose. Estimation of
glucose in CSF rhinorrhoea may not be helpful if it is mixed with blood.
3. Radiology:
a. High resolution CT scanning is very useful in traumatic cases to identify the site of
defect.
b. MRI is very useful in non traumatic cases to find the underlying pathology such as
tumor, encephalocele or hydrocephalus.
4. Intrathecal dyes and markers: Examination of nose with flexible nasopharyngo-
scope/30-degree endoscope especially after injection of dyes or markers (indigo, carmine,
methylene blue, 5 % fluorescein or radioactive tracer) into the lumbar theca may help to
localize the exact site of leakage of CSF.
Treatment:
A. Conservative:
1. Antibiotic prophylaxis: Majority of traumatic CSF leaks resolve within 7-14 days
with conservative measures. Until leak closes, the patient is at risk of developing
pneumococcal meningitis. So, all the patients with CSF rhinorrhoea should be given
adequate antibiotic prophylaxis. For adults, oral penicillin 500 mg six hourly and
sulfonamide 500 mg six hourly is given.
2. Strict bed rest and elevation of head end of bed is advised. Any straining such as
coughing, sneezing, nose blowing, bending forward, lifting heavy weight and strain-
ing on stool is avoided. Patient’s bowel is kept soft. Medical means are adopted to
decrease intracranial tension.
3. Spinal drain may be introduced to decrease intracranial pressure which helps
to increase the healing process.
B. Surgical:
Surgical treatment of CSF leakage is considered if after addressing the underlying cause
(tumor or hydrocephalous), the leak is profuse or it does not close spontaneously after 7-
10 days with conservative measures.
Exploration and surgical closure of the site of leakage is carried out. It is done either
endoscopically (FESS), trans-septally or extracranially. Defect is covered with fascia lata
or temporalis fascia.
The exact approach depends on the site of the fistula i.e., whether anterior, middle or
posterior cranial fossa. The surgical intervention is done either by otologist or by
neurosurgeon.
SELF ASSESSMENT
196 Part C Nose and Paranasal Sinuses
Q. 1 The most common cause of CSF rhinorrhoea is
a. congenital d. trauma

b. idiopathic e. tumor
c. inflammatory
Q. 2. The most common site for CSF rhinorrhoea is
a. cribriform plate d. petrous bone

b. frontal sinus e. sphenoid sinus

c. occipital bone
Q. 3. CSF rhinorrhoea may be seen after

a. fracture of cribriform plate sinus

b. fracture of floor of anterior cranial d. transsphenoidal hypophysectomy

fossa
e. all of the above

c. fracture of posterior wall of frontal


Q. 4. All are true about CSF rhinorrhoea except

a. decreased by Valsalva maneuver d. increased by lifting weight

b. increased by bending forward e. patient presents with clear watery

nasal discharge
c. increased by coughing
Q. 5. All are true about CSF rhinorrhoea except

a. continuous headache due to low in- c. halo sign is negative

tracranial pressure
d. sweet taste in mouth

b. double target sign is positive e. salty taste in mouth


Q. 6. Following contain β2 transferrin
a. aqueous humor d. all of the above

b. CSF e. none of the above

c. perilymph
Q. 7. CSF rhinorrhoea is diagnosed by
a. beta-2 microglobulin d. thyroxine

b. beta-2 transferrin e. tri-iodothyronine


c. thyroglobulin
Q. 8. The most recent investigation for CSF rhinorrhoea is estimation of

a. β2-transferrin d. Potassium

b. β2-trace protein e. Sodium

c. Glucose
Q. 9. To localize CSF leak, an intrathecal injection of fluorescein dye showed staining of
the pledget in the olfactory sulcus, the likely site of CSF leak is:
a. Cribriform plate b. Ethmoid sinus
CONCEPT BOOK OF DISEASES OF EAR, NOSE & THROAT BY FAHIM AWAN
197 Part C Nose and Paranasal Sinuses

c. Frontal sinus e. Sphenoid sinus

d. Maxillary sinus
Q. 10. All are true about treatment of CSF rhinorrhoea except:

a. Always managed surgically d. Introduction of spinal drain

b. Antibiotic prophylaxis e. Strict bed rest

c. Avoid nose blowing


Q. 11. All are true regarding surgical treatment of CSF rhinorrhoea except:
a. Can be done endoscopically
b. Can be done extracranially
c. Can be done trans-septally
d. Defect is covered with fascia lata or temporalis fascia
e. Usually done within 7-10 days

Q. 12. Clear watery nasal discharge rich in glucose is due to:


a. Allergic rhinitis d. Vasomotor rhinitis

b. Atrophic rhinitis e. Viral rhinitis

c. CSF rhinorrhoea
Scenario: A 30 years old female had road traffic accident 02 hours back and presented with
clear watery nasal discharge, nasal obstruction, salty taste in mouth and headache.
1. What is the most probable diagnosis?
2. What investigation you will advise?
3. What is your treatment plan?
----------------------------------------- ANSWERS--------------------------------
--------------- ---------------------
Q.1. d 2. a 3. e 4. a 5. c 6. d 7. b
8. b 9. a 10. a 11. e 12. c
Scenario: 1. CSF rhinorrhoea 2. β2 transferrin, β trace protein, high resolution CT, MRI,
intrathecal dyes and markers 3. Conservative, antibiotic prophylaxis, strict bed rest and
elevation of head, spinal drain, surgical exploration and surgical closure of the site of
leakage.
--------------------------------------------------------------------------------------------------------------------------
-
INFLAMMATIONS OF NOSE AND PARANASAL SINUSES
BOIL (FURUNCLE) OF NOSE
Definition:
Boil is defined as an acute localized Staphylococcus
aureus infection of the hair follicle (pilosebaceous
gland).
Etiopathology:
Trauma by nose picking or plucking the vibrissae is the
usual causative factor. Usually, the infection is self
limiting but occasionally may spread to cause cellulitis of
nasal tip, upper lip or septal abscess. There may be
spontaneous evacuation of pus in 04 or 05 days.
Pathophysiology of boil nose
198 Part C Nose and Paranasal Sinuses

In cases of recurrent boil, diabetes or other causes of immunosuppression must be ruled


out.
Clinical features:
1. Severe pain because skin is tightly adherent to underlying tissue
through rete pegs. If the infection spreads into the subcutaneous
tissue, there may be swelling in the vestibule or at the tip of nose.
2. Nose may be unsightly, edematous, red and tender. There may be
complaint of facial pain and headache. A case of boil nose

Complications: Boil nose may give rise to following


complications;
1. Cellulitis of nose and upper lip.
2. Septal/alar abscess
3. Cavernous sinus thrombosis as a result of spread of infection from the danger area.

Investigations:
Estimation of fasting blood glucose is required in recurrent cases to rule out diabetes melli-
tus.
Treatment:
1. Systemic antibiotics such as flucloxacillin, erythromycin or cephalosporins are the drugs
of first choice.
2. Analgesics are given to relieve pain.
3. Avoid nose picking and squeezing the boil (‘respect the boil on the nose’) as this may
cause spread of infection to the cavernous sinus.
4. Incision and drainage through nasal cavity under full aseptic measures is carried out if
there is formation of abscess.
5. In recurrent furunculosis, it is very important to exclude diabetes mellitus and other
causes of immunosuppression.
6. Staphylococci should be eliminated from the external auditory canal, nasal vestibule and
nail beds by applying neomycin or gentamicin cream twice daily.
SELF ASSESSMENT
Q. 1. Organism responsible for boil nose is

a. Haemophilus influenzae d. Staphylococcus aureus


b. Klebsiella pneumoniae e. Streptococcus pneumoniae
c. Pseudomonas aeruginosa
Q. 2. Organism usually responsible for nasal vestibulitis is
199 Part C Nose and Paranasal Sinuses

a. E. coli d. Staphylococcus aureus

b. Influenza virus e. Streptococcus pyogenes

c. Pseudomonas
Q. 3. Boil nose may be due to

a. diabetes mellitus d. steroids

b. nose picking e. all of the above

c. plucking vibrissae
Q. 4. Which of the following is complication of boil nose?

a. Cavernous sinus thrombosis d. All of the above

b. Cellulitis e. None of the above

c. Septal abscess
Q. 5. Treatment of boil nose includes all except

a. analgesics d. squeezing the boil

b. incision & drainage e. systemic antibiotics

c. removal of underlying cause


Scenario: A young male presented to ENT OPD with severe pain at the tip of nose for last
one day. On examination nose is swollen, red and tender. Patient gave history of nose picking
03 days ago.
1. What is your clinical diagnosis?
2. What complications can arise if left untreated?
3. How will you manage this patient?
ANSWERS-----------------------------------
-------------------------------------------------------- ------------------
Q.1. d 2. d 3. e 4. d 5. d
Scenario: 1. Boil nose 2. Septal abscess, cavernous sinus thrombosis 3. Systemic antibiotics,
analgesics.
--------------------------------------------------------------------------------------------------------------------------
RHINOSINUSITIS
Definition:
The term rhinosinusitis usually means inflammation of mu-
cous membrane of nose and paranasal sinuses.
Etiopathology:
Because rhinitis and sinusitis usually coexist, the correct
and accepted terminology is now rhinosinusitis. Common
causes of rhinosinusitis are;
1. Infective: Infection causing rhinosinusitis
200 Part C Nose and Paranasal Sinuses

a. Acute:
• Viral
• Bacterial

b. Chronic:

Bacterial

Fungal
2. Mechanical: e.g., trauma, tumor, DNS, nasal polyp, adenoids etc.
3. Autonomic imbalance: e.g., honeymoon rhinitis, emotional rhinitis (anxiety, stress,
anger, grief, humiliation).
4. Immune deficiency: e.g., HIV, hypogammaglobulinemia.
5. Mucociliary clearance abnormalities: e.g., Kartagener’s syndrome also known as
primary ciliary dyskinesia (sinusitis, bronchiectasis, obstructive azoospermia,
dextrocardia and situs inversus due to defect in cilia). Young’s syndrome (sinusitis,
bronchitis and bronchiectasis, obstructive azoospermia due to physicochemical
abnormalities of secretions).
6. Granulomatous conditions: e.g., specific and nonspecific granulomas.
7. Hormonal: e.g., puberty, pregnancy, hypothyroidism
8. Iatrogenic: e.g., rhinitis medicamentosa (RM). It is a condition of rebound nasal
congestion brought on by extended use of topical decongestants (e.g., oxymetazoline,
phenylephrine, xylometazoline and nephazoline nasal sprays) and certain oral
medications (e.g., sympathomimetic amines and various 2-imidazolines) that constrict
blood vessels in the mucous membrane of the nose.
The most common cause of rhinosinusitis is common cold. Other causes of
rhinosinusitis include swimming, diving and dental infections.
SELF ASSESSMENT
Q. 1. Prolonged use of vasoconstrictor nasal drops may cause
a. allergic rhinitis d. rhinitis sicca

b. atrophic rhinitis e. vasomotor rhinitis


c. rhinitis medicamentosa
Q. 2. Which of the following drugs is linked with rhinitis medicamentosa?
a. Cocaine d. Oxymetazoline/xylometazoline

b. Intranasal steroid spray e. Sodium chromoglycate


c. Ipratropium bromide
Q. 3. All of the following are associated with Kartagener (immotile cilia) syndrome
except:
201

a. bronchiectasis d. situs inversus

b. chronic sinusitis e. subfertility

c. cleft palate
Q. 4. All of the following components are part of Young’s syndrome except
a. bronchiectasis d. primary ciliary dyskinesia

b. bronchitis e. sinusitis

c. obstructive azoospermia
-------------------------------------------------------- ANSWERS-----------------------------------------------------
Q.1. c 2. d 3. c 4. d
--------------------------------------------------------------------------------------------------------------------------
-
COMMON COLD (CORYZA)
Etiopathology:
It is a viral infection, which is conveyed by contact or airborne
droplets. It is usually complicated by a secondary bacterial
infection.
The common cold is caused by rhinovirus and coronavirus.
However, other viruses include adenovirus, influenza virus,
parainfluenza virus and respiratory syncytial virus.
After viral infection there is transient ischemia of the mucosa.
It is followed by swelling, hyperemia and profuse secretion of Presentation of coryza
clear seromucinous fluid. Then there is secondary bacterial
infection due to rapid growth of resident flora of the nose. Then
the rhinorrhoea becomes mucopurulent.
The organisms usually found are:
1. Streptococcus hemolyticus
2. Pneumococcus
3. Staphylococcus aureus
Stages of rhinosinusitis:
There are following stages of rhinosinusitis;
1. Ischemic stage: After an incubation period of 1-3 days, there is burning sensation in the
nasopharynx. It is followed by sneezing and shivering. Sense of smell is altered or lost.
2. Hyperaemic stage: In a few hours there is profuse rhinorrhoea and varying degrees of
nasal obstruction. Fever may also occur.
3. Stage of secondary infection: As secondary infection and leucocytic invasion occurs,
the discharge becomes yellow or green. It also thickens, owing to a high mucin content,
and

CONCEPT BOOK OF DISEASES OF EAR, NOSE & THROAT BY FAHIM AWAN


202 Part C Nose and Paranasal Sinuses

will stiffen a handkerchief.


4. Stage of resolution. Resolution occurs in 5-10 days.
Clinical features:
These vary greatly but generally, include;
1 Nasal obstruction
2. Nasal discharge
3. Facial pain and headache
4. Constitutional symptoms like fever, malaise and anorexia
5. Tenderness over the affected sinus
Complications:
Secondary infection may spread throughout the mucosa and lymphatic tissues of the
whole respiratory tract. There are following possible complications of common cold;
1. Nasopharyngitis, pharyngitis and tonsillitis
2. Eustachian salpingitis and otitis media
3. Lymphadenitis
4. Lower respiratory tract complications such as bronchitis and pneumonia.
Treatment:
A. Prophylactic:
Contact with known causes must be avoided. Vaccines for prevention of common cold may
be used.
B. Therapeutic:
General treatment consists of:
1. Rest and warmth. Ideally the patient should stay in bed.
2. Analgesics. Codeine and aspirin are of value.
3. Pseudoephedrine by mouth relieves congestion.
4. Antibiotics should be given when there is secondary infection.
5. Antihistamines and vitamin C are of doubtful value.
6. Local treatment consists of:
a. Inhalation of steam: Tincture benzoin co. or menthol may be added.
b. Vasoconstrictors: In the form of drops or sprays, give quick but temporary relief from
the nasal obstruction. These should not be used for prolong period otherwise rhinitis
medicamentosa may occur.

CONCEPT BOOK OF DISEASES OF EAR, NOSE & THROAT BY FAHIM AWAN


203 Part C Nose and Paranasal Sinuses

Differential diagnosis:
1. Allergic rhinitis: It is characterized by sneezing, nasal discharge and nasal obstruction
on exposure to certain allergens.
2. Vasomotor rhinitis: It is often described by the patient as ‘cold’, but the rhinorrhoea is
characteristically spasmodic and the condition is apyrexial. The discharge remains clear,
does not stiffen a handkerchief, and may contain an excess of eosinophils unless
secondarily infected.
3. Influenzal rhinitis: In it, constitutional symptoms like fever, malaise and anorexia are
much more severe.
SELF ASSESSMENT
Q. 1. All may cause common cold except

a. Adenovirus d. Influenza virus

b. Coronavirus e. Rhinovirus
c. E.B. virus
Q. 2. All of the following are characteristics of headache of sinus origin except
a. accompanied by vomiting or flashing c. aggravated by stooping
lights before the eyes d. presents with diurnal variation
b. aggravated by standing e. relieved by nasal decongestion
Q. 3. “Office headache” is typically seen in infection of which of the following sinuses?
a. Cavernous sinus d. Maxillary sinus

b. Ethmoid sinus e. Sphenoid sinus


c. Frontal sinus
Q. 4. Headache in occiput or vertex is caused by infection of which of the following
sinuses?
a. Ethmoid sinus d. Sphenoid sinus

b. Frontal sinus e. Superior sagittal sinus


c. Maxillary sinus
Q. 5. Purulent discharge above the middle turbinate indicates
a. anterior ethmoid sinusitis d. posterior ethmoid sinusitis

b. frontal sinusitis e. sphenoid sinusitis

c. maxillary sinusitis
Scenario: A lady presented to ENT OPD with excessive sneezing, watery rhinorrhoea, nasal
obstruction, headache and low-grade fever for last 02 days. She also has bodyaches and
pain.
1. What is the most likely diagnosis?
2. What is the management?
204
ANSWERS-----------------------------------------------
-------------------------------------------------------- ------
Q.1. c 2. a 3. c 4. d 5. d
Scenario: 1. Rhinosinusitis 2. Antibiotics, analgesics, decongestants, steam inhalation, antral
wash out if not resolved
--------------------------------------------------------------------------------------------------------------------------
ALLERGIC RHINITIS
Definition:
Allergic rhinitis is an IgE mediated
hypersensitivity disease of the mucous
membrane of the nasal airway. It is characterized
by sneezing, nasal discharge and nasal
obstruction. Pathogenesis of allergic rhinitis

Etiopathology:
IgE is composed of two heavy chains (epsilon) and two light chains (kappa and lambda). IgE
receptors with a high affinity have been identified on mast cells and basophils. Antigen
molecules are thought to interact with two adjacent cells bound IgE antibody molecules so
forming a bridge composed of IgE-antigen-IgE. This is known as cross linkage. This cross
linkage allows the formation of calcium channel and that calcium influx triggers the events
that lead to release of different chemical mediators. These mediators are histamine, heparin,
leukotriene C4, prostaglandin D2, thromboxane and platelet activating factor (PAF). Most of
the effects are caused mainly by histamine as is indicated by clinical efficacy of
antihistamines in allergic reactions.
Mast cells are abundant in the submucosa. These are also found in the epithelium.
Basophils are often found in blown secretions from patients with rhinitis. Water soluble
allergens which are readily reached from air borne particles are free to interact with IgE
sensitized mast cells (and basophils) which in turn lead to release of pharmacological
mediators of allergy i.e., hypersensitivity.
Rapid release of these mediators leads to increased permeability of the epithelium so
allowing allergens access to deeper mast cells. Local lymph nodes are the main site of
production of serum IgE antibodies to allergens.
Allergen:
Allergen is a substance which elicits an allergic response. Allergens are usually proteins or
glycoproteins. These are freely soluble and so easily absorbed through mucous membranes.
Allergens may be;
1. Inhalants e.g., pollens, house dust mite, moulds and animal dander. Pollens are the most
common cause of allergic rhinitis. These are the male fertilizing part of flowering plants.
That is why allergic manifestations due to pollen are very common in spring season.
Mites are very small arthropods having four pairs of legs. Excreta of mites and

CONCEPT BOOK OF DISEASES OF EAR, NOSE & THROAT BY FAHIM AWAN


205
chitin layer is the source of allergen.
2. Ingestants e.g., fish, egg, milk, sea foods, drugs
3. Contactants e.g., powders
There are two clinical types of allergic rhinitis;
A. Seasonal/Intermittent allergic rhinitis:
Symptoms appear in a particular season.
B. Perennial/persistent allergic rhinitis:
Symptoms are present throughout the year.
Clinical features:
Symptoms of allergic rhinitis vary from minor
inconvenience to severe incapacitating symptoms affecting
the quality of life. Usually, the patient has two or more of Allergic rhinitis
the following symptoms;
1. Sneezing: It is usually episodic and is due to release of histamine.
2. Nasal discharge: It is clear and watery. It is in response to serous and seromucinous
glands to mast cells/basophils derived mediators.
3. Nasal obstruction: It is due to vasodilatation and edema.
4. Itching of nose, eyes and palate due to release of histamine.
5. Some patients have tightness of chest, wheezing and skin manifestations.
On examination;
Nasal patency is inadequate.
Nasal mucosa is edematous, having bluish tinge due to venous stasis or may be red due to
secondary infection.
There may be other allergic manifestations in the body epithelia especially asthma and
skin diseases. Occasionally nasal polypi may be present with aspirin sensitivity and bronchial
asthma which is known as Samter’s triad.
Complications:
Allergic rhinitis may give rise to following complications;
1. Recurrent rhinosinusitis
2. Nasal polypi
3. Serous otitis media
4. Bronchial asthma

Investigations:
206
Allergic rhinitis is primarily a clinical diagnosis.
1. Eosinophil count is increased on differential leukocyte count (DLC).
2. Serum IgE level is increased.
3. Nasal secretions show basophils.
4. Nasal scrapings show mast cells.
5. Skin tests such as patch test, scratch test, prick test, or intradermal test help to identify
specific allergen.
6. Radioallergosorbent test (R.A.S.T.) is in vitro test. This test is more quantitative and
qualitative but is more expensive. It is performed when the patient is is on anti-allergic
drugs or having skin disease.
Treatment:
1. Avoidance of allergens:
This is the most important step in the management of a patient with allergic rhinitis. Bed
room is the most important source of allergen exposure because of the house dust mite.
Pillow and mattress should be enclosed in air tight plastic or fabric encasing. Feather and
synthetic pillows should be avoided. Dust masks should be used during the period of
exposure to dust.
Pollens are very common source of allergens especially in spring season.
Pets should be removed if these are the source of allergen. Sometimes change of job is
indicated.
2. Drugs:
a. Antihistamines:
These decrease itching, secretions and sneezing. These may be used topically or
systemically. The main side effect of antihistamines is sleep disturbance.
b. Decongestants:
Pseudoephedrine and phenylpropanolamine are usually given orally. Xylometazoline,
oxymetazoline and phenylephrine are given topically. These are given for 7 to 10
days to relieve nasal obstruction. These may be used topically or systemically.
Prolong use of nasal decongestants may cause “rhinitis medicamentosa”.
c. Corticosteroids:
These may be used topically or systemically. Topically used steroids are budesonide,
beclomethasone etc. Systemic steroids are usually prescribed for a short time.
d. Mast cell stabilizers:
These stabilize the mast cells and prevent their degranulation. These are used only
207
prophylactically. Commonly used mast cell stabilizers are sodium cromoglycate and
ketotifen. These may be used topically or systemically.
e. Anticholinergics:
e.g., ipratropium bromide. This atropine like nasal spray is useful against watery
nasal discharge.
Side effects are glaucoma, prostatism, dry mouth and dry eyes.
f. Antileukotrienes:
e.g., montelukast. These are effective against nasal congestion.
3. Immunotherapy:
Immunotherapy involves repeated subcutaneous or sublingual administration of
diluted allergen extracts. These are given once or twice weekly for 6-8 weeks. It is
followed by “maintenance” 4-8 weekly injection for 3-5 years.
4. Psychological issues:
Psychological issues should be addressed appropriately.
5. Vidian neurectomy:
Anastomosis of greater superficial petrosal nerve (carrying parasympathetic fibers)
with deep petrosal nerve (carrying sympathetic fibers) is known as vidian nerve or nerve
of the pterygoid canal. Resection of this nerve i.e., vidian neurectomy is done as a last
resort to relieve symptom of excessive nasal discharge.

Differential diagnosis:
1. Vasomotor rhinitis
2. Infective rhinitis
3. Nasal polypi
4. DNS
SELF ASSESSMENT
Q. 1. Which of the following statement is true regarding allergic rhinitis?
a. IgA mediated response d. IgG mediated response

b. IgD mediated response e. None of the above

c. IgE mediated response


Q. 2. Allergic rhinitis is

a. Type 1 reaction d. Type IV reaction

b. Type II reaction e. None of the above

c. Type III reaction


Q. 3. Usual symptom of allergic rhinitis is
208

a. discharge from nose and eyes d. sneezing

b. itching of nose and eyes e. all of the above


c. nasal obstruction
Q. 4. Watery nasal discharge associated with sneezing and itching in the nose and eyes is
due to
a. allergic rhinitis c. CSF rhinorrhoea

b. chronic maxillary sinusitis d. foreign body of nose

e. rhinitis medicamentosa
Q. 5. Samter’s triad consists of

a. aspirin sensitivity d. all of the above

b. bronchial asthma e. none of the above


c. nasal polypi
Q. 6. Increased level of which immunoglobulin (in nasal discharge) will help in the
diagnosis of allergic rhinitis?
a. IgA d. IgM

b. IgD e. All of the above


c. IgE
Q. 7. All are investigations of allergic rhinitis except
a. eosinophil count d. RAST

b. nasal smear e. skin test


c. PCR
Q. 8. All are varieties of skin test in allergic rhinitis except
a. intradermal test d. scratch test

b. patch test e. subcutaneous test


c. prick test
Q. 9. The most important step in the treatment of allergic rhinitis is
a. antihistamines d. immunotherapy

b. avoidance of allergens e. vidian neurectomy

c. corticosteroids
Q. 10. Sodium cromoglycate acts by
a. decreasing the number of eosinophils d. increasing the mast cell permeability

b. decreasing the number of mast cells e. stabilizing the mast cell permeability
c. increasing the number of mast cells
Q. 11. The best drug which provides protection against nasal allergy when used just
before exposure to allergen is
a. corticosteroid nasal spray b. prednisolone orally
209 Part C Nose and Paranasal Sinuses

c. pseudoephedrine e. xylometazoline
d. sodium cromoglycate
Q. 12. Following statement is true about vidian nerve except
a. anastomosis of greater superficial glands
petrosal nerve with deep petrosal nerve d. controls tone of blood vessels in nasal
b. carries secretomotor fibers to nasal mucosa
glands e. section of this nerve is done in atrophic
c. carries secretomotor fibers to lacrimal rhinitis
Scenario: A 17 years old girl complains of sneezing, nasal congestion, watery rhinorrhoea,
and itching throughout the year. On clinical examination she has bluish nasal mucosa and
enlarged inferior turbinate.
1. What is the most probable diagnosis?
2. How can you confirm the diagnosis?
3. What precautions she should take to avoid the house dust mite?
--------------------------------------------------------ANSWERS-----------------------------------------------------
Q.1. c 2. a 3. e 4. a 5. d 6. c 7. c
8. e 9. b 10. e 11. d 12. e
Scenario: 1. Allergic rhinitis 2. Eosinophilic count, serum IgE level, skin tests, RAST
3. Pillow and mattress should be enclosed in air tight plastic or fabric encasing, feather and
synthetic pillows should be avoided, dust masks should be used.
-------------------------------------------------------------------------------------------------------------------------
VASOMOTOR RHINITIS (VMR)
Synonyms: Intrinsic rhinitis, Nonspecific rhinitis, Nasal hyperreactivity

Definition:
When all other causes of rhinitis are ruled out, then the condition is termed as
vasomotor rhinitis.
Etiopathology:
Autonomic nervous system is under the control of hypothalamus and, therefore,
emotions play a great role in vasomotor rhinitis. It is said to be due to overactivity
of parasympathetic system. Nasal mucosa is hyperactive and responds to several
nonspecific stimuli e.g., changes in temperature, humidity, blasts of air etc.
Clinical features:
It clinically resembles allergic rhinitis. There is complaint of sneezing, nasal discharge and
nasal obstruction throughout the year. Usually, one or the other symptoms may predominate.
On examination nasal mucosa is congested and hypertrophied.
Investigations:
210
VMR is diagnosis by exclusion. All the tests of nasal allergy are negative.
Treatment:
1. Avoidance of factors which provoke symptoms e.g., cold air, change of temperature/
humidity etc.
2. Avoidance of anxiety, fatigue and emotional instability.
3. Antihistamines
4. Nasal decongestants (topical or systemic)
5. Steroids (topical or even systemic) are used for a limited time
6. Vidian neurectomy
Differential diagnosis:
1. Allergic rhinosinusitis
2. Recurrent rhinosinusitis
3. DNS
SELF ASSESSMENT
Q. 1. Vasomotor rhinitis is also known as

a. intrinsic rhinitis d. all of the above

b. nonspecific rhinitis e. none of the above


c. nasal hyperreactivity
Q. 2. The cause of vasomotor rhinitis is
a. autonomic imbalance d. nasal decongestants

b. IgE e. physicochemical abnormality of mucus

c. mucociliary clearance abnormality


Q. 3. Vasomotor rhinitis is due to

a. parasympathetic overactivity d. sympathetic underactivity

b. parasympathetic underactivity e. none of the above

c. sympathetic overactivity

Q. 4. A large number of eosinophils in nasal smear are seen in all of the following
conditions except
a. allergic rhinitis d. Samter’s triad

b. atopic triad e. vasomotor rhinitis


c. eosinophilic nonallergic rhinitis
Q. 5. Vidian nerve section has usually been used in the treatment of
a. excessive epiphora b. excessive salivation
211 Part C Nose and Paranasal Sinuses

c. excessive watery rhinorrhoea e. None of the above


d. All of the above
Q. 6. All are usually included in the treatment of vasomotor rhinitis except
a. avoidance of provoking factors d. nasal decongestants

b. antibiotics e. vidian neurectomy

c. antihistamines
Q. 7. Vidian neurectomy is done in

a. rhinitis medicamentosa d. all of the above

b. rhinitis sicca e. none of the above

c. vasomotor rhinitis
Scenario: A young student in most of his class tests suddenly starts having nasal obstruction,
nasal discharge and sneezing for last 01 year. All the tests of nasal allergy are negative.
1. What is the most probable clinical diagnosis?
2. How will you confirm your diagnosis?
3. Describe the treatment plan of this patient?
------------------------------------------------- ANSWERS--------------------------------------
------- ---------------
Q.1. d 2. a 3. a 4. e 5. c 6. b 7. c
Scenario: 1. Vasomotor rhinitis 2. VMR is diagnosis by exclusion. All the tests of nasal
allergy are negative 3. Avoidance of factors which provoke symptoms, avoidance of anxiety,
fatigue and emotional instability, antihistamines, nasal decongestants (topical or systemic),
steroids (topical or even systemic) are used for a limited time, vidian neurectomy.
--------------------------------------------------------------------------------------------------------------------------
-
ATROPHIC RHINITIS
Synonym: Ozaena
Definition:
It is defined as chronic nasal disease characterized by progressive atrophy of mucosa and
underlying bone of the turbinates.
Etiopathology:
Exact etiology of atrophic rhinitis is unknown. Any condition that causes prolonged drying of
nasal mucous membrane gives rise to atrophic rhinitis.
Factors which are regarded as possible causes of atrophic rhinitis are chronic
rhinosinusitis, anemia, poor nutrition, excessive surgical resection of the nasal mucous
membrane, syphilis, endocrine imbalance and autoimmune diseases.
There is chronic inflammation of the nasal mucosa and various constituents of nasal
cavities. It leads to endarteritis and periarterial fibrosis of the terminal arterioles. Ciliated
212
columnar epithelium of nasal cavities is replaced by stratified squamous epithelium. There is
degeneration of the nasal epithelium and seromucinous glands with formation of thick
adherent crusts in the nose. There is secondary infection with saprophytic organisms.
Usual causative organisms are Klebsiella ozaenae, Diphtheroid bacillus, Proteus vulgaris
and E. coli. The condition may extend to pharynx (leading to atrophic pharyngitis) and larynx
(atrophic laryngitis).
Clinical features:
1. Patient’s attendants complain that there is foul smell (also called
ozaena) from the patient which is not noticed by the patient
himself.
2. Patient presents with nasal obstruction due to excessive
crusting and destruction of nerve endings.
3. Removal of crusts leads to epistaxis. A case of atrophic rhinitis
4. There may be complaint of anosmia because of nasal obstruction caused by excessive
crusting and destruction of nerve endings.
On examination;
1. Patient is generally, depressed. There is bad smell from the patient.
2. Nasal cavities are abnormally patent and roomy.
3. There are greenish, yellowish and blackish crusts within the nasal cavities. The
turbinates are generally, curled up (shriveled).
4. There may be septal perforation and saddle nose.
Investigations:
Diagnosis is usually on clinical grounds. Source of infection is excluded by X-ray paranasal
sinuses. Serological tests are required to exclude syphilis.
Hemoglobin, serum iron and protein should also be estimated.
Treatment:
A. Conservative:
1. Removal of underlying cause such as sinusitis, anemia and nutritional deficiency.
2. Alkaline nasal douches [prepared by dissolving a teaspoonful of a mixture of sodium
bicarbonate, sodium diborate and sodium chloride (in a proportion of 1:1:2) in half a pint
(280 ml) of warm water] are performed very frequently. It loosens the crusts and removes
thick tenacious discharge. These crusts are removed with forceps or suction.
3. 25% glucose in glycerin is instilled in the nasal cavities to inhibit proteolytic
organisms which are responsible for foul smell. Decongestants nasal drops are avoided.
4. Estradiole spray helps to increase vascularity of nasal mucosa.

CONCEPT BOOK OF DISEASES OF EAR, NOSE & THROAT BY FAHIM AWAN


213 Part C Nose and Paranasal Sinuses

5. Human placental extract: Topical or systemic injections have also shown


improvement.
6. Potassium iodide given orally improves nasal secretions.
B. Surgical:
1. Young’s operation: It is the most commonly performed surgical procedure for atrophic
rhinitis. Complete surgical closure of one or both nasal cavities for about six months is
commonly very successful. In order to avoid discomfort, incomplete surgical closure of
both nasal cavities is performed which is known as modified Young’s operation.
2. Lautenslager’s operation: Submucous injection of paraffin is done to displace the
lateral nasal wall medially.
3. Surgical narrowing of nasal cavities with various materials such as teflon strips,
plastipore, bone and cartilage inserted submucosally into the septum, lateral wall or floor
of nasal cavity.
Differential diagnosis:
1. Rhinitis sicca
2. Wegner’s granuloma
3. Syphilis

SELF ASSESSMENT
Q. 1. Possible cause of atrophic rhinitis is

a. anemia d. excessive surgical resection of nasal

b. chronic sinusitis mucosa

c. endocrine imbalance e. all of the above

Q. 2. Causative organism in atrophic rhinitis is


a. Diphtheroid bacillus d. Proteus vulgaris

b. E. coli e. all of the above

c. Klebsiella ozaenae
Q. 3. Changes in atrophic rhinitis include
a. ciliated columnar epithelium to stratified c. mucosal atrophy
squamous epithelium d. periarteritis
b. endarteritis e. all of the above
Q. 4. Symptom of atrophic rhinitis is
a. anosmia d. nasal obstruction
b. epistaxis e. all of the above
c. foul smell from patient
214
Q. 5. Usual clinical features of atrophic rhinitis include all except
a. curled up inferior turbinates d. saddle nose

b. green, yellow and black crusts e. septal perforation


c. narrow nasal cavities
Q. 6. All are features of atrophic rhinitis except
a. crusting limited to anterior part of na-sal c. discomfort & irritation in nasal cavity
cavity d. may extend to larynx
b. crusting in entire nasal cavity e. may extend to pharynx
Q. 7. All are conservative treatment of atrophic rhinitis except
a. 25% glucose in glycerine nasal drops d. potassium iodide

b. alkaline nasal douches e. Young’s operation

c. placental extracts
Q. 8. All are surgical management of atrophic rhinitis except
a. Lautenslager’s operation d. surgical narrowing of nasal cavities

b. Sistrunk’s operation e. Young’s operation


c. sub mucosal insertion of teflon strips
Q. 9. Which operation is mostly employed in atrophic rhinitis?
a. Lautenslenger’s operation d. Young’s operation

b. Sistrunk operation e. None of the above

c. Various implants
Scenario A: A 60 years old lady presented with bilateral nasal obstruction for last two years.
Patient’s relatives complain of foul smell from the patient for last one year. Examination
shows roomy nasal cavities containing blackish crusts.
1. What is the most probable diagnosis?
2. What is the differential diagnosis?
3. How will you manage this case?
B: A villager lady presented with nasal obstruction, nasal bleeding and at times nasal
discharge having black coloration for the last one year. Nose examination showed yellow,
green and black crusts.
1. What is the most probable diagnosis?
2. Name two other diseases which can be included in differential diagnosis in this case?
3. Mention one diagnostic step/test for each of the above three diseases.
4. How will you manage this lady?
-------------------------------------------------------- ANSWERS-----------------------------------------------------
Q.1. e 2. e 3. e 4. e 5. c
215 Part C Nose and Paranasal Sinuses
6. a 7. e 8. b 9. d
Scenario A: 1. Atrophic rhinitis 2. Rhinitis sicca, syphilis, Wegener’s granuloma
3. Conservative; Removal of underlying cause, alkaline nasal douches, 25% glucose in
glycerin, estradiole spray, human placental extract, potassium iodide given orally improves
nasal secretions. Surgical; a. Young’s operation b. Lautenslager’s operation c. Surgical
narrowing of nasal cavities with various materials such as teflon strips, plastipore, bone and
cartilage inserted submucosally.
B: 1. Atrophic rhinitis 2. Syphilis, Wegener’s granuloma 3. Atrophic rhinitis diagnosis is
usually on clinical grounds, source of infection is excluded by X-ray paranasal sinuses,
serological tests are required to exclude syphilis, cANCA for Wegener’s granulomatosis
4. Removal of underlying cause, correction of anemia and nutritional deficiencies, alkaline
nasal douches and removal of crusts by suction is encouraged.
--------------------------------------------------------------------------------------------------------------------------
-
RHINITIS SICCA
It is a mild form of atrophic rhinitis which affects only anterior part of nasal cavities.
Etiopathology:
The exact cause of rhinitis sicca is unknown. It is more common in people having dry, hot
and dusty occupations. e.g., iron and gold smiths, bakers, oven workers. There is deficiency
and inactivity of seromucinous glands. Alcoholism, anemia, nutritional deficiencies and
constitutional diseases are found to be contributing factors.
Clinical features:
1. Discomfort and irritation in the nasal cavities.
2. Crusting, removal of which may lead to epistaxis.
Characteristic smell of atrophic rhinitis is absent in rhinitis sicca.
On examination;
1. Dry and whitish mucous membrane of nose.
2. Crusting limited to anterior part of nasal cavities especially septum. A case of rhinitis sicca
There may be ulceration of the anterior part of the nasal septum. Occasionally there
may be septal perforation.
Investigations:
These are the same as those for atrophic rhinitis.
Treatment:
1. Removal of underlying cause.
2. Correction of anemia and nutritional deficiencies.
3. Alkaline nasal douches and removal of crusts by suction is encouraged.
216
SELF ASSESSMENT
Q. 1. All are causes of rhinitis sicca except
a. anemia d. hot environment

b. dry environment e. humid environment


c. dusty environment
Q. 2. Following is contributing factor of rhinitis sicca.
a. Alcoholism d. All of the above

b. Constitutional diseases e. None of the above


c. Nutritional deficiency
Q. 3. All are usual clinical features of rhinitis sicca except
a. crusting brane
b. discomfort d. ozaena
c. dry and whitish nasal mucous mem- e. septal perforation
Q. 4. Rhinitis sicca is characterized by
a. crusts in anterior part of nasal cavities c. foul smell from the patient
d. large roomy nasal cavities
b. degenerative changes in entire nasal e. saddle nose
mucosa
Q. 5. All are included in the treatment of rhinitis sicca except
a. alkaline nasal douches d. removal of underlying cause

b. correction of anemia e. Young’s operation

c. correction of nutritional deficiencies


Scenario: A middle-aged man working in iron industry where environmental conditions are
dry, hot and dusty, presented with the complaint of crusting in nose and bleeding while
removing crusts. Crusts are limited to anterior part of nasal cavities.
1. What is your clinical diagnosis?
2. What is your treatment plan?
-------------------------------------------------------- ANSWERS-----------------------------------------------------
Q.1. e 2. d 3. d 4. a 5. e
Scenario: 1. Rhinitis sicca 2. Removal of underlying cause, correction of anemia and
nutritional deficiencies, alkaline nasal douches.
RHINOLITH
Definition:
It is the formation of concretions/stone around foreign body, blood clot or mucous in the
nasal cavity.
217 Part C Nose and Paranasal Sinuses

Etiopathology:
Rhinolith may be;
1. Exogenous (foreign bodies)
2. Endogenous (blood clot or inspissated mucopus)
The concretions are formed of calcium and magnesium phosphate and carbonate. These are
often arranged in concentric layers. With the passage of time, it tends to enlarge, filling the
whole nasal cavity. It may cause pressure necrosis of the nasal septum and lateral nasal wall.
Rhinolith may be friable or hard.
Clinical features:
1. Unilateral foul-smelling bloodstained nasal discharge is
pathognomonic of rhinolith. The discharge may be mucoid or
mucopurulent.
2. Unilateral nasal obstruction is a very common symptom. A case of rhinolith

On examination;
1. Greenish black mass with irregular surface is seen in the nasal cavity.
2. On palpation with a probe there is stony hard gritty sensation.
Investigations:
X-ray paranasal sinuses shows radiopaque shadow in the nasal cavity, whereas ipsilateral
group of paranasal sinuses may be hazy due to associated blockade of the sinus ostia.
Treatment:
Removal of rhinolith is carried out through anterior nares usually under general anesthesia.
Sometimes it may be necessary to break the rhinolith and remove it piecemeal. All the
granulations around must be removed thoroughly.
Occasionally lateral rhinotomy may be required for its removal.
SELF ASSESSMENT
Q. 1. All of the following salt may be present in rhinolith except
a. calcium carbonate d. magnesium carbonate

b. calcium oxalate e. magnesium phosphate


c. calcium phosphate
Q. 2. The most common presentation of rhinolith is
a. bilateral epistaxis d. swelling of nose
b. facial pain e. unilateral foul-smelling bloodstained
c. headache nasal discharge
Q. 3. All of the following are usual clinical features of rhinolith except
218

a. bilateral nasal bleeding d. gritty stony hard sensation on probe

b. foul smell from the patient test

c. greenish black mass in the nasal cavity e. unilateral nasal obstruction

Scenario: A 05 years old boy brought by mother with history of right sided foul-smelling
bloodstained nasal discharge and nasal obstruction for last 06 months. On probe test there is
gritty sensation.
1. What is the most likely diagnosis?
2. How will you confirm your diagnosis?
3. How will you manage this case?
-------------------------------------------------------- ANSWERS-----------------------------------------------------
Q.1. b 2. e 3. a
Scenario: 1. Rhinolith 2. X-ray paranasal sinuses shows radiopaque shadow in the nasal
cavity 3. Removal under general anesthesia
--------------------------------------------------------------------------------------------------------------------------
-

COMPLICATIONS OF RHINOSINUSITIS
Definition:
Complication of rhinosinusitis may be defined as spread of infection beyond the bony walls
of paranasal sinuses.
Etiopathology:
Complications may occur as a result of acute or chronic rhinosinusitis. The former is more
common in children and the latter in adults.
Complications may occur as a result of:
1. Closed rhinosinusitis: These occur when ostium of sinus is blocked. The complications
occur on the face, orbit or cranium.
2. Open rhinosinusitis: These occur when ostium of sinus is open. Infection from paranasal
sinuses spreads into nasopharynx, middle ear, larynx, trachea and bronchi.
3. Systemic effects of rhinosinusitis: These occur in the form of bacteremia, septicemia and
abscesses in different parts of the body.
Complications of rhinosinusitis may be divided into;
A. Local complications
B. Orbital complications
C. Intracranial complications
A. Local complications
1. Cellulitis of subcutaneous tissues overlying frontal, ethmoidal and maxillary sinuses may
219
occur.
2. Abscess of soft tissues may also occur rarely.
3. Osteomyelitis of the overlying bones may occur as a
result of sinus infection.
4. Mucocele, pyocele or Pott’s puffy tumor may also
occur very rarely. Pott’s puffy tumor is a rare clinical
entity characterized by subperiosteal abscess associated
with osteomyelitis. It is characterized by an osteomyelitis
of the frontal bone, either direct or through hematogenous
spread.
B. Orbital complications
1. Cellulitis of eyelids: Pott’s puffy tumor

It is usually caused by ethmoiditis. It is simple swelling and redness


of the soft tissue of the eyelids. It is of two types;
a. Preseptal cellulitis:
Inflammation does not spread beyond the orbital septum.
b. Postseptal cellulitis:
Inflammation extends beyond the orbital septum but is limited to
eyelid.
Clinical features:
The condition is dramatic, frightening and painful but is much less

dangerous than orbital cellulitis. Cellulitis of left eyelid


a. Eyelids are swollen, red and tender.
b. Conjunctiva is normal.
c. Eyeball is in its normal position.
d. Eye movements and vision is normal.
2. Extraperiosteal abscess:
There is abscess formation superficial to the periosteum of the orbit usually the lamina
papyracea. There is displacement of the eye forwards, laterally, downwards or upwards
depending on particular sinus involved.

3. Orbital cellulitis/abscess:
It is a very dangerous condition which may prove rapidly fatal
if cavernous sinus thrombosis and meningitis follow.

In this condition infection extends from the orbital perios-


A case of orbital cellulitis
teum to enter the orbital fat where it can spread between the
orbital muscles, nerves and blood vessels.
Clinical features:
220
a. There is severe pain around the eye.
b. There is tenderness on slight pressure over the eyeball.
c. Eyeball is bulging i.e., proptosed.
d. Eye lids are swollen but not red.
e. Conjunctiva is swollen and engorged.
f. There is progressive limitation of eye movements i.e., ophthalmoplegia.
g. There may be decrease in vision. Color vision particularly red color perception is
affected first. Ishihara chart is used for assessment of color vision. The condition
closely resembles cavernous sinus thrombosis but is distinguished by its unilaterality,
relatively less toxicity, presence of pain, tenderness and the absence of papilledema.
4. Superior orbital fissure syndrome:
Rarely infection can spread from sphenoid sinus to superior orbital fissure. In this syn-
drome there is deep orbital pain, frontal headache and progressive paralysis of 6th, 3rd
and 4th cranial nerves. Fifth cranial nerve is spared.
5. Orbital apex syndrome:
In this syndrome there is involvement of optic nerve and maxillary division of
trigeminal nerve in addition to nerves involved in superior orbital fissure syndrome.
Investigations:
High resolution CT/MRI in both axial and coronal planes is required to find out the exact site
and extent of the disease.
Treatment of orbital complication:
The aims of management are;
a. To control and eliminate the orbital complication.
b. To control and eliminate the primary rhinosinusitis.
a. Medical:
The likely causative organisms are Streptococcus pneumoniae, Staphylococcus aureus,
Moraxella catarrhalis and anaerobes.
1. Intravenous cephalosporins with metronidazole is the first choice.
2. Systemic steroids may be administered to accelerate the resolution of inflammatory
process.
3. Topical nasal decongestants e.g., 0.5% ephedrine, 2 drops 6 hourly help in drainage of
underlying sinuses.
4. In case of cavernous sinus thrombosis, anticoagulants are given in addition.
Consultants like ophthalmologist, neurosurgeon, pediatrician or neurologist must be
involved during the management.
221
b. Surgical:
Surgical treatment is given in patients who fail to improve with medical treatment for 24 - 48
hours. It is also given in those with subperiosteal or orbital abscess.
It may be considered into;
1. Surgery for rhinosinusitis:
The principle is drainage of any pus and establishing ventilation of the causative sinus.
The surgery is usually done endoscopically preferably by FESS.
2. Surgery for the complications identified:
Exact surgical procedure depends on the particular complication identified but it includes
establishing wide drainage of the complication jointly with the ophthalmologist. The
surgery may be done externally or endoscopically (FESS).
C. Intracranial complications
Sinogenic intracranial complications usually arise from frontal sinus infection. Usual
intracranial complications are:
1. Extradural abscess
2. Subdural abscess
3. Meningitis
4. Brain abscess
5. Cavernous sinus thrombosis
All these complications have been discussed previously in complications of suppurative
otitis media. Only cavernous sinus thrombosis needs special mentioning.
CAVERNOUS SINUS THROMBOSIS
Etiopathology:
It is a very dangerous condition which has got a very high mortality rate. Usual causative
organisms are Staphylococcus aureus, Streptococcus and Pneumococcus. The disease usually
starts as cavernous sinus thrombophlebitis and if there is delay in treatment or patient is
immunocompromised, it readily progresses to cavernous sinus thrombosis. It usually arises
from extension of infection from sphenoid sinus.
Spread of infection and thus, thrombosis of the cavernous sinus may also occur as a result
of infection in the danger area of the face i.e., the lower part of nasal pyramid, upper lip and
lower medial part of cheek. Thus, boil nose, septal abscess and maxillary sinusitis are the
most common causes of cavernous sinus thrombosis.
Differences between cavernous sinus thrombosis and orbital cellulitis
222 Part C Nose and Paranasal Sinuses

S. Features Cavernous sinus Orbital cellulitis


No. thrombosis
1 Laterality Usually, bilateral Usually, unilateral
Relatively more
2 Toxicity toxic Relatively less toxic
Usually,
3 Pain absent Usually, present
Usually,
4 Tenderness absent Usually, present
5 Papilledema Usually, present Usually, absent

Spread of infection from the danger area is by two routes;


1. From facial vein, angular vein and superior opthalmic vein to
the cavernous sinus.
2. From the pterygoid plexus, innominate vein to the cavernous
sinus.
Clinical features:
Almost all the clinical features are bilateral due to connection of the A case of cavernous
two cavernous sinuses with intercommunicating sinus. sinus thrombosis

Initially there is swelling, pain and tenderness in the upper medial part of cheek and upper lip.
Then there is diplopia due to paresis/paralysis of the nerves which pass through the
cavernous sinus (i.e., 6th nerve) and the nerves which pass in the lateral wall of the cavernous
sinus (i.e., 3rd, 4th, 5a and 5b).
There may be mild pain in the eyes along with sharp rise in body temperature, headache,
nausea and vomiting.
There is proptosis of the eyeballs. There is hyperemia and chemosis (swelling) of the
conjunctiva. This is followed by papilledema and visual loss.

Investigations:
1. MRI venogram confirms the diagnosis by showing absent venous flow from the
cavernous sinuses.
2. High resolution contrast CT of the brain shows a filling defect in cavernous sinus.

Treatment:
1. Patient is admitted in intensive care unit. Urgent treatment is started to prevent the
spread of infection and thrombosis.
2. Broad-spectrum antibiotics are started intravenously on urgent basis.
3. Anticoagulant like heparin is given to prevent spread of thrombus. Heparin is given in a
dose of 5000 units in bolus form. Then continuous infusion is given in a dose of 18 units/
223
kg/hour with monitoring of clotting profile.
4. Systemic steroids are given in high doses to reduce edema and to improve symptoms.
Underlying condition such as boil nose, septal abscess or infected sinus should be
addressed simultaneously.
Differential diagnosis:
1. Orbital cellulitis is a unilateral condition. There is no papilledema. The patient is
usually less toxic.
2. Orbital mucormycosis may also give rise to similar picture. The patient is usually
non toxic.
TOXIC SHOCK SYNDROME (TSS)
Occasionally patients with rhinosinusitis may develop systemic septic condition which is
known as toxic shock syndrome. It is a life-threatening complication of certain types of
bacterial infections.
It results from toxins produced by Staphylococcus aureus but the condition may also be
caused by toxins produced by group A Streptococci. It is characterized by nausea, vomiting,
purulent secretions, rash and hypotension. It is treated by removal of underlying cause,
antibiotics, adequate hydration and maintenance of blood pressure.
SELF ASSESSMENT
Q. 1. Acute sinusitis without complication is treated initially with
a. antral lavage (FESS)

b. antibiotics for two weeks d. inferior meatal antrostomy

c. functional endoscopic sinus surgery e. wait & see


Q. 2. All of the following are usual complications of sinusitis except
a. mucocele d. osteomyelitis

b. meningitis e. pyocele
c. meningocele
Q. 3. Acute infection of which of the following sinuses causes swelling of both up-
224 Part C Nose and Paranasal Sinuses

per and lower lids?

a. Ethmoid sinus d. Maxillary sinus

b. Frontal sinus e. Sphenoid sinus


c. Lateral sinus
Q. 4. All are features of cellulitis of eyelid except
a. conjunctiva is normal d. eyelids are swollen and red

b. eyeball is proposed e. vision is normal


c. eye movements are normal
Q. 5. Orbital complications most commonly follow infection of which of the following
sinuses
a. ethmoid sinus d. sphenoid sinus

b. frontal sinus e. superior sagittal sinus


c. maxillary sinus
Q. 6. Features of advanced orbital cellulitis include all except
a. conjunctiva is swollen d. eye movements are normal

b. eyeball is proposed e. visual loss


c. eyelids are swollen but not red
Q. 7. The most common complication of sinusitis is
a. cavernous sinus thrombosis d. Pott’s puffy tumor

b. meningitis e. subdural abscess


c. orbital cellulitis
Q. 8. The maximum number of cranial nerves affected in the orbital complication of
sinusitis is
a. cavernous sinus thrombosis d. orbital cellulitis

b. orbital abscess e. superior orbital fissure syndrome


c. orbital apex syndrome
Q. 9. Which cranial nerve is not involved in superior orbital fissure syndrome?
a. third cranial nerve d. sixth cranial nerve

b. fourth cranial nerve e. None of the above


c. fifth cranial nerve
Q. 10. Osteoma most commonly involves which of the following sinuses?
a. Anterior ethmoid d. Posterior ethmoid
b. Frontal e. Sphenoid
c. Maxillary
Q. 11. Pott’s puffy tumor is mostly related to
225 Part C Nose and Paranasal Sinuses

a. cavernous sinus thrombosis d. pyogenic infection of frontal sinus

b. infected cell in middle turbinate e. pyogenic infection of maxillary sinus


c. pyogenic infection of ethmoid sinus
Q. 12. All are intracranial complications of rhinosinusitis except
a. brain abscess d. meningitis

b. cavernous sinus thrombosis e. orbital cellulitis


c. extradural abscess
Q. 13. All are true about cavernous sinus thrombosis except
a. dangerous condition d. unilateral condition
b. high mortality rate e. usually develops through superior oph-
c. rapidly deteriorating condition. thalmic vein
Q. 14. The condition which usually causes bilateral proptosis is
a. cavernous sinus thrombosis d. mucormycosis

b. hypothyroidism e. orbital cellulitis


c. maxillary carcinoma
Q. 15. All are features of cavernous sinus thrombosis except
a. paralysis of 3rd cranial nerve d. paralysis of 6th cranial nerve

b. paralysis of 4th cranial nerve e. paralysis of 7th cranial nerve


c. paralysis of 5a & 5b cranial nerve
Q. 16. Investigation of choice for cavernous sinus thrombosis is
a. CBC d. USG neck

b. MRI venogram e. X-ray neck


c. plain CT
Q. 17. Treatment of cavernous sinus thrombosis includes all except
a. admission in ICU d. removal of thrombus

b. anticoagulant e. systemic steroids

c. broad-spectrum antibiotics
Scenario A: A 20 years old girl came to emergency department with history of nasal
blockage and discharge for last two weeks. She developed pain in right eye with proptosis,
impaired vision and ophthalmoplegia for last 03 days. Temperature is 99 degrees F. X-ray
paranasal sinus show haziness of ipsilateral sinuses.
1. What is the most likely diagnosis?
2. What investigations you will advise?
3. What is the treatment of this patient?
B: A 25 years old lady presented with throbbing pain and swelling on left cheek area for last

CONCEPT BOOK OF DISEASES OF EAR, NOSE & THROAT BY FAHIM AWAN


226 Part C Nose and Paranasal Sinuses

one week. Anterior rhinoscopy revealed congestion of nasal mucosa and pus in the middle
meatus. Pain and swelling is not being relieved by ordinary treatment.
1. What is the clinical diagnosis?
2. What are the ways of confirmation of the diagnosis?
3. Which surgical procedure would you like to adopt?
4. Which usual complications can occur from the disease?
--------------------------------------------------------ANSWERS-----------------------------------------------------
Q.1. b 2. c 3. a 4. b
5. a 6. d 7. c 8. c 9. c 10. b
11. d 12. e 13. d 14. a 15. e 16. b
17. d
Scenario A: 1. Orbital abscess 2. HRCT with contrast 3. a. Medical: antibiotics, steroids,
topical decongestants. b. Surgical: surgery for rhinosinusitis i.e., antral washout (AWO),
surgery for orbital abscess.
B: 1. Left acute maxillary sinusitis 2. X-ray PNS, CT scan of nose and PNS
3. Antral wash out 4. Orbital cellulitis/abscess, orbital apex syndrome, cavernous sinus
thrombosis, toxic shock syndrome.
--------------------------------------------------------------------------------------------------------------------------
-
ANTRAL LAVAGE
Synonyms: Antral washout, Proof puncture
Indications:
Diagnosis and treatment of acute and subacute sinusitis, where
medical treatment has failed.
Contraindications:
1. Patient less than 03 years of age because yet maxillary sinus is
not fully developed. Antral lavage
2. Thick bony wall
3. Acute maxillary sinusitis untreated by antibiotics
4. Previous trauma which may have disrupted the orbital floor

Procedure:
Reverse Trendelenburg position with 15 degrees head flexion is
made. A pack soaked in 4% xylocaine with adrenaline is placed in the
inferior meatus. After 20 minutes, pack is removed. Tilley Lichtwitz
trocar and cannula is taken and inferior meatus is punctured with the above Tilley Lichtwitz trocar
instrument at a distance of half an inch behind the anterior end of inferior and cannula
turbinate as high as possible.
227
The direction of trocar and cannula is kept outer to the
lateral canthus of the ipsilateral eye. Then the trocar is
removed and aspiration of the sinuses is done with cannula
for any secretions or pus and any aspirate is sent for
microscopy, culture and sensitivity. Sinuses are washed with
normal saline and continuous watch is made for any
proptosis or swelling of cheek. Then the cannula is removed.

Complications:
1. Hemorrhage
2. Air embolism
Antral lavage
3. Penetration of the orbital floor and proptosis
4. Penetration of soft tissues of cheek and cellulitis
INFERIOR MEATAL ANTROSTOMY
Synonyms: Intranasal antrostomy, Antronasal antrostomy, Nasoantral antrostomy
Indications:
1. Acute, recurrent and subacute sinusitis which has failed to respond to conservative
medications and usually one or two antral washouts.
2. Chronic sinusitis

Contraindications:
1. Acute maxillary sinusitis without a cover of antibiotics.
2. Any systemic disease such as diabetes, hypertension etc. in which general
anesthesia or surgery is contraindicated.
Procedure:
It is usually carried out under general anesthesia but may also be performed
under local anesthesia. Reverse Trendelenburg position with 15 degrees head Myle’s
retrograde
flexion is made. Throat (epilaryngeal) pack is placed. Inferior turbinate is elevated antrum
perforator
upwards with a Hills’s elevator then the inferior meatus is punctured with Myle’s
retrograde antrum perforator at the same site which is used for
antral washout. Myle’s perforator is removed and the same hole is
enlarged with different instruments to make a window of at least 2
cm x 1 cm in size. Edges of the antrostomy are made smooth. Any
disease from the maxillary sinus is removed via antrostomy. The
inferior turbinate is repositioned at the end of the procedure.
Throat pack (also known as epilaryngeal pack) is removed.
Complications:
1. Hemorrhage (through inferior meatal branch of lateral
sphenopalatine artery)
2. Damage to anterior superior alveolar nerve leading to
altered dental sensation Inferior meatal antrostomy
228
3. Complete closure of antrostomy hole (if the window is made smaller).
CALDWELL-LUC’S OPERATION
Synonym: Sublabial antrostomy, Radical antrostomy
Indications:
1. Chronic maxillary sinusitis to remove
irreversibly damaged mucosal lining of maxillary
sinus and to facilitate the aeration and gravitational
drainage via an inferior meatal antrostomy.
2. Antrochoanal polyp over the age of 12 years
3. Removal of foreign body from maxillary sinus Caldwell-Luc’s operation

4. Closure of oroantral fistula


5. Dental cyst
6. Stabilization of orbital floor fracture
7. Stabilization of fracture of body of zygoma
8. As an approach to pterygopalatine fossa e.g.,
a. Ligation of maxillary artery
b. Vidian neurectomy
Contraindications:
1. Children less than 12 years of age as damage to the secondary dentitions may occur.
2. Disfigurement of facial skeleton.
Procedure:
It is usually performed under general anesthesia. Reverse
Trendenlenberg position with 15 degrees head flexion is made. Throat Mallet/hammer
(epilaryngeal) pack is placed. Incision is made down to the bone in the
labiogingival sulcus.
The incision starts from lateral border of lateral incisor to first molar
tooth. Mucoperiosteal flap is elevated. A window of 1.5 cm diameter is
made
with the help of hammer and gouge well above the roots of the teeth. A
large inferior meatal antrostomy 2 cm x1 cm in size is made. Procedure is
completed and wound is closed in layers with loose stitches. Throat pack Jenken gouge
(also known as epilaryngeal pack) is removed.
Complications:
1. Hemorrhage
2. Soft tissue swelling
3. Pain
229
4. Paraesthesia of the upper part of cheek due to damage to infraorbital nerve.
5. Alteration in dental sensation and discoloration of the teeth due to damage to teeth
apices and their innervation.
6. Oroantral fistula
SELF ASSESSMENT
Q. 1. Antral lavage is also known as
a. antral washout d. Both a & b

b. inferior meatal antrostomy e. Both a & c


c. proof puncture
Q. 2. All are indications of antral lavage except
a. acute sinusitis d. Both a & b

b. chronic sinusitis e. Both a & c


c. subacute sinusitis
Q. 3. All are contraindications of antral lavage except
a. acute rhinosinusitis untreated by an- c. disrupted orbital floor
tibiotics d. patient less than 3 years of age
b. acute rhinosinusitis where medical e. thick bony wall
treatment has failed
Q. 4. Usual complications of antral lavage include
a. air embolism d. penetration of soft tissue of cheek

b. hemorrhage e. all of the above


c. penetration of orbital floor
Q. 5. Inferior meatal antrostomy is also called
a. antronasal antrostomy d. all of the above

b. intranasal antrostomy e. none of the above


c. nasoantral antrostomy
Q. 6. Indication of inferior meatal antrostomy is
a. acute rhinosinusitis d. subacute rhinosinusitis

b. chronic rhinosinusitis e. all of the above


c. recurrent rhinosinusitis
Q. 7. Contraindication of inferior meatal antrostomy is
a. acute maxillary sinusitis without a cover c. hemostatic diathesis
of antibiotics d. all of the above
b. any systemic disease such as hypertension e. none of the above
and diabetes
230
Q. 8. Caldwell-Luc operation is also known as
a. inferior meatal antrostomy d. both a & b

b. radical antrostomy e. both b & c


c. sublabial antrostomy
Q. 9. Indication of Caldwell-Luc operation is
a. chronic maxillary sinusitis d. removal of foreign body from maxillary
b. closure of oroantral fistula sinus
c. dental cyst e. all of the above
Q. 10. Indication of Caldwell-Luc antrostomy is
a. approach to pterygopalatine fossa d. stabilization of body of zygoma fracture
b. recurrent antrochoanal polyp over the
age of 12 years e. all of the above
c. stabilization of orbital floor fracture
Q. 11. Contraindication of Caldwell-Luc operation is
a. approach to pterygopalatine fossa d. children over 12 years of age

b. children less than 12 years of age e. dental cyst


c. chronic maxillary sinusitis
Q. 12. Possible complication of Caldwell-Luc operation is
a. dental anesthesia d. paraestheisa of cheek

b. hemorrhage e. all of the above


c. oroantral fistula
Q. 13. In Caldwell-Luc operation, entry into the maxillary sinus is made through
a. canine fossa d. middle meatus

b. incisive fossa e. nasal cavity


c. maxillary alveolus
Q. 14. All are true of oroantral fistula except
a. antrum is often infected d. most important cause is dental extraction
b. food regurgitates into the nose e. treated after Caldwell-Luc operation
c. liquid food can be sipped easily with
straw
Scenario: A 30 years old patient presents with regurgitation of fluids through nasal cavity
following extraction of last upper premolar tooth three days ago.
1. What is the most probable clinical diagnosis?
2. How will you confirm the diagnosis?
3. How will you treat this patient?
Part C Nose and Paranasal
231 Sinuses
---------------------------------------------- ANSWERS-----------------------------------
---------- ------------------
Q.1. e 2. b 3. b 4. e 5. d 6. e 7. d
8. e 9. e 10. e 11. b 12. e 13. a 14. c
Scenario: 1. Oroantral fistula 2. Methylene blue dye test 3. Caldwell-Luc operation, flap
reconstruction.
--------------------------------------------------------------------------------------------------------------------------
-
ETHMOIDECTOMIES
There are different types of ethmoidectomies;
A. Intranasal ethmoidectomy
B. External ethmoidectomy (Lynch Howarth procedure)
C. Transantral ethmoidectomy (Jansen Horgan procedure)
A. Intranasal ethmoidectomy:
Indications:
Chronic sinusitis in association with nasal polyps.
Contraindication:
Only seeing eye
Procedure:
Intranasal ethmoidectomy
It may be performed under local or general anesthesia.
Reverse Trendelenburg position with 15o head flexion is made. Throat (epilaryngeal) pack is
placed. Inferior turbinate is outfractured and ethmoidal air cells are exenterated with Tilley
Henckel’s forceps.
The surgeon should consistently watch for the appearance of any yellow orbital fat. This
is confirmed by placing the removed tissue in a bowl of saline. The fat floats on the surface of
the saline, whereas other tissues sink in the saline. The displaced turbinate is repositioned and
anterior nasal packing done. Throat pack (also known as epilaryngeal pack) is removed.
Complications:
1. Injury to lamina papyracea with periorbital hematoma
2. Proptosis
3. Visual loss
4. Injury to dura leading to CSF rhinorrhoea
B. Transantral ethmoidectomy:
Synonym: Jansen Horgan’s procedure
Indications:
232
1. Chronic inflammation affecting the maxillary as well as ethmoidal sinus.
2. Orbital decompression
Contraindication:
Only seeing eye
Procedure:
After general anesthesia, reverse Trendelenburg position with 15o head flexion is made.
Throat (epilaryngeal) pack is placed. After Caldwell-Luc’s procedure, closed Tilley
Henckel’s forceps is pushed upwards, backwards and medially at the upper and inner angle of
the antrum towards the opposite parietal eminence. Posterior and middle ethmoidal air cells
are exenterated. Anterior nasal packing done.
Throat pack (also known as epilaryngeal pack) is removed.
Complications:
These are similar to those of Caldwell-Luc’s procedure and intranasal ethmoidectomy.
C. External ethmoidectomy:
Synonyms: External frontoethmoidectomy, Lynch Howarth procedure
Indications:
1. Chronic infection of the frontal and ethmoidal sinuses
2. Orbital decompression and drainage in cases of orbital cellulitis
3. Frontal sinus mucocele
4. Ligation of anterior ethmoidal artery for epistaxis
5. Trans-ethmoidal hypophysectomy
6. Dacryocystorhinostomy
7. Repair of CSF leak
8. Decompression of malignant exophthalmos
Procedure:
After general anesthesia, reverse Trendelenburg position with
15o head flexion is made. Throat (epilaryngeal) pack is placed. External ethmoidectomy
A curvilinear incision is made in the skin medial to medial
canthus of
eye. Periosteum is divided and elevated with periosteum elevator.
Flaps and lacrimal sac retracted. Anterior ethmoidal vessels are
ligated. Medial wall of the orbit is perforated and ethmoidal air cells are
exposed which are exenterated under direct vision. Medial floor of
frontal sinus is also breached. A fenestrated silastic tube 01 cm in
Tilley Henckel forcep
diameter is placed to maintain the patency of frontal sinus. The periosteum,
subcutaneous tissue and skin are sutured in layers. Throat pack (also known
as epilaryngeal pack) is removed.
233 Part C Nose and Paranasal Sinuses

Complications:
1. Hemorrhage
2. Damage to medial palpebral ligament
3. Damage to dura
4. Damage to orbit
5. Visual loss
6. Mucocele formation
SELF ASSESSMENT
Q. 1. Surgical approach to ethmoid may be

a. external approach d. transantral approach

b. FESS e. all of the above

c. intranasal approach

Q. 2. Indication of intranasal ethmoidectomy is


a. acute ethmoiditis d. subacute ethmoiditis

b. chronic ethmoiditis e. both a & b


c. only seeing eye
Q. 3. All are complication of intranasal ethmoidectomy except
a. injury to lamina papyracea d. oroantral fistula

b. injury to dura e. visual loss


c. orbital hematoma
Q. 4. Transantral ethmoidectomy is also called
a. external frontoethmoidectomy d. Paterson’s procedure

b. Jansen Horgan’s procedure e. all of the above


c. Lynch Howarth procedure
Q. 5. Indication of transantral ethmoidectomy is
a. chronic maxillary and ethmoidal si-nusitis c. repair of orbital floor fracture
b. orbital decompression d. both a & b
e. both b & c
Q. 6. Contraindication of transantral ethmoidectomy include
a. chronic maxillary and ethmoidal si-nusitis
b. only seeing eye
c. orbital decompression
d. all of the above
e. none of the above
234
Q. 7. Complication of transantral ethmoidectomy
a. injury to lamina papyracea d. visual loss

b. injury to dura e. all of the above


c. orbital hematoma
Q. 8. External ethmoidectomy is also called
a. external frontoethmoidectomy d. both a & b

b. Jansen Horgan’s procedure e. both a & c


c. Lynch Howarth procedure
Q. 9. Indication of external ethmoidectomy includes
a. anterior ethmoidal artery ligation d. orbital decompression

b. chronic frontal & ethmoidal sinusitis e. all of the above


c. frontal sinus mucocele
Q. 10. Indication of external ethmoidectomy includes
a. Dacryocystorhinostomy c. Repair of CSF leak
b. Decompression of malignant exoph- d. Transethmoidal hypophysectomy
thalmos e. all of the above
Q. 11. Complication of external ethmoidectomy includes:
a. damage to dura d. All of the above

b. damage to medial palpebral ligament e. None of the above


c. hemorrhage
Q. 12. Complication of external ethmoidectomy includes
a. damage to orbit d. all of the above

b. mucocele formation e. none of the above


c. visual loss
Q. 13. While performing external ethmoidectomy, anterior ethmoidal artery is located at
frontoethmoidal suture line. Its distance from anterior lacrimal crest is
a. 12 mm d. 30 mm

b. 18 mm e. 36 mm
c. 24 mm
Q. 14. Surgically frontal sinus can be approached through any of the following incisions
except
a. coronal d. sublabial

b. eyebrow e. Weber Ferguson

c. medial canthal
-------------------------------------------------------- ANSWERS-----------------------------------------------------
Part C Nose and Paranasal
235 Sinuses
Q.1. e 2. b 3. d 4. b 5. d 6. b 7. e
8. e 9. e 10. e 11. d 12. d 13. c 14. d
--------------------------------------------------------------------------------------------------------------------------
-
GRANULOMATOUS DISEASES OF NOSE AND PARANASAL
SINUSES
Definition:
A granuloma is an organized collection of macrophages (epithelial cells) which tend to fuse
to form multinucleated giant cells.
Etiopathology:
Granulomatous diseases are divided into;
A. Nonspecific granulomas: These are the granulomas in which specific causative organism
is unknown e.g., Wegener’s granuloma, Stewart’s granuloma and sarcoidosis.
B. Specific granulomas: These are the granulomas in which specific causative organism is
known e.g., tuberculosis, lupus, leprosy, syphilis, mucormycosis, rhinoscleroma,
rhinosporidiosis etc.
A. Nonspecific granulomas:
These are the granulomas in which specific causative organism is unknown e.g., Wegener’s
granuloma, Stewart’s granuloma and sarcoidosis.
WEGNER’S GRANULOMA
Synonyms: Granulomatosis with polyangitis (GPA)

Etiopathology:
It is a systemic granulomatous disease of unknown etiology. It is thought to be a
hypersensitivity disorder. There is necrotizing vasculitis of small and medium sized
vessels. There is also focal and proliferative glomerulonephritis.
It primarily affects;
1. Nose and paranasal sinuses
2. Lower respiratory tract
3. Kidneys A case of
Nose and paranasal sinuses are involved in 80% of cases. But it may affect any Wegner’s
organ like palate, orbit and middle ear. In untreated cases, renal failure may occur. granuloma
Death occurs within six months.

Clinical features:
1. Nasal obstruction, bloodstained nasal discharge and crusting.

CONCEPT BOOK OF DISEASES OF EAR, NOSE & THROAT BY FAHIM AWAN


236
2. Cough and Haemoptysis due to involvement of lungs.
3. Hematuria due to involvement of kidneys.
4. There may be complaint of malaise, pyrexia and weight loss.
On examination there may be;
1. Saddle nose
2. Granulomatous lesions in the nose and sinuses is the usual finding.
3. Ulceration of nasal mucosa with crust formation.
4. Perforation of nasal septum and palate may also be present.

Investigations:
1. (cytoplasmic-Anti Neutrophil Cytoplasmic Antibodies) test is diagnostic of Wegener’s
granuloma. It is positive in 95% of patients with Wegener’s granuloma.
2. Biopsy is important for tissue diagnosis. On histology there is epithelioid necrotizing
granulomatous inflammation, vasculitis and necrosis.
3. Blood urea and creatinine should also be estimated to detect renal involvement.
4. Urinalysis may show red cells, casts and albumin. There may be decreased creatinine
clearance and serum creatinine level is increased.
5. ESR is markedly increased. Regular estimation of ESR may monitor the success or
otherwise of treatment.
6. Chest X-ray may show localized areas of infarction which may proceed to cavity
formation.
7. CT/MRI is required to find out exact extent of lesion.
Some investigations may be required for the purpose of differential diagnosis.
Treatment:
1. Steroids are given at a dose 60-80 mg/day. It shows rapid improvement in symptoms.
2. Cytotoxic drugs such as cyclophosphamide 2 mg/kg body weight or azathioprine 3 mg/
kg body weight is necessary for long term control.
3. Nasal symptoms are managed by topical preparations including steroids, 25% glucose in
glycerine drops and alkaline/saline nasal douches.
4. Saddle nose and septal perforation are treated by plastic surgery after complete cure.
SELF ASSESSMENT
Q. 1. Wegener’s granuloma is also called
a. granulomatosis with polyangitis b. lethal midline granuloma

(GPA)
c. midfacial lymphoma
237 Part C Nose and Paranasal Sinuses
d. midline destructive granuloma e. Stewart’s granuloma
Q. 2. All are true regarding Wegener’s granuloma except
a. granulomatous disease of nose, lungs c. systemic granulomatous disease
and kidneys d. unknown etiology
b. non-specific granuloma e. viral etiology

Q. 3. Histological features of Wegener’s granuloma include


a. focal glomerulonephritis vessels
b. necrotizing vasculitis of medium sized d. proliferative glomerulonephritis
vessels e. all of the above
c. necrotizing vasculitis of small sized
Q. 4. Wegener’s granuloma primarily affects all except
a. heart d. nose and paranasal sinuses

b. kidneys e. palate and orbit


c. lower respiratory tract
Q. 5. Usual clinical features of Wegener’s granuloma include all except
a. bloodstained nasal discharge d. hematuria

b. cough and hemoptysis e. nasal obstruction


c. hematemesis
Q. 6. Common clinical features of Wegener’s granuloma include all except
a. cervical lymphadenopathy c. perforation of palate
b. granulomatous lesions in nose and d. perforation of septum
sinuses e. saddle nose
Q. 7. Diagnostic investigation of Wegener’s granuloma is
a. blood urea nuses

b. c-ANCA test d. ESR

c. CT scan of nose and paranasal si- e. serum creatinine


Q. 8. Investigation of Wegener’s granuloma includes all except
a. biopsy d. p-ANCA

b. chest X-ray e. urine analysis


c. CT scan
Q. 9. All are features of urine analysis in Wegener’s granuloma except
a. albumin d. red cells

b. casts e. white cells


c. proteinuria
Q. 10. Treatment of Wegener’s granuloma includes all except:
238 Part C Nose and Paranasal Sinuses

a. Azathioprine d. Radiotherapy

b. Cyclophosphamide e. Steroids
c. Nasal douching
Q. 11. Death in Wegener granulomatosis is usually due to
a. cardiac failure d. renal failure

b. hepatic failure e. respiratory failure

c. multiple organ failure


Scenario: A 30 years old patient presents with bloodstained nasal discharge, hemoptysis and
hematuria for last 03 months. On examination there is granular lesion in right nasal cavity.
His ESR is 60 mm in first hour.
1. What is the most probable clinical diagnosis?
2. What is the differential diagnosis?
3. How will you confirm your diagnosis?
------------------------------------------------------
-- ANSWERS-----------------------------------------------------
Q.1. a 2. e 3. e 4. a 5. c 6. a
7. c 8. d 9. e 10. d 11. d
Scenario: 1. Wegener’s granulomatosis 2. Stewart’s granuloma, sarcoidosis, tuberculosis
3. c-ANCA test, biopsy.
--------------------------------------------------------------------------------------------------------------------------
STEWART’S GRANULOMA
Synonyms: Midline destructive granuloma, Midline lethal granuloma,
Natural killer lymphoma, Midfacial lymphoma, Angiocentric lymphoma, T-cell
lymphoma
Etiopathology:
Exact etiology of Stewart’s granuloma is unknown. It is a lymphoma
but usually presents with midface destruction. It can occur at any age. It
most commonly affects fifth to sixth decade.
Clinical features:
These are similar to those of Wegner’s granuloma but there is no
pulmonary, renal or other systemic involvement. It is characterized by A case of
slow progressive destruction of nose and midfacial region. Stewart’s
granuloma

Investigations:
1. Biopsy of the lesion is diagnostic. Histopathology shows polymorphic lymphoid tissue
with angiocentric and angioinvasive features. There is no vasculitis.
239
2. CT/MRI is required to find out exact extent of the disease.
Treatment:
Surgical debridement with full course of local radiotherapy is given in a dose of
5000 to 5500 centigrays.
Chemotherapy is added in cases of recurrent or disseminated disease.
Surgical reconstruction or prosthesis is applied when the disease subsides.
SELF ASSESSMENT
Q. 1. Stewart’s granuloma is also called
a. midfacial lymphoma d. all of the above

b. midline destructive granuloma e. None of the above

c. natural killer lymphoma


Q. 2. Stewart’s granuloma is also called

a. angiocentric lymphoma d. all of the above

b. midline lethal granuloma e. none of the above


c. T-cell lymphoma
Q. 3. All of the following are histological features of midline lethal granuloma except
a. angiocentric features phritis

b. angioinvasive features. d. no vasculitis

c. focal and necrotizing glomerulone- e. polymorphic lymphoid tissue


Q. 4. Usual clinical feature of midline lethal granuloma is
a. hematuria d. renal involvement
b. hemoptysis e. slow progressive destruction of nose
c. pulmonary involvement and midfacial region
Q. 5. Investigation of choice in midline lethal granuloma is
a. angiography d. p-ANCA test

b. biopsy e. ultrasound
c. c-ANCA test
Q. 6. Treatment of choice in midline lethal granuloma is
a. azathioprine d. steroids

b. cyclophosphamide e. surgery

c. radiotherapy
Scenario: A 50 years old patient presents with slow progressive destruction of nose and
midfacial region for last 02 years. Biopsy of the lesion shows polymorphic lymphoid tissue
with angiocentric and angioinvasive features.
240
1. What is the most probable clinical diagnosis?
2. What is the differential diagnosis?
3. How will you manage this patient?
------------------------------------------------------
-- ANSWERS-----------------------------------------------------
Q.1. d 2. d 3. c 4. e 5. b 6. c
Scenario: 1. Stewart’s granuloma 2. Wegener’s granuloma, sarcoidosis, leprosy
3. Surgical debridement with full course of local radiotherapy, chemotherapy is added in
cases of recurrent or disseminated disease.
--------------------------------------------------------------------------------------------------------------------------
SARCOIDOSIS
Synonym: Besnier Boeck disease, Boeck sarcoid
Etiopathology
It is a chronic non-caseating systemic granulomatous disease of unknown etiology. On
histology it resembles tuberculosis but there is no caseation. It is a multi system disease
primarily affecting lower respiratory tract but may affect upper respiratory tract as well.
It may affect any organ or system of the body such as lymph nodes, nose, eyes, skin,
lungs, liver, spleen and small bones of feet. It affects young adults between third and fifth
decade with a female dominance of 2:1.
Clinical features:
1. Serosanguinous or mucopurulent nasal discharge are common
complaints.
Strawberry skin
2. There may be nasal obstruction, crusting and epistaxis. in sarcoidosis
On examination;
1. Skin lesions are common with a purple discoloration known as lupus pernio. Skin or
mucosa may show bluish red tiny 01 mm nodules (strawberry appearance). It is seen
on the nose, ears, cheeks, lips, and forehead. It is pathognomonic of sarcoidosis.
2. Nasal mucosa may be dry, ulcerated and covered with crusts.
3. There may be anterior septal perforation.
Salivary gland swelling associated with sarcoidosis and uveitis is known as Heerfordt
syndrome.
Investigations:
No single test is pathognomonic for sarcoidosis. Diagnosis depends on combination of
clinical features, hematology, histology and imaging.
1. Serum angiotensin converting enzyme (ACE) is raised. It is secreted by epithelioid
granuloma in active sarcoidosis.
241
2. Biopsy of the lesion shows non-caseating epithelioid granuloma. Giant cells may be
present which contain asteroid intracytoplasmic inclusion bodies (Schaumann bodies).
3. Kveim-Siltzback skin test is positive.
4. ESR and serum immunoglobulin particularly IgG are raised.
5. Serum and urinary calcium is raised.
6. Full blood count may show anemia, leukopenia, thrombocytopenia and eosinophilia.
7. X-ray chest shows diffuse infiltrate in lungs with hilar lymphadenopathy.
8. X-rays shows cystic punched out lesions in nasal bones, hands and feet.
9. CT of nose and paranasal sinuses shows exact extent of disease.
10. Gallium-67 scan is very helpful in localizing involved lymph nodes.
Treatment:
There is no specific treatment for sarcoid. Spontaneous remission has been observed in some
patients with limited disease.
1. Systemic steroids are the mainstay of treatment. Steroids may be given intralesionally or
systemically to relieve symptoms
2. 25% glucose in glycerine drops and nasal douches may relieve nasal symptoms.
3. Cosmetic and reconstructive surgery is carried only when the disease has subsided
completely.

SELF ASSESSMENT
Q. 1. Sarcoidosis is also known as

a. Besnier Boeck disease d. Both a & b

b. Boeck sarcoid e. Both b & c


c. Hansen’s disease
Q. 2. Which statement is true about sarcoidosis?
a. affects only lower respiratory tracts d. nonspecific granuloma

b. affects only upper respiratory tracts e. specific granuloma


c. caseating granuloma
Q. 3. Which statement is true about sarcoidosis?
a. Chronic subepithelial inflammatory d. Infection with Mycobacterium
granuloma of upper respiratory tract. e. Non caseating granuloma with histo-
b. Fungal infection of ear logical picture similar to tuberculosis
c. Fungal infection of nasal mucosa
Q. 4. All are usual clinical features of sarcoidosis except
242 Part C Nose and Paranasal Sinuses

a. apple jelly nodules d. nasal obstruction

b. epistaxis e. strawberry appearance

c. mucopurulent nasal discharge


Q. 5. Clinical feature of sarcoidosis is

a. anterior septal perforation c. lupus pernio

b. bluish red nodules on nasal skin or d. strawberry skin

mucosa e. all of the above


Q. 6. Investigation of sarcoidosis includes
a. biopsy d. urinary & serum calcium level

b. Kveim-Siltzback skin test e. all of the above


c. serum angiotensin converting enzyme
Q. 7. Treatment of sarcoidosis includes all except
a. cosmetic & reconstructive surgery d. topical steroids

b. radiotherapy e. 25% glucose in glycerin


c. systemic steroids
Q. 8. Salivary gland swelling associated with sarcoidosis and uveitis is known as
a. Heerfordt syndrome d. Sjogren’s syndrome

b. Mickulicz syndrome e. Young’s syndrome

c. Sicca syndrome
Scenario: A 40 years old woman presented with the complaints of mucopurulent nasal
discharge, nasal obstruction and crusting for last two years. Histology of lesion seems like
tuberculosis but there is no caseation.
1. What is the most probable clinical diagnosis?
2. How will you investigate this case?
3. How will you manage this patient?
----------------------------------------- ANSWERS------------------------------------------
--------------- -----------
Q.1. d 2. d 3. e 4. a 5. e 6. e 7. b 8. a
Scenario: 1. Sarcoidosis 2. Serum ACE level, biopsy, Kevium Siltzbech skin test, ESR, urine
calcium 3. Systemic steroids, nasal douches.
--------------------------------------------------------------------------------------------------------------------------
-
B. Specific granulomas:
These are the granulomas in which specific causative organism is known e.g., tuberculosis,
lupus, leprosy, mucormycosis, syphilis, etc.
TUBERCULOSIS
243
Etiopathology:
It is a systemic granulomatous disease caused by acid-fast
bacillus Mycobacterium tuberculosis. Tuberculosis in the head
and neck region is often secondary to pulmonary tuberculosis. Scrofula derma

In the neck it affects lymph nodes and patient usually presents with cold abscess or
discharging sinus known as scrofula derma.
In the oral cavity or pharynx, it may present as a stellate shaped ulcer with overhanging
edges. In the larynx it usually affects only the posterior part.

Clinical features:
1. Bloodstained nasal discharge and crusting are common
symptoms. On examination;
2. Granulomatous lesion involving anterior part of nasal cavity.
3. There may ulceration and perforation of cartilaginous portion of nasal septum.

Investigations:
1. ESR is usually elevated.
2. FNAC of neck swelling may suggest tuberculosis.
3. Sputum for AFB for three consecutive days is usually positive.
4. X-ray chest may be done to find out pulmonary focus.
5. Microscopy of discharge after ZN (Ziehl-Neelsen) staining and culture of organism
in LJ (Lowenstein-Jensen) medium may be helpful.
6. Biopsy of the lesion shows caseating epithelioid granuloma with Langhan’s giant cells.
7. P.C.R. (Polymerase Chain Reaction) for Mycobacterium tuberculosis is conclusive.
8. T spot T.B. test is highly specific for tuberculosis. It is in vitro diagnostic test for the
detection of effector T cells that respond to stimulation by Mycobacterium tuberculosis
antigens.
9. MycoDot test is a new simple, rapid (20 minutes) and reliable serodiagnostic technique
that can detect anti-mycobacterial antibodies in the serum or blood. It offers a low cost, single
visit aid in the diagnosis of tuberculosis, with good sensitivity and excellent specificity.

Treatment:
Antituberculous drugs are given as for pulmonary tuberculosis.
SELF ASSESSMENT
Q. 1. Tuberculosis is
a. caused by alkali-fast bacilli d. nonspecific granuloma

b. caused by Mycobacterium leprae e. specific granuloma


c. non caseating granuloma
Q. 2. All are usual clinical features of tuberculosis except
244 Part C Nose and Paranasal Sinuses

a. bloodstained nasal discharge d. perforation of bony portion of nasal

b. cervical lymphadenopathy septum

c. nasal crusting e. scrofula derma

Q. 3. All are investigations of tuberculosis except


a. ESR staining

b. FNAC of neck swelling d. shick test

c. microscopy of discharge after ZN e. X-ray chest


Q. 4. All are investigations of tuberculosis except
a. biopsy showing Langhans giant cells d. mycoDot test

b. culture of organism in L J medium e. PCR for Mycobacterium tuberculosis

c. monospot test
Scenario: A young male complains of nasal discharge and crusting for last 01 year. Anterior
rhinoscopy shows lesion limited to anterior part of nasal cavity with perforation of
cartilaginous part of nasal septum. Patient also gives history of pulmonary tuberculosis.
1. What is your clinical diagnosis?
2. What investigations will you carry out?
3. How will you manage this patient?
-------------------------------------------------------- ANSWERS-----------------------------------------------------
Q.1. e 2. d 3. d 4.c
Scenario: 1. Tuberculosis of nose 2. ESR, biopsy, PCR. 3. Antituberculous drugs.
--------------------------------------------------------------------------------------------------------------------------
LUPUS VULGARIS
It is a chronic indolent form of tuberculosis which affects only skin
and mucous membranes. Mucocutaneous junction of nasal septum is
the most common affected site. There is formation of apple jelly
nodules on the skin.
Investigations and treatment are the same as for pulmonary Apple jelly nodules
tuberculosis.
SELF ASSESSMENT
Q. 1. Which statement is not true about lupus vulgaris?
a. Affects only skin b. Affects mucocutaneous junction
c. An indolent form of tuberculosis d. Apple jelly nodules on the skin
e. Treatment is the same as for pulmo-
nary tuberculosis
--------------------------------------------------------ANSWERS-----------------------------------------------------
Q.1.a
245
--------------------------------------------------------------------------------------------------------------------------
LEPROSY
Synonym: Hansen’s disease
Etiopathology:
It is a chronic granulomatous disease caused by acid-fast bacillus Mycobacterium leprae.
There are following types of leprosy;
1. Tuberculoid leprosy: It is a localized disease. There are anesthetic cutaneous patches.
There may be isolated cranial nerve paralysis.
2. Lepromatous leprosy: It is a systemic disease. There is diffuse infiltration of skin,
nerves and mucosal surfaces. The parts of the body where temperature is low e.g., edges
of pinna, chin, nose and eyebrows are usually affected.
3. Borderline leprosy: This type presents midway between the above two types. The first
two types are immunologically stable.
Clinical features:
1. Bloodstained nasal discharge
2. Nasal obstruction is a common presenting symptom.
On examination;
1. Skin may show painless ulceration because of nerve involvement
causing analgesia.

A case
of leprosy

2. Nodular thickening of the nasal mucosa.


3. There is perforation of the cartilaginous septum. There may also be destruction of
anterior nasal spine with retraction of columella.
4. There may also be saddle nose and manifestations of atrophic rhinitis.
Investigations:
1. Microscopy of the nasal discharge or scrapings of nasal mucosa usually
reveal Mycobacterium leprae.
2. Biopsy of the lesion is usually diagnostic. Acid-fast leprae bacilli can be seen in the
foamy appearing histiocytes called lepra cells.
Treatment:
1. Dapsone is given at least for two months.
2. Rifampicin is given at least for ten days.
3. Associated conditions like atrophic rhinitis are treated on their own merits.
4. Reconstructive surgery is done only when the disease has subsided completely.
246
SELF ASSESSMENT
Q. 1. Leprosy is also called
a. Besnier Boeck disease d. both a & b

b. Hansen’s disease e. both b & c


c. Hodgkin’s disease
Q. 2. Destruction of anterior nasal spine is found in
a. Leprosy d. Syphilis

b. Lupus e. Tuberculosis

c. Sarcoidosis
Q. 3. Varieties of leprosy are

a. borderline d. all of the above

b. lepromatous e. none of the above


c. tuberculoid
Q. 4. All are usual clinical features of leprosy except
a. bloodstained nasal discharge d. painless ulceration of skin

b. nasal obstruction e. perforation of bony septum


c. nodular thickening of nasal mucosa
Q. 5. All are investigations of leprosy except
a. biopsy of lesion d. detection of lepra cells

b. detection of acid-fast bacilli e. microscopy of nasal discharge


c. detection of alkali-fast bacilli
Q. 6. All are treatments of lepromatous leprosy except
a. address associated condition like c. radiotherapy
atrophic rhinitis d. reconstructive surgery after cure
b. dapsone e. rifampicin
Scenario: A middle-aged man presented with painless ulceration of nose. There is history of
nasal obstruction and bloodstained nasal discharge for last 03 years. On examination there
is destruction of anterior nasal spine and perforation of cartilaginous septum.
1. What is the most probable clinical diagnosis?
2. What investigations you will carry out?
3. How will you manage this patient?
-------------------------------------------------------- ANSWERS-----------------------------------------------------
Q.1. b 2. a 3. d 4. e 5. c 6. c
Scenario: 1. Leprosy 2. Microscopy of nasal discharge and biopsy 3. Dapsone, rifampicin,
associated conditions like atrophic rhinitis are treated.
--------------------------------------------------------------------------------------------------------------------------
247
MUCORMYCOSIS
Etiopathology:
It is a rapidly lethal fungal disease of nose and paranasal sinuses.
Usual causative fungus is Mucor circinelloides. It affects diabetic individuals or whose
immunity is decreased because of malnutrition, uremia, acidosis, steroids, antimetabolites or
antibiotics. The fungus has an affinity for arteries. It dissects internal elastic lamina of the
tunica media and leads to extensive endothelial damage and thrombosis.
Mucormycosis usually commences in the nose and spreads by direct extension and
intravascular propagation to involve paranasal sinuses, orbit and brain.
Clinical features:
1. Nasal obstruction
2. Nasal discharge are usual presenting symptoms.
On examination;
1. Anterior rhinoscopy shows blackish necrotic turbinate resembling
dried clotted blood, which is pathognomonic of mucormycosis. A case of mucormycosis

2. Unilateral gangrene of the hard and soft palate occurs due to


involvement of palatine arteries.
3. If the orbit is involved, patient may complain of orbital symptoms (like chemosis of
conjunctiva, diplopia, proptosis and blurring of vision).
Investigations:
1. Biopsy must be taken to confirm the diagnosis.
2. X-ray/CT/MRI shows thickening of the lining of the sinuses and spotty destruction of the
bony wall. There is no fluid level.

Treatment:
It consists of;
1. Control of original precipitating factor such as diabetes
2. Debridement and local drainage
3. Amphotericin B: It is given in a dose of 0.25-0.5 mg/kg/day infused over 02-06 hours.
Total adult dose of amphotericin B is one gram. It is extremely nephrotoxic.
Dose of newly introduced liposomal amphotericin B is 3-6 mg/kg/day. It is relatively less
nephrotoxic.
4. Heparinization
SELF ASSESSMENT
Q. 1. Mucormycosis is

a. bacterial infection b. fungal infection


248 Part C Nose and Paranasal Sinuses

c. viral infection e. none of the above


d. all of the above
Q. 2. All are true about mucormycosis except
a. causes distant metastasis d. involves the orbit

b. commences in the nose e. spreads by direct extension


c. extends by intravascular propagation
Q. 3. The pathognomonic feature of mucormycosis is
a. blackish necrotic turbinate d. nasal obstruction

b. cervical lymphadenopathy e. orbital involvement


c. nasal discharge
Q. 4. Investigation of mucormycosis includes
a. biopsy d. all of the above

b. CT scan e. none of the above

c. MRI
Q. 5. Treatment of mucormycosis is

a. amphotericin B d. debridement

b. control diabetes e. heparinization

c. control immunosuppresion
Scenario: A 50 years old female known diabetic for past 15 years is admitted in the medical
ward for management of diabetic ketoacidosis. Blackish necrotic turbinate is noted in right
nasal cavity along with right sided proptosis.
1. What is the most probable clinical diagnosis?
2. How will you prove your diagnosis?
3. How will you manage this case?
-------------------------------------------------------- ANSWERS-- ---------------------------------------------------
Q.1. b 2. a 3. a 4. d 5.a
Scenario: 1. Mucormycosis 2. History, examination, tissue biopsy, X-ray, CT, MRI
3. Control of diabetes mellitus and immunosuppresion, debridement, amphotericin B,
heparinization.
--------------------------------------------------------------------------------------------------------------------------
RHINOSCLEROMA
Etiopathology:
It is a progressive granulomatous disease usually commences in the nose. Eventually it may
extend down to involve nasopharynx, oropharynx and even larynx. It is caused by gram
negative frisch bacillus known as Klebsiella rhinoscleromatis.
249
Investigations:
Histologically submucosa is infiltrated with plasma
cells, lymphocytes, eosinophils, Mikulicz cells and
Russell bodies. Mikulicz cells have a central nucleus
and a vacuolated cytoplasm containing frisch bacilli.
Russell bodies are homogenous eosinophilic
inclusion bodies found in plasma cells.
Clinical features:
Histopathology of rhinoscleroma
These may resemble atrophic rhinitis. There may be
bluish red nodules in the nose.
Treatment:
1. Surgical debridement is carried out initially.
2. Ciprofloxacin is given in a dose of 250 to 500 mg 12 hourly for 04 to 06
weeks. Alternatively, trimethoprim-sulphamethoxazole may be given.

3. Steroids may be given to reduce inflammation and fibrosis.


A case of rhinoscleroma
4. Cosmetic and reconstructive surgery is carried only when the
disease has subsided completely.
SELF ASSESSMENT
Q. 1. All are true about rhinoscleroma except
a. caused by Klebsiella rhinoscleromatis d. nonspecific granuloma

b. characterized by Mikulicz cells e. progressive granulomatous disease


c. characterized by Russell bodies
Q. 2. The most important investigation of rhinoscleroma is
a. CT scan d. ultrasound neck

b. detection of frisch bacilli e. X-ray PNS


c. FNAC of neck node
Q. 3. Mikulicz cells and Russell bodies are diagnostic histopathological features of
a. chordoma d. non-Hodgkin’s disease

b. leprosy e. rhinoscleroma
c. mucormycosis
Q. 4. All are treatments of rhinoscleroma except
a. ciprofloxacin d. surgical debridement

b. cosmetic and reconstructive surgery e. trimethoprim and sulphamethoxazole

c. cytotoxic drugs
Scenario: A 50 years old patient presents with progressive bilateral nasal obstruction for last
02 years. On examination there are bluish red nodules in both nasal cavities.
Histopathological
250
examination shows Mikulicz cells and Russell bodies.
1. What is the most probable clinical diagnosis?
2. What is the differential diagnosis?
3. What is your treatment plan?
-------------------------------------------------------- ANSWERS-----------------------------------------------------
Q.1. d 2. b 3. e 4.c
Scenario: 1. Rhinoscleroma 2. Stewart’s granuloma, Wegener’s granuloma
3. Surgical debridement, ciprofloxacin, trimethoprim-sulphamethoxazole, steroids.
--------------------------------------------------------------------------------------------------------------------------
-
RHINOSPORIDIOSIS
It is a chronic granulomatous disease caused by Rhinosporidium seeberi and affects both
human and animals.
Etiopathology:
It is common in younger age group particularly men. It can also be found
in animals i.e., cows, bulls, horses and dogs, where men and animals share the same infected
ponds. It is considered to be a fungus, but it is that this is an aquatic protozoan parasite
belonging to a normal group of parasites, mesomycetozea.
Clinical features:
1. The disease mostly affects nose and nasopharynx. Other sides
such as skin, lips, palate, conjunctiva, epiglottis, larynx, trachea,
bronchi, vulva and vagina may also be affected.
2. Painless nasal obstruction, secondary to nasal polyp.
3. Polyps are unilateral and pedunculated.
4. Epistaxis
5. Nasal discharge

On examination:
Clinical picture of
1. Friable leafy polypoidal mass, pink to purple in color attached to rhinospordiosis
nasal septum or lateral wall.
2. The mass is very vascular and easily bleed on touch.

Diagnosis:
1. It is made on biopsy showing several sporangia, oval or round
in shape filled with spores.
2. CT scan of nose and paranasal sinuses confirms the extent of
disease.

Treatment:
1. Wide excision of mass with electrocautery to the base of
lesion. Histopathological picture of
rhinospordiosis
2. Dapsone arrests the maturation of sporangia and promotes the stromal fibrosis.

TUMORS OF NOSE, PRANASAL SINUSES AND NASOPHARYNX


BENIGN TUMORS OF NOSE AND PARANASAL SINUSES
Benign tumors of nose and paranasal sinuses are hemangioma, squamous papilloma, inverted
papilloma, schwannoma, neurofibroma and osteoma.
HEMANGIOMA
Synonym: Bleeding polyp of the septum
Clinical features:
1. Unilateral, recurrent nasal bleeding is the most common
presenting symptom.
2. There may be complaint of ipsilateral nasal obstruction.
On examination; Right septal hemangioma
1. Anterior rhinoscopy shows solitary, reddish, polypoidal lesion,
which is usually attached to anterior part of nasal septum.
2. Occasionally it may be sessile and ulcerated.
Treatment:
1. Polyp is excised from the base along with a margin of healthy mucoperichondrium.
2. Cautery is applied to the base. The excised tissue must be sent for histopathology to rule
out some sinister pathology.
SELF ASSESSMENT
Q. 1. Bleeding polyp of nose is also known as
a. antrochoanal polyp d. mucormycosis

b. hemangioma of nasal septum e. rhinoscleroma


c. juvenile angiofibroma
Q. 2. The most common benign tumor of nose is

CONCEPT BOOK OF DISEASES OF EAR, NOSE & THROAT BY FAHIM AWAN


251 Part C Nose and Paranasal Sinuses

a. glioma d. neurofibroma

b. hemangioma e. schwannoma
c. inverted papilloma
Q. 3. All are usual clinical features of bleeding polyp of septum except
a. recurrent unilateral nasal bleeding d. smooth reddish sessile lesion

b. septal perforation e. unilateral nasal obstruction


c. smooth reddish polypoidal lesion
Q. 4. Treatment of bleeding polyp of nose is
a. ethmoidectomy d. simple polypectomy

b. excision of base of polyp e. wide bore aspiration

c. radiotherapy
Scenario: A 30 years old patient presents with episodes of right sided epistaxis and
progressive ipsilateral nasal obstruction for last 01 year. On examination there is reddish
polypoidal lesion arising from right Little’s area. Nasopharyngeal examination is
unremarkable.
1. What is your provisional diagnosis?
2. How will you confirm your diagnosis?
3. How will you treat this patient?
-------------------------------------------------
------- ANSWERS -----------------------------------------------------
Q.1. b 2. b 3. b 4. b

Scenario: 1. Septal hemangioma 2. Examination and probe test

3. Excision of polyp and cauterization of base.


--------------------------------------------------------------------------------------------------------------------------
TRANSITIONAL CELL PAPILLOMA
Synonyms: Inverted papilloma, Schneiderian papilloma, Ringertz tumor
Etiopathology:
It arises from middle meatus of nose. It arises from nonolfactory mucosa of nose
(Schneiderian membrane).
It is a benign but locally aggressive tumor. Human papilloma virus is
thought to be responsible for its etiology. It is usually unilateral.
It has got a tendency to invade surrounding structure especially orbit
and brain. There is strong male predominance with a ratio of 5:1. It
commonly presents in the 5th decade. It has got a tendency to recur.
It may be present simultaneously with carcinoma in 5-10% of cases.
It has a potential for malignant change in 2-3% of cases.
Transitional cell papilloma
Clinical features:
CONCEPT BOOK OF DISEASES OF EAR, NOSE & THROAT BY FAHIM AWAN
252 Part C Nose and Paranasal Sinuses

1. Unilateral nasal obstruction with bloodstained nasal discharge/epistaxis are common


presenting symptoms. Further symptoms depend on exact extent of the tumor into orbit or
brain.
2. Nasal cavity shows unilateral irregular fleshy mass arising from lateral nasal wall in
the middlemeatus.
Investigations:
1. Biopsy is very important to establish a tissue diagnosis. Histologically there are deep
invaginations of surface epithelium into the stroma with finger like projections and
intact basement membrane. That is why it is known as inverted papilloma. There is
formation of subepithelial crypts.
2. CT/MRI is required to find out exact extent of the tumor.

Treatment:
1. Medial maxillectomy is the treatment of choice. It is done either by lateral rhinotomy
or midfacial degloving approach. Endoscopic removal (FESS) is preferred nowadays.
Histopathological examination of all the material removed surgically is very important to
rule out any malignant change or associated malignancy.
2. Life long follow up is absolutely essential to find out any recurrence or malignant
change.
Radiotherapy is contraindicated as it may induce malignant change.
SELF ASSESSMENT
Q. 1. Transitional cell papilloma is also called

a. inverted papilloma d. all of the above

b. Ringertz tumor e. none of the above


c. Schneiderian papilloma
Q. 2. All are true of inverted papilloma of nose except
a. arises from middle meatus d. more common in male

b. may be associated with malignancy e. radiotherapy is the treatment of

choice
c. may change into malignancy
Q. 3. All are usual clinical features of transitional cell papilloma except
a. bloodstained nasal discharge d. unilateral irregular fleshy mass

b. may extend to orbit e. unilateral nasal obstruction


c. perforation of palate
Q. 4. Investigation of transitional cell papilloma includes
a. biopsy d. all of the above
b. CT e. none of the above

c. MRI
CONCEPT BOOK OF DISEASES OF EAR, NOSE & THROAT BY FAHIM AWAN
253 Part C Nose and Paranasal Sinuses
Q. 5. All are management of transitional cell papilloma except
a. FESS d. medial maxillectomy

b. lateral rhinotomy e. radiotherapy

c. life long follow-up


Scenario A: A 50 years old male presents with right sided nasal obstruction and right sided
bloodstained nasal discharge for last one year. On anterior rhinoscopy there is fleshy mass in
the right nasal cavity.
1. What is your provisional diagnosis?
2. How will you confirm your diagnosis?
3. How will you treat this patient?
B: A young patient presented with mass in the right nasal cavity causing moderate nasal
obstruction for last one and half year. Plain CT scan in axial and coronal planes was
obtained. The biopsy report revealed that the lining epithelium had inverted pattern with
finger like projections and intact basement membrane.
1. What is the pathological diagnosis?
2. What is the most common site of origin of this pathology?
3. Mention three characteristic features attributed to this pathology?
4. What is the management strategy?
-------------------------------------------------------- ANSWERS-----------------------------------------------------
Q.1. d 2. e 3. c 4. d 5. e
Scenario A: 1. Transitional cell papilloma 2. Biopsy 3. FESS, medial maxillectomy.
B: 1. Inverted papilloma 2. Middle meatus of nose 3. Bloodstained unilateral nasal
discharge, irregular fleshy mass 4. Medial maxillectomy.
--------------------------------------------------------------------------------------------------------------------------
MALIGNANT TUMORS OF NOSE AND PARANASAL SINUSES
Etiopathology:
Eighty percent of malignant tumors of nose and paranasal sinuses are squamous cell
carcinomas. Other less common tumor is adenocarcinoma.
Exact cause of these tumors is unknown. Inhaled carcinogens are thought to be
causative factors. Risk factors include leather, nickel and chromium industry. Polycyclic
hydrocarbons and mustard gas are also contributing factors.
Sixty percent of these tumors arise from maxillary sinus, 30 % from nasal cavities and 10
% from ethmoids. Tumors arising from nasal cavities and ethmoid sinuses have got poor
prognosis because of their close proximity to the cranial cavity and orbit. Male to female ratio
is 2:1. Males over the age of 50 years are usual candidates. Adenocarcinoma of ethmoid
sinuses is 10 times more common in wood workers.
CONCEPT BOOK OF DISEASES OF EAR, NOSE & THROAT BY FAHIM AWAN
254 Part C Nose and Paranasal Sinuses

Clinical features:
Clinical features depend on the exact site of origin and direction of extension of the tumor.
1. Progressive nasal obstruction is a common presenting symptom. There may be
associated bloodstained nasal discharge or epistaxis.
2. If there is orbital extension then epiphora, proptosis, diplopia and visual loss may occur.
3. Inferior extension into upper alveolus and palate may lead to ill fitting dentures or
frank ulceration of oral cavity.
4. Trismus usually indicates involvement of pterygoid muscles.
Neck node metastasis is uncommon.
Investigations:
1. Biopsy from the nasal cavity or the sinus is essential to establish tissue diagnosis.
2. CT/MRI may determine exact extent of the tumor and site, size, number and level of
lymph node involvement.
Treatment:
Combination of chemotherapy, radiotherapy and surgery is used. Full course of
radiotherapy, followed by radical surgery at an interval of 04 to 06 weeks is the usual
approach. Surgery is usually in the form of total maxillectomy through Weber-Ferguson
incision.
Some clinicians prefer primary surgery followed by postoperative radiotherapy.
Faciomaxillary surgeon is important in reconstruction of oral cavity defect.
SELF ASSESSMENT
Q. 1. The most common malignant tumor of nose and paranasal sinuses is
a. adenocarcinoma d. sarcoma

b. adenoid cystic carcinoma e. squamous cell carcinoma


c. lymphoma
Q. 2. Risk factors associated with squamous cell carcinoma of the paranasal sinuses
include all except
a. furniture industry d. nickel and chromium industry

b. leather industry e. polycyclic hydrocarbons


c. mustard gas
Q. 3. Paranasal sinus most often involved in malignancy is
a. anterior ethmoid sinus d. posterior ethmoid sinus

b. frontal sinus e. sphenoid sinus


c. maxillary sinus
Q. 4. The most common type of carcinoma of the sinonasal tract in wood worker
CONCEPT BOOK OF DISEASES OF EAR, NOSE & THROAT BY FAHIM AWAN
255 Part C Nose and Paranasal Sinuses

is

a. adenocarcinoma d. squamous cell carcinoma

b. adenoid cystic carcinoma e. transitional cell carcinoma


c. olfactory neuroblastoma
Q. 5. Exposure to wood dust is causative factor mainly for the adenocarcinoma of
a. ethmoidal sinus d. petrosal sinus

b. frontal sinus e. sphenoid sinus


c. maxillary sinus
Q. 6. Generally, maxillary sinus carcinoma may cause all eye symptoms except
a. diplopia d. proptosis

b. enophthalmos e. visual loss


c. epiphora
Q. 7. Generally, maxillary sinus carcinoma may present with problems of oral cavity like
a. difficulty in fitting denture d. non healing socket after extraction

b. frank ulceration of oral cavity e. All of the above


c. loosening of tooth
Q. 8. In carcinoma of maxillary sinus all of the following statements are usually true except
a. early lymphatic metastasis d. treatment of choice is combination of
b. invasion of ethmoid sinuses surgery and radiotherapy
c. invasion of orbit e. trismus indicates extension to ptery-goid
muscles
Q. 9. Widening of nasal root is seen in all except
a. angiofibroma d. nasopharyngeal carcinoma

b. ethmoidal malignancy e. septal abscess


c. nasal polypi
Q. 10. Best treatment for squamous cell carcinoma of maxilla—T2 N0 M0 is
a. partial maxillectomy d. total maxillectomy and chemoradio-
b. radiotherapy therapy
c. total maxillectomy and chemotherapy e. total maxillectomy
Scenario A: A 52 years old male presents with progressive nasal obstruction and purulent
nasal discharge for last 01year. He looks physically weak and emaciated. Nasal examination
reveals granular ulcerative mass in the right nasal cavity spreading towards the floor of
nose.
1. Name four conditions that can be included in differential diagnosis?
2. How will you investigate the case?
CONCEPT BOOK OF DISEASES OF EAR, NOSE & THROAT BY FAHIM AWAN
256 Part C Nose and Paranasal Sinuses
3. How will you manage the case?
B: A 55 years old male presents with progressive right sided nasal obstruction, bloodstained
nasal discharge, right sided proptosis and pain in face for last 06 months. Examination also
reveals loose right upper molar teeth.
1. What is the most probable clinical diagnosis?
2. How will you access depth of lesion?
3. What is the treatment of choice?
-------------------------------------------------------- ANSWERS-----------------------------------------------------
Q.1. e 2. a 3. c 4. a 5. a
6. b 7. e 8. a 9. e 10. d
Scenario A: 1. Nasal malignancy, inverted papilloma, Wegner’s granuloma, tuberculous
granuloma 2. Biopsy, CT, MRI 3. Total maxillectomy and radiotherapy.
B: 1. Carcinoma right maxillary sinus 2. CT, MRI 3. Total maxillectomy and radiotherapy.
--------------------------------------------------------------------------------------------------------------------------

NASOPHARYNGEAL ANGIOFIBROMA
Synonym: Juvenile nasopharyngeal angiofibroma
Etiopathology:
It is the most common benign tumor of nasopharynx. It is a benign yet locally aggressive
vascular tumor. It arises from the region of sphenopalatine foramen (nasopharynx). It spreads
along natural foramina and fissures. It may extend into nasal cavity, nasopharynx,
pterygopalatine fossa, infratemporal fossa, orbit and cranium. It may also erode bone.
As the name indicates it is composed of vascular and fibrous tissue. The vessels are
endothelial lined spaces with no muscular or elastic coat.
Exact etiology of its origin is unknown. However, there are different theories for its
pathogenesis:
1. Fibroblastic theory; It suggests abnormal growth or response of connective tissue.
2. Hormonal theory; It suggests estrogen-androgen imbalance as is indicated by its
occurrence in early adolescent males.
3. Hamartomatous theory; It suggests hamartomatous proliferation
of vascular tissue.

Clinical features:
1. Severe recurrent epistaxis and nasal obstruction in an early
teenager boy are the most common presenting symptoms.
Frog face deformity
in angiofibroma
CONCEPT BOOK OF DISEASES OF EAR, NOSE & THROAT BY FAHIM AWAN
257 Part C Nose and Paranasal Sinuses

2. Facial swelling, broadening of nasal bridge (frog face deformity), proptosis and
diplopia are also common presentations.
3. Other symptoms are due to eustachian tube dysfunction e.g., tinnitus, hearing loss and
earache.
4. On examination a characteristic smooth, lobulated, reddish,
polypoidal mass is usually visible in the nasal cavity or
nasopharynx. Flexible nasopharyngoscopy/30-degree
endoscopy may be required occasionally.
Investigations:
1. Contrast CT shows exact extension of the tumor into
surrounding structures. It also shows anterior bowing of
posterior wall of maxilla and posterior bowing of pterygoid
plates i.e., Holman-Miller sign or antral sign.
2. Angiography shows “tumor blush” and determines the
vascularity of the tumor, its feeding vessels and relation of the
tumor to the adjacent vessels.
Major arterial supply is from ipsilateral maxillary artery
and ascending pharyngeal artery. Collateral may develop from
contralateral maxillary artery, contralateral ascending
pharyngeal artery and internal carotid system respectively.
Biopsy is contraindicated in cases of suspected
angiofibroma.
Treatment:
1. Surgery is the treatment of choice. Exact approach depends
on the extent of tumor. Usual approaches are transpalatal

approach, Moure’s lateral rhinotomy approach, maxillary swing Holman-Miller sign on CT


approach or sublabial midfacial degloving approach.
Preoperative embolization of the feeding vessel within 24 hours is done to
decrease intraoperative bleeding.
2. Radiotherapy is reserved for unresectable lesions.
3. Androgens and estrogens are used as adjuncts in inoperable and recurrent tumors.

Nasopharyngeal angiofibroma has got a habit of recurrence even after complete

removal.

SELF ASSESSMENT
Q. 1. The most common benign tumor of nasopharynx is
a. adenoma d. hemangioma

b. angiofibroma e. papilloma
c. chondroma
Q. 2. Juvenile angiofibroma commonly arises from
a. maxillary sinus b. nasal cavity
CONCEPT BOOK OF DISEASES OF EAR, NOSE & THROAT BY FAHIM AWAN
258 Part C Nose and Paranasal Sinuses

c. posterior wall of nasopharynx e. sphenopalatine foramen


d. posterosuperior wall of nasopharynx
Q. 3. Major arterial supply of angiofibroma is
a. contralateral ascending pharyngeal c. ipsilateral ascending pharyngeal artery
artery d. ipsilateral lingual artery
b. contralateral maxillary artery e. ipsilateral maxillary artery
Q. 4. The usual earliest symptom of nasopharyngeal angiofibroma is
a. broadening of nasal bridge d. proptosis
b. diplopia e. severe recurrent epistaxis
c. facial swelling
Q. 5. Angiofibroma of nasopharynx
a. affects adolescent girls d. nose pinching is very effective to control
b. causes no bone destruction bleeding
c. composed of lymphoid tissue e. vascular tumor
Q. 6. Which of the following is true about angiofibroma of nasopharynx?
a. Does not erode bone d. Occurs in third decade

b. Equal incidence in male and female e. Recurs even after complete removal
c. Malignant tumor
Q. 7. A 15 years old boy has unilateral nasal obstruction and recurrent severe epi-staxis for
last 06 months There is also swelling of ipsilateral cheek. The diagnosis is
a. angiofibroma d. lymphoma of nasopharynx

b. cancer of nasopharynx e. maxillary sinusitis

c. inverted papilloma
Q. 8. Antral sign is a feature of

a. acoustic neuroma d. glomus tumor

b. asthesioneuroblastoma e. nasopharyngeal angiofibroma

c. coalescent mastoiditis

Q. 9. All are diagnostic investigations of nasopharyngeal angiofibroma except


a. angiography d. MRI

b. contrast CT e. punch biopsy


c. flexible nasopharyngoscopy
Q. 10. To decrease intraoperative bleeding in juvenile angiofibroma, preoperative
embolization is done within
a. 06 hours b. 12 hours
CONCEPT BOOK OF DISEASES OF EAR, NOSE & THROAT BY FAHIM AWAN
259 Part C Nose and Paranasal Sinuses

c. 24 hours e. 72 hours
d. 48 hours
Q. 11. Juvenile angiofibroma responds to following modalities of treatment except
a. androgen therapy d. radiotherapy

b. estrogen therapy e. surgical excision


c. immunotherapy
Q. 12. Histological diagnosis of angiofibroma of nasopharynx is made by
a. excisional biopsy d. incisional biopsy

b. exfoliative cytology e. punch biopsy

c. FNAC
Scenario A: A 15 years old boy presents with recurrent massive epistaxis, progressive nasal
obstruction and proptosis for last 01 year. Anterior rhinoscopy reveals clot in left nasal
cavity and posterior rhinoscopy shows reddish mass in the postnasal space.
1. What is the most probable diagnosis?
2. Briefly describe the investigations?
3. What is the treatment plan of this patient?
B: A 15 years old boy presented with progressive nasal obstruction, massive epistaxis
and proptosis for last 18 months. Anterior rhinoscopy revealed mass in left nasal cavity
and posterior rhinoscopy showed reddish lobulated mass in the postnasal space as well.
There is bulging of ipsilateral cheek for last six months.
1. What is the most probable clinical diagnosis?
2. What are different theories of its pathogenesis?
3. What measure can be done to decrease peroperative bleeding?
4. How will you treat this patient?
----------------------------------------------------
---- ANSWERS-----------------------------------------------------
Q.1. b 2. e 3. e 4. e 5. e 6. e
7. a 8. e 9. e 10. c 11. c 12. a
Scenario A: 1. Nasopharyngeal angiofibroma 2. CT scan with contrast, angiography
3. Surgery according to extension.
B: 1. Nasopharyngeal angiofibroma 2. Fibroblastic theory, hormonal theory, hamartomatous
theory 3. Preoperative embolization of the feeding vessel within 24 hours
4. Surgery- approach is according to extension.
--------------------------------------------------------------------------------------------------------------------------

NASOPHARYNGEAL CARCINOMA (NPC)


260
Etiopathology:
It is more common in male with a ratio of 2-3:1.
It has got bimodal age distribution i.e.; first peak is seen by the end
of second decade and second peak is seen in fourth decade of life. Large
number of factors are known to cause nasopharyngeal carcinoma. A few of
them are;
1. Geography and genetic:
Southern Chinese and other South East Asians are affected 10 to 50
times higher than other countries. A susceptible genetic constitution
plays an important role.
2. Viral:
Epstein-Barr (EB) virus.
3. Environmental:
Chinese herbal medicines, salted fish, nitrosamines, incense and joss stick smoking are
thought to play an important role in causing nasopharyngeal carcinoma.
Cigarette smoking is also an important factor.

Histopathology:
According to WHO nasopharyngeal carcinoma is classified as follows;
Type 1. Keratinizing carcinoma
Type 2. Nonkeratinizing differentiated carcinoma
Type 3. Nonkeratinizing undifferentiated
carcinoma
Nasopharyngeal carcinoma (NPC) may present as ulcerative, infiltrative or exuberant
polypoidal form. It usually arises from fossa of Rosenmuller (also known as pharyngeal
recess). The tumor extends into nasal cavity, nasopharynx, pterygopalatine fossa,
infratemporal fossa and orbit. It extends from foramen lacerum and
foramen ovale to the middle cranial fossa.
Clinical features: NPC with cervical lymph

1. Cervical lymphadenopathy:
node enlargement

The most common presentation of nasopharyngeal carcinoma is painless


cervical lymph node enlargement. The first lymph node to be involved in
nasopharyngeal carcinoma is lymph node of Rouviere (Krause) but this
is impalpable because it is present in the retropharyngeal space.
The first palpable lymph node is jugulodigastric and/or apical node under the sternomastoid
muscle.
The classical ‘silent’ site or clinical blind spot in head and neck, notorious for metastatic
neck node is nasopharynx.
261
2. Nasal symptoms:
Nasal obstruction and bloodstained nasal discharge are common presentations. Ozaena occurs
as a result of tumor necrosis.
3. Aural symptoms:
Earache, conductive hearing loss and tinnitus occur due to otitis media with effusion.
Nasopharyngeal carcinoma must be ruled out in unilateral otitis media with effusion in adults.
4. Neurological symptoms:
Any cranial nerve may be affected.
The most common nerves to be involved are V, VI, IX and X. Cervical sympathetic
chain may be involved leading to Horner’s syndrome.
Pain and headache may occur in NPC indicating its intracranial spread or
sphenoiditis.
Severe pain is the hallmark of terminal disease.
5. Distant metastasis:
Fifty percent occur in thoracolumbar spine and 30% in skeletal muscles.
Trotter’s triad/sinus of Morgagni syndrome:
It is seen in nasopharyngeal carcinoma which spreads laterally to involve the sinus of
Morgagni. It is characterized by;
1. Unilateral conductive hearing loss (due to Eustachian tube obstruction).
2. Ipsilateral paralysis of soft palate.
3. Neuralgia of 2nd branch of trigeminal nerve.

Investigations:
1. CT demonstrates the extent of bone destruction.
2. MRI is required to find out three-dimensional extent of the tumor into surrounding
structures like brain, orbit etc. It may also indicate the site, size, number and level of
lymph node involvement.
3. Biopsy after endoscopic evaluation of the tumor is required to find out exact nature of
carcinoma and its degree of differentiation. It is usually done under local anesthesia.
4. Positron Emission Tomography (PET) shows metastasis anywhere in the body.
5. PTA and tympanometry to diagnose otitis media with effusion and to detect the
otological effects of radiotherapy and chemotherapy.
Treatment:
1. Radiotherapy is the mainstay of treatment in nasopharyngeal carcinoma.
2. Chemotherapy is usually used along with radiotherapy.
Neck dissection is carried out for radioresistant neck nodes. Myringotomy and grommet
insertion are required in severe cases of OME.
Differential diagnosis:
CONCEPT BOOK OF DISEASES OF EAR, NOSE & THROAT BY FAHIM AWAN
262 Part C Nose and Paranasal Sinuses

1. Nasopharyngeal angiofibroma
2. Lymphoma
3. Adenocarcinoma
SELF ASSESSMENT
Q. 1. Nasopharyngeal carcinoma is the most common in
a. Europe d. North America

b. North Asia e. Southern China


c. North East Asia
Q. 2. Nasopharyngeal carcinoma is associated with
a. CMV d. HCV

b. EBV e. HPV
c. HBV
Q. 3. All are true about nasopharyngeal carcinoma except
a. always seen above 40 years. d. may be seen by the end of second de-cade
b. cervical lymph node enlargement is the of life
presenting feature e. squamous cell carcinoma is seen more
c. involves multiple cranial nerves often than adenocarcinoma
Q. 4. Which of the following is true about nasopharyngeal carcinoma except?
a. Causes unilateral otitis media with ef- c. Genetic factors play an important role
fusion d. Metastasize to cervical lymph nodes
b. E.B. virus is one of the causative factors e. Treatment of choice is surgery
Q. 5. Velopharyngeal insufficiency is usually seen in all except
a. cleft palate d. post adenoidectomy

b. nasopharyngeal tumor e. submucous cleft palate


c. palate paralysis
Q. 6. All are true of nasopharyngeal carcinoma except
a. fossa of Rosenmuller is the the most c. neck node enlargement is the common-est
common site of origin presenting feature
b. intracranial spread through foramen d. predominant in females
lacerum e. presentation may be with epistaxis
Q. 7. Chances of lymphatic metastasis is highest in cancer of
a. buccal mucosa e. paranasal sinuses

b. glottic region

c. hard palate
d. nasopharynx
CONCEPT BOOK OF DISEASES OF EAR, NOSE & THROAT BY FAHIM AWAN
263 Part C Nose and Paranasal Sinuses
Q. 8. Nasopharyngeal carcinoma causes Horner’s syndrome as a result of infiltra-tion of
a. 3rd cranial nerve d. 9th nerve

b. 5th cranial nerve e. cervical sympathetic chain


c. 7th cranial nerve
Q. 9. A 45 years old patient presents with left sided conductive hearing loss, left sided
facial pain in the distribution of 2nd branch of trigeminal nerve and asymmetry of soft
palate for last 03 months. The most probable clinical diagnosis is
a. Aspirin triad d. Trotter’s triad

b. Atopic triad e. Wallenberg’s syndrome


c. Samter’s triad
Q. 10. Node of Rouviere/Krause is part of
a. jugulodigastric group of lymph nodes d. retropharyngeal group of lymph nodes

b. jugulo omohyoid group of lymph nodes e. supraclavicular group of lymph nodes


c. parapharyngeal group of lymph nodes
Q. 11. The classical ‘silent’ site or clinical blind spot in head and neck, notorious for
metastatic neck node is
a. base of tongue d. pyriform fossa

b. larynx e. vallecula
c. nasopharynx
Q. 12. Treatment of choice in nasopharyngeal carcinoma is
a. chemotherapy d. surgery

b. radiotherapy e. surgery followed by radiotherapy

c. sclerotherapy
Scenario A: A 40 years old male presents to you with progressively increasing painless
swelling in upper part of right side of neck for last two months. He also complains of right
sided nasal obstruction and bloodstained nasal discharge.
1. What is the most probable clinical diagnosis?
2. What is the differential diagnosis?
3. How will you treat this patient?
B: A 45 years old man presents with huge swelling on right side of neck for last 06 months.
He also complains of sanguineous discharge from nose, ear blockage and decrease hearing
in right ear.
1. What is your provisional diagnosis?
2. What is the differential diagnosis?
3. How will you investigate this patient?
CONCEPT BOOK OF DISEASES OF EAR, NOSE & THROAT BY FAHIM AWAN
264 Part C Nose and Paranasal Sinuses
4. How will you treat this patient?
------------------------------------------------- ANSWERS--------------------------------------------------
------- ---
Q.1. e 2. b 3. a 4. e 5. b 6. d
7. d 8. e 9. d 10. d 11. c 12. b
Scenario A: 1. Nasopharyngeal carcinoma 2. Inverted papilloma, antrochoanal polyp,
adenocarcinoma 3. Radiotherapy, chemotherapy.
B: 1. Nasopharyngeal carcinoma 2. Inverted papilloma, antrochoanal polyp,
adenocarcinoma
3. CT, MRI, biopsy 4. Radiotherapy, chemotherapy.
--------------------------------------------------------------------------------------------------------------------------
-
MISCELLANEOUS DISEASES OF NOSE AND PARANASAL
SINUSES
NASAL POLYPS
Synonyms: Ethmoidal polyps/polypi/polyposis, Mucous polyps/polypi/polyposis

Etiopathology:
Nasal polyps result from prolapsed mucosal lining of ethmoidal sinuses. Nasal mucosa in the
region of middle meatus becomes edematous due to collection of extracellular fluid causing
polypoidal change. Polyps present in the nasal cavity as grape like structures having a “body”
and a “stalk”.
Exact etiology of nasal polypi is unknown. These are found in association with certain
conditions like:
1. Inflammatory conditions of nasal mucosa.
2. Aspirin hypersensitivity e.g., aspirin triad or Samter’s triad. It is the association of
nasal polyps, bronchial asthma and aspirin hypersensitivity.
3. Cystic fibrosis
4. Vasomotor rhinitis
5. Allergic rhinitis and allergic fungal sinusitis
6. Kartagener’s syndrome which consists of ciliary dyskinesia, sinusitis, bronchiectasis
and situs inversus.
7. Young’s syndrome which consists of sinusitis, bronchiectasis and obstructive
azoospermia.
Nasal polyp is a disease of adults. There is strong male predominance. Nasal polyps are
multiple and often bilateral. These have smooth surface and are pearly white or yellowish
in color. These tend to recur. These may involve all paranasal sinuses, orbit and brain. The
polyps have poor blood and nerve supply. Any child with nasal polypi should be regarded as
having cystic fibrosis until proved otherwise.

CONCEPT BOOK OF DISEASES OF EAR, NOSE & THROAT BY FAHIM AWAN


265 Part C Nose and Paranasal Sinuses

Clinical features:
Patient may present with;
1. Nasal obstruction
2. Nasal discharge
3. Sneezing
4. Decreased or absent sense of smell Anterior rhinoscopy of a
case of nasal polypi
5. Pain over bridge of nose, forehead and cheeks due to attendant
sinusitis.
On examination;
1. Multiple pearly white glistening polyps are usually visible in both
nasal cavities on anterior rhinoscopy. These are insensitive to touch
because of poor nerve supply.
2. Hyponasal voice (rhinolalia clausa).
Endoscopic view of
3. Mouth breathing. nasal polypi
4. Telecanthus i.e., increased distance between two medial canthi. On
patency test there is decreased nasal airway.
5. Broadening of nasal bridge.
Small polyps are usaully diagnosed on nasal endoscopy.

Investigations:
Diagnosis is usually made on clinical grounds.
1. Small polyps are usually diagnosed on nasal endoscopy.
2. X-ray paranasal sinuses may show haziness of sinuses, mucosal thickening or fluid level.
3. CT shows exact anatomy and extension of the disease into different sinuses, orbit or the
cranium.
4. Histological examination of the tissue must be carried out to rule out malignancy.
Treatment:
Nasal polyps tend to recur and may extend to different sites. Treatment options are;
A. Conservative:
Fifty percent of patients do respond to steroids. Trial of topical and systemic corticosteroids
must be given unless there is contraindication to use of steroids e.g., diabetes, hypertension,
tuberculosis, pregnancy, infection, peptic ulceration etc. Adult dose of prednisolone is 40-50
mg/day for up to 03 weeks. Antibiotics are usually given to control superimposed infection.
266
B. Surgical:
Depending on exact extension of the disease following surgical options are possible;
1. Intranasal polypectomy
2. Intranasal ethmoidectomy
3. Transantral ethmoidectomy
4. External ethmoidectomy
5. FESS (Functional Endoscopic Sinus Surgery)
Differential diagnosis:
1. Antrochoanal polyp
2. Nasopharyngeal angiofibroma
3. Inverted papilloma
4. Meningocele
5. Meningoencephalocele
6. Cystic fibrosis
SELF ASSESSMENT
Q. 1. Nasal polyp is also called

a. antrochoanal polyp d. Both a & b

b. ethmoidal polyp e. Both b & c


c. mucous polyp
Q. 2. Nasal polypi in a child should raise the suspicion of
a. Addison’s disease d. Potter’s syndrome

b. Cystic fibrosis e. Usher’s syndrome


c. Familial polyposis coli
Q. 3. All are true about nasal polyp except
a. arises from ethmoidal sinuses d. multiple

b. bilateral e. usually occurs in adults


c. bleed on touch
Q. 4. Multiple simple nasal polypi may arise from any of the following structures except
a. bulla ethmoidalis d. middle turbinate

b. inferior turbinate e. uncinate process


c. middle meatus
Q. 5. All of the following conditions cause multiple nasal polypi except
267 Part C Nose and Paranasal Sinuses

a. allergic rhinitis d. rhinitis sicca

b. aspirin hypersensitivity e. vasomotor rhinitis


c. cystic fibrosis
Q. 6. Which of the following statement is not correct for ethmoidal polypi?
a. Allergy is an etiological factor d. May be associated with bronchial asth-

ma
b. Bilateral condition

c. Insensitive to touch e. Occur in the first decade of life


Q. 7. Differential diagnosis of unilateral nasal polyp may be

a. angiofibroma d. meningocele

b. antrochoanal polyp e. all of the above

c. inverted papilloma

Q. 8. Investigation of nasal polyps is

a. CT scan d. all of the above

b. nasal endoscopy e. none of the above


c. X-ray PNS
Q. 9. The most effective medical treatment of nasal polypi is
a. antibiotics d. steroids

b. antihistamines e. topical decongestants


c. non-steroidal anti-inflammatory, drugs
Q. 10. Initial treatment of nasal polypi is
a. conservative with steroids d. intranasal polypectomy

b. FESS e. transantral ethmoidectomy


c. intranasal ethmoidectomy
Q. 11. Treatment of recurrent nasal polypi is
a. Caldwell-Luc operation d. intranasal antrostomy

b. Denker’s operation e. Young’s operation


c. ethmoidectomy
Scenario: A 35 years old patient presents with progressive bilateral nasal obstruction and
discharge for last 02 years. There is no complain of epistaxis. On examination, there are
bilateral multiple glistening white pedunculated masses which are insensitive to touch.
1. What is the most probable clinical diagnosis?
2. What is the differential diagnosis?
3. What are different treatment options?
--------------------------------------------------------ANSWERS-----------------------------------------------------
CONCEPT BOOK OF DISEASES OF EAR, NOSE & THROAT BY FAHIM AWAN
Part C Nose and Paranasal
268 Sinuses
Q.1. e 2. b 3. c 4. b 5. d 6. e
7. e 8. d 9. d 10. a 11. c
Scenario: 1: Bilateral ethmoidal polypi 2: Antrochoanal polyp, nasopharyngeal
angiofibroma, inverted papilloma, meningocele 3: Medical treatment including steroids,
surgical plan according to extension.
--------------------------------------------------------------------------------------------------------------------------
ANTROCHOANAL POLYP
Synonym: Killian’s polyp

Etiopathology:
Exact etiology of its origin is unknown. It arises from maxillary
sinus, comes out of maxillary ostium and hangs either in the nose
or posterior nares. It is a trilobed structure with an antral, nasal
Antrochoanal polyp
and choanal part. It may push soft palate downwards and
hanging in
forwards. It is usually unilateral and solitary. It is more common oropharynx
in children and young adults.
Its incidence is higher in male.
Clinical features:

1. Unilateral nasal obstruction, especially during breathing out due to ball valve effect.
When the polyp is very large, it may cause bilateral nasal obstruction by pushing the
septum towards opposite side. There may be anterior or posterior nasal discharge due to
associated sinusitis.
2. Hearing loss and earache due to obstruction of eustachian tube orifice may occur.
On examination;
1. Rhinolalia clausa may occur in cases of large polyp. The polyp may be visible on
anterior rhinoscopy.
2. On oral cavity examination, there may be limitation or loss of palatal movement due to
mass effect. If the polyp is large enough, it may be visible hanging down into
nasopharynx or even oropharynx.
3. On posterior rhinoscopy polyp may be visible as solitary smooth pedunculated mass in
the nasopharynx.
4. Flexible nasopharyngoscopy or 30-degree endoscopy and CT scan may be required
occasionally for diagnosis.
Investigations:
Diagnosis is usually made on clinical grounds.
1. Small polyps are usually diagnosed on nasal endoscopy.

CONCEPT BOOK OF DISEASES OF EAR, NOSE & THROAT BY FAHIM AWAN


269 Part C Nose and Paranasal Sinuses

2. X-ray paranasal sinuses shows soft tissue mass or opacity in nasal cavity and ipsilateral
maxillary sinus.
3. X-ray soft tissue nasopharynx lateral view shows, soft tissue shadow in nasopharynx.
4. CT scan may be required occasionally.
5. Histological examination of the tissue must be carried out to rule out malignancy.

Treatment:
There is no conservative treatment for antrochoanal polyp. Both nasal and antral parts of
the polyp are removed surgically. Surgical options are;
1. Caldwell-Luc operation is the treatment of choice. It is done above the age of 12
years. This operation below the age of 12 years may damage secondary dentition
present in the anterior wall of maxilla.
2. Intranasal polypectomy is done if patient is below the age of 12 years. Although simple
intranasal polypectomy may lead to recurrence.
3. FESS: Recent trend is to perform FESS for all age groups.
Differences between ethmoidal polyps and antrochoanal polyp
Features Ethmoidal polyps Antrochoanal polyp
Origin Ethmoid sinuses Maxillary sinus
Laterality Bilateral Unilateral
Number Multiple Solitary
Age Common in adults Common in children
Growth Grows anteriorly to anterior nares Grows posteriorly to choana
Size, shape Small and grapes like Trilobed with antral, nasal and choanal
parts
Recurrence Common Uncommon
Treatment Initially medical then surgical Surgical

Differential diagnosis:
1. Hypertrophied middle turbinate
2. Ethmoidal polypi
3. Nasopharyngeal angiofibroma
4. Meningocele
5. Meningoencephalocele
SELF ASSESSMENT

CONCEPT BOOK OF DISEASES OF EAR, NOSE & THROAT BY FAHIM AWAN


270 Part C Nose and Paranasal Sinuses
Q. 1. Antrochoanal polyp is also called
a. ethmoidal polyp d. both a & b

b. Killian’s polyp e. both b & c

c. mucous polyp
Q. 2. Antrochoanal polyp arises from

a. ethmoidal sinus d. middle meatus

b. frontal sinus e. sphenoid sinus


c. maxillary sinus
Q. 3. All are true about antrochoanal polyp except
a. arises from maxillary sinus d. multiple

b. common in children e. unilateral


c. common in young adults
Q. 4. The main manifestation of antrochoanal polyp is
a. anosmia d. sneezing

b. headache e. unilateral nasal obstruction


c. proptosis
Q. 5. A 14 years old boy presents with a unilateral nasal mass which comes out of nasal
cavity on nose blowing. There is no history of bleeding. It is soft to feel. The most likely
diagnosis is
a. angiofibroma d. nasopharyngeal carcinoma

b. antrochoanal polyp e. rhinosporidiosis


c. cystic fibrosis
Q. 6. Following statement is correct about antrochoanal polyp
a. arises from maxillary sinus and grows c. often a bilateral condition
posteriorly d. often affects adults
b. best treated medically e. often affects elderly
Q. 7. Treatment of antrochoanal polyp is
a. antihistamine d. systemic steroids

b. immunotherapy e. topical steroids


c. surgery
Q. 8. The management of antrochoanal polyp in children is
a. antibiotics d. intranasal polypectomy

b. Caldwell-Luc operation e. to wait and watch


c. corticosteroids
Q. 9. Treatment of antrochoanal polyp in adults is

CONCEPT BOOK OF DISEASES OF EAR, NOSE & THROAT BY FAHIM AWAN


271 Part C Nose and Paranasal Sinuses

a. Caldwell-Luc operation d. intranasal antrostomy

b. decongestants e. polypectomy
c. ethmoidectomy
Q. 10. The most modern treatment of antrochoanal polyp in any age group is
a. Caldwell-Luc operation d. lateral rhinotomy

b. functional endoscopic sinus surgery e. transantral ethmoidectomy

c. intranasal polypectomy
Scenario A: A 09 years old boy complains of right sided nasal obstruction for last 01 year.
According to him he finds difficulty in breathing out through nose. The X-ray of paranasal
sinuses shows hazy right maxillary sinus. He is apyrexial and leukocyte count is 6000 per
microliter.
1. What is the most likely diagnosis?
2. What further radiological investigation is required?
3. What are various treatment options?
B: A 20 years old male presents with right sided progressive nasal obstruction for last 02
years. Patient also complains of a mass occasionally coming out of right nasal cavity There
is no complaint of bloodstained nasal discharge. On examination there is greyish mass in
right nasal cavity which is seen hanging in the nasopharynx on posterior rhinoscopy.
1. What is the most probable clinical diagnosis?
2. How will you confirm your diagnosis?
3. How will you manage this patient?
ANSWERS--------------------------------------------
-------------------------------------------------------- ---------
Q.1. b 2. c 3. d 4. e 5. b
6. a 7. c 8. d 9. a 10. b
Scenario A: 1. Antrochoanal polyp 2. X-ray PNS, X-ray soft tissue nasopharynx, CT scan
3. FESS, intranasal polypectomy.
B: 1. Antrochoanal polyp 2. X-ray PNS, X-ray soft tissue nasopharynx, CT scan
3. FESS, Caldwell-Luc operation.
--------------------------------------------------------------------------------------------------------------------------
-
EPISTAXIS
Synonyms: Nasal hemorrhage/bleeding
Definition:
Epistaxis is a Greek word which means “bleeding from nose”.
Etiopathology:
In 90% of cases bleeding occurs through James Little’s area. This is situated in the
anteroinferior part of nasal septum just above the nasal vestibule. Anastomosis of following
four arteries occur here to form Kiesselbach plexus:
CONCEPT BOOK OF DISEASES OF EAR, NOSE & THROAT BY FAHIM AWAN
272
a. Anterior ethmoidal artery
b. Septal branch of superior labial artery
c. Septal branch of sphenopalatine artery
d. Greater palatine artery
Another anastomosis of veins is formed inferior to posterior end of inferior turbinate. It
is known as Woodruff’s plexus/naso-nasopharyngeal plexus. Retrocolumellar vein at
the anterior end of Little’s area is a common cause of venous bleeding in young persons.
Causes:
A. Idiopathic (primary)
It constitutes 80% cases of epistaxis.
B. Local causes:
1. Traumatic
a. Non-surgical trauma e.g., road traffic accidents, nose picking.
b. Surgical trauma e.g., septoplasty, SMR, turbinectomy, cautery etc.
2. Infective: Infection of nose, paranasal sinus or nasopharynx such as;
a. Viral: e.g., viral rhinitis
b. Bacterial: e.g., acute and chronic bacterial rhinitis
c. Fungal: e.g., fungal rhinosinusitis
3. Neoplastic:
a. Benign tumors; e.g., hemangioma, nasopharyngeal angiofibroma
b. Malignant tumors; e.g., nasopharyngeal carcinoma
4. Granulomatous:
a. Specific granulomas
b. Nonspecific ganulomas
5. Atmospheric condition:
Hot dry climate, high altitude
6. Long standing foreign body of nose which may be living e.g., maggots, leeches
or non living e.g., rhinolith
7. D.N.S, septal perforation
C. Systemic causes:
273
1. Acute infectious diseases e.g., typhoid, measles and any high-grade fever
2. Hypertension and arteriosclerosis
3. Diseases of liver and kidney e.g., jaundice, renal failure
4. Drugs like anticoagulants e.g., heparin, warfarin and antiplatelets e.g., aspirin
5. Hemostatic diatheses e.g., clotting disorders like hemophilia or bleeding disorder like
thrombocytopenia
6. Raised venous pressure e.g., right sided heart failure, mediastinal syndrome
7. Barometric pressure changes e.g., descent during flight
8. Hereditary hemorrhagic telangiectasia
9. Vicarious menstruation i.e., epistaxis occurs during menstruation due to endometrial
tissue in the nose (endometriosis).

Types of epistaxis:
1. Anterior epistaxis:
In this, bleeding is from anterior nasal cavity. It usually occurs in
children and adults.
Anterior epistaxis
2. Posterior epistaxis:
In this, bleeding is from the choana and throat. This type of epistaxis usually occurs in
older people.
Differences between anterior and posterior epistaxis
Features Anterior epistaxis Posterior epistaxis
Incidence More common Less common
Site Little’s area Posterosuperior part of nasal cavity
Vessel Arterial Venous
Plexus Kiesselbach plexus Woodruff’s plexus
Age Children or young After 40 years of age
Cause Mostly trauma Mostly spontaneous
Intensity Usually mild and recurrent Usually severe and nonrecurrent

Investigations:
These highly depend on findings of history and examination.
Treatment:
In cases of epistaxis, it is very important to know the cause, duration and frequency of bleeding.
General condition of the patient is assessed and nose is examined thoroughly. Such a nasal
274 Part C Nose and Paranasal Sinuses

examination locates the source of bleeding and helps to exclude certain causes such as benign
or malignant tumors, granuloma etc.
Vital signs are recorded. Intravenous line is maintained. Blood grouping, cross matching
and assessment of patient for blood transfusion is made depending on condition of patient.
Underlying cause e.g., clotting disorders like hemophilia or bleeding disorders like
thrombocytopenia must be identified and addressed accordingly.
A. Immediate:
The following sequence of immediate measures are done to stop epistaxis;
1. Pinching of nose:
The patient is asked to sit, bend forwards and pinch the nose. This
simple measure stops bleeding in majority of cases because in 90% of
cases bleeding occurs from Little’s area which is compressed by simple
pinching for at least 10 minutes.
2. Ice or cold packs: Pinching of nose

These are applied over the nose, face and head. This constricts the blood vessels and stops
bleeding.
3. Anterior nasal packing:
Nasal cavities are examined and anesthetized with 4% xylocaine spray or
with packs soaked in 4% xylocaine. Any clot in the nasal cavity is removed
by suction.
Anterior nasal packing with packs (of 2.5 cm width for adults and 1.2
cm in children) soaked in antibiotics and liquid paraffin is done to stop
bleeding. Anterior nasal pack is removed slowly after 24-48 hours.
Alternatively, different catheters with balloon to compress the blood vessels
may be inserted.
It is very important to mention that in hemostatic diatheses any
surgical intervention like anterior or posterior nasal packing should be Anterior nasal packing
avoided. In these cases, nasal packing may do more harm than good to the
patient. However, in severe cases of epistaxis light packing may be done with spongostan.
4. Posterior nasal packing:
Packing of post nasal space is done with
post nasal plug or a Foley’s catheter is
passed through anterior nasal cavity and
the balloon in post nasal space is inflated
with 5-7 ml of saline. Then anterior
nasal packing is done against post nasal
plug/inflated balloon. The post nasal
plug/balloon does not allow the anterior
nasal packs to fall into oropharynx.
The post nasal plug is removed or
balloon is deflated after 48 hours. The Balloon for epistaxis Posterior nasal packing
procedure is very effective in stopping
posterior bleeding.
B. Curative and preventive:
275
These measures are adopted when;
a. Immediate treatment fails.
b. There is recurrent nasal hemorrhage even after removal of
the underlying cause such as infusion of platelets in
thrombocytopenia, factor VIII extracts in hemophilia,
vitamin K injection in vitamin K deficiency.
1. Cauterization of bleeding point:
Under local or general anesthesia, it is done;
a. Chemically with 20% silver nitrate.
b. Electrically with unipolar/bipolar diathermy or galvanic cautery.
2. S.M.R.
If septal spur is the cause of epistaxis, then S.M.R. or even simple
elevation of mucoperichondrial flaps may be required.
3. Embolization:
It is done by interventional radiologist. Gel foam or polyvenyl alcohol is embolized into
maxillary artery through femoral artery catheterization.
4. Arterial ligation:
Following options are available;
a. Ligation of external carotid artery in the neck.
b. Ligation of maxillary artery (in posterior epistaxis) in the pterygopalatine fossa
through Caldwell-Luc approach.
c. Transnasal Endoscopic SphenoPalatine Artery Ligation (TESPAL).
d. Ligation of anterior ethmoidal artery (in anterior epistaxis) through a curvelinear
incision medial to medial canthus of eye (also known as Lynch Howarth incision).
SELF ASSESSMENT
Q. 1. The most common cause of epistaxis is

a. granuloma d. neoplasm

b. idiopathic e. trauma
c. inflammation
Q. 2. The most common site of epistaxis in the anterior part of nose is
a. inferior turbinate d. tip of nose

b. James Littles’s area e. Woodruff’s area

c. middle turbinate
Q. 3. The most common site of epistaxis is
a. anteroinferior part of nasal septum b. inferior turbinate

CONCEPT BOOK OF DISEASES OF EAR, NOSE & THROAT BY FAHIM AWAN


276 Part C Nose and Paranasal Sinuses

c. lateral wall of nose above the middle d. nasopharynx

turbinate
e. posteroinferior part of nasal septum
Q. 4. Little’s area (Kiesselbach’s area) is situated at
a. anterior inferior quadrant of lateral wall tum
of nose d. posterior inferior quadrant of lateral wall
b. anterior inferior quadrant of nasal of nose
septum e. posterior superior quadrant of lateral
c. anterior superior quadrant of nasal sep- wall of nose
Q. 5. Kiesselbach’s plexus is formed by all the following arteries except
a. anterior ethmoidal artery d. sphenopalatine artery

b. greater palatine artery e. superior labial artery


c. posterior ethmoidal artery
Q. 6. Which of the following artery belongs to internal carotid system?
a. anterior ethmoidal artery d. sphenopalatine artery

b. greater palatine artery e. superior labial artery

c. nasopalatine artery
Q. 7. Repeated digital trauma to nose causes epistaxis due to bleeding from
a. anterior facial vein d. retrocolumellar vein

b. greater palatine artery e. sphenopalatine artery


c. posterior ethmoidal artery
Q. 8. The venous plexus near the posterior end of inferior turbinate is known as
a. Keisselbach’s plexus d. pterygoid plexus

b. Little’s area e. Woodruff’s plexus


c. nasal valve
Q. 9. All are immediate treatment of epistaxis except
a. anterior nasal packing d. pinching of nose

b. arterial ligation e. posterior nasal packing


c. ice cold packs
Q. 10. Resistant posterior epistaxis requires ligation of
a. anterior ethmoidal artery d. internal jugular vein

b. common carotid artery e. sphenopalatine artery


c. internal carotid artery
Scenario A: A 15 years old boy presented in casualty with severe left sided nose bleed for
last one hour. His vitals are stable, anterior rhinoscopy revealed an active bleeding point at
Little’s area on left side of the septum.

CONCEPT BOOK OF DISEASES OF EAR, NOSE & THROAT BY FAHIM AWAN


277 Part C Nose and Paranasal Sinuses
1. Enumerate local causes of epistaxis.
2. What are various options to deal with this bleeding point?
B: A 10 years old boy presented in emergency with severe nose bleed for last 01 hour. On
clinical examination he looks pale and has rapid thready pulse. Anterior rhinoscopy revealed
clotted blood in right nasal cavity.
1. Enumerate general causes of epistaxis.
2. What investigations will you do to make a diagnosis in this patient?
C: A 40 years old patient presented to accident and emergency department with active nose
bleeding for the last 01 hour.
1. What are basic principles for managing active nose bleeding?
2. What are the various methods to control nose bleeding?
3. How will you assess the general condition of the patient and treat it?
ANSWERS--------------------------------------------------
-------------------------------------------------------- ---
Q.1. b 2. b 3. a 4. b 5. c
6. a 7. d 8. e 9. b 10. e
Scenario A: 1. Traumatic, infective, neoplastic, granulomatous, atmospheric, DNS with spur,
long standing F.B 2. Pinching, cold sponging, chemical cautery, anterior nasal packing,
posterior nasal packing.
B: 1. Acute infective disease, hypertension, liver and kidney disease, drugs like
anticoagulants, hemostatic diathesis 2. Investigate the patient according to cause of epistaxis.
C: 1. Rule out the cause of epistaxis 2. Pinching, cold sponging, chemical cautery, anterior
nasal packing, posterior nasal packing, ligation of vessel, embolization

3. Record vitals, take detail history especially about amount of blood loss, detail examination
of nose, maintain i.v line, send blood for grouping and cross matching.
--------------------------------------------------------------------------------------------------------------------------
-
HEREDITARY HEMORRHAGIC TELANGIECTASIA (HHT)
Synonyms: Osler’s disease, Rando Osler Weber/ROW disease
Etiopathology:
It is an autosomal dominant hereditary disorder. It is
characterized by formation of multiple telangiectasia in the mucous
membrane and skin especially of the head. The telangiectasia
malformations may also occur in the lungs, liver and gut.
Hereditary hemorrhagic
Hemorrhage may occur from any telangiectasia.
telangiectasia
Clinical features:
1. Patient may present with recurrent epistaxis due to lesions in the nose.
2. On examination there are small scattered purplish or reddish lesions on skin as well as on
CONCEPT BOOK OF DISEASES OF EAR, NOSE & THROAT BY FAHIM AWAN
278 Part C Nose and Paranasal Sinuses

nasal mucosa.

Treatment:
A wide variety of options are available for the treatment of epistaxis secondary to hereditary
hemorrhagic telangiectasia.
Remember the mnemonic ‘OSLER’.
Oestrogen therapy, Septodermoplasty/injection of Sclerosing agent, Laser (Argon, KTP or
Nd: YAG), Excision of affected segment, Radiotherapy/Rhinectomy.
More recent options include arterial ligation and arterial embolization.
Repeated blood transfusion may be required to correct anemia and to maintain
hemodynamics. In more severe cases of recurrent epistaxis, Young’s operation may be
considered.
SELF ASSESSMENT
Q. 1. Hereditary hemorrhagic telangiectasia is also called
a. Osler’s disease d. both a & b

b. Rando Osler Weber disease e. both b & c


c. Von Willebrand disease
Q. 2. Cause of epistaxis in hereditary hemorrhagic telangiectasia is
a. autosomal dominant inheritance d. sex linked recessive inheritance

b. autosomal recessive inheritance e. none of the above


c. sex linked dominant inheritance
Q. 3. Cause of epistaxis in hereditary hemorrhagic telangiectasia is
a. decreased number of platelets d. none of the above

b. deficiency of factor VIII e. all of the above


c. deficiency of factor IX
Q. 4. All are treatment options of hereditary hemorrhagic telangiectasia except
a. chemotherapy d. radiotherapy

b. excision of affected segment e. septodermoplasty


c. laser
Q. 5. All are treatment options of hereditary hemorrhagic telangiectasia except
a. arterial embolization d. injection of sclerosing agent

b. arterial ligation e. oestrogen therapy

c. immunotherapy
Scenario: A twenty years old patient presented with recurrent nasal bleeding for last ten
years. On examination there are small scattered purplish/reddish lesions on skin as well as
on nasal mucosa.
CONCEPT BOOK OF DISEASES OF EAR, NOSE & THROAT BY FAHIM AWAN
279 Part C Nose and Paranasal Sinuses
1. What is your provisional diagnosis?
2. How will you assess the general condition of the patient?
3. What are various methods to manage nose bleeding?
ANSWERS------------------------------------------------
-------------------------------------------------------- -----
Q.1. d 2. a 3. d 4. a 5. c
Scenario: 1. Hereditary hemorrhagic telangiectasia 2. Detailed history especially about
amount of blood loss, detailed examination of nose, record vitals, maintain i.v line, send
blood for grouping and cross matching 3. Pinching of nose, cold sponging, chemical cautery,
anterior nasal packing, posterior nasal packing, ligation of vessels, embolization.
--------------------------------------------------------------------------------------------------------------------------
SEPTAL HEMATOMA
Definition:
It is a collection of blood beneath the perichondrium/periosteum of the nasal septum.
Etiopathology:
It is nearly always traumatic in origin and may be due to;
1. Direct blow or fall on the nose.
2. Surgery on the nose e.g., septoplasty, SMR.
3. Hemostatic diathesis (rarely). A case of septal hematoma
Clinical features:
1. History of trauma to nose is usually available.
2. Bilateral nasal obstruction is the most common
presenting symptom.
3. There may be sense of pressure over nasal bridge.
On examination there is;
1. Bilateral, smooth, reddish, rounded septal swelling
which may extend to the lateral nasal wall.
2. On palpation with a probe, septal swelling is soft,
fluctuant and compressible.
Diagramatic view of septal hematoma
3. Wide bore needle aspiration also confirms the
presence of blood.
Complications:
1. Septal cartilage has got no blood supply of its own. It gets its nutrients and oxygen from
overlying perichondrium. If untreated septal cartilage may undergo avascular necrosis
within 2-3 days leading to saddle nose or septal perforation.
2. Septal hematoma if infected, may change into septal abscess.
3. There may be permanent thickening of the septum because of fibrous organization.
280 Part C Nose and Paranasal Sinuses

4. If this type of injury occurs during childhood, it may affect development of whole of the
middle-third of the face with resultant maxillary hypoplasia.
Treatment:
1. Aspiration with wide bore needle may be sufficient
in early cases where septal hematoma is small, but it
may have to be repeated.
2. Incision and drainage of the hematoma is done
with excision of small piece of mucoperichondrium
on one side. Incision is made horizontal to avoid
disruption of mucociliary transport. A corrugated
drain is placed between the flaps for drainage.
3. Anterior nasal packing, splints or quilting sutures
are applied to obliterate the potential space.
4. Systemic antibiotics are given to prevent secondary
infection.
5. Revaluation is done after 2-3 days to detect any Drainage steps of septal hematoma
recollection.
SELF ASSESSMENT
Q. 1. In septal hematoma, blood accumulates mainly in
a. submucous space d. none of the above

b. subperichondrial space e. all of the above


c. subperiosteal space
Q. 2. The most common cause of septal hematoma is
a. granuloma d. neoplasia

b. hemostatic diathesis e. trauma


c. inflammation
Q. 3. Chondrocyte in avascular cartilage die in
a. 12-24 hours d. 5-7 days

b. 2- 3 days e. 7-9 days


c. 4-5 days
Q. 4. A young patient following nasal trauma one day back presents in emergency.
Diagnosis of septal hematoma is made. Predominant symptom of septal hematoma is
a. bilateral nasal obstruction d. loss of smell

b. bleeding from nose


c. excessive sneezing e. severe pain over forehead

Q. 5. All are usual clinical features of septal hematoma except


a. bilateral reddish rounded septal c. fluctuation is positive
swelling
d. rise in body temperature
b. bilateral nasal obstruction
e. swelling is soft in nature

CONCEPT BOOK OF DISEASES OF EAR, NOSE & THROAT BY FAHIM AWAN


281 Part C Nose and Paranasal Sinuses
Q. 6. A 02 years old child fell from bed about 06 hours back. Casuality medical offi-cer has
seen bilateral smooth rounded swelling in nasal cavities. The diagnosis is
a. choanal atresia d. septal hematoma

b. deflected nasal septum e. septal perforation

c. septal abscess
Q. 7. Treatment of septal hematoma is

a. anterior nasal packing c. incision & drainage

b. excision of small piece of mucoperi- d. insertion of drain

chondrium
e. all of the above
Scenario: A 05 years old baby was hit on nose by a fellow child. He developed complete
blockage of nose within one hour when he was brought to the ENT surgeon. There is no
epistaxis. Anterior rhinoscopy showed smooth reddish diffuse swelling in both nasal cavities.
1. What is the most probable clinical diagnosis?
2. What complications can arise from this condition?
3. How will you treat this patient?
ANSWERS------------------------------------------------
-------------------------------------------------------- -----
Q.1. b 2. e 3. b 4. a 5. d
6. d 7. e
Scenario: 1. Septal hematoma 2. Avascular necrosis of septal cartilage, saddle nose
deformity, septal abscess, fibrous thickening of septum, maxillary hypoplasia 3. Wide bore
needle aspiration, incision and drainage, anterior nasal packing, systemic antibiotics.
--------------------------------------------------------------------------------------------------------------------------
SEPTAL ABSCESS
Etiopathology:
It is usually secondary to infection of septal hematoma. Occasionally it may also occur
spontaneously after nasal furunculosis, measles or typhoid.
Clinical features:
1. Bilateral nasal obstruction is usual presenting symptom. A case of septal
abscess
2. Severe throbbing pain over bridge of nose.
3. Rise in the body temperature and pulse rate which distinguish septal abscess from
septal hematoma.
On examination;
1. Skin of nose is red and swollen.
2. Nasal cavities show smooth, dull, purplish or whitish, rounded bilateral septal
swelling.
3. Fluctuation can be elicited with a probe.
282 Part C Nose and Paranasal Sinuses

4. Occasionally submandibular lymph nodes may be enlarged and tender.


Complications:
1. Cavernous sinus thrombosis and meningitis; As the septal abscess is lying in the
danger area of the face, cavernous sinus thrombosis and meningitis may occur.
2. Saddle nose or supratip depression; It results from septal cartilage necrosis.
3. Septal perforation may also occur.

Treatment:
1. Admission in hospital is carried out and vitals monitored regularly.
2. Urgent incision and drainage as for septal hematoma, is carried out. Necrosed piece of
cartilage is removed. At the same time conchal cartilage graft may be used to avoid
external nasal deformity. Incision may require to be open daily for 2-3 days to drain pus.
3. Parenteral antibiotics are given to prevent spread of infection.
4. Anti-inflammatory, analgesics are given to relieve pain.
5. In future most of the patients may need surgical correction of the nasal deformity caused
by septal abscess.
SELF ASSESSMENT
Q. 1. All are the usual clinical features of septal abscess except
a. bilateral whitish rounded septal swelling c. nasal obstruction
d. normal body temperature
b. fluctuation is positive e. severe throbbing pain
Q. 2. Regarding septal abscess
a. causes destruction of nasal bones d. leads to infratip depression

b. destroys septal cartilage e. leads to orbital cellulitis


c. follows furuncle of ear
Q. 3. Usual complications following septal abscess include all except
a. cavernous sinus thrombosis d. saddle nose

b. meningitis e. severe epistaxis


c. septal perforation
Q. 4. All are treatments of septal abscess except
a. admission in hospital d. parenteral antibiotics

b. anti-inflammatory, analgesics e. urgent incision and drainage

c. cytotoxic drugs
Scenario: A 23 years old boy following a fight received blow on nose. After 04 days he
approached ENT consultant with swelling of nose and severe bilateral nasal obstruction.
CONCEPT BOOK OF DISEASES OF EAR, NOSE & THROAT BY FAHIM AWAN
283 Part C Nose and Paranasal Sinuses

There is complaint of high-grade fever. On examination of nasal cavities there is diffuse


swelling of either side of nasal septum.
1. What is the most likely diagnosis?
2. What are possible complications of this entity?
3. What is the treatment of this patient?
-------------------------------------------------------- ANSWERS-----------------------------------------------------
Q.1. d 2. b 3. e 4.c
Scenario: 1. Septal abscess 2. Cavernous sinus thrombosis, meningitis, nasal deformity,
septal perforation 3. Incision and drainage, antibiotics, anti-inflammatory, analgesics.
--------------------------------------------------------------------------------------------------------------------------
SEPTAL PERFORATION
Etiopathology:
1. Trauma such as;
a. Septal surgery
b. Repeated cautery
c. Digital trauma (nose picking)
d. Bullet or penetrating injuries to the nasal septum
e. Customs (as piercing of septum)
2. Tumors of nose and paranasal sinuses
3. Granulomatous diseases. Tuberculosis, lupus, leprosy cause perforation
in cartilagenous part while syphilis causes perforation in bony part. Septal
perforation
4. Rhinolith or neglected foreign body may lead to pressure necrosis of
nasal septum.
5. Chronic infections such as tuberculosis, syphilis, leprosy etc.
6. Maggots
7. Chemicals such as cocaine, topical steroids, topical
decongestants and certain industrial chemicals such as
chromium, arsenic and mercury.
8. Atrophic rhinitis and rhinitis sicca
9. Idiopathic
Clinical features:
1. There may be no symptom at all and septal perforation may
be discovered incidentally.
2. Whistling during respiration through nose occur in anterior and small Septal
perforations. perforation

3. Blockage of nose secondary to crusting occur in posterior or large perforations.


CONCEPT BOOK OF DISEASES OF EAR, NOSE & THROAT BY FAHIM AWAN
284 Part C Nose and Paranasal Sinuses

4. There may be irritation in the nasal cavities and recurrent epistaxis.


5. Perforation is usually visible on anterior rhinoscopy. Sometimes nasal endoscopy is
required for diagnosis of small perforations.
Investigations:
1. History is of great importance in the diagnosis of traumatic and occupational cases.
2. When the margins of the perforation look raised and hypertrophic, a biopsy must be
taken to rule out granulomatous diseases especially, Wegener’s granuloma.
Treatment:
It is very important to cure the causative disease process and
then to encourage natural heal-ing. If the perforation is
asymptomatic and idiopathic, no treatment should be given.
A. Conservative treatment:
1. Alkaline nasal douches and 25% glucose in glycerin nasal
drops are given in cases of excessive crusting.
2. Silver nitrate cautery of the bleeding granulation may be
done.
3. Prefabricated silastic buttons or obturators are simple,
safe and reliable method of closing almost any septal
perforation.
B. Surgical treatment: Silastic buttons for closure of septal perforation

Surgical closure of the perforation may be carried out


with grafts or local flaps.
SELF ASSESSMENT
Q. 1. The most common cause of perforation of nasal septum is

a. cocaine sniffing c. granulomatous disease

b. chemicals d. trauma

e. tumor of nose and paranasal sinuses


Q. 2. Usual symptoms of nasal septal perforation include all except
a. anosmia d. recurrent nasal bleeding

b. crusting e. whistling
c. nasal obstruction
Q. 3. Whistling sound in nasal septal perforation is usually caused by
a. large anterior perforation d. small anterior perforation

b. large central perforation e. small posterior perforation


c. large posterior perforation
Q. 4. Treatment of nasal septal perforation is
a. flaps c. obturators

b. grafts d. prefabricated silastic buttons

CONCEPT BOOK OF DISEASES OF EAR, NOSE & THROAT BY FAHIM AWAN


285 Part C Nose and Paranasal Sinuses
e. all of the above

Scenario: A young patient complains of whistling sounds during respiration through nose for
last 09 months. He has history of septal surgery 01 year before.
1. What is the most probable clinical diagnosis?
2. How will you treat this patient?
-------------------------------------------------------- ANSWERS-----------------------------------------------------
Q.1. d 2. a 3. d 4.e
Scenario: 1. Septal perforation 2. A. Conservative; alkaline nasal douches, silver nitrate
cautery of the bleeding granulation, prefabricated silastic buttons or obturators are simple,
safe and reliable method of closing almost any septal perforation.
B. Surgical treatment; surgical closure of the perforation may be carried out with grafts or
local flaps.
--------------------------------------------------------------------------------------------------------------------------
MAGGOTS
Synonym: Myiasis
Definition:
Larvae of a common house fly (genus Chrysomyia) are known as maggots.
Etiopathology:
It is a disease of temperate climate where flies are abundant.
Houseflies are commonly attracted towards debilitated, neglected,
unconscious, or addicted patients. Flies lay about 500 eggs in her
lifetime, in several batches of about 75-150 at one time. These eggs
hatch into larvae (maggots) within 24 hours. If sufficient food is
available in the form of crusts, dead or decaying matter, larvae do
not undergo further metamorphosis. A case of maggot nose
Maggots are about 1.5 cm in length. These are common in
the nose in patients with atrophic rhinitis and purulent nasal discharge. These eat away the
mucosa, septum, turbinate, palate and cribriform plate. Maggots may spread to paranasal
sinuses and through the nasolacrimal duct to the lacrimal sac. Because of superadded
infection, there is foul-smelling discharge and bleeding from the affected site.
These are also common in patients with aural discharge, infected tracheostomy wound or
chronic malignant ulcer. Maggots may occur at any age and in either gender.
Clinical features:
1. Severe irritation, sneezing and lacrimation are the presenting symptoms in the initial 3-
4 days.
NCEPT BOOK OF DISEASES OF EAR, NOSE & THROAT BY FAHIM AWAN
286 Part C Nose and

2. Maggots may be coming out of nose.


3. There may be complaint of diffuse swelling around the nose and eye, nasal obstruction,
epistaxis, facial pain and headache.

On examination;

1. Foul smell from the patient.


2. Edema of nose cheeks and eyelids.
3. Necrotic material, congested edematous and ulcerated mucosa with embedded maggots.
4. There may be septal or palatal perforation.
Death may result from sepsis, meningitis or suicide.
Treatment:
1. Patient is admitted in hospital and isolated with a mosquito net to avoid contact with
flies.
2. Urgent removal of the maggots with suction or forceps is carried out under direct vision
with headlight or endoscope. Hidden maggots may be removed after a spray or wick
soaked in maggot oil [equal parts of turpentine oil (which irritates the maggots to come
out of the hidden site) and chloroform (which anesthetizes the maggots)].
3. Saline douching is done to remove crusts, slough and dead maggots.
4. Analgesics are given to relieve pain.
5. Antibiotics are given to prevent associated infection. Underlying cause such as poor
hygiene or atrophic rhinitis is addressed.
SELF ASSESSMENT
Q. 1. Maggots are also known as

a. larva of mosquito d. pupae form of mosquito

b. myiasis e. none of the above

c. pupae form of housefly


Q. 2. Maggots are

a. adult form of housefly d. pupae form of housefly

b. larva form of housefly e. pupae form of mosquito


c. larva form of mosquito
Q. 3. All are usual clinical features of maggots except
a. diffuse swelling of nose c. flies coming out of nose

b. facial pain & headache d. maggots coming out of nose


CONCEPT BOOK OF DISEASES OF EAR, NOSE & THROAT BY FAHIM AWAN
287 Part C Nose and Paranasal Sinuses

e. nasal obstruction and epistaxis


Q. 4. Maggot oil contains

a. chloroform d. both a & b

b. glycerine e. both a & c


c. turpentine oil
Q. 5. All are treatment options of maggots except
a. analgesics d. steroids

b. antibiotics e. urgent removal

c. maggot oil
Scenario: A 60 years old, emaciated female was brought to ENT OPD with complaint of
epistaxis, nasal obstruction and severe pain in the nose for last three days. Her examination
revealed multiple whitish moving foreign bodies in the nose with septal and palatal
perforations.
1. What is the most probable clinical diagnosis?
2. What are predisposing factors for the condition?
3. How will you treat this patient?
-------------------------------------------------------- ANSWERS-----------------------------------------------------
Q.1. b 2. b 3. c 4. e 5. d
Scenario: 1. Nasal myiasis 2. Debilitated, neglected, unconscious, or addicted patients are at
more risk 3. Admission in hospital, urgent removal of maggots using maggot oil, saline
douching, analgesics, antibiotics.
--------------------------------------------------------------------------------------------------------------------------
FOREIGN BODY OF NOSE
Etiopathology:
These may enter the nose by different routes:
1. Through the anterior nares
2. Through the posterior nares by vomitus
3. Through penetrating injuries
4. May arise de novo as a result of inflammatory reaction around A case of foreign body nose
dried crust, blood or mucous.
Foreign bodies are mostly seen in children. Anything small enough to pass the anterior nares
may be pushed into the nose by the child himself or the playmates. Pieces of paper, beads,
buttons, eraser, pebbles, sponge, button batteries and cotton wool are common foreign bodies.
Clinical features:
288 Part C Nose and Paranasal Sinuses

1. History of introduction of a foreign body is usually obtained.


2. Pain may occur soon after introduction.
3. There is complaint of unilateral nasal obstruction.
4. There may also be complaint of sneezing with irritation. If foreign body is present for
long time, it may lead to unilateral foul-smelling nasal discharge which may very
occasionally be bloodstained nasal discharge.
On examination;
1. Foreign body is commonly visible near the floor of nasal cavity.
2. Probe test may be helpful in identifying the foreign body.

Complications:
1. Rhinitis
2. Sinusitis
3. Rhinolith formation

Investigations:
Diagnosis is usually made on history and examination of nose after using nasal decongestant
drops/sprays.
X-ray paranasal sinus shows radiopaque foreign body. In long standing cases there is
haziness of the ipsilateral maxillary and other sinuses secondary to associated sinusitis.
Treatment:
Sometimes there may be spontaneous expulsion of the foreign body. Forceful
nose blowing may expel the foreign body.
1. Whenever possible try to get a template of the foreign body. It helps in
making the strategy and choosing proper instruments for its safe removal. It
also helps in anticipating the possible complications of the foreign body and its
removal.
2. Removal under direct vision through anterior nares with the help of Foreign body hook
foreign body hook/forceps/sucker with or without local anesthesia. If
retrieval of the foreign body fails, it may be pushed into the nasopharynx
under general anesthesia and thus, may be extracted through oral route.
3. Nasoendoscopy is very helpful in locating and removing a small foreign body.
SELF ASSESSMENT
Q. 1. Source of foreign body of nose may be

a. anterior nares d. posterior nares

b. De novo e. All of the above

c. penetrating injuries
CONCEPT BOOK OF DISEASES OF EAR, NOSE & THROAT BY FAHIM AWAN
289 Part C Nose and Paranasal Sinuses
Q. 2. All are usual clinical feature of foreign body nose except
a. bilateral nasal obstruction d. sneezing

b. irritation e. unilateral nasal discharge


c. nasal bleeding
Q. 3. All are included in the treatment of foreign body nose except
a. Caldwell-Luc operation d. suction

b. to get a template of foreign bod e. use of foreign body hook or forceps

c. nasal endoscopy
Scenario: A 04 years old child presents with mucopurulent discharge from right side of nose
for last two days. There is preceding history of playing pebbles with his playmates. On
examination right nasal cavity is severely congested.
1. What is the most probable clinical diagnosis?
2. What are possible complications of this entity?
3. How will you treat this patient?
------------------------------------
-------------------- ANSWERS-----------------------------------------------------
Q.1. e 2. a 3. a
Scenario: 1. Foreign body nose 2. Rhinitis, sinusitis, rhinolith formation 3. Removal under
general anesthesia.
--------------------------------------------------------------------------------------------------------------------------
RHINOPHYMA
Synonym: Potato tumor of nose, Elephantiasis of nose
Definition:
It is a condition associated with benign nodular enlargement of tip of nose due to hypertrophy
of the sebaceous glands of skin.
Etiopathology:
It is usually seen in long standing cases of acne rosacea. It occurs almost exclusively in
middle-aged male.

Clinical features:
Patient presents with nodular swelling of lower half of nose
especially nasal tip. This gives an unsightly appearance of nose. There
may also be nasal obstruction.
Treatment: A case of
rhinophyma
Removal of the lesion is carried out with surgical knife or CO2 laser. Decortication of
CONCEPT BOOK OF DISEASES OF EAR, NOSE & THROAT BY FAHIM AWAN
290 Part C Nose and Paranasal Sinuses

thickened skin or dermabrasion has also been tried.


SELF ASSESSMENT
Q. 1. Rhinophyma is also called

a. elephantiasis of nose d. both a & c

b. potato tumor of ear e. both b & c

c. potato tumor of nose


Q. 2. Rhinophyma is a disorder of

a. ceruminous gland d. sweat glands

b. epidermis e. none of them


c. sebaceous glands
Q. 3. All of the following statements about rhinophyma are true except
a. due to hypertrophy of sebaceous d. occurs almost exclusively in middle
Gland Age

b. more common during childhood e. tip of the nose is usually affected

c. occurs almost exclusively in males


Scenario: A middle-aged man presented with very ugly looking nodular swelling of lower
half of nose especially nasal tip for last 07 years. Patient has also got acne rosacea.
1. What is the most probable clinical diagnosis?
2. What are the treatment options?
-------------------------------------------------------- ANSWERS-----------------------------------------------------
Q.1. d 2. c 3. b
Scenario: 1. Rhinophyma 2. Removal of the lesion with surgical knife or CO2 laser,
decortication of thickened skin or dermabrasion.
--------------------------------------------------------------------------------------------------------------------------
-
CYSTIC FIBROSIS
Synonyms: Mucoviscidosis, Salty baby syndrome
Etiopathology:
It is an autosomal recessive disease in which there is primary defect (mutation) in the long
arm of chromosome number 7. Respiratory and digestive systems are the most commonly
affected. Twenty percent of patients with cystic fibrosis form nasal polypi.
All the clinical and diagnostic features are because of abnormalities of secretions of
exocrine glands. There is high level of sodium and chloride in the sweat.
Plasma immunoreactive trypsin remains elevated for first three months of life. So, its
estimation may be used for neonatal screening.

CONCEPT BOOK OF DISEASES OF EAR, NOSE & THROAT BY FAHIM AWAN


291 Part C Nose and Paranasal Sinuses

Clinical features:
A. Neonates:
In neonates it may present in different forms such as;
1. Meconium ileus
2. Meconium perforation and peritonitis
3. Obstructive jaundice A diagram of
4. Neonatal screening raised immunoreactive trypsin cystic fibrosis

5. Positive sweat test


B. Infants:
In infants it may present in different forms such as:
1. Failure to thrive
2. Recurrent loose stools
3. Recurrent lower respiratory tract infection
C. Children and adults:
In children and adults, it may present in different forms such as;
1. Nasal polyps and sinusitis
2. Recurrent wheeze, cough and purulent sputum production
3. Male infertility
4. Biliary cirrhosis
5. Diabetes mellitus
6. Heat exhaustion
7. Anemia and edema
8. Rectal prolapse
Investigations:
Sweat test is the only diagnostic test for cystic fibrosis. This is done with the use of
pilocarpine ionotophoresis and collection of sweat for sodium and chloride estimation. A
value of this salt more than 100 mM is diagnostic of cystic fibrosis in the presence of
clinical symptoms. Normal value of this salt is 20-30 mM.
Treatment:
Patient needs not only medical treatment but psychological, genetic, educational and
occupational issues should also be addressed.
Antibiotic treatment is frequently required for the infection of nose, paranasal sinuses,

CONCEPT BOOK OF DISEASES OF EAR, NOSE & THROAT BY FAHIM AWAN


292 Part C Nose and Paranasal Sinuses

nasopharynx, middle ear and chest. Meconium ileus, alimentary problem, diabetes mellitus,
infertility and psychological problems should be addressed on their own merits.
Prognosis:
Seventy percent of newborn infants with cystic fibrosis are expected to reach adult life if
managed properly.
SELF ASSESSMENT
Q. 1. Cystic fibrosis is also known as

a. Kartagener’s syndrome d. both a & b

b. mucoviscidosis e. both b & c


c. salty baby syndrome
Q. 2. The primary genetic defect in cystic fibrosis is in the
a. long arm of chromosome No. 7 d. short arm of chromosome No. 8

b. long arm of chromosome No. 8 e. short arm of chromosome No. 9


c. shot arm of chromosome No. 7
Q. 3. Which of the following is found in excess in the sweat of the patient with cystic
fibrosis?
a. Calcium d. Sodium chloride

b. Phosphorus e. Zinc
c. Potassium
Q. 4. Glue like glassy white to yellow secretions obstruct the nasal lumen seen in cystic
fibrosis. This can be diagnosed by sodium and chloride estimation of these substances in
excess of 100 mM in
a. blood d. sweat

b. nasal secretion e. urine


c. serum
Q. 5. The common neonatal presentation of cystic fibrosis
a. hepatosplenomegaly d. nasal polypi

b. male infertility e. peritonitis

c. meconium ileus
Scenario: A 8 years old female child presents with history of chronic sinusitis and multiple
nasal polypi since birth. She has recurrent attacks of chest infection and malabsorption
syndrome. There is no situs inversus or dextrocardia.
1. What is the most probable clinical diagnosis?
2. Name single test for its diagnosis?
3. How will you treat this patient?

CONCEPT BOOK OF DISEASES OF EAR, NOSE & THROAT BY FAHIM AWAN


Part C Nose and Paranasal
293 Sinuses
-------------------------------------------------------- ANSWERS-----------------------------------------------------
Q.1. e 2. a 3. d 4. d 5. c
Scenario: 1. Cystic fibrosis 2. Sweat test 3. Patient needs not only medical treatment but
psychological, genetic, educational and occupational issues should also be addressed.
Antibiotic treatment is frequently required.
--------------------------------------------------------------------------------------------------------------------------

MUCOCELE
Definition:
It is a mucous containing cyst completely filling an air sinus and
capable of expansion.
Etiopathology:
It occurs as a result of acute infection, chronic infection, tumor or
trauma. Opening of an air sinus may become permanently stenosed.
Continued secretions in the obstructed sinus lead to increase in
pressure within the sinus cavity with consequent slow expansion.
A number of bone resorbing substances such as
Case of frontal sinus mucocele
prostaglandins (PGE2), leukotrienes and cytokines are produced by
mucocele. This phenomenon most commonly occurs in the frontal
sinus where only 02 mm wide duct passes for a distance of about 05-10 mm.
As the sinus expands, floor of the frontal sinus that is orbital roof gives away and
mucocele expands into the orbit and pushes the eyeball downwards and forwards.
Mucocele of the frontal sinus usually presents in 90% of cases on the superomedial
quadrant of orbit. Anterior wall of the frontal sinus may be pushed forwards and patient
presents with forehead swelling.
In children, a mucocele from the ethmoid sinus is more common and the mucocele
expands into the orbit. The lateral wall of the sinus i.e., lamina papyracea is pushed laterally,
the eyeball is pushed forwards and laterally.
Mucocele of the maxillary sinus and sphenoid sinus is extremely rare.
Clinical features:
These depend highly on the exact sinus and particular wall involved.
1. In cases of frontal sinus mucocele, there is downwards, forwards and lateral
displacement of eyeball. There may be swelling over the forehead.
2. In case of ethmoidal sinus mucocele, there may be forwards and lateral displacement
of the eyeball.
3. On palpation there may be ‘egg-shell crackling sensation’.
4. Diplopia may occur very rarely as there is gradual displacement of eyeball.
Investigations:
1. Digital X-ray of paranasal sinuses shows characteristic
radiological appearances of the
94 Part C Nose and Paranasal Sinuses

affected sinuses.
On radiology there is;
CT scan of mucocele
a. Expansion of affected sinus
b. Loss of translucency
c. Loss of scalloping
d. Marginal sclerosis

2. CT/MRI are required to differentiate mucocele from benign and malignant tumors of the
sinus.
Treatment:
In majority of cases endonasal approach is used. In complicated or extensive mucocele an
external ethmoidectomy (Lynch-Howarth) approach, endonasal approach or FESS is used.
Following steps are carried out in surgery;
1. Mucocele is drained.
2. All the lining of the affected sinus is removed.
3. Permanent wide drainage of the sinus is established.
4. In case of frontal sinus mucocele
a. One cm wide fenestrated tube is placed for 3-5 months.
Intersinus septum is breached to allow drainage into contralateral sinus if the
blockage of the sinus recurs.
Differential diagnosis:
1. Fungal rhinosinusitis
2. Pseudotumor
3. Benign and malignant tumors of orbit, nose, paranasal sinuses and nasopharynx.

SELF ASSESSMENT
Q. 1. In 90% cases mucocele of frontal sinus usually presents on the
a. inferolateral quadrant of orbit d. superomedial quadrant of orbit

b. inferomedial quadrant of orbit e. superolateral quadrant of orbit


c. posterolateral quadrant of orbit
Q. 2. In children mucocele is more common in
a. ethmoidal sinus d. maxillary sinus
b. frontal sinus e. sphenoidal sinus
c. lateral sinus
Q. 3. In adults mucocele is more common in
a. ethmoidal sinus b. frontal sinus
CONCEPT BOOK OF DISEASES OF EAR, NOSE & THROAT BY FAHIM AWAN
295 Part C Nose and Paranasal Sinuses

c. lateral sinus e. sphenoidal sinus


d. maxillary sinus
Q. 4. Fronto-ethmoidal mucocele secretes all the following bone resorbing substances
except
a. cytokines d. prostacyclins

b. leukotrienes e. prostaglandins
c. PGE2
Q. 5. Usual clinical feature of mucocele includes
a. displacement of eyeball sinus

b. eggshell crackling sensation d. swelling over forehead

c. erosion of the bone of floor of frontal e. all of the above


Q. 6. Radiology of frontoethmoidal mucocele shows
a. loss of scalloping d. all of the above

b. loss of translucency e. none of the above


c. marginal sclerosis
Q. 7. In treatment of frontoethmoidal mucocele, following step is carried out during
surgery.
a. All the mucosa of affected sinus is placed

Removed
d. Permanent wide drainage is estab-
b. Mucocele is drained lished

c. One cm wide fenestrated tube is e. All of the above


Scenario: A 30 years old patient presented in ENT OPD with forehead swelling for last 01
year. On inspection eyeball is pushed downwards and forwards. On palpation, there is
eggshell crackling sensation.
1. What is the most probable clinical diagnosis?
2. What investigation will you carry out?
3. What are the positive findings on radiological investigations?
--------------------------------------------------------ANSWERS-----------------------------------------------------
Q.1. d 2. a 3. b 4. d
5. e 6. d 7. e
Scenario: 1. Frontoethmoidal mucocele 2. X-ray, CT, MRI 3. Loss of translucency, loss of
scalloping, marginal sclerosis.
--------------------------------------------------------------------------------------------------------------------------
-

CONCEPT BOOK OF DISEASES OF EAR, NOSE & THROAT BY FAHIM AWAN


296 Part D Oral Cavity and Pharynx

PART D
ORAL CAVITY AND
PHARYNX
Chapter 11
SYMPTOMATOLOGY AND EXAMINATION
OF THROAT
SYMPTOMATOLOGY OF THROAT
Every system and organ in the body has got its own set of symptoms and signs. More than
90% of patients presenting to ENT department, present with one or more of the following
complaints;
1. Sore throat (pain in throat)
2. Odynophagia (painful swallowing)
3. Dysphagia (difficulty on swallowing)
4. Dysphonia (difficulty in phonation)
5. Dyspnea (difficulty in respiration)
6. Stridor (noisy respiration)
7. Bloodstained spitting or frank bleeding from oral cavity
8. Swelling in neck, oral cavity or throat
9. Speech defects
1. Sore throat: (pain in throat)
This is an important symptom in inflammatory conditions of the throat. Whenever a patient
complains of pain in the throat, he should be categorically asked whether he feels pain or is it
just a sense of irritation. The former is typical of inflammation, whereas the latter is met with
in allergy of the throat and should surgeon to avoid surgery on throat unless clearly indicated
by the local condition of the tonsils. The patient should be questioned regarding exact
duration, site, progress, intensity, persistence of pain. Duration of each episode and the length
of the intervals in between the episodes should be asked.
2. Odynophagia: (pain on swallowing)
297 Part D Oral Cavity and Pharynx

In inflammatory conditions of the throat the patient feels pain during swallowing.
3. Dysphagia: (difficulty in swallowing)
In tumors of the pharynx, the food gets obstructed on its way down producing obstructive
dysphagia. Since pharynx is a very wide cavity, dysphagia in tumors of the pharynx is a late
symptom.
In paralysis of the palate and the pharynx, the food cannot be swallowed properly and it
may find its way into the nose from inability of the palate to shut off the nasopharynx from
oropharynx. This condition is called nasal regurgitation.
In paralysis of the larynx, the food may also find its way into the larynx, as larynx
cannot be shut off adequately on attempted swallowing, there is a severe bout of coughing.
This condition is called tracheobronchial aspiration.
In anemias and atrophic conditions of the pharyngeal mucosa, there is dysphagia due to
lack of secretions of mucous glands which normally lubricate food bolus.
4. Dysphonia: (difficulty in phonation)
Congenital, traumatic, inflammatory/granulomatous conditions and tumors of larynx,
hypopharynx and oropharynx may lead to dysphonia. Unilateral or bilateral paralysis of the
recurrent or superior laryngeal nerve may also lead to dysphonia.
5. Dyspnea: (difficulty in respiration)
Congenital, traumatic, inflammatory/granulomatous conditions and tumors of larynx,
hypopharynx and oropharynx may lead to dyspnea. Bilateral recurrent laryngeal nerve
paralysis may also lead to very severe dyspnea.
6. Stridor: (noisy respiration)
It is harsh, high pitch breath sound produced at or below the level of larynx due to narrowed
air passages. Different congenital, traumatic, inflammatory, granulomatous, neurological
conditions and tumors of the larynx, hypopharynx, oropharynx may lead to stridor.
7. Bloodstained spitting or frank bleeding from oral cavity:
Different traumatic, inflammatory, granulomatous conditions and tumors of oral cavity,
oropharynx, hypopharynx and larynx may lead to bloodstained spitting or frank bleeding
from oral cavity.
8. Swelling in neck, oral cavity or throat:
Different congenital, traumatic, inflammatory, granulomatous conditions and tumors of the
larynx, hypopharynx, oropharynx or thyroid may lead to swelling in neck, oral cavity or
throat.
9. Speech defects:
Pharynx is an important organ for giving resonance to the speech. Thus, in cases of extreme
hypertrophy of the tonsils or a tumor in the pharynx, the speech appears to be very dull.

CONCEPT BOOK OF DISEASES OF EAR, NOSE & THROAT BY FAHIM AWAN


298 Part D Oral Cavity and Pharynx

EXAMINATION OF THROAT
After completing all prerequisites of examination, throat is examined as follows;
1. Inspection:
Any artificial denture should be removed. Inspection is done of the oral cavity and
oropharynx i.e., lips, gums, teeth, upper alveolus, lower alveolus, tongue, movements of
tongue, floor of mouth, hard palate, soft palate and movement of soft palate. Clinician has to
look for;
a. Congenital anomalies
b. Anatomical landmarks
c. Symmetry
d. Any other positive finding (enlarged tonsils, ulcer, swelling, white patch etc.)
Depress anterior two third of the tongue and inspect anterior faucial pillar, tonsils, posterior
faucial pillar, posterior pharyngeal wall. Check movements of the soft palate by asking the
patient to phonate Aah. Check gag reflex by touching separately posterior pharyngeal wall on
each side of the midline.
2. Indirect laryngoscopy (IDL):
Indirect laryngoscopic mirror (singly angulated) is warmed
in the same manner as the mirror for posterior rhinoscopy.
The patient is asked to open his mouth and protrude the Indirect laryngoscopy mirror
tongue.
The tongue is held with a gauze between the thumb below
and index finger above of the left hand. Upper lip is retracted
upwards with the middle finger.
Indirect laryngoscopic
Pre-tested warm mirror is introduced into the oral cavity and examination
soft palate/uvula are retracted up with the indirect laryngoscopic
mirror without touching the posterior pharyngeal wall. View is obtained of the following
structures;
Base of the tongue, valleculae, median glossoepiglottic fold, lateral glossoepiglottic fold,
epiglottis, arytenoids, aryepiglottic folds, pyriform sinuses, false cords, true cords and
subglottis. Movements of the vocal cords are noted by
asking the patient to phonate Eee.
Any pooling of saliva in the pyriform sinus is noted.
3. Check for laryngeal crepitus:
Larynx is held between thumb and index finger and moved
side to side. Normally a sensation is felt which is known as
laryngeal crepitus. It is absent in postcricoid tumor and
retropharyngeal abscess.
4. Lymph nodes:
Checking laryngeal crepitus
Draining lymph nodes should be examined in detail.
299 Part D Oral Cavity and Pharynx

5. Palpation of lesion:
If there is any lesion in the oral cavity or oropharynx, it should be palpated bidigitally
(bimanually). A gloved finger palpates from inside and the other hand from outside.
6. Redrape the patient and pay thanks.
7. Flexible nasopharyngoscopy or 70-degree endoscopy:
In doubtful cases or cases where detailed examination is required,
flexible nasopharyngoscopy or 70-degree endoscopy under local
asesthesia is done for examination of oropharynx, hypopharynx and
larynx. Stroboscopy is done to detect detailed fine movements of
vocal cords. 70-degree endoscopic
examination of larynx
SELF ASSESSMENT
Q. 1. The length of the rima glottidis in male is
a. 1.5 cm d. 4.5 cm

b. 2.5 cm e. 5.5 cm

c. 3.5 cm
Q. 2. The length of glottis in female is

a. 1.6 cm d. 4.6 cm

b. 2.6 cm e. 5.6 cm
c. 3.6 cm
Q. 3. The vocal cord vibration is best analyzed by
a. direct laryngoscopy d. stroboscopy

b. indirect laryngoscopy e. 70-degree endoscopy


c. microlaryngoscopy
-------------------------------------------------------- ANSWERS-----------------------------------------------------
Q.1. b 2. a 3. d
--------------------------------------------------------------------------------------------------------------------------
CONCEPT BOOK OF DISEASES OF EAR, NOSE & THROAT BY FAHIM AWAN
300 Part D Oral Cavity and Pharynx

Chapter 12
DISEASES OF ORAL CAVITY
AND PHARYNX
CLEFT PALATE
Etiopathology:
It occurs due to failure of fusion of maxillary and medial nasal processes
shelf in the midline during intrauterine life. It occurs alone or in
combination with cleft lip. It is classified into;
1. Complete (hard and soft palate)
2. Incomplete (a hole in the palate)
It is subclassified as primary or secondary and unilateral, bilateral or Cleft palate
midline cleft.
The resulting opening between the mouth and nose leads to
velopharyngeal insufficiency.
Submucosal cleft palate is one in which there is a triad of bifid uvula, a
furrow along the midline of the soft palate and a notch in the back of hard
palate. There is bluish line in the midline of soft palate due to absence of
muscle layer.
Clinical features:
1. Nasal regurgitation of fluids is the most common presentation.
2. There is also complaint of recurrent ear infection.
3. In neonates, infants and children there may be complaint of failure
to thrive.
Treatment:
1. Surgical repair of lip is performed after 03 months of age.
2. Repair of cleft is carried out between 6 and 12 months of age.

Several procedures may be required for complete velopharyngeal Grades of cleft palate
closure. Delayed repair may result in impaired speech, recurrent ear
problems in the form of acute, chronic or serous otitis media. There may be gross cosmetic
facial defects. Recently prosthodontic closure with acrylic obturator has been introduced as
an alternative.
SELF ASSESSMENT
Q. 1. All are true of submucous cleft palate except
a. absence of muscle layer in the middle b. bifid uvula
of soft palate
c. furrow along the midline of soft palate
CONCEPT BOOK OF DISEASES OF EAR, NOSE & THROAT BY FAHIM AWAN
301 Part D Oral Cavity and Pharynx

e. notch in posterior border of hard

palate
d. lack of movements in soft palate.
Q. 2. All are clinical features of cleft palate except
a. aspiration d. nasal regurgitation of fluid

b. failure to thrive in children e. recurrent ear infection

c. failure to thrive in infants


Q. 3. Ideal age for repair of cleft lip is

a. after 3 months d. after 4 years

b. after 6 months e. after 6 years


c. after 2 years
Q. 4. Ideal age for repair of cleft palate is
a. between 1-3 months d. between 4-5 years

b. between 6-12 months e. between 5-6 years

c. between 2-3 years


Scenario: An 11 months old child presented in ENT OPD with history of failure to thrive. His
mother gave history of feeding difficulty and nasal regurgitation of fluids since birth. He is
also having recurrent right ear discharge for last 06 months.
1. What is the most probable clinical diagnosis?
2. How will you confirm diagnosis?
3. Write down the treatment options.
-------------------------------------------------------- ANSWERS-----------------------------------------------------
Q.1. d 2. a 3. a 4.b
Scenario: 1. Cleft palate 2. History and clinical examination 3. Surgical repair of cleft palate
--------------------------------------------------------------------------------------------------------------------------
ULCERS OF ORAL CAVITY
There are a number of causes of ulceration of oral cavity. Some of the causes include;
1. Infections e.g., viral, bacterial, fungal.
2. Immune disorders e.g., aphthous ulcers, Behcet’s syndrome.
3. Trauma e.g., sharp edges of broken tooth, artificial denture.
4. Tumors e.g., squamous cell carcinoma, salivary gland tumors.
5. Mucocutaneous disorders e.g., pemphigus vulgaris, mucous
membrane pemphigoid, lichen planus
6. Blood disorders e.g., leukemia, agranulocytosis
7. Drug allergy e.g., Steven Johnson syndrome.
CONCEPT BOOK OF DISEASES OF EAR, NOSE & THROAT BY FAHIM AWAN
302 Part D Oral Cavity and Pharynx

8. Nutritional deficiency e.g., vit B 2 (riboflavin), vit B 12, vit C, nicotinic acid and folic
acid.
9. Miscellaneous conditions e.g., radiotherapy and chemotherapy.
1. Infections:
A. Viral:
a. Herpes labialis: This is common after a cold, local infections,
emotional upset, heat and various unknown factors. Patient presents with
A case of
pricking sensation. On examination there are clusters of vesicles.
herpes labialis
Vesicles may discharge and then there is crust formation. Treatment is
done by topical application of 1% idoxuridine and acyclovir.

b. Herpetiform ulcers: Causative organism is herpes simplex type-1. It is very common in


children. It begins with vesicles, distributed singly or in clumps over the oral mucosal
membrane. Vesicles are rounded, 3-4 mm in size and rupture to form ulcers. These are
often painful. Antiviral agents (idoxuridine, acyclovir) may be used.
c. Hand, foot and mouth disease: It is caused by “A” strain of Coxsackie virus. This is
viral infection which is common among school children. There is ulceration of mucosa
and vesicular rash on extremities. The incubation period is 3-10 days. It is highly
contagious and there is often a mini epidemic in school when every child has disease.
Usually, no treatment is required.
B. Bacterial:
i. Vincent’s angina
ii. Specific bacterial infections e.g., tuberculosis and syphilis
C. Fungal:
Candidiasis: Synonym: moniliasis
Gram positive hyphae of candida are the normal commensal in about 40% of oral cavitie. The
most common strain is candida albicans. It is a “disease of the diseased”.
It is often seen in diabetics. It may be seen in patients on broad-spectrum antibiotics,
steroids, cytotoxic drugs or radiations. It may also be seen in patients with systemic
malignancy.

a. Acute candidiasis: It appears as white, soft, creamy yellow plaques on the oral
mucous membrane. If the plaques are rubbed off, a red area of mucous remains. It is
treated by topical nystatin or imidazole.
b. Chronic hypertrophic candidiasis: It is also known as
candidial leukoplakia. It appears as thick plaques of
keratotic epithelium. The plaques cannot be rubbed off. It
mostly affects angles of mouth. Underlying conditions such
as diabetes, malnutrition and immunosuppression must be
Oral candidiasis
corrected.

CONCEPT BOOK OF DISEASES OF EAR, NOSE & THROAT BY FAHIM AWAN


303 Part D Oral Cavity and Pharynx

2. Immune disorders:
a. Aphthous ulcers
b. Behcet’s syndrome: Synonym “Oculo-Oro-Genital (OOG)”
syndrome. It is characterized by anterior uveitis, genital and
oral ulcers. Oral ulcers are usually similar in appearance to the
major aphthae. If symptoms are severe then systemic steroids are
given.

A case of Behcet’s
syndrome

3. Trauma:
This is usually caused by biting or by sharp edges of broken tooth or artificial denture. If it
does not heal even on removal of initiating factors, it should be biopsied.
4. Tumors:
Malignancy of the oral cavity may present as chronic ulcer. The most common malignancy of
oral cavity is squamous cell carcinoma. Other tumors causing oral ulcers are mixed salivary
gland tumors and non-Hodgkin lymphoma.
5. Mucocutaneous disorders:
A. Pemphigus vulgaris: It is a rare disease
caused by immunological failure. It is
characterized by vesicles or bullae on the skin
and/or mucous membrane. Histologically
epithelial cells lose their attachment to each
other---a phenomenon known as “Acantholysis”.
Mouth lesion usually precede the skin lesion.
Nikolsky’s sign is diagnostic (striking the mucus
membrane causes the bullae or vesicles to
appear). Disease may be mild or severe. In the
severe cases, there may be widespread ulceration
in the mouth and skin. After the rupture of vesicles, A case of pemphigus vulgaris
underlying ulcers are painful. Treatment is carried
out by corticosteroids and azathioprine which must be continued throughout life.
B. Mucous membrane pemphigoid: It is not as fulminant as pemphigus
vulgaris. Bullae and erosions of mucus membrane are found but skin is usually not
affected. Immunological failure is thought to be causative factor. The disease mainly
affects women. Mouth being the most common site. It also affects pharynx, larynx
and soft palate. After the vesicles rupture, the membrane heals with scaring.
Treatment is with systemic corticosteroids.
C. Lichen planus: The condition is very common particularly in women beyond
middle age. Exact cause is unknown but an immunological
cause is suspected. In the mouth the lesions are usually
symmetrical. These are commonest on the buccal mucosa and
A case of lichen planus
tongue. Symptoms may vary in different patients. It may be
detected as an incidental finding. There may be complaint of
roughness of mouth. Lesions may be painful and chewing
food may be difficult.

CONCEPT BOOK OF DISEASES OF EAR, NOSE & THROAT BY FAHIM AWAN


304 Part D Oral Cavity and Pharynx

On examination lesions form whitish striae with lace like pattern on buccal mucosa
or skin. Histologically, there is lymphocytic infiltration and liquefaction
degeneration of basal layer. There are three types of lichen planus:
I. Strial
II. Atrophic
III. Erosive
Treatment includes betnisol tablets 0.5 mg 1-10 tab per day in three divided doses.
6. Blood disorders:
Due to weakness of defense mechanisms for example in leukemia and agranulocytosis,
infections dominate and cause oral ulceration.
7. Drug allergy:
A. Steven Johnson syndrome: Synonym: Acute erythema multiforme. There are split, crusted
and bleeding lips.
B. Certain drugs, penicillin, phenytoin etc may cause mouth ulcers.
8. Nutritional deficiency:
Deficiency of vit B2 (riboflavin), vit B12, vit C, nicotinic acid and folic acid may cause oral
ulceration.
9. Miscellaneous conditions:
Radiotherapy and chemotherapy can lead to oral ulceration.
APHTHOUS ULCERS
There are three types of aphthous ulcers;
1. Minor aphthae:
These are also known as Miculickz aphthae. These are small shallow ulcers, 2-3 mm in
diameter. These have a yellowish floor and a sharply defined red margin. These affect non-
keratinized areas of moveable oral mucosa. The exact etiology is unknown but is said to be
because of immunological abnormalities. These are more common in women than men.
These are more common in sedentary people as compared to manual workers.
2. Major aphthae:
These ulcers are several centimeters in diameter and persist for
many months. A biopsy should be taken to differentiate them from
carcinoma.
3. Herpetiform aphthae:
These ulcers are much less regular in size and shape. These are A case of
characterized by a very large number of small ulcers. These ulcers may aphthous ulceration
305
become confluent with each other.
Treatment:
1. Mouth washes containing tetracyclines, and 0.2% chlorhexidine are given.
2. Corticosteroids are applied locally.
3. Sodium chromoglycate reduces the intensity of pain.
4. Levamisole (ketrax) 40 mg: Three tablets are given as single dose which enhances both
cellular and humoral immune response. It reduces the number, frequency and severity of
aphthous ulcers.
5. Dietary deficiencies if any should be corrected.

SELF ASSESSMENT
Q. 1. Behcet’s syndrome is characterized by the following except
a. genital ulcers c. parotid swelling

d. uveitis
b. oral ulcers e. none of the above
Q. 2. All are true about pemphigus vulgaris except
a. acantholysis d. immunological disorder

b. affects skin and/or mucous membrane e. Nikolsky’s sign is negative


c. causes vesicles or bullae
Q. 3. Deficiency of all the following nutrients may cause oral ulceration except
a. nicotinic acid d. vitamin B 12

b. vitamin A e. vitamin C
c. vitamin B 2
Q. 4. Acute erythema multiforme is otherwise known as
a. Eagle’s syndrome d. Usher’s syndrome

b. Kartagener’s syndrome e. Young’s syndrome


c. Stevens-Johnson syndrome
Q. 5. White striae forming lace like pattern on buccal mucosa are due to
a. erythroplakia d. linea alba

b. leukoplakia e. pemphigus vulgaris


c. lichen planus
Q. 6. All are true about aphthous ulceration except
a. immunological abnormality is a con- c. Recurrent
tributing factor d. Red margin
b. more common in men e. yellowish floor
306 Part D Oral Cavity and Pharynx
Q. 7. All are true about aphthous ulcers except
a. absence of constitutional symptoms d. pain
b. involvement of moveable mucosa e. shallow ulcers with yellow floor and a
c. marked cervical lymphadenopathy red halo around them
Scenario: A 22 years old medical student has history of recurrent painful ulcer in the oral
cavity during her class tests and annual examinations for last 03 years.
1. What is the most probable clinical diagnosis?
2. Discuss etiopathology of this entity?
3. Describe briefly its management.
-------------------------------------------------------- ANSWERS-----------------------------------------------------
Q.1. c 2. e 3. b 4.c
5. c 6. b 7. c
Scenario: 1. Aphthous ulcer 2. Infections, immune disorders, trauma, nutritional deficiency,
drug allergy 3. Mouth wash, corticosteroid, sodium chromoglycate, levamisole.
--------------------------------------------------------------------------------------------------------------------------

INFLAMMATIONS OF MOUTH AND PHARYNX


PHARYNGITIS
Definition:
Inflammation of pharyngeal mucosa is known as pharyngitis. It is divided into nonspecific
and specific pharyngitis. Nonspecific pharyngitis is extremely common and is caused by
viruses, bacteria, fungi and several other factors like post nasal drip, gastroesophageal reflux,
spices, condiments, hot or cold drinks, immunosuppression etc. Nonspecific pharyngitis is
again, divided into acute and chronic pharyngitis.

1. Acute pharyngitis:
Definition:
It is an acute inflammatory condition of the pharynx.

Etiopathology:
Acute pharyngitis is very common and occurs due to different etiological factors like viral,
bacterial, fungal or others. Viral causes are more common. Acute streptococcal pharyngitis
(due to group A beta-haemolytic streptococci) has received more importance because of its
etiology in rheumatic fever and post streptococcal glomerulonephritis.

Clinical features:
Acute pharyngitis may occur in different degrees of severity. Mild infections present with

CONCEPT BOOK OF DISEASES OF EAR, NOSE & THROAT BY FAHIM AWAN


307 Part D Oral Cavity and Pharynx

discomfort in the throat, malaise and low-grade fever. Pharynx in these cases is congested but
there is no lymphadenopathy.
Moderate and severe infections present with pain in throat,
dysphagia, headache, malaise and high-grade fever. Pharynx in these
cases shows erythema, exudate and enlargement of tonsils and
lymphoid follicles on posterior pharyngeal wall.
Very severe cases show edema of soft palate and uvula with
enlargement of cervical lymph nodes. A case of acute
pharyngitis

It is not possible on clinical examination, to differentiate viral from


bacterial infections. Viral infections are generally, mild and are accompanied by rhinorrhoea
and hoarseness while the bacterial ones are severe. Gonococcal pharyngitis is mild and may
even be asymptomatic.
Investigations:
Culture of throat swab is helpful in the diagnosis of bacterial pharyngitis. It can detect 90%
of group A Streptococci. Diphtheria bacillus is cultured on special media. Swab from a
suspected case of gonococcal pharyngitis should be cultured immediately without delay.
Failure to get any bacterial growth suggests a viral etiology.

Treatment:
1. General measures: Bed rest, plenty of fluids, warm saline gargles and analgesics form
the mainstay of treatment. Local discomfort in the throat in severe cases can be relieved
by lignocaine before meals to facilitate swallowing.
2. Specific treatment: Streptococcal pharyngitis (group A, beta-haemolytic
streptococcus) is treated with penicillin G, 200,000 to 250,000 units orally four times a
day for 10 days or benzathine penicillin G, 600,000 units once i.m. for patient < 60 lb in
weight and 1.2 million units once i.m. for patient > 60 lb. In penicillin sensitive
individuals, erythromycin 20-40 mg/kg body weight daily, in divided oral doses for 10
days is equally effective.
Diphtheria is treated by diphtheria antitoxin and administration of penicillin or
erythromycin. Gonococcal pharyngitis responds to conventional doses of penicillin or
tetracycline.
Viral infections causing pharyngitis:
Herpangina
Infectious mononucleosis
Cytomegalovirus
Measles and chickenpox
2. Chronic pharyngitis:
308 Part D Oral Cavity and Pharynx

Definition:
It is a chronic inflammatory condition of the pharynx.

Etiopathology:
It is characterized by hypertrophy of mucosa, seromucinous glands, subepithelial
lymphoid follicles and even the muscular coat of the pharynx. It is of two types;
1. Chronic catarrhal pharyngitis.
2. Chronic hypertrophic (granular) pharyngitis.
A large number of factors are responsible for chronic pharyngitis. These include;
a. Chronic irritants (smoking, chewing tobacco, pan, drinking)
b. Laryngopharyngeal reflux
c. Postnasal discharge.
d. Environmental pollution.
e. Mouth breathing

Clinical features:
Severity of symptoms in chronic pharyngitis vary from person to person.
1. Discomfort or pain in the throat is the most common presentation. This is specially
noticed in the morning.
2. Foreign body sensation in the throat leads to a constant desire to swallow or clear his
throat to get rid of this “foreign body”.
3. Cough: Throat is irritable and there is tendency to cough. Mere opening of mouth may
induce retching or gaging.
4. Tiredness of voice: Patient cannot speak for long and has to make undue effort to speak
as throat starts aching. The voice may also lose its quality and may even crack. On
examination findings depend on the type of pharyngitis.
Chronic catarrhal pharyngitis: There is congestion of posterior pharyngeal wall with
engorgement of vessels. Faucial pillars may be thickened. There is increased mucous
secretion which may cover pharyngeal mucosa.
Chronic hypertrophic (granular) pharyngitis: Pharyngeal wall
appears thick and edematous with congested mucosa and dilated
vessels.
Posterior pharyngeal wall may be studded with reddish nodules
(hence, the term granular pharyngitis). These nodules are due to
hypertrophy of subepithelial lymphoid follicles normally seen in
pharynx. Lateral pharyngeal bands become hypertrophied. Uvula may
be elongated and appear edematous.
A case of granular pharyngitis
309 Part D Oral Cavity and Pharynx

Treatment:
1. Etiological factor should be sought and removed.
2. Benzydamine mouth washes/warm saline gargles, especially in the morning are
soothing and relieve discomfort.
3. Mandl’s (compound iodine) paint may be applied to the pharyngeal mucosa.
4. Cautery of lymphoid granules. Throat is sprayed with local anesthetic and granules are
touched with 10-25% silver nitrate. Electrocautery or diathermy of nodules may require
general anesthesia.
5. Voice rest and speech therapy is essential for those with faulty voice production.
Hawking, clearing the throat frequently or any other such habit should be stopped.

Fungal pharyngitis:
Candida infection of oropharynx can occur as an extension of oral thrush. It is seen in the
patients who are immunosuppressed, debilitated or taking high doses of antibiotics. Usually,
patient complains of pain in the throat with dysphagia. Treatment of choice is oral nystatin.
01 ml i.e., 10000 units given 6-hourly at least for 2 weeks.
SELF ASSESSMENT
Q. 1. Infections of the pharynx in adults are virtually confined to
a. hypopharynx d. oropharynx

b. nasopharynx e. all of the above

c. oral cavity
Q. 2. Chronic pharyngitis is common in

a. drinkers d. smokers

b. exposed to atmospheric pollution e. all of the above


c. mouth breathers
Q. 3. What is not a usual clinical feature in chronic pharyngitis?
a. Fever d. Painful swallowing

b. Foreign body sensation e. Raw and painful throat

c. Lateral pharyngeal bands


-------------------------------------------------------- ANSWERS-----------------------------------------------------
Q.1. d 2. e 3. a
--------------------------------------------------------------------------------------------------------------------------

ADENOIDITIS
310 Part D Oral Cavity and Pharynx

Definition:
It is defined as inflammation of the nasopharyngeal tonsils, sufficient
to produce symptoms.
Etiopathology:
Adenoids are also known as nasopharyngeal tonsils. Physiological
enlargement of adenoids usually starts between the ages of 5-7 years.
Most of the lymphoid tissue atrophy with the onset of puberty e.g.,
thymus. Atrophy of adenoids usually begins by the age of 10 years A diagram of adenoids
and is complete before the age of 15-16 years.
Recurrent episodes of rhinitis, sinusitis or tonsillitis may lead to adenoid hypertrophy.

Clinical features:
These may be due to simple enlargement, inflammation or both. It
is the size of the adenoids relative to the nasopharyngeal space that is
of importance and not its absolute size. Clinical manifestations of
adenoids may be nasal, aural or general.
A. Nasal manifestations:
1. Nasal obstruction leads to mouth breathing, noisy breathing, Adenoid facies
snoring and toneless voice. There may even be obstructive sleep
apnea syndrome.
2. Nasal discharge is due to associated rhinosinusitis and choanal obstruction.
3. Epistaxis can occur due to acute infection of adenoids.
4. Adenoid facies may develop in long standing cases i.e., dull looking triangular face,
open mouth posture, prominent crowded upper teeth, broad upper lip, pinched nose and
high arch palate.
B. Aural manifestations:
1. Eustachian tube obstruction leads to hearing loss.
2. Otitis media, which may be in the form of serous/acute/recurrent/ chronic otitis media.
C. General manifestations:
Mental dullness, lack of concentration (aprosexia), apathy, nocturnal enuresis, sleep apnea,
night terrors and pulmonary hypertension leading ultimately to cor pulmonale. Very large
adenoids may occasionally be visible through the open mouth.
Posterior rhinoscopic examination is usually diagnostic in patients over the age of three
years. Flexible nasopharyngoscopy or zero-degree endoscopy may reveal status of adenoids.

CONCEPT BOOK OF DISEASES OF EAR, NOSE & THROAT BY FAHIM AWAN


311 Part D Oral Cavity and Pharynx

Investigations:
X-ray soft tissue nasopharynx lateral view for adenoids, may show narrowing of the
nasopharyngeal air space.

Treatment:
A. Conservative:
1. Removal of underlying cause such as allergy, immunity deficiency should be corrected.
2. Antibiotics, nasal decongestant drops may give relief when symptoms are not severe.
Patient should be instructed in nose blowing and nasal douching.
B. Surgical:
1. Adenoidectomy is performed under general anesthesia, when medical treatment fails or
symptoms are very severe.
2. Myringotomy with or without grommet insertion may be considered if patient has got
otitis media with effusion.

Differential diagnosis:
1. Foreign body nose
2. Meningoencephalocele
3. Angiofibroma
4. Thornwald’s cyst
Differential diagnosis of a pediatric nasopharyngeal mass:
Pediatric nasopharyngeal masses are rare. These cause respiratory obstruction and create
problems in diagnosis and management. Besides adenoid hypertrophy (which is unusual in
early infancy) and antrochoanal polyp, the differential diagnosis of a nasopharyngeal mass
includes:
1. Teratoid e.g., dermoids, teratoma, epignathi
2. Neuroectodermal e.g., encephalocele, brain heterotopia, meningioma
3. Dysontogenetic e.g., chordoma, craniopharyngioma
4. Miscellaneous e.g., cysts, haemangioma, hamartoma, rhabdomyosarcoma
SELF ASSESSMENT
Q. 1. Adenoiditis is inflammation of
a. inner Weldeyer’s ring d. outer Weldeyer’s ring

b. nasopharyngeal tonsils e. palatine tonsils

c. oropharyngeal tonsils

CONCEPT BOOK OF DISEASES OF EAR, NOSE & THROAT BY FAHIM AWAN


312 Part D Oral Cavity and Pharynx
Q. 2. Adenoids usually completely regress by the age of
a. 3-5 years d. 15-16 years

b. 5-7 years e. 18-20 years


c. 7-9 years
Q. 3. Nasal manifestation of adenoiditis is
a. epistaxis d. sleep apnea syndrome

b. nasal discharge e. all of the above


c. nasal obstruction
Q. 4. Aural manifestation of adenoiditis includes
a. acute otitis media d. serous otitis media

b. chronic otitis media e. all of the above


c. eustachian obstruction
Q. 5. Nasopharyngeal obstruction due to adenoids can lead to all except
a. acute otitis media d. sinusitis

b. cor pulmonale e. serous otitis media

c. proptosis
Q. 6. Adenoid facies include all except

a. closed mouth posture d. open mouth posture

b. crowding of upper teeth e. prominent upper teeth

c. high arched palate


Q. 7. Adenoid facies include all except

a. broad upper lip d. pinched nose

b. dull looking face e. triangular face


c. narrow upper lip
Q. 8. Long standing obstruction due to enlarged tonsils and adenoids can cause
a. bundle branch block d. left ventricular hypertrophy

b. cardiac ischemia e. right atrial failure


c. cor pulmonale
Q. 9. The voice in adenoid hypertrophy is
a. hoarse d. rhinolalia clausa

b. hot potato voice e. rhinolalia aperta


c. husky voice
Q. 10. Treatment of adenoiditis is

a. adenoidectomy c. orrection of underlying cause such

as allergy
b. antibiotics
CONCEPT BOOK OF DISEASES OF EAR, NOSE & THROAT BY FAHIM AWAN
313 Part D Oral Cavity and Pharynx

d. nasal decongestants e. all of the above


Q. 11. A 06 years old child presented with history of recurrent upper respiratory tract
infection, nasal obstruction, snoring and hearing loss for last 02 years. Management will
be
a. adenoidectomy with myringotomy d. tonsillectomy

b. myringotomy e. tonsillectomy with myringotomy

c. myringotomy with grommet


Q. 12. Thornwaldt cyst is seen in

a. base of tongue d. nasopharynx

b. floor of mouth e. oropharynx


c. larynx
Q. 13. Thornwaldt disease or cyst refers to
a. abscess of pharyngeal bursa d. tumor of notochord

b. chordoma e. tumor of Rathke’s pouch

c. craniopharyngioma
Scenario A: A 07 years old girl presented with mouth breathing, recurrent upper respiratory
tract infections and nasal discharge for last 06 months. X-ray lateral view neck showed a soft
tissue mass in the nasopharynx.
1. What is the most probable clinical diagnosis?
2. How will you confirm your diagnosis?
3. How are you going to manage this patient?
B: A 09 years old boy presents in ENT OPD with his mother with complaints of mouth
breathing, snoring and occasional bleeding from nose for the last 02 months. On examination
patient has got dull looking triangular face.
1. What is the most probable clinical diagnosis?
2. How will you confirm your diagnosis?
3. How are you going to manage this patient?
C: A 06 years old child presented with persistent nasal discharge and mouth breathing for
last 01 year and now he started feeling decrease in hearing. There is no history of ear
discharge or earache.
1. What pathology is likely to cause symptoms of ear in this child?
2. What investigations you want to request in this case in order to reach the diagnosis?
3. How will you manage this case?
-------------------------------------------------------- ANSWERS-----------------------------------------------------
Q.1. b 2. d 3. e 4. e 5. c 6. a

CONCEPT BOOK OF DISEASES OF EAR, NOSE & THROAT BY FAHIM AWAN


Part D Oral Cavity and
314 Pharynx
7. c 8. c 9. d 10. e 11. a 12. d 13. a
Scenario A: 1. Adenoiditis 2. Flexible nasopharyngoscopy 3. Antibiotic, decongestants,
adenoidectomy.
B: 1. Adenoiditis 2. Flexible nasopharyngoscopy 3. Antibiotic, decongestants, adenoidectomy
C: 1. Eustachian tube obstruction due to adenoiditis 2. X-ray nasopharynx lateral view, PTA,
tympanometry 3. Adenoidectomy, myringotomy with or without grommet insertion if there is
otitis media with effusion.
--------------------------------------------------------------------------------------------------------------------------
-
ADENOIDECTOMY
Indications:
1. Nasal obstruction and nasal discharge due to recurrent or chronic adenoiditis
2. Acute/recurrent/chronic/serous otitis media
3. Recurrent epistaxis due to adenoiditis
4. Recurrent rhinosinusitis due to adenoiditis
Contraindications:
1. Acute upper or lower respiratory tract infections
2. Cleft palate (to avoid nasal twang and nasal regurgitation of food and fluids)
3. Hemostatic diathesis
a. Bleeding disorders e.g., thrombocytopenia
b. Clotting disorders e.g., hemophilia
Procedure:
After orotracheal intubation for general anesthesia, patient is made to lie
supine on the table (head and neck should not be extended).
Boyle Davis mouth gag with blade applied. Throat (epilaryngeal) pack
placed. A Nelton catheter passed through the nasal cavity and delivered
out through the oral cavity and the soft palate retracted. Inspection of
adenoids and nasopharynx is done through warm indirect laryngoscopic
mirror. Palpation of the adenoids and nasopharynx is done to differentiate
adenoids from other pathologies like meningocele and chordoma.

Boyle Davis mouth


gag with different
blades
Adenoid curette with guard is inserted into nasopharynx till it touches posterior border of
the nasal septum. Adenoid tissue is engaged in adenoid curette and a
single central and two lateral sweeps are made. These sweeps usually
curette out whole the adenoids.
Postnasal space is packed with ribbon gauze for 6-8 minutes. Posterior
nasal pack is removed and hemostatis ensured. Nelton catheter and throat
Adenoid curette
CONCEPT BOOK OF DISEASES OF EAR, NOSE & THROAT BY FAHIM AWAN
315 Part D Oral Cavity and Pharynx

pack (also known as epilaryngeal pack) is removed. Boyle Davis mouth gag is removed.
A simple way of removing the adenoids is FESS (Functional Endoscopic Sinus Surgery)
and microdebrider.
Complications:
1. Hemorrhage usually indicates inadequate removal. So, adenoids should be removed
thoroughly. Perioperative hemorrhage is managed by placing a pack for a few minutes.
Postoperative hemorrhage is managed by nasal decongestant drops. If bleeding is severe,
it is managed by placing a post nasal pack in the nasopharynx under general anesthesia.
2. Injury to pharyngeal opening of eustachian tube may occur.
3. Dislocation of the atlantoaxial joint (if vigorous curettage of adenoids is performed).
4. Palatal insufficiency may occur if adenoidectomy is performed in a patient with
submucous cleft palate.
5. Nasopharyngeal stenosis as a result of scar formation.
Postoperative care:
It is same as that for tonsillectomy.
SELF ASSESSMENT
Q. 1. Adenoidectomy is indicated in all of the following except
a. nasal obstruction due to allergic c. otitis media with effusion
rhinitis d. recurrent otitis media in children
b. nasal obstruction due to adenoid e. recurrent rhinosinusitis
hyperplasia
Q. 2. All are contraindications of adenoidectomy except
a. acute respiratory tract infection c. cleft palate
b. bleeding disorder like thrombocytope- d. clotting disorder like hemophilia
nia e. otitis media with effusion
Q. 3. Before adenoidectomy nasopharynx is palpated to identify
a. adenoids d. meningocele

b. antrochoanal polyp e. all of the above


c. chordoma
Q. 4. Adenoidectomy may be performed with
a. adenoid curette d. Hajek’s forceps

b. Asch’s forceps e. Walsham’s forceps


c. crocodile forceps
Q. 5. Usual complications of adenoidectomy include
a. dislocation of atlantoaxial joint b. hemorrhage
CONCEPT BOOK OF DISEASES OF EAR, NOSE & THROAT BY FAHIM AWAN
Part D Oral Cavity and
316 Pharynx

c. injury to eustachian tube e. all of the above

d. nasopharyngeal stenosis
-------------------------------------------------------- ANSWERS-----------------------------------------------------
Q.1. a 2. e 3. e 4. a 5. e
--------------------------------------------------------------------------------------------------------------------------
TONSILLITIS
Definition:
It is defined as an inflammation of palatine tonsils (also known as pharyngeal tonsils).
Clinically it is divided into;
A. Acute tonsillitis
B. Chronic tonsillitis
A. Acute tonsillitis
Etiopathology:
Acute tonsillitis is a disease of children and young adults although it may present at any age.
It is rare in infants and after 50 years of age. It may be caused by viruses or bacteria. The
most common causative organisms are Beta hemolytic Streptococcus. Less common
causative organisms are Staphylococci, Pneumococci and H. influenzae.
Acute inflammation of the tonsil is classified as follows;
1. Acute catarrhal tonsillitis:
In this, infection is limited to the surface epithelium of the tonsil.
2. Acute follicular tonsillitis:
In this, infection spreads into crypts of the tonsil. Openings of the crypts
are visible as whitish or yellowish dots, where exudate from the crypts
coalesce to form a membrane on the surface and is then known as
membranous tonsillitis. A case of acute
3. Acute parenchymatous tonsillitis: follicular tonsillitis

In this, infection spreads into the substance of the tonsil and the tonsil is enlarged.
Clinical features:
These vary with the severity of infection.
1. Sore throat is the most common presentation.
2. There may be complaint of odynophagia.
3. General symptoms like fever, anorexia, malaise and bodyaches may also occur.
Occasionally referred earache may occur due to stimulation of glossopharyngeal nerve.
CONCEPT BOOK OF DISEASES OF EAR, NOSE & THROAT BY FAHIM AWAN
317 Part D Oral Cavity and Pharynx

There may be abdominal symptoms due to mesenteric lymphadenitis secondary to


swallowed infected saliva. This may mimic acute appendicitis.
Generally, adults suffer more frequently from local symptoms. Whereas children
usually develop general symptoms.
On examination;
1. Tonsils may be red, containing whitish or yellowish spots on the opening of the crypts or
a membrane covering the tonsils.
2. Jugulodigastric lymph nodes may be enlarged and tender.

Complications:
Acute tonsillitis may give rise to following complications;
A. Local complications:
1. Chronic tonsillitis. In acute tonsillitis, infection may persist in the tonsil and may change
into chronic tonsillitis.
2. Peritonsillar abscess, parapharyngeal abscess, retropharyngeal abscess and acute
otitis media may occur as a result of spread of infection to adjacent structures.
B. Distant complications:
1. Rheumatic fever
2. Acute glomerulonephritis rarely.
Investigations:
Diagnosis of acute tonsillitis is usually clinical.
Throat swab is sent for culture and sensitivity in doubtful or resistant cases.
Treatment:
1. Bed rest is advised.
2. Antibiotics like penicillin or erythromycin are given for 7-10 days.
3. Analgesics and antipyretics are given to relieve pain and fever.
Differential diagnosis:
1. Diphtheria:
It is slow in onset as compared to acute tonsillitis. Sore throat is also less as compared to
acute tonsillitis. In cases of diphtheria, membrane extends beyond the tonsil on to the faucial
pillars and soft palate, whereas in acute tonsillitis, membrane is localized only to the tonsils.
In cases of diphtheria, membrane is dirty grey in color and tightly adherent. Removal
of the membrane leaves a bleeding surface. Smear and culture of throat swab/membrane
shows Corynebacterium diphtheriae.
CONCEPT BOOK OF DISEASES OF EAR, NOSE & THROAT BY FAHIM AWAN
318 Part D Oral Cavity and Pharynx

2. Infectious mononucleosis: Synonyms: Glandular fever, Kissing fever


It is a disease of lymphoreticular system. Appearance of the tonsil is similar to that of
acute tonsillitis. All the cervical, axillary, inguinal lymph nodes and spleen are enlarged.
Blood smear shows more than 5% lymphocytes.
Paul- Bunnel (monospot) test is positive.
3. Agranulocytosis:
There are widespread ulcerative and necrotic lesions in the oral cavity and oropharynx.
Patient is debilitated and severely ill. Total leukocyte count is reduced to less than 2000 per
cubic millimeter.
Differences between acute tonsillitis and pharyngeal diphtheria
Clinical
S.No. features Acute tonsillitis Pharyngeal diphtheria
1 Onset Quick within 24 hours Relatively slow (1-3 days)
2 Age Any Generally, 2-5 years
Patient toxic but alert, face Marked toxicity, general
3 Toxicity
flushed senses dull, face pale
Uni or
4 Always bilateral Most often unilateral
bilateral
Usually present, true, dull
If present, false, bright yellow,
grey, unilateral, extending
limited to tonsils only, soft and
5 Membrane on to palate and adherent.
easily removable. If detached,
leaves behind If detached, leaves behind
no bleeding surface. bleeding surface.
6 Neck nodes Enlarged and very tender Enlarged and tender
7 Fever Generally, above102F Generally, below102F
8 Albuminuria Slight Marked
Quick, rhythmic and of good Quick and may be weak and
9 Pulse
volume irregular
10 Bacteriology Beta-hemolytic Streptococcus Corynebacterium diphtheriae

Differential diagnosis of white patch on the tonsils


1. Membranous tonsillitis
2. Diphtheria
3. Infectious mononucleosis

CONCEPT BOOK OF DISEASES OF EAR, NOSE & THROAT BY FAHIM AWAN


319 Part D Oral Cavity and Pharynx

4. Agranulocytosis
5. Leukemia
6. Aphthous ulcers
7. Malignancy of tonsil
8. Candida infection of tonsil
B. Chronic tonsillitis
Etiopathology:
Long standing infection of the tonsils may occur as a result of; A case of chronic
tonsillitis
1. Subclinical infection of the tonsil
2. Complication of acute tonsillitis

Chronic post nasal discharge secondary to chronic sinusitis is usually a predisposing factor.
Chronic tonsillitis usually occurs in children or young adults.
Clinical features:
1. Patient usually presents with recurrent episodes of sore throat.
2. There may be chronic irritation in the throat.
3. If the tonsils are large, there may be difficulty in swallowing and choking sensation
during sleep.
4. There may be bad taste in mouth or foul breath (halitosis) due to pus in crypts.
On examination signs of chronic tonsillitis are positive i.e.
a. Tonsils are usually enlarged.
b. Crypts are prominent.
c. Anterior faucial pillars give an appearance of a flare (also known as anterior
faucial flare).
d. Jugulodigastric lymph nodes are enlarged.
Investigations:
Diagnosis of chronic tonsillitis is mainly clinical. No investigation is required for diagnostic
purposes. Investigations may be required for differential diagnosis, identification of any
complication or fitness of the patient for general anesthesia and surgery.
Treatment:
1. Removal of the underlying cause such as rhinitis and sinusitis. Nutrition and hygiene
should be improved.
2. Tonsillectomy should be performed as a last resort if the tonsils interfere with daily
routine life, deglutition, respiration and sleep.

CONCEPT BOOK OF DISEASES OF EAR, NOSE & THROAT BY FAHIM AWAN


320 Part D Oral Cavity and Pharynx

Complications:
These are same for acute as well as chronic tonsillitis.
SELF ASSESSMENT
Q. 1. Acute or chronic tonsillitis is usually diagnosed on
a. culture and sensitivity of blood d. X-ray

b. clinically e. none of the above


c. throat swab
Q. 2. Usual clinical feature of acute tonsillitis is
a. anorexia d. sore throat

b. odynophagia e. all of the above


c. rise in body temperature
Q. 3. Local complication of tonsillitis includes
a. otitis media d. retropharyngeal abscess

b. parapharyngeal abscess e. all of the above


c. peritonsillar abscess
Q. 4. Distant complication of tonsillitis is
a. glomerulonephritis d. septicemia

b. rheumatic fever e. all of the above


c. rheumatic endocarditis
Q. 5. Differential diagnosis of white patch on tonsil is
a. agranulocytosis d. membranous tonsillitis

b. diphtheria e. all of the above

c. infectious mononucleosis
Q. 6. Tonsillar lymph node is

a. Dolphin’s lymph node d. submandibular lymph node

b. jugulo omohyoid lymph node e. submental lymph node

c. jugulo diagastric lymph node


Q. 7. Bed of tonsil is formed by

a. hyoglossus d. palatopharyngeus

b. inferior constrictor e. superior constrictor


c. middle constrictor
Q. 8. Anterior pillar of tonsils is formed by
a. hyoglossus c. palatopharyngeus

b. palatoglossus d. styloglossus

CONCEPT BOOK OF DISEASES OF EAR, NOSE & THROAT BY FAHIM AWAN


321 Part D Oral Cavity and Pharynx
e. superior constrictor
Q. 9. Posterior pillar of tonsils is formed by
a. hyoglossus d. styloglossus

b. palatoglossus e. superior constrictor


c. palatopharyngeus
Q. 10. Usual clinical features of stylalgia (Eagle’s syndrome) include all except
a. earache lowing

b. foreign body sensation d. retro-orbital neuralgia

c. pain in the throat aggravated on swal- e. tenderness in tonsillar fossa


Scenario: 10 years old child has recurrent attacks of acute tonsillitis more than 05 attacks
per year for last 02 consecutive years. Examination shows anterior faucial flare, debris
coming out of crypts and enlarged jugulodigastric lymph nodes.
1. What is your clinical diagnosis?
2. What is the treatment of this patient?
3. What are other indications of the same operation?
4. What is primary hemorrhage, secondary hemorrhage and reactionary hemorrhage?
--------------------------------------------------------ANSWERS-----------------------------------------------------
Q.1. b 2. e 3. e 4. e 5. e
6. c 7. e 8. b 9. c 10. d
Scenario: 1. Chronic tonsillitis 2. Tonsillectomy 3. Chronic tonsillitis, recurrent tonsillitis,
peritonsillar abscess, suspicion of malignancy, as an approach for glossopharyngeal
neurectomy 4. Bleeding during surgery is primary hemorrhage, after 24 hour of surgery is
secondary hemorrhage and bleeding within 24 hour of surgery is reactionary.
--------------------------------------------------------------------------------------------------------------------------
-
TONSILLECTOMY
Indications:
1. Chronic tonsillitis
2. Recurrent tonsillitis; If there are 4-5 episodes of acute tonsillitis per year for the last two
consecutive years.
3. Peritonsillar abscess; Tonsillectomy is done 4-6 weeks after resolution of peritonsillar
abscess.
4. Enlarged tonsils if causing mechanical obstruction to respiration and swallowing.
5. Doubtful malignancy (especially lymphoma) of tonsil (for biopsy purpose).
6. As an approach procedure (to reach the tonsillar bed);
322 Part D Oral Cavity and Pharynx

a. Glossopharyngeal neurectomy in glossopharyngeal neuralgia.


b. For removal of styloid process in Eagle’s syndrome.
Contraindications:
1. Acute respiratory tract infection
2. Haemoglobin below 10 g/dl
3. Hemostatic diathesis;
a. Bleeding disorders e.g., thrombocytopenia
b. Clotting disorders e.g., haemophilia
4. Systemic diseases e.g., hypertension, diabetes, bronchial asthma etc.
5. Active period of menstruation
Procedure:
After general anesthesia with endotracheal intubation, patient is
made to lie supine on the operation table. Tonsillectomy position
(head extended at neck and neck extended at chest) is made with
a sand bag under the shoulders.
After applying head light, surgeon sits on head side of the
table. Boyle Davis mouth gag is applied and held by an assistant
or a suspension apparatus like Draffin bipod and McGaurans Tonsillectomy in progress
plate. Throat pack (also known as epilaryngeal pack) is placed.
The tonsil is held from the upper pole and stretched medially by means of a tonsil holding
forceps.
An incision is made into capsule of the tonsil, medial to the
medial border of anterior faucial pillar. The tonsil is continuously Draffin bipod with McGaurans plate
pulled medially and dissection is carried out with tonsil dissector
in the plane between the tonsil and the superior constrictor muscle. As the
lower pole is reached, tonsillar snare is applied at the glossotonsillar
sulcus to crush the blood
vessels so as to decrease hemorrhage. At the time of dissection, a wet
piece of gauze is placed in the dissected area to decrease hemorrhage.
Continuous suction with Yankauer pharyngeal sucker is done to keep the Tonsil holding forcep
surgical field clear. Hemostasis of the tonsillar fossa is secured with
unipolar or bipolar diathermy. It may also be done with absorbable
sutures like catgut or vicryl. Tonsil dissector with pillar
Similarly, dissection and hemostasis is carried out on opposite side. retractor
Blood collected in the nasopharynx is sucked out at the end with a
Nelton catheter passed through the nasal cavities. Throat pack (also
known as epilaryngeal pack) is removed. Boyle Davis mouth gag is
removed. Sand bag is removed at the end.
Disection of tonsil

Complications of tonsillectomy:
CONCEPT BOOK OF DISEASES OF EAR, NOSE & THROAT BY FAHIM AWAN
323 Part D Oral Cavity and Pharynx

Complications of tonsillectomy may be divided into;


A. Peroperative complications
B. Postoperative complications
A. Peroperative complications:
a. Hemorrhage: Bleeding during surgery is known as primary hemorrhage. It is
because of recent infection, previous peri-tonsillar abscess, hemostatic diathesis, a
large tonsillar branch of the facial artery or an aberrant internal carotid artery.
b. Trauma: Damage to carious teeth, dislocation of temporomandibular joint and
excessive resection of mucosa of the soft palate.
B. Postoperative complications:
a. Immediate complications:
1. Hemorrhage: Bleeding which occurs within 24 hours of surgery is known as
reactionary hemorrhage but usually occurs within first 6-8 hours. It occurs due to
changes in blood pressure or excessive venous pressure induced by coughing or
retching. It is managed by removal of blood clots from tonsillar fossa and application
of slight pressure with a gauze soaked in 1:1000 adrenaline. If bleeding does not stop,
the patient is prepared for second general anesthesia and the bleeding points are re-
stitched. Any blood in the stomach is aspirated with the nasogastric tube. If the
patient is hypovolemic, cross-matched blood should be transfused.
2. Aspiration: Blood, secretions or tissues may be aspirated into lungs.
b. Intermediate complications:
1. Hemorrhage: Bleeding which occurs after 24 hours of surgery is known as
secondary hemorrhage. Usually, it occurs between 5th to 6th postoperative day.
The cause of secondary hemorrhage is infection of the tonsillar fossa. Management is
done by removing clots from the tonsillar fossa and application of pressure with a
gauze soaked in 1:1000 adrenaline. A coagulation screen should be performed
because a minor coagulation defect which did not cause problem in the initial stage
may became problematic later on. Systemic antibiotics (intravenous penicillin or
erythromycin) are given to treat the infection. Hemoglobin estimation should be done
and blood should be cross-matched.
2. Edema of the uvula.
3. Pulmonary complications may occur because of inhalation of blood or secretions.
4. Subacute bacterial endocarditis: Tonsillectomy leads to transient bacteremia at
the time of surgery. If the patient has an abnormal heart valve or a septal defect,
subacute bacterial endocarditis may occur. So, surgery in immediate and
postoperative period should be covered by systemic penicillins.

CONCEPT BOOK OF DISEASES OF EAR, NOSE & THROAT BY FAHIM AWAN


324 Part D Oral Cavity and Pharynx

c. Late complications:
Postoperative scaring: Careless traumatic surgery with loss of the soft palate may
lead to scarring and limited mobility of the soft palate. it leads to nasal regurgitation of
fluids and hypernasal voice.
SELF ASSESSMENT
Q. 1. Indications of tonsillectomy include
a. chronic tonsillitis obstruction

b. doubtful malignancy d. peritonsillar abscess

c. large tonsils causing respiratory e. all of the above


Q. 2. Concerning bed of tonsil;
a. 7th cranial nerve can be approached scess
through it d. place to drain quinsy
b. formed by middle constrictor e. styloid process can be excised
c. place to drain parapharyngeal ab- through it
Q. 3. Contraindication of tonsillectomy includes
a. acute upper or lower respiratory tract c. hemophilia
infection d. thrombocytopenia
b. Hb below 10 gm/dl e. all of the above
Q. 4. Usual complications of tonsillectomy include
a. aspiration of blood or secretions c. hemorrhage

b. dislocation of temporomandibular d. subacute bacterial endocarditis

Joint
e. all of the above
Q. 5. If tonsillectomy is carried out within three weeks of an acute attack, then
a. dissection is easy morrhage

b. dissection plane is easily identifiable d. less chances of complications

c. increased chances of secondary hae- e. tonsillar capsule is inflamed


Q. 6. Tonsillectomy can be done in all the following conditions except
a. acute attack of tonsillitis or pharyn- c. palatopharyngoplasty for sleep apnea
gitis d. recurrent tonsillitis
b. biopsy for lymphoma or carcinoma e. second attack of peritonsillar abscess
Q. 7. Which of the following is a contraindication of tonsillectomy?
a. Hemostatic diathesis d. Respiratory tract infection

b. Hypertension and diabetes e. All of the above


c. Menstruation

CONCEPT BOOK OF DISEASES OF EAR, NOSE & THROAT BY FAHIM AWAN


325 Part D Oral Cavity and Pharynx
Q. 8. Earache following tonsillectomy is referred through
a. glossopharyngeal nerve d. vagus nerve

b. hypoglossal nerve e. vestibulocochlear nerve


c. mandibular branch of trigeminal nerve
Q. 9. A 05 years old patient is scheduled for tonsillectomy. On the day of surgery, he had
runny nose, temperature 38°C and dry cough. Which of the following should be the most
appropriate decision for surgery?
a. Get X-ray chest before proceeding for c. Proceed for surgery if chest is clear
surgery d. Postpone surgery for 3 weeks and
b. Proceed for surgery and give antibi- patient to be put on antibiotics
otics e. Surgery should be cancelled
Scenario: A 20 years old boy after 07 days of his tonsillectomy comes to emergency with
bleeding from mouth, fever and pain.
1. What is your diagnosis?
2. What investigations you will advise?
3. How will you treat this case?
ANSWERS--------------------------------------------------
-------------------------------------------------------- ---
Q.1. e 2. e 3. e 4. e 5. c
6. a 7. e 8. a 9. d
Scenario: 1. Secondary hemorrhage 2. Blood complete examination for Hb and TLC
3. Removal of blood clots, adrenaline-soaked packs, systemic antibiotics.
--------------------------------------------------------------------------------------------------------------------------
DIPHTHERIA
It is an infection of pharynx, larynx or nasal cavities caused by Corynebacterium diphtheriae.
Etiopathology:
Corynebacterium diphtheriae is a gram-positive bacillus. It produces exotoxin. Spread of
infection is by infected droplets of nasal, nasopharyngeal or pharyngeal secretions.
The disease varies in severity depending upon the immunity of the host and also the
virulence of the infective organism. So, the clinical picture may vary from asymptomatic
carrier state to a rapidly fatal toxic disease.
Clinical features:
In pharyngeal/laryngeal diphtheria patient presents with;
1. Severe sore throat
2. Odynophagia

CONCEPT BOOK OF DISEASES OF EAR, NOSE & THROAT BY FAHIM AWAN


Pseudomembrane in
diphtheria
326 Part D Oral Cavity and Pharynx

3. Dysphonia
4. Dyspnea and stridor
On examination;
1. Tightly adherent greyish pseudomembrane on tonsils, faucial pillars, soft palate and
posterior pharyngeal wall is visible. Bleeding occurs when trying to remove the
pseudomembrane.
2. There is tender cervical lymphadenopathy.
Complications:
1. The membrane may spread to the larynx causing rapidly increasing airway obstruction
and death.
2. Most of the deaths in diphtheria are due to toxemia which causes myocarditis, cardiac
conduction defects and arrhythmias producing acute circulatory failure.
3. Neurological complications may appear 3-6 weeks after the onset of diphtheria and
include paralysis of soft palate, diaphragm and extraocular muscles.
4. The exotoxins may also produce fatal thrombocytopenia.
Investigations:
Diagnosis is usually made on clinical grounds but may be confirmed by smear and culture
of throat swab/membrane and isolation of Corynebacterium diphtheriae.
Treatment:
1. Admission in hospital and isolation. It is done on urgent basis.
2. Maintenance of airway with endotracheal intubation or tracheostomy must be done if
there is risk of airway obstruction.
3. Strict bed rest is given to avoid the risk of myocarditis.
4. Benzyl penicillin 500,000 units intramuscular, every 6 hours for 6 days is effective
against diphtheria bacilli. Erythromycin may be given if patient is allergic to penicillin.
5. Diphtheria antitoxin; Its dose varies from 20,000 units to 100,000 units, through
intravenous infusion. It neutralizes free toxin circulating in blood.
Prevention:
Active immunization against diphtheria is done by injection of toxoid in three doses,
beginning at 45 days of age.
Differential diagnosis:
1. Acute membranous tonsillitis
2. Infectious mononucleosis
3. Agranulocytosis
CONCEPT BOOK OF DISEASES OF EAR, NOSE & THROAT BY FAHIM AWAN
327 Part D Oral Cavity and Pharynx

4. Leukemia
SELF ASSESSMENT
Q. 1. The causative organism of diphtheria is

a. Corynebacterium diphtheriae c. Mycobacterium

b. Haemophilus influenzae d. Staphylococcus aureus

e. Streptococcus pneumoniae
Q. 2. Corynebacterium diphtheria is
a. gram negative bacillus d. gram positive bacillus

b. gram positive coccus e. None of the above


c. gram negative coccus
Q. 3. All are usual clinical features of diphtheria except
a. non tender cervical lymphadenopathy

b. pseudomembrane on tonsil and faucial


pillars d. severe sore throat

c. pseudomembrane on tonsil and palate e. very toxic look


Q. 4. Complication of diphtheria includes
a. airway obstruction d. toxemia

b. myocarditis e. all of the above


c. thrombocytopenia
Q. 5. Common complications of faucial diphtheria include all except
a. cardiac arrythmias and eye muscles
b. myocarditis and cardiac conduction d. thrombasthenia
defects e. toxemia
c. paralysis of diaphragm, soft palate
Q. 6. Treatment of diphtheria include all except
a. aminoglycosides d. isolation
b. bed rest e. maintenance of airway by endotrache-al
c. diphtheria antitoxin intubation or tracheostomy
Scenario A: A 09 years old boy comes with history of fever and painful swallowing for last
03 days. On examination, there is greyish membrane over the right tonsil which also extends
to the soft palate.
1. What is the most likely diagnosis?
2. What usual complications can occur in this patient?
3. What is the treatment plan for this patient?
B: A 03 years old non-immunized child presented to ENT department through casualty
department with history of fever, sore throat, dyspnea and hoarseness of voice for last 05

CONCEPT BOOK OF DISEASES OF EAR, NOSE & THROAT BY FAHIM AWAN


328 Part D Oral Cavity and Pharynx
days. His clinical examination revealed toxic look and temperature 101 degrees F. Tonsils
are swollen and covered with tightly adherent dirty grey membrane along with markedly
enlarged jugulodigastric lymph nodes.
1. What is the most likely diagnosis?
2. How will you investigate?
3. How will you treat the child?
C: A six years old boy is brought to the emergency department with low-grade fever for last
four days. He has difficulty in breathing for last one day. Oral cavity shows a dull membrane
on the left tonsil extending onto the soft palate. Removal of membrane leaves behind bleeding
surface. There is swelling of the neck as well.
1. What is the most probable clinical diagnosis?
2. How will you investigate?
3. How will you manage the patient?
---------------------------------------------- ANSWERS-----------------------------------------------
---------- ------
Q.1. a 2. d 3. a 4. e 5. d 6. a
Scenario A: 1. Diphtheria 2. Airway obstruction, myocarditis, paralysis of soft palate,
diaphragm and extraocular muscle, thrombocytopenia 3. Admission and isolation,
maintenance of airway, strict bed rest, benzyl penicillin, diphtheria antitoxin.
B: 1. Diphtheria 2. Smear and culture of throat swab and membrane 3. Admission and
isolation, maintenance of airway, strict bed rest, benzyl penicillin, diphtheria antitoxin.
C: 1. Diphtheria 2. Smear and culture of throat swab and membrane 3. Admission and
isolation, maintenance of airway, strict bed rest, benzyl penicillin, diphtheria antitoxin.
--------------------------------------------------------------------------------------------------------------------------
PERITONSILLAR ABSCESS (PTA)
Synonyms: Peritonsillar cellulitis, Quinsy, Angina tonsillaris
Definition:
It is defined as a collection of pus between the fibrous capsule of the tonsil (usually at its
upper pole) and the superior constrictor muscle of pharynx.
Etiopathology:
Exact etiology is unknown. Usually, it is a complication of acute tonsillitis. Initially there is
peritonsillar cellulitis but if left untreated may proceed to peritonsillar abscess. Beta
haemolytic streptococcus and anaerobes are usual causative organisms. It is almost always
unilateral. It is rare in children.
Clinical features:
1. Patient may have history of previous episodes of acute tonsillitis.

Left
peritonsillar abscess
329
2. There is severe pain in the throat on the side of peritonsillar cellulitis.
3. There is plummy quality of voice and intense salivation with dribbling.
4. There may be ipsilateral referred earache. There is also rise in the body temperature.
5. There may be fetid smell.
On examination there is;
1. Asymmetry of oropharynx with hyperemia and edema of the soft palate.
2. Medial displacement of the tonsil.
3. Trismus caused by spasm of medial pterygoid muscle due to contact with pus.
4. Ipsilateral tender enlarged cervical lymph nodes.
Complications:
1. Airway obstruction may occur due to edema of larynx.
2. Spread of infection to other spaces of the neck especially parapharyngeal space may
occur.
Investigations:
1. Wide bore aspirate is sent for microbiology, culture and sensitivity.
2. CT/MRI is carried out in case of suspected complications such as spread of infection to
parapharyngeal space, retropharyngeal space or mediastinum.
Treatment:
1. In early stages of cellulitis, conservative
treatment is carried out with intravenous
antibiotics for 48 hours.
2. In late stages, patient is admitted in the
hospital for I/V antibiotics and observation for
any airway obstruction.
3. Wide bore needle aspiration is often
curative. It is carried out at a point where a
horizontal line drawn from the base of uvula
and a vertical line drawn from medial border
of anterior faucial pillar meet together.
4. Incision and drainage of the abscess under
local anesthesia is carried out if wide bore
aspiration fails or insufficient. It is done with
bistoury knife at the point mentioned above.
Efficient suction is carried out at the same
time. Even if pus is not formed, early
resolution of symptoms may be achieved with

incision and drainage. Site of drainage of quinsy


Taken into consideration the morbidity and mortality
associated with peritonsillar abscess, tonsillectomy is carried out about one month after the
acute episode of peritonsillar abscess has subsided. “Abscess tonsillectomy” also known as
“hot tonsillectomy” (incision and drainage of abscess plus tonsillectomy) is carried out by
some surgeons as a preferred method of treatment.

CONCEPT BOOK OF DISEASES OF EAR, NOSE & THROAT BY FAHIM AWAN


330 Part D Oral Cavity and Pharynx

Differential diagnosis:
1. Parapharyngeal abscess
2. Retropharyngeal abscess
3. Parapharyngeal tumor
SELF ASSESSMENT
Q. 1. Peritonsillar abscess is also called

a. angina tonsillaris d. all of the above

b. peritonsillar cellulitis e. none of the above


c. quinsy
Q. 2. In which of the following locations (spaces), there is collection of pus in quinsy?
a. peritonsillar space d. retropharyngeal space
b. parapharyngeal space e. within tonsil
c. prevertebral space
Q. 3. A peritonsillar abscess is
a. caused by gram negative organisms d. lies lateral to middle constrictor
muscle
b. collection of pus lateral to superior
constrictor e. potential source of parapharyngeal

abscess
c. drained by external cervical approach
Q. 4. In peritonsillar abscess, pus lies

a. in crypta magna d. medial to superior constrictor

b. lateral to buccopharyngeal fascia e. none of the above


c. lateral to superior constrictor
Q. 5. Usual clinical feature of peritonsillar abscess includes
a. asymmetry of oropharynx d. trismus

b. intense salivation e. all of the above


c. severe throat pain
Q. 6. A 20 years old patient presents with trismu, pain on swallowing, pain in right ear and
right jugulodigastric lymph nodes enlargement of 02 days duration. The likely diagnosis is
a. cancer of tongue d. lymphoma

b. cancer of pyriform sinus e. peritonsillar abscess


c. cancer of tonsil
Q. 7. Peritonsillar abscess is differentiated from acute tonsillitis by

CONCEPT BOOK OF DISEASES OF EAR, NOSE & THROAT BY FAHIM AWAN


331 Part D Oral Cavity and Pharynx

a. it is unilateral d. tonsil is pushed medially

b. there is dribbling of saliva e. all of the above


c. there is trismus
Q. 8. Trismus accompanying peritonsillar abscess is due to spasm of which muscle?
a. Lateral pterygoid d. Pharyngeal constrictor

b. Masseter e. Temporalis

c. Medial pterygoid
Scenario A: An 18 years old male presented with severe sore throat and odynophagia for last
seven days. Pain was mostly confined to left side of throat and there was also difficulty in
opening the mouth. Throat examination revealed a rounded, reddish diffuse bulge in the left
supratonsillar region.
1. What is the most probable clinical diagnosis?
2. What usual complications are associated with this condition?
3. How will you manage this case?
B: A 35 years old patient presents with left sided sore throat, odynophagia and high-grade
fever for the last six days. Throat examination is difficult because of inability to open the
mouth. Left sided jugulodigastric lymph nodes are palpable and painful. CBC reveals
neutrophilia and ESR was 25 mm/hour.
1. What is the provisional diagnosis?
2. Mention three conditions included in differential diagnosis.
3. What usual complications are associated with this disease?
4. Give your plan to manage this case.
C: A 25 years old young boy presented with severe odynophagia for the last seven days,
associated with high-grade fever and drooling of saliva. Pain was mostly confined to left side
of the throat and there was also difficulty in opening the mouth. Throat examination revealed
a round, reddish bulge in the left supratonsillar region and further detailed examination is
not possible due to pain and trismus.
1. What is possible clinical diagnosis?
2. How would you manage this case?
3. What complications are associated with this condition?
--------------------------------------------------------ANSWERS-----------------------------------------------------
Q.1. d 2. a 3. e 4. d 5. e 6. e 7. e 8. c
Scenario A: 1. Peritonsillar abscess 2. Airway obstruction, parapharyngeal abscess 3. I.v
antibiotics, wide bore needle aspiration, incision and drainage.
B: 1. Peritonsillar abscess 2. Parapharyngeal abscess, retropharyngeal abscess,
parapharyngeal tumors 3. Airway obstruction, parapharyngeal abscess 4. Intravenous
332
antibiotics, wide bore needle aspiration, incision and drainage.
C: 1. Peritonsillar abscess 2. Intravenous antibiotics, wide bore needle aspiration, incision
and drainage 3. Airway obstruction, parapharyngeal abscess.
--------------------------------------------------------------------------------------------------------------------------
-
PARAPHARYNGEAL ABSCESS
Definition:
It is defined as suppurative infection of parapharyngeal space.
Etiopathology:
Infection in the parapharyngeal space may spread from;
1. Pharynx e.g., infection of tonsils, adenoids, peritonsillar abscess,
retropharyngeal abscess.
2. Teeth e.g., infection of lower last molar tooth.
3. Foreign body penetration
4. Ear e.g., petrositis
A case of parapharyngeal abscess
The abscess may occur at any age but is more frequent in
adolescents and adults.
Clinical features:
These are very similar to those of peritonsillar abscess.
1. History of causative factors is usually present.
2. Pain, odynophagia, dysphagia, dyspnea and trismus (because of
involvement of medial pterygoid muscle) are usual symptoms.
There is rise in body temperature between 101-102ºF.
CT of left
3. Difficulty in flexing and turning the neck. parapharyngeal
abscess
On examination;
1. Pharyngeal wall and tonsil pushed medially. The swelling is more inferiorly placed and
the soft palate is less edematous as compared to peritonsillar abscess.
2. Paralysis of 9th, 10th, 11th, 12th cranial nerves and cervical sympathetic chain.
3. There may be tender swelling on the side of neck behind the middle third of
sternomastoid muscle.

Complications:
1. Airway obstruction due to edema of larynx.
2. Spread of infection into other spaces of neck.
3. Spread of infection into the mediastinum.
4. Thrombosis of internal jugular vein.
CONCEPT BOOK OF DISEASES OF EAR, NOSE & THROAT BY FAHIM AWAN
333 Part D Oral Cavity and Pharynx

5. Rupture of carotid artery.


6. Septicemia and pyemia.
Investigations:
1. CT scan
2. Wide bore needle aspiration for microbiology
3. Chest X-ray
4. Dental evaluation to rule out odontogenic source.
Treatment:
CT showing parapharyn-
1. Admission in the hospital and observation for any airway geal abscess
obstruction.
2. Parenteral antibiotics are given.
3. Incision and drainage of the abscess is performed under general anesthesia if fluctuation
is present. This may be done through the pharynx or through the neck depending upon
the point of maximum swelling but preferably by the later root. Inhalation of blood and
pus is prevented by efficient suction when incision and drainage are being done through
the pharynx.
Differential diagnosis:
1. Peritonsillar abscess
2. Retropharyngeal abscess
3. Tumors of parapharyngeal space
4. Aneurysm of the carotid artery
SELF ASSESSMENT
Q. 1. All are complications of parapharyngeal abscess except

a. airway obstruction d. thrombosis of external jugular vein

b. rupture of carotid artery e. thrombosis of internal jugular vein


c. spread of abscess into mediastinum
Q. 2. Complication of parapharyngeal abscess is

a. pyemia c. spread of abscess into mediastinum

b. septicemia d. All of the above

e. None of the above


Q. 3. Differential diagnosis of parapharyngeal abscess is

a. peritonsillar abscess d. all of the above


b. retropharyngeal abscess e. none of the above
c. tumors of parapharyngeal space
CONCEPT BOOK OF DISEASES OF EAR, NOSE & THROAT BY FAHIM AWAN
334 Part D Oral Cavity and Pharynx
Q. 4. All are treatment options of parapharyngeal abscess except
a. admission in hospital d. parenteral antibiotics

b. incision through neck e. radiotherapy

c. observation for airway obstruction


Scenario: A 30 years old male presented with odynophagia, trismus and fever for last 02
weeks. He gives a history of extraction of right lower third molar tooth for dental caries a few
days back. On examination of throat there is swelling pushing the right tonsil medially and
spreading laterally posterior to the middle third of sternomastoid.
1. What is the most probable clinical diagnosis?
2. How will you manage this case?
3. What complications are associated with this condition?
--------------------------------------------------------ANSWERS-----------------------------------------------------
Q.1. d 2. d 3. d 4. e
Scenario: 1. Parapharyngeal abscess 2. Admission, parenteral antibiotics, incision and
drainage 3. Airway obstruction, spread of infection, thrombosis of internal jugular vein,
rupture of carotid artery, septicemia and pyemia.
--------------------------------------------------------------------------------------------------------------------------
-

RETROPHARYNGEAL ABSCESS (RPA)


It is very important to differentiate between retropharyngeal space and prevertebral space.
Retropharyngeal space: It is a potential space which lies behind the pharynx between the
buccopharyngeal fascia (covering pharyngeal constrictor muscles) and the prevertebral fascia.
This space extends from the base of skull to the level of bifurcation of trachea. Abscess of
this space usually produces unilateral bulge.
Prevertebral space: It lies between the prevertebral fascia anteriorly and the vertebral
bodies posteriorly. It extends from the base of skull to coccyx. Abscess of this space usually
produces midline bulge.
Definition:
It is defined as an abscess which lies in the retropharyngeal space.
These are of two types;
A. Acute retropharyngeal abscess
B. Chronic retropharyngeal abscess
A. Acute retropharyngeal abscess:
It tends to be limited to one side of midline of pharynx by the central adherence of the fascia
in the lateral space of Gillette.
Etiopathology:
CONCEPT BOOK OF DISEASES OF EAR, NOSE & THROAT BY FAHIM AWAN
335 Part D Oral Cavity and Pharynx

The acute abscess is usually caused by;


1. Suppuration in the retropharyngeal lymph nodes which become infected from the
nasopharynx and oropharynx.
2. Spread of infection from peritonsillar abscess, parapharyngeal abscess or other
abscesses in neck.
3. Penetration of the pharyngeal wall by a sharp foreign body like bone chip.
4. Neoplasm in this region may be another causative factor.
5. Rarely infection may spread from acute suppurative otitis media to the petrous apex and
then to the retropharyngeal space.
The most common organisms are streptococcus Viridans, Staphylococcus aureus and
Klebsiella pneumoniae.
Clinical features:
Acute retropharyngeal abscess is usually seen in young children below the age of 03 years
but adults may also be affected rarely.
1. Dysphagia and dyspnea. There is difficulty in swallowing and
breathing.
2. Stridor may be present.
3. There may also be complaint of fever.
On examination of pharynx there is;
1. Unilateral bulge of posterior pharyngeal wall.
2. On palpation, there may be fluctuation. X-ray soft tissue neck
Investigations: retropharyngeal abscess

1. Transoral needle aspiration may be carried out for


microbiology as well as for immediate treatment purposes.
2. X-ray neck lateral view shows prevertebral soft tissue widening with air in it, loss of
normal curvature of cervical spine and bone destruction.

3. CT/MRI may be done to delineate the extent of the abscess.


Treatment:
1. Vertical incision of the abscess through an open mouth is carried out under local
anesthesia.
2. Parenteral antibiotics are given.
3. Tracheostomy may be considered if there is impending airway obstruction.
4. Causative factor should be removed.

CONCEPT BOOK OF DISEASES OF EAR, NOSE & THROAT BY FAHIM AWAN


336 Part D Oral Cavity and Pharynx

Differential diagnosis:
1. Acute epiglottitis
2. Peritonsillar abscess
3. Parapharyngeal abscess
B. Chronic retropharyngeal abscess:
Etiopathology:
It is usually caused by tuberculous infection. There are two types of retropharyngeal abscesses;
1. Tuberculous infection of the retropharyngeal lymph nodes due to spread of infection from
cervical lymph node. The abscess lies laterally in the space of Gillette.
2. Tuberculous caries of the cervical vertebrae, when the abscess lies centrally behind the
prevertebral fascia.
Clinical features:
This is usually slow in onset. It is usually seen in older children, adolescents and adults.
Local symptoms and signs may be minimal or absent.
1. There may be mild sore throat, slight dysphagia and cough.
2. The “cold” abscess is present in the posterior pharyngeal wall in the midline in cases of
spinal caries.
3. Enlarged painless lymph nodes may be present in the deep cervical chain.
Investigations:
X-ray neck lateral view shows similar findings are those for acute
retropharyngeal abscess. In addition, radiography may reveal calcification
in tuberculous lymph nodes or vertebral disease.
Treatment:
1. Incision of the abscess is made through neck, never through mouth.
The approach is made in front of sternomastoid muscle in the plane
between the carotid sheath and the visceral compartment of neck. If the abscess is highly
situated, it is better approached from behind the carotid sheath through an incision behind
the sternomastoid muscle.
2. Antituberculous chemotherapy must be given in preoperative and postoperative period.
3. Any spinal caries is managed by stabilization of spine otherwise neurological deficit
may occur.
Complications of retropharyngeal abscess:
1. Airway obstruction
2. Parapharyngeal abscess

CONCEPT BOOK OF DISEASES OF EAR, NOSE & THROAT BY FAHIM AWAN


337 Part D Oral Cavity and Pharynx

3. Prevertebral abscess
4. Lemierre’s syndrome resulting in internal jugular vein thrombosis, septicemia and
septic emboli
5. Mycotic aneurysm of carotid artery
6. Carotid blow out with massive hemorrhage
SELF ASSESSMENT
Q. 1. Retropharyngeal space lies
a. anterior to buccopharyngeal fascia d. posterior to prevertebral fascia

b. anterior to prevertebral fascia e. posterior to vertebral bodies

c. anterior to vertebral bodies


Q. 2. Prevertebral space lies

a. anterior to buccopharyngeal fascia d. posterior to prevertebral fascia

b. anterior to prevertebral fascia e. posterior to vertebral bodies


c. posterior to buccopharyngeal fascia
Q. 3. A unilateral bulge of posterior pharyngeal wall is usually seen in
a. peritonsillar abscess d. retropharyngeal abscess

b. parapharyngeal abscess e. submandibular abscess


c. prevertebral abscess
Q. 4. A midline bulge of posterior pharyngeal wall is usually seen in
a. peritonsillar abscess d. retropharyngeal abscess

b. parapharyngeal abscess e. submandibular abscess


c. prevertebral abscess
Q. 5. Cause of acute retropharyngeal abscess is
a. infection of last lower molar tooth d. spread of infection from peritonsillar
b. penetrating foreign body abscess
c. spread of infection from parapharyn- e. all of the above
geal abscess
Q. 6. All are usual clinical features of acute retropharyngeal abscess except
a. dysphagia d. midline bulge of posterior pharyngeal
b. dyspnea wall
c. fluctuation is positive e. stridor
Q. 7. All are treatment options of acute retropharyngeal abscess except
a. parenteral antibiotics c. removal of causative factor
b. radiotherapy
d. tracheostomy if upper airway e. vertical incision through open mouth
obstruction
CONCEPT BOOK OF DISEASES OF EAR, NOSE & THROAT BY FAHIM AWAN
338 Part D Oral Cavity and Pharynx

Q. 8. Usually cause of chronic retropharyngeal abscess is

a. spread of infection from peritonsillar c. tuberculous caries of cervical verte-


abscess brae

b. tuberculous infection of retropharyn- d. both a & b

geal lymph node


e. both b & c
Q. 9. All are treatment options of chronic retropharyngeal abscess except
a. incision through neck d. post operative antituberculous chemo-
b. incision through open mouth therapy
c. pre operative antituberculous chemo- e. stabilization of cervical spine
therapy
Q. 10. Usual complications of retropharyngeal abscess include
a. airway obstruction d. prevertebral abscess

b. Lemierre’s syndrome e. all of the above

c. parapharyngeal abscess
Scenario: A 03 years old child presented with the history of dysphagia, dyspnea and high-
grade fever for last one week. On examination of throat, there is unilateral bulging of
posterior pharyngeal wall which is fluctuant.
1. What is the most probable clinical diagnosis?
2. What investigations will you carry out?
3. How will you treat this patient?
ANSWERS--------------------------------------
-------------------------------------------------------- ---------------
Q.1. b 2. d 3. d 4. c 5. e
6. d 7. b 8. e 9. b 10. e
Scenario: 1. Acute retropharyngeal abscess 2. X-ray soft tissue neck lateral view 3. Incision
and drainage, antibiotics.
--------------------------------------------------------------------------------------------------------------------------
-
TUMORS OF ORAL CAVITY, OROPHARYNX AND
HYPOPHARYNX
TUMORS OF ORAL CAVITY
Tumors of oral cavity may be divided into:
A. Benign tumors
B. Malignant tumors
A. Benign tumors:
The common benign tumors of mouth are;
CONCEPT BOOK OF DISEASES OF EAR, NOSE & THROAT BY FAHIM AWAN
339 Part D Oral Cavity and Pharynx

1. Squamous cell papilloma: It presents as a warty finger like processes. It is treated by


simple excision.
2. Pleomorphic adenoma: In the oral cavity it most commonly arises from the hard
palate. It is excised with a healthy margin. After removal there is usually a large defect in
the palate which may require flap repair or obturator.
The other less common benign tumors are hemangioma, lymphangioma, fibroma,
neurofibroma and lipoma.
Pre-malignant lesions of the mouth:
There are certain lesions in the oral cavity which may change into malignancy. These lesions
include leukoplakia, erythroplakia, leuko erythroplakia, lichen planus, submucous fibrosis
etc.
B. Malignant tumors:
Etiopathology:
A number of factors are thought to contribute to the malignancy of the oral cavity which
include; smoking especially reverse smoking, habit of chewing raw tobacco/betel nut/pan/
gutka, alcohol consumption, syphilis and cirrhosis of liver. Human papilloma virus (HPV)
due to orogenital sex is a new known factor.
Certain dietary deficiencies and metabolic factors also play a role in oral and
hypopharyngeal cancers.
More than 90% of malignancies of the oral cavity are squamous cell carcinoma. The
remaining are minor salivary gland tumors, sarcomas, lymphomas and melanomas.
Clinical features:
Early lesions are asymptomatic.
1. Usually there is complaint of a white or red patch or a non-healing
ulcer in the oral cavity.
2. In cases of carcinoma of the tongue patient may present with
discomfort on eating spicy food.
Carcinoma of the tongue most commonly arises from lateral border. Carcinoma of
3. Trismus may occur in advanced cases due to involvement of pterygoid tongue
muscles. There may be loss or loosening of teeth. There may be
involvement of mandible.
On examination there is;
1. Ulcer with raised edges and granular floor which bleeds easily and the base is indurated.
2. There is lymph node enlargement in 50 % cases.
Investigations:
340 Part D Oral Cavity and Pharynx

1. Biopsy under local anesthesia is taken. It determines not only the type of carcinoma but
also its degree of differentiation (well differentiated, moderately differentiated or poorly
differentiated).
2. CT shows exact extent of the tumor and site, size, number and level of lymph node
involvement.
3. OPG (orthopantomogram) shows evidence of bony invasion.
4. Panendoscopy: (oropharyngoscopy, hypopharyngoscopy, laryngoscopy,
tracheobronchoscopy, esophagoscopy and nasopharyngoscopy) under general
anaesthesia. Malignancies of head and neck region are usually multiple. These are
associated with malignancy elsewhere in the body. This second malignancy is known as
second primary. The second primary may exist simultaneously (known as synchronous)
or may occur later (known as metachronous) with the first primary. The incidence of
second primary is again highest in the head and neck region. So panendoscopy is
performed to rule out or find out second primary.
5. Radionucleotide scanning of the bone and liver is used to exclude metastasis.
6. Staining with toluidine blue leads to uptake of the dye by the carcinoma.

Classification of primary tumor (T) of oral cavity


Tis Carcinoma in situ
T0 No primary tumor
T1 Tumor < than 2 cm
T2 Tumor > 2 cm but < 4 cm
T3 Tumor > 4 cm
T4 Tumor with extension to bone, muscle, skin, etc.
TX Primary tumor cannot be assessed
Classification of neck node metastasis (N), distant metastasis (M) and TNM
classification is almost the same as that for carcinoma of larynx.
Treatment:
It is extremely variable and depends on a number of factors. The factors are exact site and
size of tumor, its extension into surrounding structures, general health, nutritional status,
smoking, drinking habits, patient’s mental and psychological make up.
1. Radiotherapy:
T1 and T2 lesions in all sites within the oral cavity respond equally well to surgery or
radiotherapy. 6500 cGy are given in fractions over 6 to 7 weeks. Lesions of the lateral border
of the tongue and anterior floor of the mouth may be successfully treated with interstitial
implants to deliver a dose upto 10,000 cGy.
341 Part D Oral Cavity and Pharynx

If neck nodes are present, surgery should be used as a first line of treatment.
2. Surgery:
For T3 and T4 lesions, radical surgery is usually done.
Standard procedures for oral cancer are partial or total glossectomy and mandibulectomy.
Dissection of the mandible with adjacent floor of the mouth is called the “commando
operation” (Combined dissection of the neck, Mandibulectomy and Oropharyngeal
resection).
In majority of cases of squamous cell carcinoma of oral cavity, a combination of surgery
and radiotherapy is required.
SELF ASSESSMENT
Q. 1. Premalignant lesions of the oral cavity include all except
a. erythroplakia d. leukoerythroplakia

b. fordyce’s spots e. submucous fibrosis


c. leukoplakia
Q. 2. All are true of carcinoma cheek except
a. can be caused by jagged carious c. often adenoid cystic variety
teeth d. often associated with leukoplakia
b. has recognized association with chewing e. often squamous cell carcinoma
betel nuts
Q. 3. Cause of oral cancer includes
a. alcohol d. smoking

b. betel nut chewing e. all of the above


c. HPV due to orogenital sex
Q. 4. Mixed salivary tumor (pleomorphic adenoma) of minor salivary glands most
commonly involves
a. hard palate d. oropharynx

b. lip e. tongue
c. nasopharynx
Q. 5. The most common site of tongue cancer is
a. base of the tongue d. tip of the tongue

b. dorsal surface e. ventral surface

c. lateral border
Q. 6. Clinical feature of oral cancer
a. discomfort on eating spicy food d. white/red patch on oral mucosa

b. non-healing ulcer e. all of the above

c. trismus
CONCEPT BOOK OF DISEASES OF EAR, NOSE & THROAT BY FAHIM AWAN
342 Part D Oral Cavity and Pharynx
Q. 7. Investigation of oral cancer includes
a. biopsy d. panendoscopy

b. CT e. all of the above


c. OPG
Q. 8. Dye used in supravital staining in oral cancer is
a. congo red d. methylene blue

b. fluorescein e. toluidine blue

c. gentian violet
-------------------------------------------------------- ANSWERS-----------------------------------------------------
Q.1. b 2. c 3. e 4. a
5. c 6. e 7. e 8. e
--------------------------------------------------------------------------------------------------------------------------
TUMORS OF OROPHARYNX
The oropharynx is that part of pharynx which extends from the hard palate above to the hyoid
bone below. Oropharynx is further subdivided into palatine arch and oropharynx proper.
Etiopathology:
1. Smoking and alcohol are known etiological factors.
2. Human papilloma virus (HPV) due to orogenital sex is a new known
factor. Oropharyngeal tumors may arise from;
1. A lining of squamous epithelium which may give rise to squamous cell carcinoma. It is
the most common malignancy of oropharynx.
2. Tonsils and collection of minor lymphoid tissue on the base of the tongue, which may
give rise to lymphoma and lymphoepithelioma.
3. Collection of minor salivary glands, which may give rise to salivary gland tumors.

Clinical features:
1. Sore throat, dysphagia and odynophagia are common presentations.
2. Change of voice, (hot potato voice) and referred earache.
On examination of oropharynx;
1. Lesion may be exophytic or ulcerative. Exophytic type spreads superficially and the
ulcerative type infiltrates deeply.
2. On palpation the lesion is indurated.
3. Cervical lymph node enlargement is found in 50 % cases.
Investigations:
CONCEPT BOOK OF DISEASES OF EAR, NOSE & THROAT BY FAHIM AWAN
343 Part D Oral Cavity and Pharynx

1. CT shows exact extent of the tumor and site, size, number and level of lymph node
involvement.
2. Panendoscopy (oropharyngoscopy, hypopharyngoscopy, laryngoscopy,
tracheobronchoscopy, esophagoscopy and nasopharyngoscopy) under general
anaesthesia.
3. Biopsy under general anesthesia is done to confirm the diagnosis and to find out
extension of the disease into surrounding structures.

Classification of primary tumor (T) of oropharynx


TX Primary tumor cannot be assessed
T0 No evidence of primary tumor
Tis Carcinoma in situ
T1 Tumor < 2 cm in greatest dimension
T2 Tumor > 2 cm but < 4 cm in greatest dimension
T3 Tumor > 4 cm in greatest dimension
T4a Tumor invades the larynx, deep/extrinsic muscle of tongue, medial
pterygoid, hard palate or mandible
T4b Tumor invades lateral pterygoid muscle, pterygoid plate,
lateral nasopharynx, skull base or carotid artery
Classification of neck node metastasis (N), distant metastasis (M) and TNM
classification is almost the same as that for carcinoma of larynx.
Treatment:
Most patients with an oropharyngeal tumor without a palpable lymph node of neck should be
treated with radiotherapy.
If there is lymph node involvement or the tumor is large then surgery is a better option.
In majority of cases a combination of surgery and radiotherapy is given.
Surgery usually involves extensive resection of tumor along with surrounding healthy
tissues, followed by reconstruction with flaps or grafts.
SELF ASSESSMENT
Q. 1. Cause of oropharyngeal carcinoma is
a. alcohol d. all of the above

b. HPV due to orogenital sex e. none of the above

c. smoking
CONCEPT BOOK OF DISEASES OF EAR, NOSE & THROAT BY FAHIM AWAN
344 Part D Oral Cavity and Pharynx
Q. 2. The most common benign neoplasm of tonsil is
a. fibroma d. neuroma

b. haemangioma e. papilloma
c. neurofibroma
Q. 3. The most common malignancy of tonsil is
a. adenocarcinoma d. rhabdomyosarcoma

b. Hodgkin’s lymphoma e. squamous cell carcinoma


c. non-Hodgkin’s lymphoma

Q. 4. Usual clinical feature of oropharyngeal carcinoma includes


a. cervical lymphadenopathy d. sore throat

b. dysphagia e. all of the above

c. referred earache
-------------------------------------------------------- ANSWERS-----------------------------------------------------
Q.1. d 2. e 3. e 4. e
--------------------------------------------------------------------------------------------------------------------------
TUMORS OF HYPOPHARYNX
Hypopharynx is also known as laryngopharynx. It extends from the level of the hyoid bone
to the lower border of the cricoid cartilage. Anatomically it is divided into three sites.
According to UICC classification these sites are; (remember three Ps)
1. Posterior pharyngeal wall; It extends from the level of the floor of the valeculla to the
level of the cricoarytenoid joint.
2. Pyriform fossa/sinus; It extends from the pharyngoepiglottic fold to the upper end of
the esophagus.
3. Postcricoid area; It extends from the level of the arytenoid cartilage to the inferior
border of the cricoid cartilage.
Etiopathology:
Benign tumors of hypopharynx are very rare. These include fibrolipoma and leiomyoma.
These are removed by lateral pharyngotomy approach.
The most common malignant tumor of hypopharynx is squamous cell carcinoma.
1. Smoking and alcohol are known etiological factors.
2. Human papilloma virus (HPV) due to orogenital sex is a new known factor. But is
thought to play less role than in oropharynx. Malignancy of hypopharynx is common in
the 5th to 6th decade.

CONCEPT BOOK OF DISEASES OF EAR, NOSE & THROAT BY FAHIM AWAN


345 Part D Oral Cavity and Pharynx

The incidence of hypopharyngeal carcinoma is much higher in men. Only postcricoid


carcinoma is more common in women and it presents in much earlier age group. Postcricoid
carcinoma is associated with Plummer Vinson syndrome.
Malignancies of hypopharynx have notorious propensity to metastasize to cervical lymph
nodes and in fact an enlarged lymph node may be the presenting symptom.

Clinical features:
1. Dysphagia or odynophagia are common presentations.
2. Dysphonia or dyspnea occur due to involvement of larynx.
3. There may be referred earache. There may also be complaint of weight loss.
On examination there may be:
1. Exophytic, fungating lesion in the region of hypopharynx with indirect laryngoscopic
mirror.
2. Fixation of the vocal cords or pooling of saliva in the pyriform sinuses is an important
sign which may indicate postcricoid carcinoma.
3. Laryngeal crepitus is absent in the postcricoid tumors.
4. Widening of laryngeal framework may be felt in advanced postcricoid carcinoma.
5. Cervical lymph node enlargement
Investigations:
1. CT/MRI shows the exact extension of the tumor and its relation to the neighbouring
structures. It may also indicate the site, size, number and level of lymph node involvement.
2. Panendoscopy under general anaesthesia is extremely important to find out the vertical
and horizontal extent of the disease. It is also useful in finding out second primary.
3. Biopsy is the last but not the least important part of endoscopy.
Classification of primary tumor (T) of hypopharynx
T1 Tumor < 2 cm and limited to one subsite of hypopharynx
T2 Tumor > 2 cm but < than 4 cm or invades more than one subsite of
hypopharynx or an adjacent site
T3 Tumor > 4 cm or with fixation of hemilarynx
T4 Tumor with extension to bone, muscle, skin, etc.
TX The minimum requirements to assess the tumor cannot be met
Classification of neck node metastasis (N), distant metastasis (M) and TNM
classification is almost the same as that for carcinoma of larynx
Treatment:

CONCEPT BOOK OF DISEASES OF EAR, NOSE & THROAT BY FAHIM AWAN


346 Part D Oral Cavity and Pharynx

Treatment of hypopharyngeal tumors is highly variable.


1. For limited malignancy of certain sites, local resection may be possible.
2. For extensive tumors, a total laryngopharyngectomy with gastric pull up may be
required.
3. Postoperative radiotherapy is usually required in majority of cases.
SELF ASSESSMENT
Q. 1. The following is component of hypopharynx

a. postcricoid area d. all of the above

b. posterior pharyngeal wall e. none of the above


c. pyriform sinus
Q. 2. All are usual clinical features of postcricoid carcinoma except
a. dysphagia d. pooling of saliva

b. fixation of vocal cord e. widening of laryngeal framework

c. laryngeal crepitus present


Scenario: A 48 years old female comes to ENT OPD with complaint of progressive
dysphagia for last one year. She also has complaint of slight hoarseness for past 6 months.
IDL examination shows slight pooling of saliva in pyriform sinus, restricted movements of
right vocal cord. She gives wasted appearance and has bilateral neck swellings.
1. What is the most probable clinical diagnosis?
2. What investigations would you order to confirm diagnosis?
3. What would be your management plan?
--------------------------------------------------------ANSWERS-----------------------------------------------------
Q.1. d 2. c
Scenario: 1. Hypopharyngeal malignancy 2. CT/MRI, panendoscopy under general
anaesthesia and biopsy 3. Total laryngopharyngoesophagectomy with gastric pull up,
postoperative radiotherapy.
--------------------------------------------------------------------------------------------------------------------------
MISCELLANEOUS DISEASES OF ORAL CAVITY AND PHARYNX
LUDWIG’S ANGINA
Definition:
It is rapidly developing infection of submandibular space.
Etiopathology:
Submandibular space is divided by the mylohoid muscle into sublingual space above and
submaxillary space below. Either of the one or both spaces may get infected in Ludwig’s
angina.
Causes of Ludwig’s angina are;
347 Part D Oral Cavity and Pharynx

1. Dental infection; Most commonly it is secondary to infection of the lower premolar or


molar tooth.
Roots of premolar usually lie above the attachment of mylohyoid and cause infection of
sublingual space.
Roots of molar usually lie below the attachment of mylohyoid and cause infection of
submaxillary space. Usual causative organisms in Ludwig’s angina of dental origin are
Streptococcus viridans and E. coli.
2. Submandibular sialadenitis may also lead to Ludwig’s angina occasionally.

Clinical features:
1. Usually there is preceding history of dental pain.
2. Pain, salivation and trismus are common
presentations. On examination; Left sided
Ludwig’s angina
1. When infection is in the submaxillary space, there is swelling of upper
part of neck giving a ‘bull neck’ appearance.
2. When infection is in the sublingual space, the floor of mouth become swollen and may
rise to the level of biting edge of teeth. Tongue is pushed upwards.
3. On palpation swelling is tender and hard.
Investigations:
1. OPG (orthopantomogram) is carried out to find out causative diseased tooth.
2. Pus for culture and sensitivity. If there is discharge of pus, it should be sent for
culture and sensitivity.
3. CT/MRI is carried out if complications of Ludwig’s angina are suspected.
Treatment:
1. Admission in hospital and observation for airway obstruction.
2. Parenteral antibiotics are started.
3. Incision and drainage are carried out if there is no response to antibiotics for 24-48
hours or if fluctuation is present.
If infection is localized to sublingual space it is carried through intraoral incision. If
infection is localized to submaxillary space it is carried through neck crease incision
under local anesthesia.
4. Causative tooth should get proper attention by dental surgeon.
SELF ASSESSMENT
Q. 1. Space primarily involved in Ludwig’s angina includes
a. danger area of face b. parapharyngeal space
CONCEPT BOOK OF DISEASES OF EAR, NOSE & THROAT BY FAHIM AWAN
348 Part D Oral Cavity and Pharynx

c. peritonsillar space e. submandibular space


d. retropharyngeal space
Q. 2. Ludwig’s angina can be best described by the following statement
a. abscess formation in the space of acute oral ulceration
Gillette d. pain in neck which simulates angina
b. acute ulcerative and necrotizing gin- pectoris
givitis e. rapidly spreading cellulitis of sub-
c. autoimmune disorder causing an mandibular space
Q. 3. Usual causative organism in Ludwig’s angina
a. E. coli d. both a & b

b. Streptococcus pneumoniae e. both a & c


c. Streptococcus viridans
Q. 4. The most common source of infection in Ludwig’s angina is
a. dental infection d. penetrating injury

b. lingual tonsillitis e. submandibular sialadenitis


c. local trauma
Q. 5. Generally, Ludwig’s angina may be secondary to all except
a. infection of lower molars d. soft tissue infection of floor of mouth

b. infection of lower premolars e. tonsillar infection


c. infection of upper molar
Q. 6. All are features of Ludwig’s angina except
a. bull neck d. trismus

b. profuse gingival bleeding e. tongue pushed upwards


c. salivation
Q. 7. The most important investigation to find out causative diseased tooth in Lud-wig’s
angina
a. culture and sensitivity of pus d. OPG

b. microscopy of pus e. X-ray paranasal sinuses


c. MRI
Q. 8. All are treatments of Ludwig’s angina except
a. admission in hospital d. prescribed antibiotic and advise follow
b. incision & drainage up after one week
c. parentral antibiotics e. treatment of causative tooth
Scenario: A 20 years old girl presented with diffuse swelling in submental and right
submandibular region for last four days. On examination skin over the swelling is red and
tender. First right lower molar tooth is carious.
CONCEPT BOOK OF DISEASES OF EAR, NOSE & THROAT BY FAHIM AWAN
349 Part D Oral Cavity and Pharynx
1. What is the most probable clinical diagnosis?
2. What are possible causative organisms?
3. How will you manage this case?
-------------------------------------- ANSWERS---------------------------------------
------------------ --------------
Q.1. e 2. e 3. e 4. a 5. c 6. b 7. d 8. d
Scenario: 1. Ludwig’s angina 2. Streptococcus viridian and E. coli 3. Admission in hospital,
maintenance of airway, parenteral antibiotics, incision and drainage.
--------------------------------------------------------------------------------------------------------------------------
VINCENT’S ANGINA
Synonyms: Trench mouth, Acute Necrotizing Ulcerative Gingivitis (ANUG)
Definition:
It is a progressive painful infection with ulceration, swelling and
sloughing of tissues of the oral cavity and pharynx due to spread of
infection from the gums.
Etiopathology:
It is a painful infection of gums with sudden onset. It is distinguished from A case of
more common chronic periodontitis which is rarely painful. Fusiform Vincent’s angina
bacilli (anaerobes Gram negative) and Spirochetes (Borrelia vincentii)
are
the usual causative organisms. Predisposing factors include poor orodental hygiene, smoking,
malnutrition, psychological stress and immunosuppression. Although the condition has a
rapid onset and is debilitating, it usually resolves quickly.
Clinical features:
1. Severe gingival pain
2. Profuse gingival bleeding that requires little or no provocation
3. Interdental papillae or ulceration with necrotic slough
4. Interoral halitosis
5. Bad metallic taste

Investigations:
Diagnosis is usually made on clinical grounds. In doubtful cases smear for Fusiform bacilli
and Spirochetes (Borrelia vincentii) is diagnostic.
Treatment:
1. Oral antibiotics including metronidazole are given.
2. Strong analgesics are given to relieve pain.
3. Irrigation and debridement of necrotic area, improvement of orodental hygiene and use

CONCEPT BOOK OF DISEASES OF EAR, NOSE & THROAT BY FAHIM AWAN


350 Part D Oral Cavity and Pharynx

of oral rinse.
4. Underlying predisposing cause must be addressed.
Prognosis:
If untreated then infection may lead to rapid destruction of periodentum and can spread as
necrotizing stomatitis or cancrum oris (Noma).
It is especially dangerous in people with immunosuppression. It can spread to
neighbouring tissues in teeth, lips and bones of the jaw.

SELF ASSESSMENT
Q. 1. Vincent’s angina is also called
a. acute necrotizing ulcerative gingivitis d. both a & b

b. aphthous ulcer e. both a & c


c. trench mouth
Q. 2. Usual causative organism in Vincent’s angina is
a. Borrelia vincentii d. both a & b

b. E. coli e. both a & c


c. Fusiform bacilli
Q. 3. Condition in which smear clinches the diagnosis is
a. aphthous ulcers d. Quinsy

b. infectious mononucleosis e. Vincent’s angina


c. Ludwig’s angina
Q. 4. Clinical feature of Vincent’s angina is
a. interdental papillae or ulceration d. severe gingival pain

b. interoral halitosis e. all of the above


c. profuse gingival bleeding
Q. 5. All are treatments of Vincent’s angina except
a. aminoglycosides d. metronidazole

b. analgesics e. removal of underlying cause

c. irrigation and debridement


Scenario: A 45 years old male presented necrosis of gingival margins with slough for last
few days. Gingiva are red and edematous.
1. What is the most probable diagnosis?
2. What are your differential diagnoses?
3. How will you treat this patient?
CONCEPT BOOK OF DISEASES OF EAR, NOSE & THROAT BY FAHIM AWAN
351 Part D Oral Cavity and Pharynx
-------------------------------
------------------------- ANSWERS-----------------------------------------------------

Q.1. e 2. e 3. e 4. e 5. a
Scenario: 1. Vincent’s angina 2. Apthous ulcer, traumatic ulceration 3. Oral antibiotics,
analgesics, irrigation and debridement.
--------------------------------------------------------------------------------------------------------------------------
RANULA
Definition:
It is a uni or multilocular mucous filled cyst in the floor of mouth on one side of frenulum.
The term ranula is derived from a Latin word “rana” which means “belly of a small frog’’.
Etiopathology:
It is considered to arise from sublingual salivary gland due to obstruction of its duct. It is of
two types;
1. Simple ranula: It is limited to floor of mouth.
2. Plunging or burrowing ranula: It extends through the mylohyoid
muscle and presents as a swelling in neck as well.

Clinical features: Right sided ranula


1. Patient presents with painless, progressive, translucent, bluish, cystic swelling in the
floor of mouth on one side of frenulum. It may push the tongue upwards.
2. In cases of plunging ranula there is an associated swelling in the upper part of neck as
well.

Investigations:
Ultrasound/CT/MRI may be required rarely to find out exact extent of ranula.
Treatment:
1. Simple ranula is often treated by complete excision of ranula along with sublingual
gland through an intraoral incision.
2. Plunging ranula is treated by marsupialization of ranula along with excision of the
sublingual gland.
As the ranula is very thin walled, it is difficult to excise it completely. So incomplete removal
may lead to recurrence.
Differential diagnosis:
1. Benign lymphoepithelial cyst
2. Dermoids
SELF ASSESSMENT
352 Part D Oral Cavity and Pharynx
Q. 1. A bluish cystic translucent swelling in the floor of mouth on one side of frenulum is
a. calculus in submandibular duct d. mucocele
b. dermoid cyst e. ranula
c. epidermoid cyst
Q. 2. All are true about ranula except
a. plunging ranula presents as swelling in mouth
the upper part of neck d. treated by intraoral incision
b. presents as bluish cystic swelling e. usually presents in center of floor of
c. simple ranula is limited to floor of mouth
Q. 3. All are treatments of ranula except
a. combined with excision of sublingual c. incomplete removal may lead to re-
gland currence.
b. combined with excision of submandib- d. plunging ranula is marsupialized
ular gland e. simple ranula is excised
Scenario: A 15 years old boy presents to you with progressive, cystic, bluish swelling on one
side of floor of mouth for last 06 months.
1. What is the most probable clinical diagnosis?
2. What is the differential diagnosis?
3. How will you manage this patient?
--------------------------------------------------------ANSWERS-----------------------------------------------------
-
Q.1. e 2. e 3. b
Scenario: 1. Ranula 2. Benign lypmhoepithelial cyst, dermoids 3. Excision if simple ranula
and marsupialization if plunging ranula.
--------------------------------------------------------------------------------------------------------------------------
SUBMUCOUS FIBROSIS (SMF)
Definition:
It is deposition of fibrous tissue in the submucosal space of the oral cavity and pharynx.
Etiopathology:
It is found usually in poor socioeconomic people.
1. Betel nut, pan, gutka or tobacco chewing are major causative
agents.
2. Smoking and alcohol are also contributing factors.
3. Anemia, vitamin A, zinc and certain micronutrient
deficiencies may also be causative factors.
A case of submucosis fibrosis
353
Subepithelial inflammatory response to the irritants is presumed to be the cause of
submucous fibrosis. There is formation of fibroelastic connective tissue in the lamina propria
associated with atrophy of epithelium.
Patient usually presents between third and fourth decade of life. It is a pre-malignant
condition with a tendency to undergo malignant change in 3-6% of cases.
Clinical features:
1. There is complaint of progressive sore mouth, intolerance to chillies and spices.
2. In late cases there may be inability to open the mouth (trismus) and protrude the
tongue.
On examination;
1. There is trismus and blanching of oral mucosa.
2. There are fibrous bands over the soft palate, buccal mucosa and faucial pillars.
Staging:
There are following stages of submucous fibrosis.
1. Burning sensation in mouth
2. Limitation of jaw movements
Sequelae of limitation of jaw
movements:
A. Poor orodental hygiene
B. Emaciation
C. Limitation of palatal movement which may lead to;
a. Nasal regurgitation of fluids.
b. Otitis media with effusion (conductive hearing loss, earache and tinnitus).
Treatment:
A. Conservative:
1. Removal of underlying factors. Betel nut, pan, gutka, tobacco chewing, smoking
and alcohol should be avoided.
2. Correction of anemia, vitamin, zinc and nutritional deficiencies.
3. Topical steroid injections into the affected site at a dose of 4 mg biweekly for 8 to
12 weeks may show some improvement. It may be combined with hylase injections
1500 IU/ml.
4. Antioxidants are given for a long time.
5. Physiotherapy of jaw should be encouraged.
B. Surgical:
354 Part D Oral Cavity and Pharynx

1. Removal of the fibrous tissue and skin grafting is of some success.


2. Laser excision of the fibrous bands may be carried out.
SELF ASSESSMENT
Q. 1. All are usual causative factors of submucous fibrosis except
a. alcoholism d. smoking

b. betel nut chewing e. tobacco chewing


c. human papilloma virus
Q. 2. All are true about submucous fibrosis except
a. malignant condition lies

b. premalignant condition d. presentation with sore mouth

c. presentation with intolerance to chil- e. presentation with trismus


Q. 3. On examination of submucous fibrosis, there are fibrous bands over
a. buccal mucosa d. all of the above

b. facial pillar e. none of the above


c. soft palate
Q. 4. The most common site involved in submucous fibrosis is
a. floor of mouth d. lingual mucosa

b. hard palate mucosa e. soft palate mucosa


c. labial mucosa
Q. 5. A young man presents with progressive trismus and intolerance to chillies for last 02
years. Pharyngeal examination shows white blanched area over soft palate, pillar and
pterygomandibular area. The most likely diagnosis is
a. aphthous ulcers d. scleroma of pharynx

b. atrophic pharyngitis e. submucous fibrosis


c. iron deficiency anemia
Q. 6. A 60 years old female presents with progressive trismus and burning sensation in
mouth for last 04 years. She has history of betel nut chewing. The most likely diagnosis is
a. aphthous ulcers d. leukoplakia

b. erythroplakia e. submucous fibrosis


c. geographic tongue
Q. 7. All are treatment options of submucous fibrosis except
a. avoidance of betel nut chewing d. skin grafting
b. cytotoxic therapy e. topical steroid injection

c. laser excision

CONCEPT BOOK OF DISEASES OF EAR, NOSE & THROAT BY FAHIM AWAN


355 Part D Oral Cavity and Pharynx

Scenario: A 45 years old female visited OPD for the treatment of burning sensation in throat
and trismus for the last two years. She has a habit of pan chewing and alcohol consumption.
Oral cavity examination reveals poor orodental hygiene and whitish oral cavity mucosa in
the vestibule.
1. What is the patient pathologically suffering from?
2. Enumerate four predisposing factors in this patient.
3. What is the pathological significance of this condition?
4. How will you manage this case?
-------------------------------------------------------- ANSWERS-----------------------------------------------------
Q.1. c 2. a 3. d 4. e 5. e
6. e 7. b
Scenario: 1. Submucous fibrosis 2. Betel nut, pan, gutka, smoking, alcohol, vitamins, anemia,
zinc and nutritional deficiencies 3. It is premalignant condition 4. Removal of underlying
factors, correction of anemia, vitamins, zinc, topical steroid injection. Surgical removal of
fibrous tissue and grafting, laser treatment.
--------------------------------------------------------------------------------------------------------------------------
-
GLOBUS PHARYNGEUS
Synonyms: Globus pharyngis, Globus hystericus, Globus syndrome
Definition:
It is a feeling of a lump or tightness in the throat. Latin “Globus” means a Sensation of
ball. It is a functional disorder. lump in throat
Clinical features:
Patient is usually young or middle-aged female. Patient presents with a sensation of lump in
the throat or intermittent (pseudo) dysphagia. The dysphagia is between the meals and
there is no dysphagia during meal.
Generally, history of anxiety, stress from any source such as family, social, employment,
education, social or psychological is positive.
Clinical examination is unremarkable.
Investigations:
Diagnosis is made on high index of suspicion and by exclusion of other causes of dysphagia.
Radiological and endoscopic findings are unremarkable.
Treatment:
Treatment is directed towards reassurance, alleviation of anxiety and relaxation training.
Differential diagnosis:
1. Cricopharyngeal spasm
CONCEPT BOOK OF DISEASES OF EAR, NOSE & THROAT BY FAHIM AWAN
356 Part D Oral Cavity and Pharynx

2. Pharyngeal pouch
3. Postcricoid carcinoma
SELF ASSESSMENT
Q. 1. Globus pharyngeus is also called

a. globus hystericus d. all of the above

b. globus pharyngis e. none of the above


c. globus syndrome
Q. 2. All are true about globus pharyngeus except
a. complaints are persistent lump in throat
b. presentation with dysphagia occuring d. positive history of anxiety
between meals rather than during meal e. positive history of emotional instability
c. patient presents with sensation of
Q. 3. Diagnosis of globus pharyngeus is made on
a. endoscopic findings d. manometric studies

b. exclusion of other causes e. X-ray barium swallow


c. fluoroscopy
Q. 4. Treatment of globus pharyngeus includes
a. alleviation of anxiety d. all of the above

b. reassurance e. none of the above

c. relaxation training
Scenario: A 45 years old female presented with the history of intermittent dysphagia and
feeling of lump in the neck for last 06 months. On examination and investigation, no
abnormality is present;
1. What is the most probable clinical diagnosis?
2. How will you treat this patient?
-------------------------------------------------------- ANSWERS-----------------------------------------------------
Q.1. d 2. a 3. b 4.d
Scenario: 1. Globus pharyngeus 2. Treatment is directed towards reassurance, alleviation of
anxiety and relaxation training.
--------------------------------------------------------------------------------------------------------------------------

PARAPHARYNGEAL SPACE
Synonyms: Pterygomaxillary space, Lateral pharyngeal
space, Pharyngomaxillary space
Anatomy:
357 Part D Oral Cavity and Pharynx

Parapharyngeal space is roughly in the shape of a “five-sided inverted pyramid” with base
towards sphenoid bone and apex towards lesser horn of hyoid bone.
Relations of the space are;
1. Anteriorly----- --pterygoid muscles
2. Posteriorly---- --cervical spine covered by prevertebral muscles and fascia
3. Medially-------- --pharynx with tonsillar fossa inferiorly and eustachian tube superiorly
4. Laterally-------- --ascending ramus of mandible
5. Posterolaterally---deep lobe of parotid gland
Parapharyngeal space is divided into anterior (pre-styloid) and posterior (post-styloid)
compartments by styloid process and three muscles and two tendons attached to it (Riolan’s
bouquet).
Almost all the important structures i.e., internal carotid artery, internal jugular vein, last
four cranial nerves and cervical sympathetic chain lie in post-styloid compartment.
Parapharyngeal space is connected with retropharyngeal space in an area just medial to
carotid sheath.
SELF ASSESSMENT
Q. 1. Parapharyngeal space is also called

a. lateral pharyngeal space d. all of the above

b. pterygomaxillary space e. none of the above


c. pharyngomaxillary space
Q. 2. Regarding parapharyngeal space all the structures lie in poststyloid compartment
except
a. cervical sympathetic chain d. internal carotid artery

b. external carotid artery e. internal jugular vein


c. glossopharyngeal nerve
Q. 3. Regarding parapharyngeal space following structure lie in poststyloid compartment
a. accessory nerve d. all of the above

b. hypoglossal nerve e. none of the above

c. vagus nerve
-------------------------------------------------------- ANSWERS-----------------------------------------------------
Q.1. d 2. b 3. d
--------------------------------------------------------------------------------------------------------------------------
-

PARAPHARYNGEAL SPACE TUMORS


These tumors present in the upper neck near the angle of mandible or retromandibular area.
358 Part D Oral Cavity and Pharynx

These may also be seen intraorally displacing the tonsil, lateral pharyngeal wall and soft
palate medially.
Sources of parapharyngeal space tumors;
1. Originate from various types of cells and tissues present in this space.
2. By direct extension of tumor arising from various surrounding structures.
3. Through lymphogenous and hematogenous spread.
Types of parapharyngeal space tumors;
1. Salivary gland tumors----------50%
2. Neurogenic tumors-------------30%
3. Variety of the other tumors---20%
A. Salivary gland tumors:
There are two types of salivary gland tumors in the parapharyngeal space:
1. Which arise de-novo from salivary gland tissue in the parapharyngeal space.
2. Parotid tumors which arise from deep lobe of the parotid gland.
B. Neurogenic tumors:
1. Nerve sheath tumors:
a. Schwannoma:
All peripheral sensory and motor axons are covered by schwan cells. Tumor arising
from these cells is called schwannoma. This is the most common neurogenic tumor in
parapharyngeal space. This tumor is capsulated. This is usually solitary and is seen
in relation to nerve trunks. This can readily be dissected from the nerve of origin,
most often the vagus nerve. Axons are found in the capsule and not within the bulk of
the tumor.
b. Neurofibroma:
Neurofibromas arise from schwan cells and the perineural fibroblasts. These tumors
are unencapsulated. These are usually multiple. These encompass the nerve of
origin and virtually it is impossible to dissect out the tumor without sacrificing the
nerve of origin.
c. Malignant schwannoma:

This is a malignant tumor of nerve sheath origin which infiltrates locally and also

metastasize.

2. Nerve cell tumors:


These include neurofibroma and ganglioneuroma.
3. Paraganglioma:
359 Part D Oral Cavity and Pharynx

Synonyms: Glomus tumor, Chemodectoma


These tumors arise from paraganglionic cells. These occur in people living at high altitude.
These have got a familial tendency. These arise often in females and are usually multicentric.
These tumors are often encapsulated. Malignant change occurs in less than 10% of cases.
Clinical features:
Most tumors of parapharyngeal space are benign, slow growing and produce symptoms by
causing pressure on nearby structures.
Patient may present with chronic sore throat, speech defects, dysphagia or neuralgia
like pain. There may be trismus and narrowing of pharynx leading to airway obstruction.
There may be lump on the side of upper part of neck. There may be complaint of
hearing loss, secondary to pressure on the eustachian tube.
There may be involvement of 9th, 10th, 11th, 12th cranial nerves and cervical
sympathetic chain. Involvement of cervical sympathetic chain leads to Horner’s syndrome
(ptosis, meiosis, anhydrosis, enophthalmosis and loss of ciliospinal reflex).
Bimanual ballottement of the tumor is very important part of examination to find out exact
site and extent of the tumor. Sometimes diagnosis is made incidentally during routine
examination of head and neck by inspection and palpation.
Investigation:
1. CT/MRI may determine not only exact site of origin and extent of tumor but also site,
size, number and level of lymph node involvement.
2. Fine needle aspiration cytology/incisional biopsy/ excisional biopsy is required to reach
tissue diagnosis.
Tumor derivatives from neural crest:
A. Schwann cell
a. Neurofibroma
b. Schwannoma
B. Sympathicoblast
a. Paraganglionic cells
1. Glomus tympanicum
2. Glomus jugulare
3. Glomus vagale
4. Carotid body tumors
b. Ganglionic cells
1. Benign gangliomata

CONCEPT BOOK OF DISEASES OF EAR, NOSE & THROAT BY FAHIM AWAN


360 Part D Oral Cavity and Pharynx

2. Malignant gangliomata
SELF ASSESSMENT
Q. 1. The most common primary parapharyngeal tumor is
a. neurofibroma d. schwannoma

b. paraganglioma e. vascular tumor


c. salivary gland tumor
Q. 2. The most common tumor of parapharyngeal space originates from
a. lymph nodes d. salivary tissue

b. nerves e. vessels
c. squamous epithelium
Q. 3. Usual clinical feature of parapharyngeal space tumor includes
a. dysphagia d. lump on the side of upper part of
b. hearing loss neck
c. Horner’s syndrome e. All of the above
Q. 4. A parapharyngeal mass displacing the tonsil and tonsillar fossa medially with
pulsations on intraoral palpation is due to
a. Hodgkin’s lymphoma d. pleomorphic adenoma

b. internal carotid artery aneurysm e. Schwannoma of parapharyngeal space

c. Non-Hodgkin’s lymphoma
Scenario: A 55 years old male presented in ENT OPD with the history of chronic sore throat,
speech defects and dysphagia for last 01 year. He is also having hard and fixed swelling at
level III with 9th, 10th, 11th and 12th cranial nerve palsy.
1. What is the most probable diagnosis?
2. What investigations will you carry out?
3. How will you manage this patient?
ANSWERS-----------------------------------------------
-------------------------------------------------------- ------
Q.1. c 2. d 3. e 4.b
Scenario: 1. Parapharyngeal space tumor 2. CT/MRI, FNAC 3. Surgery, chemotherapy or
radiotherapy depending upon histopathology.
--------------------------------------------------------------------------------------------------------------------------
-
SLEEP APNEA SYNDROME
Definition:
It is defined as 30 apneic spells (each spell lasting for at least 10 seconds or more) during a
period of 7 hours sleep.
361 Part D Oral Cavity and Pharynx

Etiopathology:
Sleep apnea syndrome may be central,
obstructive or mixed.
A. Central sleep apnea syndrome:
It occurs due to a failure of respiratory
drive.
B. Obstructive Sleep Apnea
Syndrome (OSAS):
It occurs due to the obstruction of upper
respiratory passages.
C. Mixed sleep apnea syndrome:
In it both the central and mixed causes are present.
Clinical features:
1. Patient presents with periods of apnea during sleep.
2. Sleep is completely restless.
3. There is loud snoring. It is to be remembered that all patients with OSAS snore heavily
but not all snorers have OSAS.
4. During sleep there is kicking of legs, slapping of arms and choking.
5. There is early morning headache, irritability and daytime somnolence. There is also lack
of concentration and memory.
6. On examination, there may be obesity, nasal polypi, antrochoanal polypi, DNS,
adenoids, enlarged tonsils, lax soft palate/uvula, nasopharyngeal tumor, retrognathia or
macroglossia.
Pickwickian syndrome is a typical example of OSAS. This is a squat obese individual with
a short fat neck who snores heavily with increasing respiratory effort until prolonged apnoeic
episodes occur with cyanosis. The patient then begins to breath again with semi-arousal only
to drift back into sleep and a further apnoeic episode. This disturbed sleep with oxygen
desaturation leads to daytime somnolence such that individual is falling off to sleep during
daytime. The patient develops right sided heart failure.
Complications:
In untreated cases of OSAS, there may be hypertension, type II diabetes, cardiac arrythmias,
angina pectoris, myocardial infarction, cerebrovascular accidents, or cor pulmonale.
Excessive daytime somnolence can lead to higher incidence of road traffic accidents (RTA).
Investigations:
These highly depend on the findings of history and examination.
1. These may simply include sinus and chest X-rays and pulmonary function tests.
2. Polysomnography in a sleep laboratory i.e., ECG, EEG (electroencephalogram), EOG
(electrooculogram), EMG (electromyogram), record of chest and abdominal respiratory
movements, nasal and oral airflow, pulse oximetry, recording of body movements with a
vibration transducer and tape recording of snoring.
3. In an awake patient, Muller’s maneuver may be performed. In this flexible
nasopharyngoscope is passed to the postnasal space and the patient is instructed to
attempt a snore with the mouth closed. With this maneuver the principal site of
obstruction can be seen directly.
4. Drug induced sedation endoscopy (DISE) is useful in evaluating the exact site and
extent of upper airway obstruction.
5. Cephalometry with CT/MRI may be used to evaluate upper airway.
Treatment:
A. Conservative:
1. Weight reduction
2. Avoidance of sedatives, hypnotics, smoking, alcohol and high altitude.
3. Prosthesis of different types may be used during sleep to relieve the site of
obstruction in obstructive sleep apnea syndrome (OSAS).
4. CPAP (Continuous Positive Airway Pressure) is very effective for the management
of OSAS.
5. APAP (Autotitrating PAP) is now available which continuously adjusts the pressure.
6. Drugs like tricyclic antidepressants, L-tryptophan and medroxyprogesterone are
given in central sleep apnea.
B. Surgical:
1. Surgical procedures such as
septoplasty, polypectomy, adenoidectomy,
tonsillectomy, UvuloPlatoPharyngoPlasty (UP3),
mandibular advancement, genial tubercle
advancement, hyoid suspension, hyoid expansion
or base of tongue resection to relieve obstruction.
2. Tracheostomy rarely may be the only
method to relieve obstructive sleep apnea
syndrome.
SELF ASSESSMENT Uvuloplatopharyngoplasty

Q. 1. Total number of apnoeic spells in OSAS in 07 hours


sleep is
a. 18 d. 30

b. 22 e. 44

c. 26
363 Part D Oral Cavity and Pharynx
Q. 2. Total duration of each apnoeic spells in OSAS is
a. at least 04 seconds d. at least 10 seconds

b. at least 06 seconds e. at least 12 seconds


c. at least 08 seconds
Q. 3. All of the following conditions are usually associated with snoring except
a. adenoids
b. angiofibroma d. laryngeal papillomatosis

c. antrochoanal polyp e. tonsillar enlargement


Q. 4. All of the following are usual symptoms of OSAS excep
a. daytime somnolence d. morning headache

b. full concentration e. snoring


c. lack of memory
Q. 5. All of the following are usual complications of OSAS except
a. angina pectoris d. myocardial infarction

b. cardiac arrythmia e. type I diabetes


c. hypertension
Q. 6. All of the following are usual complications of OSAS except
a. cerebrovascular accidents d. high incidence of road traffic acci-
b. corpulmonale dents
c. daytime alertness e. type II diabetes
Q. 7. All of the following are investigations of OSAS except
a. cephalometry d. Muller maneuver

b. DISE e. X-ray of paranasal sinuses and chest


c. monosomnography
Q. 8. Following is not included in polysomnographic study of OSAS
a. ECG d. EMG

b. ECoG e. EOG
c. EEG
Q. 9. All of the following are conservative managements of OSAS except
a. avoidance of alcohol d. smoking

b. avoidance of high altitude e. weight reduction


c. avoidance of sedatives
Q. 10. All of the following are conservative managements of OSAS except
a. APAP c. endotracheal intubation
b. CPAP d. prosthesis
364 Part D Oral Cavity and Pharynx
e. tricyclic antidepressants
Q.11. All of the following are surgical managements of OSAS except
a. base of tongue resection d. hyoid expansion

b. cricothyroidotomy e. hyoid suspension


c. genial tubercle advancement
Q. 12. All of the following are surgical managements of OSAS except
a. CPAP d. tracheostomy

b. mandibular advancement e. UP3


c. septoplasty/adenoidectomy
Q. 13. Pickwickian syndrome includes all the following except
a. heavy snoring d. short neck

b. left heart failure e. thick neck

c. obesity
Scenario: A 40 years old male presented in ENT OPD with the complaint of disturbed sleep
and daytime somnolence for last 05 years. His wife added that he has loud snoring and apnea
for which she needs to wake him up sometimes.
1. What is the most probable clinical diagnosis?
2. What are the usual causes of this condition?
3. How will you treat this patient?
---------------------------------------------- ANSWERS--------------------------------------------
---------- ---------
Q.1. d 2. d 3. d 4. b 5. e 6. c 7. c
8. b 9. d 10. c 11. b 12. a 13. b
Scenario: 1. Obstructive sleep apnea 2. Obstruction of upper airway 3. Conservative and
surgical treatment.
--------------------------------------------------------------------------------------------------------------------------
-
PHARYNGEAL POUCH
Synonyms: Zenker’s diverticulum, Posterior pharyngeal
pulsion diverticulum, Hypopharyngeal diverticulum

Definition:
Anatomy of pharyngeal
In this condition, pharyngeal mucosa (outpouching of epithelium pouch
and lamina propria) herniates through Killian’s dehiscence (also
known as ‘gateway of tears’).
Etiopathology:
Inferior constrictor muscle has two parts thyropharyngeus and cricopharyngeus. Killian’s
dehiscence is a weak area between the two. Exact cause of pharyngeal pouch is unknown.
365 Part D Oral Cavity and Pharynx

It is said to be due to cricopharyngeal spasm. It may also be due to incoordinated


contractions during the act of swallowing. It is usually seen in older age group. Herniation of
pouch starts in the midline. It is initially posterior to esophagus and then comes to lie usually
on left side. When the mouth of pouch is wider than the lumen of esophagus, food
preferentially enters the pouch. In long standing cases, carcinoma can very rarely develop in
pharyngeal pouch.

Clinical features:
1. Dysphagia which appears after a few initial boluses is the most common presentation.
The pouch gets filled with food and causes pressure on the esophagus leading to
dysphagia.
2. Gurgling sound during swallowing is sometimes noticed by patient.
3. At night when the patient is lying, undigested food may regurgitate leading to coughing,
choking and hoarseness.
4. There may be complaint of foul taste in mouth due to prolonged retention of undigested
food.
5. There may be complaint of weight loss due to prolonged dysphagia.
On examination;
1. Patient is usually emaciated.
2. There may be swelling in neck usually on left side in the lower part of anterior triangle.
3. Swelling is usually soft and reducible. It may gurgle on palpation and is known as
Boyce’s sign.
4. Spasm of coughing may be caused during palpation due to spillage of contents into the
larynx.
Investigations:
History and examination are usually diagnostic.
1. Plain X-ray soft tissue neck lateral view may show
triangular translucency in prevertebral soft tissues.
Apex of the triangle is at cricoid level while base of the
translucent triangle has a meniscus due to fluid in the
fundus of the sac with air above it.
2. Barium swallow with fluoroscopy usually confirms
the sac and its size. The sac gives the classic ‘pear
Zenker’s diverticulum with barium
shape’ appearance. swallow
3. Esophagoscopy may be required not only for diagnosis
but also, to rule out suspected carcinoma of the diverticulum.

Treatment:
366 Part D Oral Cavity and Pharynx

A small pouch with minimal symptoms may not require any intervention.
1. Diverticulectomy i.e., excision of the pouch and cricopharyngeal myotomy through a
neck incision.
2. Dohlman’s operation: The wall between the esophagus and the pouch is divided either
by diathermy or carbon dioxide laser using special endoscopic equipments.
The excised pouch should be sent for histopathological examination to rule out malignancy.
SELF ASSESSMENT
Q. 1. Pharyngeal pouch is also called
a. hypopharyngeal diverticulum c. Zenker’s diverticulum
b. posterior pharyngeal pulsion divertic- d. all of the above
ulum e. none of the above
Q. 2. Which statement is true about Killian’s dehiscence?
a. also called Lamer-Hackermann area d. lies between esophageal muscle
b. area of weakness below superior con- fibers
strictor muscle e. lies between thyropharyngeus and
c. causes laryngocele cricopharyngeus
Q. 3. All are usual clinical features of pharyngeal pouch except
a. dysphagia which appears after a few triangle
boluses d. swelling may gurgle on palpation
b. foul taste in mouth e. swelling is irreducible
c. swelling in lower part of left anterior
Q. 4. All are true about pharyngeal pouch except
a. Boyce’s sign is positive d. Heller’s myotomy may be performed
b. barium swallow shows pear shaped for its treatment
appearance e. treatment is done with excision of
c. Dohlman’s operation may be per- pouch
formed for its treatment
Q. 5. All of the following statements about Zenker’s diverticulum are correct except
a. arises from posterior part of hypo- c. traction diverticulum

pharynx
d. treated by cricopharyngeal myotomy

b. causes regurgitation of digested food e. treated by diverticulectomy

Scenario: A 38 years old male presented with complain of dysphagia after 2 to 3 bites of
food and regurgitation of undigested food at night time. The symptom is lasting for several
years. He is also having chest infection.
1. What is the most probable diagnosis?
367 Part D Oral Cavity and Pharynx
2. What are the positive findings on radiological investigations?
3. How will you treat this patient?
ANSWERS--------------------------------------------
-------------------------------------------------------- ---------
Q.1. d 2. e 3. e 4. d 5. b
Scenario: 1. Pharyngeal pouch 2. Triangular translucency in prevertebral soft tissues on X-
ray and on barium swallow with fluoroscopy ‘pear shape’ appearance 3. Diverticulectomy
and Dohlman’s procedure.
--------------------------------------------------------------------------------------------------------------------------
-

DISEASES OF SALIVARY GLANDS


MUMPS/VIRAL PAROTITIS
Definition:
It is a viral infection of the parotid gland usually caused by paramyxovirus.
Etiopathology:
Children are usually affected but occasionally adults may also have mumps. It is transmitted
by droplet infection. Incubation period is 2-3 weeks.
Clinical features:
1. There is swelling of parotid gland usually on one but occasionally on
both sides.
2. There is complaint of pain in the region of parotid gland, rise in body
temperature, malaise and anorexia.
3. On examination there is swelling and tenderness of parotid gland.
Complications:
Right sided mumps
There may be;
1. Epididymo-orchitis and oophoritis which may result in
subfertility.
2. Unilateral sensorineural hearing loss secondary to labyrinthitis.
3. Meningitis
4. Pancreatitis
Investigations:
Diagnosis is usually made on clinical grounds.
1. Serum and urinary amylase are raised during the first week.
2. Serum IgG and IgM are also usually raised.
Treatment:
368
Simple analgesics are usually sufficient to relieve pain of parotitis and orchitis. Parotid
swelling usually subsides within 7-10 days.
Prevention:
Immunization with MMR (Mumps, Measles, Rubella) is usually given at the age of 15
months.
SELF ASSESSMENT
Q. 1. All of the following are true of mumps except
a. incubation period is 2-3 weeks raised

b. mostly affects children d. transmitted by feco-oral route

c. serum and urine amylase levels are e. unilateral parotid is usually affected
Q. 2. Usual complications that can occur with mumps include all except

a. conjuctivitis d. pancreatitis

b. epididymo-orchitis e. unilateral sensorineural hearing loss.

c. meningitis
Q. 3. Treatment of choice in mumps is

a. analgesics d. hyperimmune gamma globulin

b. antiviral drugs e. steroids

c. broad-spectrum antibiotics
Scenario: A 04 years old child presented in ENT OPD with parotid swelling, high-grade
fever and malaise for past 05 days. On examination of swelling, severe tenderness is present.
Investigations shows raised levels of serum and urinary amylase and serum IgG and IgM.
1. What is your diagnosis?
2. What investigations will you carry out?
3. How will you treat this patient?
-------------------------------------------------------- ANSWERS-----------------------------------------------------
Q.1. d 2. a 3. a
Scenario: 1. Mumps 2. Serum and urinary amylase, serum IgG and IgM 3. Conservative with
analgesics.
--------------------------------------------------------------------------------------------------------------------------
-
ACUTE SUPPURATIVE PAROTITIS
Etiopathology:
The most common causative organism is Staphylococcus aureus but other gram-positive
anaerobic organisms may also be seen. It is most commonly seen in elderly, debilitated and
dehydrated patients. Infection usually travels from the oral cavity through the parotid duct. It
may involve parotid gland as well as sublingual salivary gland.
369

Clinical features:
1. Severe pain and enlargement of the parotid gland are usual symptoms.
2. On palpation parotid gland is tender.
A case of acute
3. Opening of the parotid duct (Stensen’s duct) is swollen and red.
suppurative
4. Gentle pressure on the parotid gland expresses pus from the opening of parotitis
parotid duct.
5. Patient is usually toxic.
Investigations:
1. Microscopy, culture and sensitivity of pus collected from the opening of parotid duct.

2. Total leukocyte count is usually elevated with increase in neutrophils.

Treatment:
1. Parenteral antibiotics along with adequate hydration are given.
2. Surgical drainage is carried out if there is collection of pus.
3. Good oral hygiene is advised.
4. Sialogogues are prescribed to promote salivation.
SELF ASSESSMENT
Scenario: A 30 years old man presented in ENT OPD with the complains of severe pain and
swelling in parotid region for last few days. On examination, he looks toxic, has swelling with
tenderness of parotid region and oral cavity examination shows pus coming from parotid
duct on pressing the parotid gland. TLC count is raised with elevated neutrophils.
1. What is your clinical diagnosis?
2. How will you investigate this case?
3. What are the treatment options in this patient?
--------------------------------------------------------ANSWERS-----------------------------------------------------
Scenario: 1. Acute suppurative parotitis 2. Microscopy, culture and sensitivity of pus, TLC
3. Antibiotics, surgical drainage, oral hygiene is improved, sialogogues are prescribed.
--------------------------------------------------------------------------------------------------------------------------

CHRONIC RECURRENT SIALADENITIS


Definition:
Recurrent salivary gland enlargement associated with pain and tenderness is known as
CONCEPT BOOK OF DISEASES OF EAR, NOSE & THROAT BY FAHIM AWAN

370 Part D Oral Cavity and Pharynx

chronic recurrent sialadenitis.

Etiopathology:
It is usually due to infection with Staphylococci and Streptococci.

Clinical features:
During acute episode, salivary gland (parotid or submandibular) is enlarged and tender. Pus
can be expressed from the duct of the gland. Between the acute episode, gland is firm and
slightly enlarged. Sialography shows normal duct system.

Treatment:
Acute episode is treated similar to acute bacterial sialadenitis. Between the episodes, patient
is advised to observe good oral hygiene. Sialogogues are prescribed to promote salivation.
SELF ASSESSMENT
Scenario: A 25 years old male presented to ENT OPD with the complaint of recurrent
swelling of parotid gland for last 06 months. On examination, pus is coming from duct on
pressing the gland and gland is soft to firm in consistency. Sialography shows normal duct
system.
1. What is your most probable diagnosis?
2. What is your treatment plan?
--------------------------------------------------------ANSWERS-----------------------------------------------------
Scenario: 1. Chronic recurrent sialadenitis 2. Acute episode is treated similar to acute
bacterial sialadenitis. Between the episodes, patient is advised to observe good oral hygiene,
sialogogues to promote salivation.
--------------------------------------------------------------------------------------------------------------------------
SIALECTASIS
Definition:
It is defined as dilatation of the duct system of salivary gland leading to stasis of secretions.

Etiopathology:
It is a chronic condition of major salivary gland. Sialectasis is comparable to bronchiectasis
where there is disintegration and necrosis. It results in formation of cellular debris which
causes obstruction, stenosis and dilatation of ducts along with stasis and cyst formation. This
stasis predisposes to infection. There may also be formation of salivary calculi.
Clinically sialectasis resembles chronic recurrent sialadenitis but it can be differentiated
from it by sialography which may indicate dilatation of the duct system. Causes of
sialectasis may be;
A. Granulomatous diseases
CONCEPT BOOK OF DISEASES OF EAR, NOSE & THROAT BY FAHIM AWAN
371 Part D Oral Cavity and Pharynx

B. Sjogren’s syndrome and other autoimmune diseases


C. Congenital
A. Granulomatous diseases of salivary glands:
Tuberculosis, sarcoidosis and actinomycosis may affect the
salivary glands. Investigations and treatment are done on general
principles of the disease which affects the salivary gland.
1. In tuberculous infection of the gland, surgical drainage and
debridement of the tissue along with antituberculous
treatment is given.
2. In sarcoidosis, enlargement of parotid gland, uveitis and fever
is known as Heerfordt’s syndrome or uveoparotid fever.
Uveoparotid fever
Treatment is with systemic steroids.
3. In actinomycosis, there may be abscess in the salivary gland with multiple sulfur granules
from discharging sinuses. Treatment is surgical drainage/debridement and large doses of
penicillins or tetracyclins.
B. Sjogren’s syndrome
Synonyms: Sicca syndrome

Etiopathology:
It is an autoimmune disease involving exocrine glands of body. The disease is found in all age
groups but mostly affects between the ages of 40 and 60 years. Male to female ratio is 1:9.
It is classified as:
1. Primary Sjogren’s syndrome (also known as sicca complex or Mikulikz disease); It
consists only of xerostomia and xerophthalmia.
2. Secondary Sjogren’s syndrome; It consists of xerostomia,
xerophthalmia and a connective tissue disorder usually rheu-
matoid arthritis.
Risk of developing non-Hodgkin’s lymphoma in Sjogren’s
syndrome is 1 in 6. Clinical features of
Clinical features: Sjogren’s syndrome

It is a multisystem disorder which primarily affects oral cavity, the eyes and the salivary
glands.
1. Oral symptoms include dry mouth leading to stomatitis, glossitis, oral candidiasis and
subsequent dental caries.
2. Eye symptoms are keratoconjunctivitis sicca; foreign body sensation in the eye, burning,
redness, photosensitivity and inability to tolerate contact lenses.
CONCEPT BOOK OF DISEASES OF EAR, NOSE & THROAT BY FAHIM AWAN
372 Part D Oral Cavity and Pharynx

3. Salivary gland enlargement is seen in 40% of cases.


Investigations:
Diagnosis is usually made on clinical grounds.
1. Autoantibodies to Sjogren’s syndrome i.e., SSA and SSB are positive.
2. Biopsy from the back of lower lip shows periductal lymphocytic infiltration.
3. Raised ESR
4. Positive rheumatoid factor and positive antinuclear antibodies help to exclude
associated rheumatoid arthritis or SLE.
Treatment:
Treatment is generally, symptomatic.
1. Steroids and other immunosuppressants like cyclophosphamide are given to treat
episodes of parotid swelling.
2. Artificial tears are used for eye symptoms.
3. Synthetic saliva and orally administered pilocarpine are used for oral symptoms.
4. Patient should have life long follow up for lymphoma.
SELF ASSESSMENT
Q. 1. Salivary gland swelling associated with sarcoidosis, uveitis and fever is known as
a. Heerfordt syndrome d. Sicca syndrome

b. Mikulicz syndrome e. Usher’s syndrome


c. Sjogren’s syndrome
Q. 2. All are true about primary Sjogren’s syndrome except
a. autoimmune disease d. associated with connective tissue
b. also known as Mikulicz disease disorder
c. also known as sicca complex e. characterized by xerophthalmia &
xerostomia
Q. 3. Sjogren’s syndrome is usually characterized by all except
a. bilateral parotid enlargement d. dryness of eye

b. dental caries e. facial paralysis


c. dryness of mouth
Q. 4. All of the following are true about Sjogren’s syndrome except
a. biopsy of lower lip shows periductal c. increased ESR
lymphocytic infiltration d. positive SSA
b. decreased ESR e. positive SSB

CONCEPT BOOK OF DISEASES OF EAR, NOSE & THROAT BY FAHIM AWAN


373 Part D Oral Cavity and Pharynx
Q. 5. All are true about treatment of Sjogren’s syndrome except
a. eye symptoms treated with artificial c. treated with immunosuppressants
tears d. treated with radiotherapy
b. mouth symptoms are treated synthetic e. treated with steroids
saliva
Scenario: A 50 years old female presents in ENT OPD with complaints of dry mouth, dry
eyes and rheumatoid arthritis for last three years.
1. What is the most probable clinical diagnosis?
2. What investigation would you like to confirm the diagnosis?
3. How will you treat this patient?
-------------------------------------------------------- ANSWERS-----------------------------------------------------
Q.1. a 2. d 3. e 4. b 5. d
Scenario: 1. Secondary Sjogren’s syndrome 2. Autoantibodies SSA, SSB, biopsy, ESR,
rheumatoid factor 3. Steroids, immunosuppressants, artificial tears, synthetic saliva.
--------------------------------------------------------------------------------------------------------------------------
SIALOLITHIASIS
Synonym: Salivary calculi
Definition:
It is defined as formation of stones in the parenchyma or duct system of the major salivary
gland.
Etiopathology:
Submandibular salivary gland is a mixed seromucinous gland and has secretions high in
calcium. The secretions of submandibular salivary gland are, therefore, calcified easily. So
90% of the stones are formed in the submandibular salivary gland. These stones are of high
density and are, therefore, radiopaque.
Parotid is a serous gland and has secretions low in calcium. The epithelial debris
remains as mud and usually does not form stone. 10% of stones are formed in the parotid
gland. These are of low density and radiolucent.
If main duct is blocked, then the whole gland swells up in response to secretomotor
stimulation of eating and drinking especially with citrus drinks.
Clinical features:
Symptoms are produced only when the ducts are blocked.
1. Patient presents with swelling and pain in the submandibular
region during meal times which usually subside after some time.
A case of sialoli-
2. On examination stone may be palpable by bidigital palpation of the thiasis

CONCEPT BOOK OF DISEASES OF EAR, NOSE & THROAT BY FAHIM AWAN


374 Part D Oral Cavity and Pharynx

duct or gland.
3. Stone may be visible in the submandibular duct opening occasionally.
Investigations:
1. Plain X-ray. Eighty percent of the salivary stones are radiopaque;
hence, visible on plain X-ray.
2. Sialography and ultrasonography may be required for radiolucent
stones.
Plain X-ray showing
Treatment: sialolithiasis
1. Stones in the peripheral duct system are removed intraorally by
incision and marsupialization of the duct.
2. Stones in the proximal duct and parenchyma require excision of the affected gland.
3. Recently sialendoscopic lithotripsy is being used.
Differential diagnosis:
1. Sialadenitis
2. Sialactasis
3. Tumor of salivary gland.
SELF ASSESSMENT
Q. 1. The percentage of salivary calculi seen in submandibular gland is
a. 10% d. 70%

b. 30% e. 90%
c. 50%
Q. 2. The percentage of radiolucent calculi in the salivary glands is
a. 10% d. 40%

b. 20% e. 50%
c. 30%
Q. 3. All are usual clinical features of sialolithiasis except
a. submandibular duct stone may be visible d. submandibular swelling subsides after
some time
b. submandibular gland swells up during e. 80% of the submandibular stones are
eating and drinking not visible on plain X-ray
c. submandibular stone may be palpable by
bidigitally

Q. 4. Investigation of choice in sialolithiasis is


a. angiography c. Plain X-ray
b. biopsy d. sialography

CONCEPT BOOK OF DISEASES OF EAR, NOSE & THROAT BY FAHIM AWAN


375 Part D Oral Cavity and Pharynx
e. SSA and SSB
Q. 5. All are true about treatment of sialolithiasis except
a. parenchymal stone should be treated by removed by intraoral incision
excision of submandibular gland d. submandibular duct should be marsu-
b. submandibular duct stone should be pialized
removed by external incision e. sialendoscopic lithotripsy
c. submandibular duct stone should be
Scenario: A 35 years old man presents in ENT OPD with complaint of recurrent left sided
painful swelling just below the mandible for last three months. According to patient the
swelling increases on chewing food and decreases after some time. On examination, the
swelling is 02 X 02 cm in size and is tender and hard on palpation.
1. What is the most probable clinical diagnosis?
2. What investigation would you like to confirm the diagnosis?
3. How will you treat this patient?
ANSWERS-----------------------------------------------
-------------------------------------------------------- ------
Q.1. e 2. b 3. e 4. c 5. b
Scenario: 1. Sialolithiasis 2. Plain X-ray, sialography and ultrasonography 3. A. In duct;
Incision and marsupialization of duct B. In parenchyma; Excision of gland, sialendoscopic
lithotripsy.
--------------------------------------------------------------------------------------------------------------------------
Complications of submandibular gland excision:
There may be damage to;
1. Marginal mandibular branch of facial nerve. It leads to paralysis of depressor anguli
oris.
2. Hypoglossal nerve. It leads to deviation of tongue towards the affected side.
3. Lingual nerve. It leads to loss of taste on anterior two third of the ipsilateral side of
tongue.
Complications of parotidectomy:
1. Haematoma formation
2. Facial paralysis
3. Salivary fistula
4. Frey’s syndrome (gustatory sweating). The complaint is of sweating in the preauricular
and subparotid region during meal times. It is due to misrouting of parasympathetic se-
cretomotor fibers into cutaneous nerves during the healing phase following
parotidectomy.
SELF ASSESSMENT

CONCEPT BOOK OF DISEASES OF EAR, NOSE & THROAT BY FAHIM AWAN


376 Part D Oral Cavity and Pharynx
Q. 1. All are usual complications of submandibular gland excision except
a. damage to glossopharyngeal nerve d. damage to marginal mandibular

branch of facial nerve


b. damage to hypoglossal nerve

c. damage to lingual nerve e. paralysis of tongue muscles

Q. 2. All are usual complications of parotidectomy except


a. facial nerve paresis/paralysis d. hypoglossal nerve paralysis

b. gustatory sweating (Frey’s syndrome) e. salivary fistula


c. hematoma formation
Q. 3. All of the following statements are correct except
a. damage to marginal mandibular branch lateral side of tongue
of facial nerve leads to paralysis of d. Frey’s syndrome is due to misrouting of
depressor anguli oris parasympathetic secretomotor fibers into
b. damage to hypoglossal nerve leads to cutaneous nerves during the healing phase
deviation of tongue towards the contra- e. gustatory sweating is sweating in the
lateral side preauricular and subparotid region
c. damage to lingual nerve leads to loss of during meal times
taste on anterior two third of the ipsi-
--------------------------------------------------------ANSWERS-----------------------------------------------------
Q.1. a 2. d 3. c
--------------------------------------------------------------------------------------------------------------------------
SALIVARY GLAND TUMORS
Remember rule of 80 %.
Eighty per cent of the salivary gland tumors occur in the major salivary glands. 80 % of the
major salivary gland tumors occur in the parotid gland. 80 % of the major salivary gland
tumors are benign in nature. 80 % of the parotid gland tumors are benign in nature. 80% of
the benign tumors of parotid gland are pleomorphic adenoma. 80 % of the minor salivary
gland tumors are malignant in nature.
Tumors of salivary glands are divided into benign and malignant tumors.
A. Benign tumors
a. Pleomorphic adenoma (mixed cell tumor):
It is the most common benign tumor of salivary glands. It
comprises of both epithelial and myoepithelial elements. That
is why it is also known as mixed cell tumor. It has a
pseudocapsule which is incomplete. It has got bosselations on
its surface. It is soft to firm in consistency. The tumor usually
grows slowly. Patient usually presents with slowly growing
swelling. Pain
Pleomorphic adenoma of left
377
and paralysis may indicate malignant change incidence of which is 6% (then it is known as
carcinoma ex pleomorphic adenoma).
Treatment is excision with a healthy margin. Recurrence is very common.
b. Monomorphic adenoma:
1. (Warthin’s tumor):
It occurs primarily in middle-aged or elderly men. It nearly always occurs in the tail of
parotid gland. It is susceptible to inflammation associated with upper respiratory tract
infection. Therefore, patient usually presents with intermittent swelling and pain.

Treatment is excision with a healthy margin. Recurrence is unusual.


2. Oxyphil adenoma (oncocytoma):
It is a rare benign tumor of eosinophilic cells. It usually occurs in the superficial lobe of
parotid gland. It rarely undergoes malignant change. Treatment is by surgical excision.
SELF ASSESSMENT
Q. 1. The most common benign tumor of salivary gland is
a. adenolymphoma d. Warthin’s tumor

b. oxyphil adenoma e. none of the above


c. pleomorphic adenoma
Q. 2. Which statement is not true about pleomorphic adenoma?
a. Benign tumor c. Grows slowly
b. Comprises of epithelial and myoepi- d. Non recurrent
thelial elements e. Soft to firm in consistency
Q. 3. Carcinoma arising from pleomorphic adenoma (mixed cell tumor) is known as
a. adenocarcinoma d. squamous cell carcinoma

b. carcinosarcoma e. none of the above


c. carcinoma ex pleomorphic adenoma
Q. 4. All are true about adenolymphoma except
a. also known as Warthin’s tumor d. malignant tumor
b. arises from tail of parotid gland e. primarily occurs in middle-aged or
c. increases in size during upper respi- elderly men
ratory tract infection
Scenario: A 28 years old female reported in ENT OPD with swelling in left parotid region
for last 04 years. which is painless and slowly increasing in size. On examination, it is soft to
firm in consistency, not adherent to underlying structures and not involving the facial nerve.
Biopsy shows both epithelial and myoepithelial elements.
378
1. What is the most probable clinical diagnosis?
2. What investigations will you carry out?
3. How will you treat this patient?
ANSWERS-----------------------------------------------
-------------------------------------------------------- ------
Q.1. c 2. d 3. c 4.d
Scenario: 1. Pleomorphic adenoma of parotid 2. FNAC 3. Excision with healthy margins.
--------------------------------------------------------------------------------------------------------------------------

B. Malignant tumors
a. Carcinomas:
1. Adenoid cystic carcinoma (cylindroma):
It is the most common malignant tumor of salivary glands especially in adults. It is more
common in minor than in major salivary gland. It grows slowly and has a characteristic
tendency for perineural infiltration. The most common presenting feature is pain and
swelling. There may also be complaint of paralysis of the adjacent nerves e.g., facial
paralysis in parotid gland tumor. Incidence of lymph node involvement is 20%.
Treatment is widest possible excision followed by radiotherapy.
2. Adenocarcinoma:
Adenocarcinoma is relatively uncommon tumor. Presentation and management are similar to
that of adenoid cystic carcinoma.
3. Carcinoma ex pleomorphic adenoma:
Rarely carcinoma may arise from preexisting pleomorphic adenoma. This is called
carcinoma ex pleomorphic adenoma.
b. Mucoepidermoid tumor:
It is the most common salivary malignancy in childhood but the second most common
salivary malignancy over all. It grows slowly and recurs locally. Treatment is surgical
resection followed by radiotherapy.
Investigations of salivary gland tumors:
1. FNAC (Fine Needle Aspiration Cytology)
2. Frozen section histopathology
3. CT/MRI are the mainstays of investigation for salivary gland tumors.
SELF ASSESSMENT
Q. 1. The most common malignant tumor of salivary gland in adults is
a. acinic cell carcinoma b. adenoid cystic carcinoma
379 Part D Oral Cavity and Pharynx

c. adenocarcinoma e. squamous cell carcinoma


d. mucoepidermoid carcinoma
Q. 2. The most common malignant salivary gland tumor in children is
a. acinic cell tumor d. pleomorphic adenoma

b. adenoid cystic carcinoma e. Warthin’s tumor


c. mucoepidermoid tumor
Q. 3. The salivary gland tumor having a marked tendency to invade nerves with high
frequency of pain and paralysis is
a. adenocarcinoma d. pleomorphic adenoma

b. adenoid cystic carcinoma e. squamous cell carcinoma


c. carcinoma ex-pleomorphic adenoma
Q. 4. All are true about adenoid cystic carcinoma except
a. presents with pain d. The most common malignant salivary
b. presents with paralysis gland tumor in adults
c. presents with swelling e. treated by simple excision
Q. 5. All are investigations of salivary gland tumor except
a. CT d. MRI

b. fine needle aspiration cytology e. sialography

c. frozen section histopathology


-------------------------------------------------------- ANSWERS-----------------------------------------------------
Q.1. b 2. c 3. b 4. e 5. e
--------------------------------------------------------------------------------------------------------------------------

XEROSTOMIA
Synonym: Dry mouth syndrome

Definition:
It is an excessive dryness of mouth.

Etiopathology:
It is caused by;
1. Drugs e.g., antihistamines, anticholinergics, tricyclic
antidepressants, diuretics and benzodiazepines.
A case of xerostomia
2. Mouth breathing due to nasal obstruction.
3. Certain general conditions e.g., diabetes insipidus, uremia and Sjogren’s syndrome.
380 Part D Oral Cavity and Pharynx

4. Lesions which interrupt pathways of the secretory nerves.


5. Psychological depression.
6. Radiotherapy.
Treatment:
It is directed towards removal of the underlying cause.
SELF ASSESSMENT
Q. 1. Xerostomia is also called
a. drooling d. ptyalism

b. dry mouth syndrome e. sialorrhea


c. hypersialia
Q. 2. Xerostomia can be caused by all of the following except
a. antihistamines d. Sjogren’s syndrome

b. cerebral palsy e. uremia

c. mouth breathing
------------------------------------------------
-------- ANSWERS-----------------------------------------------------
Q.1. b 2. b
--------------------------------------------------------------------------------------------------------------------------
PTYALISM
Synonyms: Sialorrhea, Drooling, Hypersalivation, Hypersialia

Definition: It is an increased flow of saliva. Average amount


of saliva produced is 1500 ml/day.
Etiopathology:
It may be caused by;
1. Inflammations or tumors of the mouth, pharynx or
esophagus.
2. Ill fitting dentures
3. Parkinsonism
4. Cerebral palsy
A case of ptyalism
5. Stroke
6. Action of certain drugs like reserpine, pilocarpine or iodides.
7. Heavy metal poisoning like mercury, arsenic, lead etc.
Treatment:
It consists of;
CONCEPT BOOK OF DISEASES OF EAR, NOSE & THROAT BY FAHIM AWAN
381 Part D Oral Cavity and Pharynx

1. Removal of the underlying cause.


2. Mouth wash and tooth brushing is advised.
3. Anticholinergic drugs like glycopyrolate are used.
4. Botulinum toxin, radiotherapy or surgery to denervate the gland is carried out.

5. Starch intake is reduced.


SELF ASSESSMENT
Q. 1. Ptyalism is also called

a. drooling d. sialorrhea

b. hypersalivation e. all of the above

c. hypersialia

Q. 2. Average amount of saliva produced per day is


a. 750 ml d. 2000 ml

b. 1000 ml e. 2500 ml
c. 1500 ml
Q. 3. Excessive salivation occurs in all except
a. inflammations of pharynx d. peritonsillar abscess

b. mouth breathing e. poor oro-dental hygiene


c. parkinsonism
Q. 4. All of the following cause excessive salivation except
a. diabetes insipidus d. peritonsillitis

b. ill fitting dentures e. ulcers in oral cavity

c. iodides taken orally


Scenario: A 25 years old female having history of excessive salivation of about 04 liters/day.
1. What are usual causative factors of excessive salivation?
2. What are different drugs causing excessive salivation?
ANSWERS--------------------------------------------------
-------------------------------------------------------- ---
Q.1. e 2. c 3. b 4.a
Scenario: 1. Inflammations or tumors of the mouth, pharynx or esophagus, ill fitting denture,
parkinsonism, cerebral palsy, stroke 2. Reserpine, pilocarpine or iodides.
--------------------------------------------------------------------------------------------------------------------------
383 Part E

PART E
LARYNX
Chapter 13
DISEASES OF LARYNX
CONGENITAL ANOMALIES OF LARYNX
LARYNGOMALACIA
Definition:
It is a congenital malformation of the larynx associated with excessive softening of skeletal
framework of larynx.
Etiopathology:
It is the most common congenital malformation of the larynx. There is an abnormal
flaccidity of the laryngeal cartilages. This allows the laryngeal structures to vibrate like a
small elongated reed. It is characterized by partial or complete collapse of supraglottic
structures on inspiration. The condition becomes more severe during first 09 months of life
and then gradually resolves. It generally, disappears by the age of 02 years.
Clinical features:
1. It is the most common congenital condition causing inspiratory
stridor mainly. The stridor is usually worse during exertion
such as crying or lying supine position. It is diminished by rest
and during sleep.
2. Voice is normal.
Endoscopic view
3. Cyanosis may occur occasionally or shortly after birth. of laryngomalacia
Investigations:
History and examination are usually diagnostic.
Inspiratory stridor without hoarseness is always suggestive of laryngomalacia when
occurring at or soon after birth.
Flexible fiber optic/direct laryngoscopy shows;
1. Exaggerated infantile type of larynx.
CONCEPT BOOK OF DISEASES OF EAR, NOSE & THROAT BY FAHIM AWAN
384 Part E

2. The epiglottis is long and narrow and folded backwards at each lateral edge so the
epiglottis becomes omega shaped.
3. There are short aryepiglottic folds and redundant mucosa over the arytenoid cartilages.
The edges of the laryngeal inlet are seen to be drawn in with each inspiration.
4. Vocal cords are absolutely normal in laryngomalacia.
Treatment:
1. Reassurance is required in majority of cases. Laryngomalacia usually resolves
spontaneously. Resolution is usually complete by the age of 02 years.
2. In very severe cases, endoscopic aryepiglottoplasty (also called supraglottoplasty) is
carried out. It may be done microsurgically or by carbon dioxide laser using an
operating microscope.
3. In extremely severe cases, tracheostomy is rarely required to relieve the airway
obstruction.
Differential diagnosis:
1. Laryngeal web
2. Laryngeal cyst
3. Subglottic hemangioma
4. Acute laryngitis
SELF ASSESSMENT
Q. 1. The most common congenital anomaly of larynx is

a. cleft larynx d. subglottic haemangioma

b. laryngeal web e. subglottic stenosis


c. laryngomalacia
Q. 2. The most common cause of stridor in newborn child is
a. diphtheria d. tracheobronchial agenesis

b. intubation trauma e. vocal cord paralysis


c. laryngomalacia
Q. 3. The stridor in laryngomalacia usually disappears in most cases after
a. 02 to 03 months d. 18 to 24 months

b. 03 to 06 months e. 36 to 48 months
c. 06 to 12 months
Q. 4. All are usual clinical features of laryngomalacia except
c. stridor getting worse on crying or
a. hoarse cry exertion
b. onset of stridor soon after birth d. stridor which is mainly inspiratory

CONCEPT BOOK OF DISEASES OF EAR, NOSE & THROAT BY FAHIM AWAN


385 Part E

e. tendency to disappear by 02 years of


Age
Q. 5. An infant after birth is noticed to have stridor which increases on crying. The most
probable diagnosis is
a. congenital laryngeal cyst d. laryngocele

b. laryngeal paralysis e. laryngomalacia


c. laryngeal web
Q. 6. Omega shape epiglottis is found in
a. acute epiglottitis d. syphilis of larynx

b. laryngomalacia e. tuberculous laryngitis


c. sarcoidosis of larynx
Q. 7. Which structure of larynx is absolutely normal in laryngomalacia?
a. Aryepiglottic folds d. All of the above

b. Epiglottis e. None of the above


c. Vocal cords
Q. 8. All are included in the differential diagnosis of laryngomalacia except
a. acute laryngitis d. laryngeal web

b. bilateral adductor paralysis


e. subglottic hemangioma
c. laryngeal cyst
Q. 9. All are included in the treatment of laryngomalacia except
a. CO2 laser d. reassurance

b. cricothyroidotomy e. to wait and observe

c. endoscopic aryepiglottoplasty
Scenario A: A one month old baby was brought to ENT consultant with history of noisy,
difficult breathing and occasional cyanosis when the baby is exerting or crying. The
symptoms are present since birth. At rest baby usually remains normal.
1. What is your probable clinical diagnosis?
2. How will you confirm your diagnosis?
3. How will you treat the baby?
B: A newborn baby was brought to the outpatient department with the complaint of stridor
which is present since birth. The stridor tends to increase during supine position. Breast
feeding of the baby is without any problem or respiratory difficulty.
1. What is the most likely diagnosis?
2. What are two other diseases which can be included in differential diagnosis?
3. How will you manage this case?

CONCEPT BOOK OF DISEASES OF EAR, NOSE & THROAT BY FAHIM AWAN


386 Part E
ANSWERS--------------------------------------------
-------------------------------------------------------- ---------
Q.1. c 2. c 3. d 4. a 5. e
6. b 7. c 8. b 9. b
Scenario A: 1. Laryngomalacia 2. Flexible fiberoptic/direct laryngoscopy 3. Reassurance,
endoscopic aryepiglottoplasty which may be done microsurgically or by carbon dioxide
laser.
B: 1. Laryngomalacia 2. Laryngeal web, laryngeal cyst 3. Reassurance, endoscopic
aryepiglottoplasty which may be done microsurgically or by carbon dioxide laser.

--------------------------------------------------------------------------------------------------------------------------
LARYNGEAL WEB
Etiopathology:
The web is due to incomplete recanalization of larynx during intrauterine life. It consists of
fibrous tissue stroma covered with epithelium in the anterior half of glottis . Occasionally
supraglottis may also be involved.
Clinical features:
Symptoms vary with the size of the web.
1. Hoarseness is the most common symptom. There may be
associated weak cry and dyspnea on exertion.
2. Stridor occurs in severe cases.

Investigations: Endoscopic view of laryngeal web

The web can be seen on flexible nasopharyngoscopy, 70-degree endoscopy or direct


laryngoscopy.
Treatment:
In milder cases no treatment is required.
1. CO2 laser ablation of the web under general anesthesia is the best treatment option.
2. Excision of the web and McNaught keel may be applied through laryngofissure
(external approach).
3. When the patient has severe symptoms like dyspnea and stridor, tracheostomy may be
performed as an emergency procedure to relieve the airway obstruction.

Differential diagnosis:
1. Laryngomalacia
2. Laryngeal cyst
3. Subglottic hemangioma
CONCEPT BOOK OF DISEASES OF EAR, NOSE & THROAT BY FAHIM AWAN
387 Part E

4. Acute laryngitis

SELF ASSESSMENT
Q. 1. The region most commonly affected in laryngeal web is
a. glottis d. trachea

b. subglottis e. all of the above


c. supraglottis
Q. 2. The most common site of laryngeal web is
a. anterior half of glottis d. Subglottis

b. laryngopharynx e. Supraglottis
c. posterior half of glottis
Q. 3. All are usual clinical features of laryngeal web except
a. dyspnea d. stridor

b. hoarseness of voice e. weak cry


c. normal voice
Q. 4. All are included in the treatment of laryngeal web except
d. Nd: YAG laser
a. CO2 laser
b. laryngofissure e. Tracheostomy

c. McNaught keel
Q. 5. All are included in the differential diagnosis of laryngeal web except
a. acute laryngitis d. laryngeal web

b. bilateral adductor paralysis e. subglottic hemangioma

c. laryngeal cyst
Scenario: A mother comes to outpatient clinic with her 01-year-old baby. She complains of
hoarseness of voice, dyspnea and abnormal cry since birth. There is no other complaint.
1. What is the most likely diagnosis?
2. How can we confirm the diagnosis?
3. What is the treatment plan?
-------------------------------------------------------- ANSWERS-----------------------------------------------------
Q.1. a 2. a 3. c 4. d 5. b
Scenario: 1. Laryngeal web 2. Flexible nasopharyngoscopy, 70-degree endoscopy or direct
laryngoscopy 3. CO2 laser ablation, excision, tracheostomy if airway obstruction.
--------------------------------------------------------------------------------------------------------------------------
CONCEPT BOOK OF DISEASES OF EAR, NOSE & THROAT BY FAHIM AWAN
388 Part E

INFLAMMATIONS OF LARYNX
ACUTE LARYNGITIS
Definition:
Acute inflammation of mucosa of larynx is known as acute
laryngitis.
Acute laryngitis on left side
Etiopathology: Normal larynx on right side
Acute laryngitis may be infective or noninfective.
1. Infective laryngitis: It is the most common form of
laryngitis. Majority of infections are caused by viruses.
The viruses usually found are rhinovirus, adenovirus, parainfluenza virus and respiratory
syncytial virus. Soon bacterial infection takes place with Streptococcus pneumoniae,
Haemophilus influenzae and Hemolytic streptococci.
2. Non infective laryngitis: It is due to vocal abuse, allergy, thermal or chemical burns to
the larynx. It may also be due to inhalation or ingestion of various substances or laryngeal
trauma such as endotracheal intubation.

Clinical features:
Symptoms are usually sudden in onset and consist of;
1. Hoarseness which may lead to complete loss of voice.
2. Discomfort or pain in throat, particularly after talking.
3. Dry, irritating cough which is usually worse at night.
4. General symptoms of rawness or dryness of throat, malaise and fever.
Laryngeal appearance varies with severity of the disease.
In early stages, there is erythema and edema of epiglottis, aryepiglottic folds, arytenoids
and ventricular bands. But the vocal cords appear white and near normal and stand out in
contrast to surrounding mucosa.
In later stages, there is hyperaemia and increase in swelling. Vocal cords also become red
and swollen. Subglottic region also gets involved. Sticky secretions are seen between the
cords and interarytenoid region. In case of vocal abuse, submucosal hemorrhages may be
seen in the vocal cords.
Treatment:

CONCEPT BOOK OF DISEASES OF EAR, NOSE & THROAT BY FAHIM AWAN


389 Part E

1. Vocal rest is the most important single step in the treatment of acute laryngitis. Use of
voice during acute laryngitis may lead to incomplete or delayed recovery.
2. Avoidance of smoking and alcohol.
3. Steam inhalation is given with tincture benzoin co. It has soothing effect and helps in
loosening viscid secretions.
4. Cough suppressant suppresses irritating cough.
5. Analgesics are given to relieve local pain and discomfort.
6. Antibiotics are given when there is secondary infection with fever and toxaemia or
purulent expectoration.
7. Steroids are useful in laryngitis following thermal or chemical burns.
Acute membranous laryngitis:
This condition is similar to acute membranous tonsillitis and is caused by pyogenic
nonspecific organisms. It may begin in the larynx or may be an extension from pharynx. It
should be differentiated from laryngeal diphtheria.
SELF ASSESSMENT
Scenario: A 35 years old male reported to ENT OPD with the hoarseness of voice after upper
respiratory tract infection 10 days ago. Hoarseness led to complete loss of voice and
discomfort and pain in throat also. Patient also complains of dry, irritating cough usually at
night. Indirect laryngoscopic examination shows vocal cords are red and swollen. Sticky
secretions are also seen in between cords and interarytenoid region.
1. What is your clinical diagnosis?
2. What is your treatment plan?
--------------------------------------------------------ANSWERS-----------------------------------------------------
Scenario: 1. Acute laryngitis 2. Voice rest, steam inhalation, cough suppressant, analgesics.
--------------------------------------------------------------------------------------------------------------------------
ACUTE EPIGLOTTITIS
Synonyms: Supraglottitis, Supraglottic laryngitis
Definition:
It is defined as an acute infection primarily of the epiglottis.

Etiopathology:
It may involve other sites such as aryepiglottic folds and false cords.
It is a potentially life-threatening condition. It usually affects children between 04 to 05
years of age but adults may also be affected. In children it progresses very rapidly and may
compromise the airway.

CONCEPT BOOK OF DISEASES OF EAR, NOSE & THROAT BY FAHIM AWAN


390 Part E

The most common pathogen is Haemophilus influenza type B. Less common organisms
are group A Streptococcus pneumoniae and Staphylococcus aureus. Rarely viruses are
implicated particularly in adults.

Clinical features:
Usual clinical features are;
1. Short history of acute painful sore throat, odynophagia and
dysphagia.
2. High-grade fever developing within few hours is a very common
presenting symptom.
Endoscopic view of
3. Hoarseness of voice and dyspnea may be the presenting symptom epiglottitis
occasionally.
On examination;
1. Tripod sign i.e., patient is sitting and bending forward as dyspnea is improved in this
position.
2. Drooling sign i.e., there is dribbling of saliva from the angle of mouth because there is
increased production of saliva secondary to inflammation and patient is unable to
swallow the saliva due to odynophagia.
3. There may be dyspnea or even stridor especially on lying supine as the inflamed
epiglottis shuts down the glottis in this position.
Local examination of a patient with epiglottitis should be avoided as simple examination with
tongue depressor may lead to life-threatening glottic spasm.
On examination;
1. Epiglottis is swollen and gives ‘cherry red appearance’.
2. There may be edema of palatine arches and uvula.
3. Palpation may show extremely tender larynx and tender cervical lymphadenopathy.
Complications:
Acute epiglottitis may give rise to;
1. Acute airway obstruction and death due to spread of edema to glottis.
2. Epiglottic abscess.
3. Thrombosis of internal jugular vein (Lemierre’s syndrome).
Investigations:
1. X-ray soft tissue neck lateral view shows “thumbprint sign”.
2. Flexible nasopharyngoscopy or 70-degree endoscopy is very
useful for diagnosis.
X-ray soft tissue neck
CONCEPT BOOK OF DISEASES OF EAR, NOSE & THROAT BY FAHIM AWAN
shows thumbprint sign
391 Part E

3. CT must be carried out in cases of suspected complications like epiglottic abscess.


4. White cell count may be significantly raised.
5. Blood cultures and throat swabs should also be taken.
Treatment:
1. Admission in hospital and observation for airway obstruction. Arrangements must be
made for endotracheal intubation or tracheostomy.
2. 100% humidified oxygen must be given.
3. I/V ampicillin is the drug of choice.
4. Adrenaline nebulization with 1:1000 at a dose of 0.5ml/Kg. It provides quick but
temporary relief of airway obstruction.
5. Steroids may be given to reduce laryngeal edema.
6. Adequate hydration of patient is maintained.
Prevention:
There is marked decrease in pediatric epiglottitis since the introduction of Haemophilus
influenza type B i.e., HiB vaccine. So, vaccination schedule should be strictly followed.
Differential diagnosis:
1. Acute retropharyngeal abscess

2. Laryngotracheobronchitis
3. Foreign body inhalation
4. Angioedema
SELF ASSESSMENT
Q. 1. Acute epiglottitis is also called

a. croup d. Both a & b

b. supraglottitis e. Both b & c


c. supraglottic laryngitis
Q. 2. Acute epiglottitis in children is mostly caused by
a. Haemophilus influenza type B d. Staphylococcus aureus

b. Parainfluenza type I and II e. Streptococcus pneumoniae


c. Respiratory syncytial virus
Q. 3. The most dangerous paediatric emergency due to respiratory infection is
a. acute epiglottitis c. chronic laryngitis

b. acute laryngitis
CONCEPT BOOK OF DISEASES OF EAR, NOSE & THROAT BY FAHIM AWAN
392 Part E

d. laryngotracheobronchitis e. subglottic laryngitis


Q. 4. All are usual clinical features of acute epiglottitis except
a. acute fever d. bipod sign

b. acute odynophagia e. drooling sign


c. acute painful sore throat
Q. 5. All are usual clinical features of acute epiglottitis except
a. cherry red epiglottis d. safe condition with no risk to life

b. hoarseness of voice e. tripod sign


c. life-threatening condition
Q. 6. Classical “thumbprint sign” on X-ray lateral view of neck is found in
a. acute epiglottitis d. quinsy

b. acute laryngitis e. retropharyngeal abscess

c. parapharyngeal abscess
Q. 7. Treatment of acute epiglottitis is

a. admission in hospital d. 100% oxygen inhalation

b. antibiotics e. all of the above


c. observation for airway obstruction
Q. 8. In a child with suspected diagnosis of epiglottitis, your first line of treatment would
be
a. to order a laryngogram d. to secure the airway by intubation
b. to order an X-ray of soft tissue neck lat- e. to take a throat swab and blood
eral view to establish the diagnosis culture and start intravenous fluid
c. perform a laryngoscopy
Q. 9. Antibiotic of choice for acute epiglottitis is
a. aminoglycosides d. chloramphenicol

b. ampicillin e. tetracycline

c. cephalosporin
Scenario A: A 05 years old fit boy started having severe sore throat which developed into
odynophagia quickly. Inspiratory stridor with drooling of saliva followed soon. The child
looks terrified and feels comfortable only in sitting and leaning forward position.
1. What is the most probable clinical diagnosis?
2. How will you secure the airway in this patient?
3. Name three other conditions which can easily mimic this clinical picture?
B: A 05 years old boy was perfectly alright in the morning before going to school but on
returning home on afternoon he had high-grade fever, unable to eat because of severe pain

CONCEPT BOOK OF DISEASES OF EAR, NOSE & THROAT BY FAHIM AWAN


393 Part E
in throat.
1. What is the most probable clinical diagnosis?
2. What is the differential diagnosis?
3. What are other clinical features of this entity?
C: A 04 years old child was brought to emergency room with severe stridor for last 02 hours.
On examination he had fever, looked very ill and was cyanotic. He was comfortable only in
sitting position. His body temperature is 102-degree F.
1. What is the most likely diagnosis?
2. Which radiological investigation will you order and what will be positive finding on it?
3. Which organism is responsible for this condition?
4. How will you treat this patient?
ANSWERS-----------------------------------------------
-------------------------------------------------------- ------
Q.1. e 2. a 3. a 4. d 5. d
6. a 7. e 8. d 9. b
Scenario A: 1. Acute epiglottitis 2. Adrenaline nebulization, i.v steroids 3. Acute
retropharyngeal abscess, laryngotracheal bronchitis, foreign body inhalation.
B: 1. Acute epiglottitis 2. Acute retropharyngeal abscess, laryngotracheal bronchitis, foreign
body inhalation, angioedema 3. Hoarseness of voice, dyspnea, stridor.
C: 1. Acute epiglottitis 2. X-ray soft tissue neck lateral view, thumbprint sign 3. Haemophillus
influenzae type B 4. Admission in hospital, maintenance of airway, 100% humidified oxygen,
i.v ampicillin, adrenaline nebulization, steroids, adequate hydration.
--------------------------------------------------------------------------------------------------------------------------

LARYNGOTRACHEOBRONCHITIS
Synonym: Croup
Definition:
It is defined as subacute inflammatory disease of the larynx and lower respiratory tract
predominantly involving subglottis, trachea and tracheobronchial tree.
Etiopathology:
It affects usually young children between the ages of 06 months to 03 years. It is the most
common cause of stridor in children. It is caused by viruses. Usual viruses are rhinovirus,
adenovirus, parainfluenza type 1 and type 2. There is characteristic subglottic edema.
Clinical features:
Clinical features of laryngotracheobronchitis are;
1. Croup

CONCEPT BOOK OF DISEASES OF EAR, NOSE & THROAT BY FAHIM AWAN


394 Part E

a. Barking cough (like seal’s bark)


b. Hoarseness
c. Low-grade fever
2. There may be shortness of breath and
stridor. Flexible laryngobronchoscopy shows:
1. Subglottic edema extending to trachea.
2. Vocal cords are slightly inflamed and epiglottis is normal.

Investigations:
Diagnosis is usually clinical. Detection of viral antigen by sampling
mucous from the nasopharynx is helpful.
X-ray soft tissue neck AP view may show narrowing of
subglottis (steeple sign).
Treatment: Steeple sign in laryngo-
tracheobronchitis
1. Admission in hospital and observation for any airway obstruction.
2. Oxygen inhalation is given.
3. Broad-spectrum antibiotics are given to prevent secondary infection.
4. Steroids are given to reduce subglottic edema.
5. Nebulization with adrenaline is done to combat bronchoconstriction.
Airway and oxygen saturation should be monitored and endotracheal intubation considered if
airway is impaired.

Differential diagnosis:
1. Acute epiglottitis
2. Foreign body inhalation
3. Angioedema

Difference between acute epiglottitis and acute laryngotracheobronchitis

Sr. Features Acute epiglottitis Acute laryngotracheobronchitis


No
1 Definition It is an acute infection of It is an acute infection of
epiglottis subglottis
2 Causative Usually, bacteria Usually, virus
pathogen
3 Severity Very dangerous. It is an Less dangerous. It is not an
emergency emergency
4 Age Mostly affects children Mostly affects children between
between the age of 4 to 5 the age of 6 months to 3 years
years
5 Course Rapid Slow
6 Incidence Rare Common
7 Fever High grade Low grade
8 Cough There is no cough There is barking cough
9 Signs Tripod and drooling signs are Tripod and drooling signs are
present not present
10 On Epiglottis gives cherry red Subglottis shows marked edema
examination appearance
11 Diagnostic Shows thumb print sign Shows steeple sign
x-ray

SELF ASSESSMENT
Q. 1. Laryngotracheobronchitis is also known as

a. acute epiglottitis d. pseudocroup

b. acute laryngitis e. supraglottic laryngitis


c. croup
Q. 2. All are usual clinical features of laryngotracheobronchitis except
a. barking cough c. dysphonea

b. dysphagia d. hoarseness of voice

CONCEPT BOOK OF DISEASES OF EAR, NOSE & THROAT BY FAHIM AWAN


395 Part E
e. low-grade fever
Q. 3. Steeple sign is seen on X-ray neck AP view in
a. acute epiglottitis d. subglottic laryngitis

b. acute laryngitis e. supraglottic laryngitis


c. laryngotracheobronchitis
Scenario: A 02 years old baby is having hoarseness, barking cough and dyspnea for last 05
days. The child is also having fever of 101 0F.
a. What is the most probable clinical diagnosis?
b. What is the differential diagnosis?
c. How will you manage this patient?
-------------------------------------------------------- ANSWERS-----------------------------------------------------
Q.1. c 2. b 3. c
Scenario: 1. Acute laryngotracheobronchitis 2. Acute epiglottitis, foreign body, angioedema
3. Admission in hospital, O2 inhalation, broad-spectrum antibiotics, steroids, nebulization
with adrenaline.
--------------------------------------------------------------------------------------------------------------------------
GRANULOMATOUS DISEASES OF LARYNX
TUBERCULOSIS OF LARYNX
Etiopathology:
Laryngeal tuberculosis is almost always secondary to open
pulmonary tuberculosis. It is due to contamination with sputum
containing acid-fast bacilli. It has a predilection for the posterior
part of larynx i.e., posterior commissure, arytenoids and the vocal Mouse nibbled appearance in
cords. tuberculosis of larynx

Infection starts in the subepithelial space. There is an


inflammatory reaction of mucosa and tubercles are found. These consist of Langhan’s giant
cells, caseation and necrosis. Covering mucosa has an irregular appearance, which sloughs
and ulcerates. The ulcers are shallow with undermined edges. There may be mouse nibbled
appearance. Sometimes tumor like swellings is found with reparative process called
tuberculomata.

Clinical features:
There is hoarseness of voice and productive cough. There may be pain in the throat and
referred earache.
On examination of larynx, there may be mucosal hyperemia, irregularity of the mucosal
surface, granuloma or ulceration in the posterior part of larynx.

CONCEPT BOOK OF DISEASES OF EAR, NOSE & THROAT BY FAHIM AWAN


396 Part E

Investigations:
9. X-ray chest usually shows focus of pulmonary tuberculosis.
10. Sputum for AFB is diagnostic.

11. MycoDot test


12. PCR for Mycobacterium tuberculosis is helpful in doubtful cases.
13. Biopsy should always be taken to rule out malignancy.
Treatment:
Antituberculous drugs are given as for pulmonary tuberculosis. Strict voice rest is advised.

Differential diagnosis:
1. Lupus vulgaris

2. Syphilis

3. Carcinoma of larynx
SELF ASSESSMENT
Q. 1. In the larynx tuberculosis usually starts in

a. cartilage layer d. subepithelial space

b. mucosal layer e. submucosal space


c. muscular layer
Q. 2. In the larynx the most common site of tuberculosis is
a. anterior commissure d. posterior commissure

b. epiglottis e. subglottis
c. false cords
Q. 3. Mouse nibbled appearance of vocal cord is seen in
a. cancer of larynx d. syphilis of larynx

b. sarcoidosis of larynx e. tuberculosis of larynx

c. scleroma of larynx
Scenario: A 35 years old lady reported to ENT OPD with the complains of hoarseness of
voice for 03 months and productive cough for 02 months. Now she is complaining of earache
which is more on left side. On indirect laryngoscopic examination, larynx is hyperemic with
irregularity of mucosal surface and ulcers on vocal cords with undermined edges and mouse
nibbled appearance of vocal cords.
1. What is the most probable clinical diagnosis?
CONCEPT BOOK OF DISEASES OF EAR, NOSE & THROAT BY FAHIM AWAN
397 Part E
2. What investigations will you carry out?
3. How will you treat this patient?
-------------------------------------------------------- ANSWERS-----------------------------------------------------
Q.1. d 2. d 3. e
Scenario: 1. Tuberculosis of larynx 2. X-ray chest, sputum for AFB, MycoDot test, PCR,
biopsy 3. Antituberculous drugs
--------------------------------------------------------------------------------------------------------------------------

SYPHILIS OF LARYNX
All stages of syphilis can manifest in the larynx. The mucosa is
swollen and infiltrated and later changes to deep punched out ulcer
with central sloughing. It has predilection for the anterior part of the
larynx i.e., epiglottis and the aryepiglottic fold.

SELF ASSESSMENT Syphilis of larynx

Q. 1. In the larynx the most common site of syphilis is


a. epiglottis and aryepiglottic fold d. subglottis

b. false cords e. ventricles


c. posterior commissure
Q. 2. Punched out ulcer in the larynx is usually seen in
a. cancer of larynx d. syphilis of larynx

b. sarcoidosis of larynx e. tuberculosis of larynx

c. scleroma of larynx
Scenario: A 50 years old non smoker male with saddle nose presented in ENT OPD with
complaint of hoarseness of voice for last 05 years. Anterior rhinoscopy reveals bony septal
perforation. On laryngeal examination there is punched out ulcers involving epiglottis and
aryepiglottic folds.
1. What is the most probable clinical diagnosis?
2. What investigations will you carry out?
3. How will you treat this patient?
--------------------------------------------------------ANSWERS-----------------------------------------------------
Q.1. a 2. d
Scenario: 1. Syphilis 2. VDRL, FTA, TPI test 3. High doses of penicillin
--------------------------------------------------------------------------------------------------------------------------

CONCEPT BOOK OF DISEASES OF EAR, NOSE & THROAT BY FAHIM AWAN


398 Part E

TUMORS OF LARYNX
PAPILLOMA OF LARYNX
Etiopathology:
It is the most common benign tumor of larynx. It is common in adults. It is usually solitary
and small in size. The usual site of origin is anterior commissure and anterior half of true
vocal cords. Rarely false cords may be involved. Its recurrence rate is much less than
juvenile laryngeal papillomatosis. It is caused by human papilloma virus.
Clinical features:
Patient usually presents with hoarseness of voice. Indirect
laryngoscopic examination shows pedunculated papilloma which may
be sucked down between the cords during inspiration and then blown
up again between the cords during expiration.
Papilloma of larynx
Treatment:
1. Endoscopic removal is carried out microlaryngoscopically or with carbon dioxide laser
when papilloma is small. Histological examination is necessary to rule out its benign
nature.
2. Laryngofissure occasionally may be required when papilloma is large.

CARCINOMA OF LARYNX
Etiopathology:
The most common malignant tumor of larynx is squamous cell carcinoma. Carcinoma of
larynx constitutes 01% of all malignancies in men.
It is more common in men as compared to women. Men over the age of 40 are usually
affected.
Tobacco and alcohol are the most common known causative factors. Combination of
tobacco and alcohol has synergistic effect (15-fold increase) in causing carcinoma of larynx.
Radiations and asbestos are also known causes of carcinoma of larynx. The most
commonly affected site in the larynx is glottis (76 %), the next frequent is supraglottis (19 %)
and least frequent is subglottis (05 %). Most frequent site involved in glottic carcinoma is
anterior 2/3rd of vocal cord.
Supraglottic carcinoma is the most aggressive of laryngeal carcinoma. The most
common site involved in supraglottic carcinoma is epiglottis. It presents in younger age
group as

CONCEPT BOOK OF DISEASES OF EAR, NOSE & THROAT BY FAHIM AWAN


399 Part E

compared to other carcinomas of larynx. It has got early lymphatic spread.


Distant metastasis in late stages of laryngeal cancer is usually seen in the lungs.

Clinical features:
1. Persistent and progressive hoarseness of voice is the
most common presenting symptom.
2. Dyspnea and stridor may occur in later stages.
3. There may be swelling in the neck (due to lymph node)
or larynx (due to direct extension of the tumor outside the
larynx). Endoscopic view of
carcinoma of larynx
4. In supraglottic carcinoma, there may be referred earache.
On examination with indirect laryngoscopic mirror;
1. There is granular, exophytic or ulcerative lesion.
2. There may be limitation or loss of vocal cord movements.
3. Cervical lymph nodes may be enlarged especially in supraglottic carcinoma.
Investigations:
1. X-ray chest is done to find out metastasis and to access the general physical status of the
patient for general anesthesia and possible surgery.
2. CT/ MRI is carried out to find three-dimensional extent of the tumor site, size, number
and level of lymph node involvement).
3. Panendoscopy (oropharyngoscopy, hypopharyngoscopy, laryngoscopy,
tracheobronchoscopy, esophagoscopy and nasopharyngoscopy) under general anesthesia
is performed.
4. Biopsy of the lesion is taken for histopathology.

Differential diagnosis:
1. Granuloma of larynx
2. Carcinoma of hypopharynx
Anatomical regions of larynx:
Anatomically there are following regions of larynx;
1. Supraglottis
2. Glottis
3. Subglottis

CONCEPT BOOK OF DISEASES OF EAR, NOSE & THROAT BY FAHIM AWAN


400 Part E

Classification of anatomical regions/sites and subsites of larynx:


S. No. Region/site Subsites
1. Supraglottis Epiglottis (both lingual and laryngeal surfaces)
Aryepiglottic fold (laryngeal aspect)
Arytenoid
Ventricular bands (false cords)
2. Glottis Vocal cords
Anterior commissure
Posterior commissure
3. Subglottis Subglottis upto lower border of cricoid cartilage
Carcinoma of larynx is grossly divided according to following classification;

Classification of primary tumor (T) of larynx


T1 Tumor limited to one region. One or both sides of the supraglottis, glottis or
subglottis.
T2 Tumor with extension to adjacent region without fixation of vocal cord.
T3 Tumor with fixation of vocal cord but limited to larynx.
T4 Tumor with direct extension beyond larynx.

Classification of neck node metastasis (N)


Nx Regional lymph nodes cannot be assessed.
N0 No regional lymph node metastasis.
N1 Metastasis in a single ipsilateral lymph node 03 cm or less in greatest
dimension.
N2 Metastasis in a single ipsilateral lymph node, more than 03 cm but not more
than 06 cm in greatest dimension, or in multiple ipsilateral lymph nodes none
more than 06 cm in greatest dimension, or in bilateral or contralateral lymph
nodes, none more than 06 cm in greatest dimension.
N2a Metastasis in a single ipsilateral lymph node, more than 03 cm but not more
than 06 cm in greatest dimension.

CONCEPT BOOK OF DISEASES OF EAR, NOSE & THROAT BY FAHIM AWAN


401 Part E

N2b Metastasis in multiple ipsilateral lymph nodes, none more than 06 cm in


greatest dimension.
N2c Metastasis in bilateral or contralateral lymph nodes, none more than 06 cm in
greatest dimension.
N3 Metastasis in a lymph node more than 06 cm in greatest dimension.
Classification of distant metastasis (M)
Mx Distant metastasis cannot be assessed
M0 No distant metastasis
M1 Distant metastasis present
TNM/clinical staging
Stage I T1, N0, M0
Stage II T2, N0, M0
Stage III T3, N0, M0 / T1, T2 or T3 with N1, M0
Stage IV A T4, N0, M0 or T4, N1, M0
Stage IV B Any T, N2, M0 or Any T, N3, M0
Stage IV C Any T any N, M1
Treatment:
General principles are;
1. T1 and T2 lesions, radiotherapy is the preferred mode of treatment. It is given at a dose
of 5500-6500 centigrays.
2. T3 and T4 lesions, surgery is the preferred mode of treatment. Most commonly
performed surgery is total laryngectomy. Partial laryngectomy may be performed in
cases of limited T3 lesions.
3. If there is extension of tumor beyond the larynx or if the resection margins are not free of
malignancy, then postoperative radiotherapy must be given.
4. If lymph nodes are involved then regardless of the stage of the tumor, surgery is the
preferred mode of treatment.
LARYNGECTOMY
Preparation:
Any nasal and orodental sepsis is eliminated. Attention should be given to general health and
chest. Any other medical condition like hypertension or diabetes should be controlled.
Psychological assessment of every patient is carried out individually. Informed consent is
taken for better cooperation of the patient during postoperative period.

CONCEPT BOOK OF DISEASES OF EAR, NOSE & THROAT BY FAHIM AWAN


402 Part E

Procedure:
After endotracheal intubation for general anesthesia, patient is made to lie supine on the table.
Sand bag is placed beneath the shoulders. Head and neck extended. Scrubbing and draping
done.
Gluck Soerenson incision made in the skin. The incision starts on the anterior border of
sternomastoid muscle, at the level of the hyoid bone, passes down along the anterior border
for about 6-7 cm and then curves across the midline at the level of the 2nd and 3rd ring of the
trachea. Skin flap elevated and strap muscles are divided in the neck as low as possible.
Suprahyoid muscles are separated from the hyoid bone. Larynx is entered through the
suprahyoid space. Constrictor muscles are separated from the posterior aspect of the thyroid
lamina. Superior thyroid and middle thyroid vascular pedicle on both sides are ligated.
Inferior thyroid pedicle on the diseased side is also ligated.
Contralateral lobe of thyroid gland i.e., healthy side of the thyroid gland freed from
specimen and conserved. Trachea divided from the larynx well below the tumor margin. A
nasogastric tube is passed and secured in position. End tracheostome fashioned and pharynx
is closed with absorbable sutures in 2 to 3 layers. Bilateral suction drains are placed and the
skin is closed in two layers.
Complications of total laryngectomy:
1. Hemorrhage
2. Wound infection
3. Pharyngocutaneous fistula
4. Necrosis of flap
5. Pulmonary and cerebral embolism
6. Parathyroid insufficiency
7. Thyroid insufficiency
8. Stomal recurrence
9. Pharyngeal recurrence
Postoperative care:
Continuous monitoring is done to check for any changes in vitals, bleeding, obstruction of
airway and any local infection.
Rehabilitation of voice after total laryngectomy
Different options are available for voice rehabilitation after total laryngectomy.
1. Esophageal speech:
The patient is trained to swallow some air into esophagus and then
expel the air into the pharynx. While expelling the swallowed air the

Blom and Singer voice


prosthesis
403 Part E

patient can utter a few words. Phonation is produced by Pharyngoesophageal segment.


2. Electrolarynx (pharyngeal vibrator):
It is a battery driven electronic device which is held by the patient
against the soft tissues of the neck. Sound is produced in the pharynx
and oral cavity and converted into speech by the device.
3. Tracheoesophageal speech:
At the time of total laryngectomy, TracheoEsophageal Puncture (TEP)
made. One-way valve e.g., Panje or Blom-Singer is placed in this
artificial fistula. The valve diverts air from trachea to esophagus and
prevents aspiration of food. Air is carried from the lungs and trachea Use of electrolarynx
to esophagus. Air entering the pharynx is converted to speech.
SELF ASSESSMENT
Q. 1. The most common benign tumor of larynx is

a. chondroma d. neurofibroma

b. fibroma e. solitary papilloma


c. hemangioma
Q. 2. The most common site of laryngeal carcinoma is
a. glottis d. suprahyoid epiglottis

b. infrahyoid epiglottis e. transglottic


c. subglottis
Q. 3. The most frequent site involved in glottic carcinoma is
a. anterior commissure d. posterior 1/2 of vocal cord

b. anterior 2/3rd of vocal cord e. posterior 1/3rd of vocal cord


c. posterior commissure
Q. 4. Persistent and progressive hoarseness of voice is the earliest symptom of carcinoma
of
a. hypopharynx d. oral cavity

b. larynx e. oropharynx
c. nasopharynx
Q. 5. The most common and earliest manifestation of carcinoma of the glottis is
a. cervical lymph node enlargement c. haemoptysis

b. dysphagia d. hoarseness

CONCEPT BOOK OF DISEASES OF EAR, NOSE & THROAT BY FAHIM AWAN


404 Part E
e. stridor
Q. 6. The lymphatic drainage of the vocal cord is to the
a. lower deep cervical node d. all of the above

b. prelaryngeal lymph node e. none of the above


c. upper deep cervical node
Q. 7. All are true of supraglottic carcinoma except
a. higher incidence of nodal metastasis d. presents in older age group
b. most aggressive of laryngeal carcino- e. the most common site in supraglottic
mas region is epiglottis
c. presents in younger age group
Q. 8. Chances of cervical nodal metastasis are highest in carcinoma of
a. anterior commissure d. subglottis

b. glottis e. supraglottis
c. posterior commissure
Q. 9. The most common primary head and neck malignancy is carcinoma of
a. hypopharynx and oropharynx d. oral cavity and hypopharynx

b. larynx and oral cavity e. oral cavity and oropharynx


c. nasopharynx
Q. 10. The highest incidence of distant metastasis in laryngeal carcinoma is seen in
a. bone d. lung

b. brain e. spleen
c. liver
Q. 11. A middle-aged male comes to OPD with the only complaint of progressive
hoarseness for the past 01 year. He is a smoker for last 30 years. On examination, reddish
areas of mucosal irregularity overlying both cords are seen. Investigations would include
all except
a. biopsy d. staining with toludine blue

b. CT/MRI e. X-ray chest


c. panendoscopy
Q. 12. Treatment of choice in case of squamous cell carcinoma larynx stage T4 N0 M0 is
a. chemotherapy d. radiotherapy

b. neck dissection e. total laryngectomy


c. partial laryngectomy
Q. 13. Phonation in esophageal speech in a case of laryngectomy is produced by
a. buccal cavity b. esophagus
CONCEPT BOOK OF DISEASES OF EAR, NOSE & THROAT BY FAHIM AWAN
405 Part E

c. larynx e. pharyngoesophageal segment


d. pharynx
Q. 14. A Blom-Singer prosthesis is used in
a. laryngectomised patient d. stapedectomy

b. obstructed airway e. total maxillectomy


c. OSAS
Q. 15. TracheoEsophageal Puncture (TEP), currently the surgical method of choice for voice
restoration after total laryngectomy was introduced by
a. Blom and Ishiki d. Singer and Ishiki

b. Blom and Singer e. Woodman and Blom


c. Ishiki and Woodman
Q. 16. Blom Singers prosthesis is fitted between
a. larynx and pharynx d. trachea and esophagus

b. larynx and trachea e. trachea and pharynx

c. pharynx and esophagus


Scenario A: A 50 years old male presents with progressive hoarseness of voice for last six
months. On indirect laryngoscopy there is an irregular mass on right vocal cord which
extends to the supraglottis. The right hemilarynx is fixed. CT shows lesion limited to larynx.
There are no neck nodes and no evidence of distant metastasis.
1. What is the most probable clinical diagnosis?
2. How will you confirm the diagnosis?
3. What is the stage of the disease?
4. How will you manage this case?
B: A 65 years old chain smoker is suffering from hoarseness of voice for last 01 year.
Recently he developed noisy breathing and at times spells of respiratory distress. But for last
two days respiratory distress was persistent compelling the family to seek medical advice in
accident and emergency department.
1. What will be your immediate action?
2. What usual complications can occur during the procedure?
3. How will you manage the patient further?

Q.1. e 2. a 3. b 4. b 5. d 6. e 7. d 8. e 9. b
10. d 11. d 12. e 13. e 14. a 15. b 16. d
Scenario A: 1. Carcinoma of larynx 2. Endoscopy and biopsy 3. Stage III
4. Total laryngectomy.
B: 1. Tracheostomy 2. Hemorrhage, air embolism, apnea, cardiac arrest, damage to cricoid,

CONCEPT BOOK OF DISEASES OF EAR, NOSE & THROAT BY FAHIM AWAN


406 Part E

trachea and recurrent laryngeal nerve 3. X-ray chest CT/MRI, panendoscopy under general
anaesthesia and biopsy, treatment include radiotherapy for T1, T2 and surgery for T3, T4.
--------------------------------------------------------------------------------------------------------------------------
-

UPPER AIRWAY OBSTRUCTION


In upper airway obstruction, different techniques are available to restore the airway. The
exact technique depends upon site of obstruction and the facilities/expertise available.
Techniques of restoring upper airway obstruction:
1. Chin lift/head tilt
2. Jaw thrust
3. Oropharyngeal airway
4. Laryngeal mask
5. Endotracheal intubation
6. Minitracheostomy
7. Tracheostomy
1.CHIN LIFT
Synonym: Head tilt
This procedure is used to prevent the tongue obstructing the
airway. This maneuver is performend
by tilting the head backwards in an unconscious patient, often Jaw thrust
Chin lift
by applying pressure to the forehead and chin.
2. JAW THURST
It is performed by placing the index and middle fingers to It
physically push the posterior aspect of lower is
Jaw upwards while their thumb pushes down on the chin to a
open mouth. When the mandible is displaced forward, it pulls
the tongue forward and prevent it from obstructing the
entrance of trachea. supraglottic airway
3. OROPHARYNGEAL AIRWAY
Synonym: OPA, Oral airway, Guedal pattern airway
It is a device used to maintain or open an unconscious patient's
airway by preventing the tongue from covering the epiglottis. device just like a large
4. LARYNGEAL MASK AIRWAY (LMA) endotracheal tube on the
proximal end, that
connects to an elliptical
mask on the distal end.
It is designed to sit in
the patient's
Laryngeal mask airway hypopharynx and cover
supraglottic structures
for proper ventilation.
It is a good alternative for bag valve mask ventilation.
5. ENDOTRACHEAL INTUBATION
Indications:
Following are indications of endotracheal intubation; Oropharyngeal airways of
1. For the purpose of general anesthesia. different sizes

2. Severe upper airway obstruction.


3. Protection of trachea from contamination e.g., head injury,
cerebrovascular accident.
4. To facilitate controlled ventilation. Endotracheal intubation

5. To facilitate tracheobronchial toilet.


Procedure:

Patient is made to lie supine; occiput is elevated about 04-06 cm off the table. The neck is
flexed on trunk and head extended on neck (also called ‘sniffling position’). The
laryngoscope is held in the left hand and is introduced in the right side of patient’s mouth. So
that the tip of the blade approaches the midline to lie in the vallecula. The Z-shape cross
section of laryngoscope causes the tongue to be pushed out of the way to left. The handle of
the laryngoscope is lifted to the direction to which it points. It must not be rotated as this
movement will damage teeth, gums and mucus membrane of the pharynx.
Elevation of the root of the tongue will indirectly elevate the posteriorly placed epiglottis.
The laryngeal opening will come in view. The endotracheal tube is passed between the vocal
cords. If the larynx is difficult to see it may be brought into view by an assistant who gently
407 Part E

presses the thyroid cartilage posteriorly. Laryngoscope is removed. The cuff of endotracheal
tube is inflated with air. The endotracheal tube is attached with the Ambo bag, ventilator or
anaesthetic circuit. For prolonged intubation the cuff of the tube is temporarily deflated for
five minutes after every one hour.
Complications:
Following are complications of endotracheal intubation;
1. Tracheal/subglottic stenosis
2. Glottic granuloma
3. Laryngotracheal synechiae
SELF ASSESSMENT
Q. 1. Indications of endotracheal intubation include

a. controlled ventilation tion

b. general anesthesia d. severe upper airway obstruction


c. protection of trachea from contamina- e. all of the above
Q. 2. After acute laryngeal injury with bleeding, first thing to be done is
a. to carry out cardiac massage d. maintain airway

b. to examine for head injury e. monitor blood pressure and pulse


c. to give intravenous antibiotic
Q. 3. Complications of endotracheal intubation include
a. glottic granuloma d. tracheal stenosis

b. laryngotracheal synechiae e. all of the above

c. subglottic stenosis
-------------------------------------------------------- ANSWERS---------------------------------------------------
Q.1. e 2. d 3. e
--------------------------------------------------------------------------------------------------------------------------
-
6. MINITRACHEOSTOMY
Synonyms: Laryngotomy, Cricothyroidotomy

In an emergency situation minitracheostomy may be a life-saving procedure. Failure to


clear or secure the airway with endotracheal tube or tracheostomy may lead to death. Entry
into the airway can be achieved rapidly through the cricothyroid
membrane because it is superficial and an early identifiable
landmark.
Indications:
Minitracheostomy may be performed when obstructed airway
cannot be secured through laryngeal route for whatever reason.
Cricothyroidotomy
408 Part E

Minitracheostomy as an elective procedure is strongly condemned because of higher


incidence of subglottic stenosis.
Procedure:
Patient’s head is placed in extreme extended position by placing a pillow beneath the
shoulders. Thyroid cartilage is stabilized with the thumb and middle finger using the non-
dominant hand. Cricothyroid space is identified with index finger. Scalpel is held in the
dominant hand. A stab incision is made through the skin and cricothyroid membrane.
With the non-dominant hand, the artery forceps is slightly opened and passed around the
scalpel blade into the airway to widen the hole. The scalpel is removed and is exchanged for
endotracheal tube or tracheostomy tube. When the facilities for mini-tracheostomy are not
available, airway can be restored with a 14-gauge intravenous needle. This can be inserted
through the cricothyroid membrane into the trachea.
Complications:
1. Hemorrhage
2. Subcutaneous emphysema
3. Subglottic stenosis
7. TRACHEOSTOMY
Definition:
The word tracheostomy is derived from two Greek
words meaning “to cut the trachea”. It is defined as “to
make a surgical hole into the trachea and to maintain
it.”
Indications:
Following are indications of tracheostomy.
A. Upper airway obstruction which may be real or
anticipated e.g.,

1. Infectious processes in the oral cavity, oropharynx,


hypopharynx or larynx Patient with tracheostomy tube

2. Tumors of oral cavity, oropharynx, hypopharynx or larynx


3. Trauma of oral cavity, oropharynx, hypopharynx or larynx
4. Foreign bodies of larynx and the trachea
B. Mechanical respiratory insufficiency e.g.,
1. Drug intoxication
2. Head and chest trauma
3. Neuroparalytic disorders
CONCEPT BOOK OF DISEASES OF EAR, NOSE & THROAT BY FAHIM AWAN
409 Part E

4. Chronic obstructive pulmonary disease (COPD)


C. Ventilatory insufficiency e.g.,
1. Congestive heart failure
2. Pulmonary edema
3. Bulbar disease
4. Chronic lung disease
Criteria for emergency tracheostomy in upper airway obstruction:
In upper airway obstruction, tracheostomy is immediately required if the patient has stridor at
rest, is restless, cannot lie flat, is using accessory muscles of respiration, has a rising pulse
rate or has recessions of supraclavicular and intercostal spaces during inspiration.
Procedure of tracheostomy:
Tracheostomy position made i.e., patient lying supine, head and neck extend with pillow
under the shoulders. Scrubbing done. Local anesthesia injected in the skin and strap muscle
midway between the suprasternal notch and lower border of cricoid.
A horizontal incision is made in the skin. Strap muscles are separated in the midline
(linea alba). Thyroid isthmus is retracted up or down (or may be divided and transfixed).
Pretracheal fascia is divided in the midline. Presence of trachea is confirmed by aspiration of
air in a syringe containing liquid in it.
A hole is made in the midline of trachea at 2nd tracheal ring or below. Tracheostomy
tube with an obturator is inserted into the trachea. This insertion may be aided by the use of a
tracheal dilator. Obturator is removed. Wound is closed in layers. Loose skin stitches are
applied to avoid surgical emphysema. Tracheostomy tube is fixed in position with skin
stitches or straps around the flexed neck.
Complications of tracheostomy:
A. Immediate i.e., which occur during surgery e.g.,
1. Hemorrhage
2. Air embolism due to injury to major veins in the neck, as these have suction effect
secondary to negative pressure within them.
3. Apnea immediately after opening the trachea is due to sudden wash of the blood
carbon dioxide level.
4. Cardiac arrest
5. Damage to cricoid, trachea and recurrent laryngeal nerve
B. Intermediate i.e., which occur during the period in which patient stays in the hospital
e.g.,
1. Dislodgement of tracheostomy tube

CONCEPT BOOK OF DISEASES OF EAR, NOSE & THROAT BY FAHIM AWAN


410 Part E

2. Surgical emphysema i.e., air in the soft tissues due to overtight skin sutures
3. Pneumothorax due to damage to apical pleura
4. Tracheoesophageal fistula
5. Tracheo-arterial fistula
6. Infection
C. Late i.e., which occur when the patient has been discharged from
hospital e.g.,
1. Subglottic/tracheal stenosis
2. Tracheocutaneous fistula
3. Difficulty with decannulation
Postoperative care of tracheostomy:
1. Care of wound: Antiseptic dressing is applied daily for 07-10 Subglottic stenosis
days and stitches removed on 7th day.
2. Care of tracheostomy tube: Repeated suction is done to avoid
blockage. A few drops of saline or sodium bicarbonate instilled 02
hourly into the tracheostomy tube to humidify inspired air and to
avoid crusting. Double layer of wet gauze or steam tent may serve the
same purpose.
3. Care of chest: Tracheostomy patients are prone to develop chest X ray showing
subglottic stenosis
infection, so regular chest physiotherapy is instituted.
4. Care of communication: Loss of voice is a major handicap in
such patients. So, a bell should be provided to the patient to call for help in case of any
emergency. Patient should be provided with pen and paper.
Handicaps of a treacheostomized/laryngectomized patient:
1. Aphonia
2. Inability to blow the nose
3. Inability to build intra-abdominal and intra-thoracic pressure such as during
explosive coughing, climbing, lifting heavy weight, parturation, etc.
4. Inability to swim
5. Exposure of the lung to cold dirty air and thus, increase in incidence of physical and
microbial injury.
Types of tracheostomy tube:
A wide variety of tracheostomy tubes are available and their selection depends on;
1. Indications of the operation
2. Postoperative care

CONCEPT BOOK OF DISEASES OF EAR, NOSE & THROAT BY FAHIM AWAN


411 Part E

Generally, the tracheostomy tubes are classified into;


A. Metallic tubes
B. Non metallic tubes
A. Metallic tubes:
It has following components;
1. Outer tube
2. Inner tube; It is always slightly longer than the outer tube so that crusts can collect
on the protruding end and cleaning of the inner tube can be performed unhurriedly
because the outer tube maintains the patency of the airway.
3. Obturator (optional); It is used for insertion of tracheostomy tube into tracheostome.
Advantages:
The metallic tubes usually suit to the patients who are not hospitalised as the patients
themselves can take care of their tracheostomy tube.
Disadvantages:
1. These do not have a cuff, therefore, cannot produce an airtight seal.
2. If these do not fit properly, the end of the tube can erode the anterior tracheal wall.
B. Non metallic tubes:
These are usually made of rubber or silastic and again these may be with or
without cuff. It has an obturator.
Advantages:
Advantage of inflatable cuff is that these can be connected to an Ambu bag,
anesthesia machine or ventilator. These do not produce mechanical damage
to the trachea.
Disadvantages: Components
of a non-
1. Inflated cuff may cause avascular necrosis as a result of over pressure metallic
tracheostomy tube
in the cuff. Salpeker tube is currently available which has two cuffs
one above the other, allowing alternate deflation and inflation of each
cuff. Use of this tube reduces chances of subglottic/tracheal stenosis.
2. It is difficult for the patients themselves to manage at home.
Decannulation:
It is defined as permanent removal of the tracheostomy tube when the condition leading to
tracheostomy is over.
Before actual removal, the tracheostomy tube is replaced with a smaller tracheostomy
CONCEPT BOOK OF DISEASES OF EAR, NOSE & THROAT BY FAHIM AWAN
412 Part E

tube. The tube is closed and the patient monitored for a couple of days. If the patient is able to
withstand the blocked tube then the removal of tracheostomy tube is carried out. If the
tracheostomy is less than 16 weeks duration, tracheocutaneous fistula heals spontaneously.
Otherwise, surgical closure of fistula may be required.

SELF ASSESSMENT
Q. 1. Minitracheostomy is also called

a. cricothyroidotomy d. Both a & b

b. high tracheostomy e. Both a & c


c. laryngotomy
Q. 2. Opening the airway between cricoid and thyroid cartilages is called
a. low tracheostomy High tracheostomy d. minitracheostomy

b. laryngofissure e. thyroidotomy
c. low tracheostomy
Q. 3. Minitracheostomy is done through
a. cricothyroid membrane d. quardangular membrane

b. cricotracheal membrane e. thyrohyoid membrane


c. cricovocal membrane
Q. 4. Apnea immediately after opening the trachea is due to
a. rise of the blood carbon dioxide level d. wash of the blood carbon dioxide
b. rise of the blood oxygen level level
c. rise of carbon mono oxide e. none of the above
Q. 5. Which of the following cartilage is like a signet ring?
a. Arytenoid d. Cuneiform

b. Corniculate e. Thyroid
c. Cricoid
Q. 6. All are criteria of emergency tracheostomy in upper airway obstruction except
a. normal pulse rate d. restlessness

b. recession of intercostal spaces e. stridor at rest

c. recession of supraclavicular spaces


Q. 7. Indication of tracheostomy is

a. mechanical respiratory insufficiency d. all of the above

b. upper airway obstruction e. none of the above


c. ventilatory insufficiency

CONCEPT BOOK OF DISEASES OF EAR, NOSE & THROAT BY FAHIM AWAN


413 Part E
Q. 8. All are immediate complications of tracheostomy except
a. air embolism d. hemorrhage

b. apnea e. subglottic stenosis

c. cardiac arrest

Q. 9. All are intermediate complications of tracheostomy except


a. air embolism d. surgical emphysema

b. dislodgement of tracheostomy tube e. tracheoesophageal fistula


c. infection of wound
Q. 10. All are late complications of tracheostomy except
a. cardiac arrest d. tracheal stenosis

b. difficulty with decannulation e. tracheocutaneous fistula


c. subglottic stenosis
Q. 11. Post operative care of tracheostomy is
a. care of chest d. care of wound

b. care of communication e. all of the above


c. care of tracheostomy
Q. 12. A tracheostomized patient with portex tracheostomy tube in the ward, developed
sudden complete blockage of the tube. Which of the following is best next step in the
management?
a. Change with metallic tube d. Suction of tube after saline instillation
b. Immediate removal of the tracheostomy e. Suction of tube after sodium bicarbonate
tube instillation
c. Jet ventilation
Q. 13. Which words cannot be spoken by a patient with a tracheostomy?
a. Adjectives d. Nouns

b. Consonants e. Verbs

c. Cannot speak at all


Scenario A: A young boy was admitted in intensive care unit with head injury. He was on
ventilatory support and his tracheostomy was performed on 14th day.
1. What are functions of tracheostomy in this patient?
2. What are indications of tracheostomy?
3. What are immediate complications of tracheostomy?
B: A 70 years old male with carcinoma of larynx and stridor for last 02 months is brought to
emergency department. An urgent tracheostomy is carried out. When trachea is opened
CONCEPT BOOK OF DISEASES OF EAR, NOSE & THROAT BY FAHIM AWAN
414 Part E
and tracheostomy tube is placed in the trachea it is observed that patient is not breathing.
1. What happened to the patient?
2. What is cause of this condition?
3. How will you manage this patient?
------------------------------------------- ---ANSWERS-----------------------------------
------------- ------------------
Q.1. e 2. d 3. a 4. d 5. c 6. a 7. d
8. e 9. a 10. a 11. e 12. b 13. c
Scenario A: 1. To reduce dead space, for positive pressure ventilation, to clear
tracheobronchial secretions 2. Upper airway obstruction, mechanical respiratory
insufficiency, ventilatory insufficiency 3. Hemorrhage, air embolism, apnea, cardiac arrest,
damage to cricoid, trachea and recurrent laryngeal nerve.
B: 1. Patient is in apnea 2. Due to sudden wash out of CO2 3. Mouth to mouth breathing.
--------------------------------------------------------------------------------------------------------------------------
-
MISCELLANEOUS DISEASES OF LARYNX AND
TRACHEOBRONCHIAL TREE
FOREIGN BODY OF LARYNX
Etiopathology:
A foreign body inhaled into the air passage can lodge in the larynx, trachea or bronchi. Site of
lodgment depends on the size, shape and nature of foreign body. A large foreign body unable
to pass through glottis will lodge in the supraglottic area while the smaller ones will pass
down through larynx into trachea or bronchi. Foreign bodies with sharp points e.g pins, fish
bones, metal nails etc can stick anywhere in the larynx or tracheobronchial tree.
Children less than 04 years of age are usually affected. The mishap occurs when the
mouth is full of food and due to sudden fright, the child takes a deep inspiration and the
foreign body is inhaled. In children peanut is the most common vegetable foreign body.
In adults, dentures or other foreign bodies may be inhaled especially during general
anesthesia. Foreign bodies of vegetable origin can cause severe inflammation thus,
aggravating dyspnea. Inhalation usually occur during play or fight while having something in
the mouth. In adults, foreign bodies like loose teeth or dentures may be aspirated during deep
sleep, general anesthesia or alcoholic intoxication.
There are different types of foreign bodies. These are
generally, classified into;
1. Irritating foreign bodies (vegetable foreign bodies) like betal
nut (areca nut), peanut, seed, pea, bean, wheat, etc. lead to
diffuse reaction leading to congestion and edema of the
tracheobronchial mucosa. These can cause airway obstruction
and later suppuration in the lungs.
2. Non irritating foreign bodies (non vegetable foreign bodies) like whistle, small toys,
ball bearings. These may remain symptomless for long time.
CONCEPT BOOK OF DISEASES OF EAR, NOSE & THROAT BY FAHIM AWAN
415 Part E

Foreign bodies of the larynx are not very common as most of them pass through the
larynx into tracheobronchial tree. Very large foreign bodies can get stuck in the larynx which
are usually fatal. Impaction of a large foreign body in glottis leads to the bronchospasm,
sudden complete airway obstruction and death.

Clinical features:
1. Usually there is history of oral intake of foreign body.
2. There is complaint of difficulty in breathing. There may be inspiratory stridor.
3. There may also be hoarseness of voice or even aphonia due to foreign body itself and
inflammation. Hoarseness is an important symptom to differentiate a foreign body in the
larynx from that in the tracheobronchial tree.
4. Cough is usually also present.
On examination; it is usually not possible to perform indirect laryngoscopy in children and
thus, one has to suspect the foreign body from the history. Examination of pharynx and lungs
should be carried out, however, to exclude foreign body in these regions.

Investigations:
1. Diagnosis is usually made on clinical grounds.
2. X-ray soft tissue neck may show radiopaque foreign body.
Treatment:
Whenever possible, try to get a template of the foreign body. It helps in
making the strategy and choosing proper instruments for safe removal of
foreign body. It also helps in anticipating the possible complications of the
foreign body and its removal.
Thin objects such as artificial nails may also be lodged into the larynx
which can be removed very easily. Large foreign bodies must be removed
urgently otherwise death may occur unless surgical airway is created within
Heimlich
maneuver
seconds.
In adults Heimlich maneuver also known as ‘bear hug’ is performed.
It involves sudden compression of upper abdomen to squeeze the air from
lungs, sufficient to expel the foreign body.
In infants back blow maneuver is performed. It involves sudden compression
of the chest from the posterior force while the anterior chest is held with head
down position. The child is held on its back on the adult’s knee and
pressed frequently on the upper abdomen to expel the foreign body.
Removal of the foreign body by performing direct laryngoscopy is
carried out. It is strongly recommended that tracheostomy is performed
before an attempt at removal of impacted foreign body is made, as
oxygenation of the already taxed heart is essential. Moreover, the
Back blow maneuver
CONCEPT BOOK OF DISEASES OF EAR, NOSE & THROAT BY FAHIM AWAN
416 Part E

foreign body may take longer time before it can be removed.


Antibiotics are started and the tracheostomy tube left in position till the inflammation in
the larynx has subsided.
FOREIGN BODY OF TRACHEOBRONCHIAL TREE
Clinical features:
These most commonly occur between the ages of one and three years.
1. The typical history is of a choking episode and turning blue (cyanosis) while the child
feeds or while he is playing with toys or small objects.
2. This is followed by vigorous coughing.
3. This may be followed by a relatively symptom free period as the foreign body lodges in
the lower airway. Partial obstruction of the one of the main stem bronchi causes the
characteristic wheeze on one side of the chest on auscultation. On inhalation of foreign
body, the positive pulmonary pressure compresses the main bronchi occluding the airway
around the offending object, preventing expulsion of air.

Complications:
If the foreign body remains untreated, it may give rise to following complications;
1. Bronchiectasis
2. Pneumothorax
3. Pneumonia
4. Mediastinal shift

Investigations:
X-ray chest may show radiopaque foreign body. It may also show complications like
bronchiectasis, pneumothorax, pneumonia, mediastinal shift or hyperinflation of one lung.
The hyperinflation occurs due to ball valve effect where negative intrathoracic pressure on
inspiration dilates the lumen around the foreign body.
Treatment:
1. Whenever possible, try to get a template of the foreign body. It helps in making the
strategy and choosing proper instruments for safe removal of foreign body. It also helps
in anticipating the possible complications of the foreign body and its removal.
2. Foreign body may be removed with rigid or flexible bronchoscopy.
3. In the acute situation, whilest emergency preparations for bronchoscopy are underway,
high flow oxygen is essential. Once foreign body is removed after bronchoscopy,
meticulous examination of the tracheobronchial tree must be carried out to exclude
further foreign bodies or other abnormalities.

CONCEPT BOOK OF DISEASES OF EAR, NOSE & THROAT BY FAHIM AWAN


417 Part E

Even after successful removal, atelectasis, pneumonia, retained fragments, airway spasm or
airway edema may still occur and patient must be monitored closely.
4. Systemic steroids are given after rigid bronchoscopy, to reduce airway edema.
5. Nebulization with vasoconstriction is done to reduce the incidence of postoperative
complications.
6. Thoracotomy and lobectomy/lung resection may be required if the foreign body is left
untreated for many years.
SELF ASSESSMENT
Q. 1. Foreign bodies of aerodigestive tract are associated with the highest mortality when
the site of impaction is
a. esophagus d. main stem bronchi

b. hypopharynx e. oropharynx
c. larynx
Q. 2. Inhaled small smooth foreign body is commonly arrested in
a. larynx d. subglottic space

b. left bronchus e. trachea


c. right bronchus
Q. 3. Following indicate foreign body in tracheobronchial tree in a child.
a. Choking d. Wheezing and gaging

b. Cyanosis e. All of the above


c. Vigorous coughing
Q. 4. A 03 years old boy presented with sudden acute respiratory distress, with coughing
and cyanosis, using accessory muscles of respiration. There is no rise in body temperature.
It is most probably due to
a. acute epiglottitis d. foreign body tracheobronchial tree

b. acute follicular tonsillitis e. vocal nodules


c. adenoid hypertrophy
Q. 5. All of the following indicate foreign body in tracheobronchial tree on X-ray chest
except
a. bronchiectasis d. mediastinal shift

b. canon ball opacities e. pneumonia

c. hyperinflated lung on one side


Scenario A: A 04 years old child was playing with small toys. He developed a sudden spell of
choking, cough and went blue. There is no other complaint.
1. What is the most likely diagnosis?
2. What complication can arise in this patient?
CONCEPT BOOK OF DISEASES OF EAR, NOSE & THROAT BY FAHIM AWAN
418 Part E
3. How will you treat this patient?
B: A 02 years old baby with severe dyspnea was brought by his parents stating that 02 hours
ago he was eating ground nuts and suddenly he became blue. There is no other complaint.
1. What is your diagnosis?
2. How will you investigate this case?
3. How will you manage this case?

c. A 4 years old boy is referred to ENT OPD with complaint of recurrent attacks of lower
respiratory tract infection for last 4 months. ENT examination is unremarkable. On chest
auscultation there is decreased air entry on left side.
1. What is your provisional diagnosis?
2. How will you investigate this patient?

ANSWERS-----------------------------------------
-------------------------------------------------------- ------------
Q. 1. c 2. c 3. e 4. d 5. b
Scenario A: 1. Tracheobronchial foreign body 2. Bronchiectasis, pneumothorax, pneumonia,
mediastinal shift 3. Get a template of foreign body, bronchoscopy, high flow oxygen,
corticosteroids, nebulization, thoracotomy and lung resection.
B: 1. Foreign body larynx 2. Clinical grounds, X-ray neck and chest 3. Get a template of
foreign body, high flow oxygen, bronchoscopy, nebulization, corticosteroids, thoracotomy
and lung resection.
C: 1. Tracheobronchial foreign body 2. Clinical grounds, X-ray neck and chest
--------------------------------------------------------------------------------------------------------------------------
-
VOCAL NODULES
Synonyms: Singer’s nodes/nodules, Teacher’s nodes/nodules, Hawker’s nodes/nodules,
Screamer’s nodes/nodules
Definition:
These are localized symmetrical swellings, on the edges of both vocal cords at the
junction of anterior one third and posterior two third (halfway along the membranous
cord). This is the area which vibrates maximally during speech.

Etiopathology:
These commonly occur in singers, teachers, hawkers, and screamers. Vocal nodules are
frequent cause of hoarseness in children who are naturally vocal and excitable.
These are manifestations of repeated trauma due to misuse of voice with tense vocal
cords. So, it is a hyperkinetic dysphonia. Certain aggravating factors are smoking, allergy,
infection and gastroesophageal reflux. There are following stages of vocal nodule formation:
1. Submucous transudation
2. Ingrowth of vessels
3. Fibrous organization
If the factors which have led to nodule persist for a long time, then the
nodules may become permanent.

Clinical features:
419 Part E

1. Progressive hoarseness of voice is the most common presenting symptom. In the initial
stages, hoarseness decreases during periods of vocal rest and increases during periods of
vocal misuse.
2. Pitch range is restricted.
3. On indirect laryngoscopy, there are whitish localized symmetrical swellings on the
edges of both vocal cords at the junction of anterior one third and posterior two third.

Investigations:
1. Flexible nasopharyngolaryngoscopy or 70-degree endoscopy is helpful in doubtful
cases.
2. Stroboscopy is diagnostic. A local edematous swelling of recent onset vibrates in phase
with the whole vocal cord on phonation, whereas an old and fibrous swelling can impede
the vibrations and only a part of the vocal cord vibrates.
Treatment:
1. Voice rest: Strict voice rest is advised at least for 02 to 03 weeks.
2. Speech therapy: Speech therapy is instituted to encourage the patient to avoid overuse
and abuse of voice. Laryngeal abuse such as whispering and shouting should be avoided.
3. Surgery: Surgery is reserved for those cases where voice therapy fails and patient re-
mains symptomatic. Microlaryngoscopic surgical removal or laser ablation of vocal
nod-ule is carried out under general anesthesia. Excision of nodule is carried out with no
exposure or damage to the underlying ligaments.
4. Aggravating factors such as smoking, allergy, infection and gastroesophageal reflux
should be addressed to reduce the chances of recurrence.
SELF ASSESSMENT
Q. 1. Vocal nodules are also called

a. Hawker’s nodes d. Teacher’s nodes

b. Screamer’s nodules e. all of the above

c. Singer’s nodes
Q. 2. Singer’s nodules are

a. allergic in origin the vocal cord

b. infective in origin d. premalignant lesion

c. localized submucous transudation on e. viral in origin


Q. 3. The site of singer`s nodules is

a. arytenoids d. preepiglottic space


b. epiglottis e. none of the above

c. junction of the anterior 1/3 with pos-


terior 2/3 of the vocal cords
420 Part E
Q. 4. All are true about vocal nodules except
a. appears white in color third of the cord

b. bilaterally symmetrical d. involves both the cords

c. characteristically occurs at the junction e. result from misuse of voice


of anterior two third with posterior one
Q. 5. A young singer develops intermittent hoarseness of voice for last 03 months.
The likely diagnosis is
a. carcinoma of larynx d. vocal cord hemorrhage

b. contact ulcer e. vocal nodules

c. laryngeal edema
Scenario: A young teacher presented with progressive hoarseness of voice for last six months
and she is engaged in teaching for last five years. Her voice is persistently hoarse and is not
being relieved with medication by primary health care physician.
1. What is the most likely diagnosis?
2. How will you investigate this case?
3. How will you manage this case?
-------------------------------------------------------- ANSWERS-----------------------------------------------------
Q.1. e 2. c 3. c 4. c 5.e
Scenario: 1. Vocal cord nodule 2. Flexible nasopharyngoscopy, stroboscopy 3. Voice rest,
speech therapy, surgery.
--------------------------------------------------------------------------------------------------------------------------
-
VOCAL FOLD EDEMA AND LARYNGEAL POLYP
Synonyms: Reinke’s edema
Definition:
It is defined as an accumulation of fluid in the subepithelial Reinke’s space of vocal cords.

Etiopathology:
It is symmetrical swelling of whole membranous part of both vocal cords.
It originates from misuse of voice with relaxed vocal cords. So, it is a hypokinetic
dysphonia. Physical trauma (voice abuse, chronic cough), chemical trauma
(laryngopharyngeal reflux, smoking and alcohol), infection and inflammation are thought to
be responsible. Slight edema resolves with time, whereas excessive edema may change into
vocal polyp.
If the accumulation and inflammation is concentrated at one point, the epithelium
balloons out, this is known as vocal fold polyp (also known as laryngeal polyp). Polyp can
occur around the whole membranous part of vocal cord but it is more common near anterior
commissure.
CONCEPT BOOK OF DISEASES OF EAR, NOSE & THROAT BY FAHIM AWAN
421 Part E

Vocal fold polyp is a pedunculated or sessile lesion, often unilateral, that arises from the
free edge of the vocal cord.
Clinical features:
1. Voice is usually harsh.
2. Falsetto voice becomes almost
impossible. On indirect laryngoscopic
examination;
Endoscopic view of
1. Vocal cords are grossly edematous in the initial stages.
left vocal polyp
2. Smooth polyp is visible near anterior commissure in later stages.
The polyp moves up and down during respiration.
Investigations:
Flexible nasopharyngoscopy/70-degree endoscopy/ stroboscopy is diagnostic in doubtful
cases.
Treatment:
1. Strict voice rest is advised for at least 48 hours. Laryngeal abuse such as whispering,
shouting and smoking should be avoided.
2. Speech therapy is directed to encourage the patients to avoid overuse and abuse of
voice.
3. Microlaryngoscopic removal of vocal fold polyp is performed. Swollen mucosa is
excised surgically with preservation of underlying vocal ligament. Excised tissue is sent
for histopathology.
Spontaneous cure may occur rarely in early stages by rupture of the polyp.
SELF ASSESSMENT
Q. 1. Reinke’s space is found in the

a. epiglottis d. true vocal cord

b. false cord e. ventricle of larynx

c. saccule of larynx
Q. 2. Reinke’s space is found in the

a. mucosal space d. subepithelial space

b. muscular space e. submucosal space


c. preepiglottic space
Q. 3. Which of the following is usually associated with misuse of voice?
a. Contact ulcer d. Quincke's edema
b. Cyst of vocal cord e. None of the above

c. Reinke’s edema
Q. 4. Vocal polyp is usually found on

CONCEPT BOOK OF DISEASES OF EAR, NOSE & THROAT BY FAHIM AWAN


422 Part E

a. anterior commissure 2/3rd of vocal cord


b. junction of anterior 1/2 and posterior 1/2 d. junction of anterior 2/3rd and posterior
of vocal cord 1/3rd of vocal cord
e. posterior commissure
c. junction of anterior 1/3rd and posterior
Scenario: A 30 years old male broker of fruit market presented with progressive hoarseness
of voice for last three months. Indirect laryngoscopic examination shows smooth polypoidal
lesion on anterior commissure which moves up and down during respiration.
1. What is the most likely diagnosis?
2. How will you investigate this case?
3. How will you manage this case?
-------------------------------------------------------- ANSWERS-----------------------------------------------------
Q.1. d 2. d 3. c 4.a
Scenario: 1. Vocal fold edema 2. Flexible nasopharyngoscopy 3. Strict voice rest, speech
therapy, microlaryngoscopic removal.
--------------------------------------------------------------------------------------------------------------------------
STRIDOR
Definition:
It is a defined as a harsh, high pitch breath sound produced at or below the level of
larynx.

Etiopathology:
It is due to narrowed air passages at or below the level of larynx. There is turbulence of air
flowing through it. Laryngeal stridor is usually inspiratory, whereas bronchial stridor is
usually expiratory.
It is a heralding symptom as well as sign which warrants immediate restoration of
airway. At the same time search should be made to find out the cause. Cause of stridor is
identified by history, clinical examination and investigations like radiology (e.g., X-ray neck
and chest, CT), endoscopy etc.

Causes:
Causes of stridor are classified into;
A. Congenital:
B. Acquired:
1. Traumatic
2. Inflammatory
3. Neoplastic
4. Paralytic
CONCEPT BOOK OF DISEASES OF EAR, NOSE & THROAT BY FAHIM AWAN
423 Part E

5. Granulomatous
6. Spasmodic
7. Mechanical
Different diseases causing stridor are;
A. Congenital:
1. Laryngomalacia is the most common cause of congenital laryngeal stridor.
2. Laryngeal web
3. Subglottic stenosis
4. Laryngeal cyst
5. Vocal cord paralysis
6. Hemangioma
7. Lymphangioma
B. Acquired:
a. Pyrexial:
1. Acute epiglottitis
2. Laryngotracheobronchitis
3. Diphtheria
b. Apyrexial:
1. Trauma to larynx
2. Tumors of larynx
3. Foreign body larynx
4. Recurrent respiratory papillomatosis
5. Scald
SELF ASSESSMENT
Q. 1. Laryngeal stridor is usually

a. expiratory d. all of the above

b. inspiratory e. none of the above

c. inspiratory as well as expiratory


Q. 2. Bronchial stridor is usually

a. expiratory d. all of the above

b. inspiratory e. none of the above


c. inspiratory as well as expiratory

CONCEPT BOOK OF DISEASES OF EAR, NOSE & THROAT BY FAHIM AWAN


424 Part E
Q. 3. All are true about stridor except
a. caused by narrowing at the level of c. harsh sound
larynx d. high pitch sound
b. caused by narrowing below the level of e. low pitch sound
larynx
Q. 4. All are causes of congenital stridor except
a. laryngeal cyst d. laryngotracheobronchitis

b. laryngomalacia e. subglottic stenosis


c. laryngeal web
Q. 5. Following is cause of pyrexial stridor
a. acute epiglottitis d. all of the above

b. diphtheria e. none of the above


c. laryngotracheobronchitis
Q. 6. All are causes of apyrexial stridor except
a. foreign body of larynx d. trauma to larynx

b. laryngotracheobronchitis e. umors to larynx

c. recurrent respiratory papillomatosis


Scenario: A 02 years old child presented in pediatric emergency room with fever and severe
inspiratory stridor for the last 24 hours in the month of December.
1. Give 03 likely causes of stridor?
2. Give 03 ways of securing his airway?
3. Give radiological investigations with positive findings in two diagnoses?
-------------------------------------------------------- ANSWERS-----------------------------------------------------
Q.1. b 2. a 3. e 4. d 5. d 6. b
Scenario: 1. Acute epiglottitis, laryngotracheobronchitis, diphtheria 2. Oropharyngeal
airway, laryngeal mask, minitracheostomy, tracheostomy 3. X-ray soft tissue neck lateral
view shows thumb sign in acute epiglottitis. X-ray soft tissue neck A.P view shows steeple
sign in laryngotracheobronchitis.
--------------------------------------------------------------------------------------------------------------------------
-
STERTOR
Definition:
It is defined as a low pitch noisy respiration caused by partial airway obstruction above
the level of larynx produced when the patient is awake. Usual sites of obstruction include
nose, nasopharynx, oropharynx or hypopharynx. The noise is produced by turbulent airflow
through the narrowed airways, causing vibration of surrounding tissues.
CONCEPT BOOK OF DISEASES OF EAR, NOSE & THROAT BY FAHIM AWAN
425 Part E

Snoring is disturbing harsh sound during sleep. It occurs when air flows through relaxed
tissues in the throat, causing the tissues to vibrate during breathing.
Causes:
A. Congenital:
1. Nasal obstruction e.g., choanal stenosis, congenital cysts of
nose, meningoencephalocele, encephalolaryngocele.
2. Facial skeletal anomalies e.g., Treacher Collins syndrome, Pierre Robin
syndrome, cleft palate.
3. Macroglossia e.g., Down syndrome, hemangioma or lymphangioma of tongue.
4. Pharyngeal swelling e.g., lingual thyroid, Thornwaldt cyst (bursa), chordoma,
craniopharyngioma.
B. Acquired:
1. Traumatic e.g., septal hematoma, septal abscess, facial fractures.
2. Inflammatory e.g., rhinitis, adenoiditis, tonsillitis
3. Neoplastic e.g., angiofibroma, nasopharyngeal carcinoma, lymphoma
4. Iatrogenic e.g., rhinitis medicamentosa, repair of cleft palate, anterior and posterior
nasal packing.
5. Miscellaneous e.g., cystic fibrosis, nasal polyps, antrochoanal polyp, DNS.
SELF ASSESSMENT
Q. 1. All are true about stertor except
a. cause by airway obstruction above the d. noise produced by turbulent airflow
level of larynx e. noise produced when the patient is
b. high pitch noise awake
c. low pitch noise
Q. 2. All are causes of congenital stertor except
a. choanal stenosis d. lingual thyroid

b. cystic fibrosis e. macroglossia


c. facial skeletal abnormality
Q. 3. All are causes of acquired stertor except
a. adenoiditis d. meningocele

b. angiofibroma e. tonsillitis

c. laryngomalacia
-------------------------------------------------------- ANSWERS-----------------------------------------------------
Q.1. b 2. b 3. c
--------------------------------------------------------------------------------------------------------------------------
CONCEPT BOOK OF DISEASES OF EAR, NOSE & THROAT BY FAHIM AWAN
426 Part E

VOCAL CORD PARALYSIS


Recurrent laryngeal nerve (RLN):
Right recurrent laryngeal nerve arises from the vagus
nerve in front of first part of subclavian artery, winds
backwards around the vessel and ascends to the side of trachea.
Right and left recurrent
Left recurrent laryngeal nerve arises from vagus in the laryngeal nerve
mediastinum at the level of arch of aorta, loops around it and thus,
ascends into the neck in the Tracheoesophageal groove. Thus, left recurrent laryngeal nerve
has a much longer course which makes it
prone to paralysis compared to right
recurrent laryngeal nerve.

Superior laryngeal nerve (SLN):


It arises from inferior ganglion of the vagus,
descends behind internal carotid artery and
at the level of greater cornu of hyoid bone
divides into external and internal branches.
The external branch supplies cricothyroid
muscle while the internal branch pierces the
thyrohyoid membrane and supplies sensory
innervation to the larynx above the vocal
cord and also hypopharynx.

Nerve supply of larynx: Vocal cords positioning in different paralysis

Vocal cords form a watershed for the blood and nerve


supply of larynx.
1. Sensory supply of larynx:
The larynx gets its sensory innervation by;
a. Internal laryngeal nerve (a branch of superior laryngeal nerve) above the level of
vocal cord.
b. Recurrent laryngeal nerve below the level of vocal cord.
2. Motor supply of larynx:
a. All the muscles of vocal cords (abductors, adductors and tensors) except cricothyroid
are supplied by recurrent laryngeal nerve.
b. Cricothyroid is supplied by external laryngeal nerve (a branch of superior laryngeal
nerve).
Etiology of vocal cord paralysis:
Vocal cord paralysis is often a manifestation of some disease and not a diagnosis itself.
Usually there are following causes of vocal cord paralysis;
1. Malignant diseases e.g., bronchogenic carcinoma is the most common cause of left
recurrent laryngeal paralysis.
2. Inflammatory diseases e.g., pulmonary tuberculosis.
3. Surgical trauma e.g., procedure on the thyroid, lungs, heart, oropharynx and
mediastinum.

CONCEPT BOOK OF DISEASES OF EAR, NOSE & THROAT BY FAHIM AWAN


427 Part E

4. Non-surgical trauma e.g., enlargement of the left atrium, aortic aneurysm or neck
trauma.
5. Neurological disorders e.g., cerebrovascular accidents, multiple sclerosis, alcoholic
and diabetic neuropathy.
Clinical features:
A. Superior laryngeal nerve paralysis:
It leads to paralysis of cricothyroid muscle. Its paralysis is
clinically unrecognized. In superior laryngeal nerve paralysis,
there is:
1. Bowing of the vocal cord Endoscopic view of unilateral
vocal cord paralysis
2. Vocal cord appears wavy as it is deprived of its tensor
cricothyroid.
3. Flapping of paralyzed vocal cord during phonation.
4. Overhanging epiglottis in cases of bilateral paralysis.
5. The voice fails to gain its strength
B. Recurrent laryngeal nerve paralysis:
It leads to paralysis of all the intrinsic muscles of larynx except cricothyroid. It causes the
vocal cord to lie in the paramedian position.
Wegner and Grossman theory:
It states that an immobile cord in the paramedian position has a total pure unilateral recurrent
laryngeal nerve paralysis, whereas an immobile cord in the lateral position has a combined
paralysis of superior and recurrent laryngeal nerve (because the adductive action of the
cricothyroid is lost).
Semon’s law is obsolete now and is of historical interest only.
Investigations:
Exact investigations depend on clinical findings.
1. Radiological: X-ray chest, X-ray base of skull and barium swallow. C.T scan of the
brain, base of skull, neck and chest.
2. Stroboscopy: It is done to assess the structure and fine movements of vocal cords.
3. Endoscopies: (oropharyngoscopy, hypopharyngoscopy, laryngoscopy,
tracheobronchoscopy, esophagoscopy and nasopharyngoscopy) under general
anaesthesia with biopsy of suspected sites.
4. Hematological:
Complete blood count, ESR, tests for diabetes, tuberculosis and syphilis.

CONCEPT BOOK OF DISEASES OF EAR, NOSE & THROAT BY FAHIM AWAN


428 Part E

Treatment:
When it is possible to treat the cause, it should be treated as a first priority. Then wait at
least 06 months for spontaneous recovery of the paralyzed vocal cord or compensation by the
opposite healthy vocal cord. When recovery is not expected or 06 months have passed after
the paralysis then the treatment of vocal cord paralysis depends on;
1. Etiology
2. Defect it causes
3. Profession of the patient
4. General state of the patient

A. Unilateral abductor paralysis:


Affected vocal cord is in paramedian position and the patient presents with hoarseness of
voice. Treatment is;
1. Speech therapy
2. Autologous collagen or fat injection of the vocal cord to push it medially.
B. Unilateral adductor paralysis:
Affected vocal cord is in lateral (cadaveric) position and the patient presents with husky
voice. There is also complaint of aspiration. Treatment is;
1. Speech therapy
2. Medialization of vocal cord in the form of type I thyroplasty.
C. Bilateral abductor paralysis:
Both the vocal cords are in the paramedian position and the patient presents with severe
respiratory distress and stridor.
1. Immediate treatment is by endotracheal intubation or tracheostomy with a
speaking valve.
2. Definitive treatment is lateralization of vocal cords in the form of type II
thyroplasty.
3. CO2 laser cordectomy may be performed endoscopically.
4. Arytenoidectomy may be performed through external approach or endoscopically.
5. Innervation procedure such as anastomosis of phrenic nerve to the recurrent
laryngeal nerve.
D. Bilateral adductor paralysis:
Both the vocal cords are in the lateral position. The patient presents with aphonia, unable to
cough and life-threatening aspiration. Usually, it is due to psychiatric disturbances.

CONCEPT BOOK OF DISEASES OF EAR, NOSE & THROAT BY FAHIM AWAN


429 Part E

If it is due to an organic lesion, treatment is;


1. Tracheostomy with a cuffed tube (to prevent aspiration) and a nasogastric tube.
2. Epiglottopexy is another solution.
3. Total laryngectomy is the only sure way to protect the lungs from aspiration if there
is no possibility of neurological recovery.
Differnt syndromes with vocal cord paralysis:
1. Vernet’s syndrome: Paralysis of 9th, 10th and 11th cranial nerves is called Vernet’s
syndrome.
2. Hugling Jackson syndrome: Paralysis of 10th, 11th and 12th cranial nerves is called
Hugling Jackson syndrome.
3. Collet Sicard syndrome: Paralysis of 9th, 10th, 11th, and 12th cranial nerves is called
Collet Sicard syndrome.
4. Villaret (posterior retropharyngeal) syndrome: Paralysis of 9th, 10th, 11th, 12th
cranial nerves plus Horner’s syndrome are collectively called Villaret’s syndrome.
5. Schmidt syndrome: Paralysis of 10th and 11th cranial nerves is called Schmidt
syndrome.
SELF ASSESSMENT
Q. 1. All the muscles of larynx are supplied by recurrent laryngeal nerve except
a. cricothyroid d. thyroarytenoid

b. interarytenoid e. vocalis
c. oblique arytenoid
Q. 2. The most common cause of left recurrent laryngeal nerve paralysis is
a. bronchogenic carcinoma d. nasopharyngeal carcinoma

b. esophageal carcinoma e. thyroid surgery


c. laryngeal carcinoma
Q. 3. Left recurrent laryngeal nerve palsy could be due to all except
a. aoritc aneurysm d. subclavian aneurysm

b. mediastinal tumors e. surgery of patent ductus arteriosus


c. mitral stenosis
Q. 4. Right recurrent laryngeal nerve paralysis could be due to all except
a. aortic aneurysm d. carcinoma of oropharynx

b. carcinoma of apex of lung e. thyroid surgery


c. carcinoma of esophagus
Q. 5. Paralysis of vagus nerve at jugular foramen will place the affected cord in
CONCEPT BOOK OF DISEASES OF EAR, NOSE & THROAT BY FAHIM AWAN
430 Part E

a. cadaveric position d. paramedian position

b. full adduction e. None of the above


c. median position
Q. 6. A 10 years old boy had hoarseness of voice secondary to an attack of diphthe-ria and
presented one month after the attack. On examination his right vocal cord was paralyzed
in paramedian position. The management of choice for paralyzed vocal cord will be
a. collagen/fat injection of right vocal d. thyroplasty type II
cord e. to wait for spontaneous recovery of
b. gel foam injection of right vocal cord vocal cord
c. thyroplasty type I
Q. 7. All are true about unilateral abductor paralysis except
a. affected vocal cord is in paramedian c. treatment is with autologous collagen or
position fat injection
b. patient presents with hoarseness of d. treatment is with speech therapy
voice e. treatment is with tracheostomy
Q. 8. All are true about unilateral adductor paralysis except
a. affected vocal cord is in lateral c. treatment is with speech therapy
(cadaveric) position d. treatment is with thyroplasty type 1
b. patient presents with husky voice e. treatment is with thyroplasty type 11
Q. 9. All are true about bilateral abductor paralysis except
a. affected vocal cords are in paramedian cordectomy
position d. treatment is with endotracheal intu-
b. patient presents with dyspnea and bation or tracheostomy
stridor e. treatment is with collagen/fat injection of
c. treatment is with arytenoidectomy and vocal cord
Q. 10. All are true about bilateral adductor paralysis except
a. affected vocal cords are in lateral or fat injection
position d. treatment is by tracheostomy with a
b. patient presents with aphonia, cough cuffed tube
and life-threatening aspiration
e. treatment is with epiglottopexy or
c. treatment is with autologous collagen total laryngectomy
Q. 11. A patient presented with stridor and dyspnea which developed after an at-tack of
upper respiratory tract infection 06 months ago. On examination he was found to have a
03 mm glottis opening with bilateral cords in paramedian position. All of the following are
used in the management except
a. arytenoidectomy b. cordectomy
CONCEPT BOOK OF DISEASES OF EAR, NOSE & THROAT BY FAHIM AWAN
431 Part E

c. collagen/fat injection e. tracheostomy


d. innervation procedure
Q. 12. Horner’s syndrome consists of all of the following features except
a. anhydrosis d. loss of ciliospinal reflex

b. dilated pupil e. ptosis


c. endophthalmos
Q. 13. The cranial nerves involved in jugular foramen syndrome or Vernet’s syndrome are
a. IX and X d. X, XI and XII

b. X and XI1 e. IX, X, XI and XII


c. IX, X and XI
Q. 14. The cranial nerve involved in Schmidt syndrome are
a. 9th and 10th d. 10th, 11th and 12th

b. 10th and 11th e. 9th and 11th


c. 9th, 10th and 11th
Q. 15. The cranial nerves involved in Villaret’s syndrome are
a. IX and X d. X and XI1

b. IX, X and XI e. none of the above


c. IX, X, XI and XII
Q. 16. Posterior retropharyngeal syndrome is otherwise known as
a. Schmidt syndrome d. Weber syndrome

b. Vernet’s syndrome e. none of the above


c. Villaret’s syndrome
Q. 17. Horner’s syndrome may be associated with
a. Collet Sicard syndrome d. Vernet’s syndrome

b. Hugling Jackson’s syndrome e. Villaret’s syndrome

c. Schmidt syndrome
Scenario: A 10 years old child developed hoarseness of voice following an attack of
diphtheria 02 months ago. On laryngeal examination right vocal cord is seen in paramedian
position.
1. What is the most probable clinical diagnosis?
2. What investigations will you carry out?
3. How will you manage this patient?
--------------------------------------------------- ANSWERS------------------------------------------------
----- -----
Q.1. a 2. a 3. d 4. a 5. a 6. e
7. e 8. e 9. e 10. c 11. c 12. b

CONCEPT BOOK OF DISEASES OF EAR, NOSE & THROAT BY FAHIM AWAN


432 Part E
13. c 14. b 15. c 16. c 17. e
Scenario: 1. Right vocal cord paralysis due to neurological complication of diphtheria
2. Hematological, radiological, stroboscopy, flexible larygoscopy 3. Watch and wait atleast
for 06 months for spontaneous recovery of the diseased cord or compensation by the
contralateral vocal cord.
--------------------------------------------------------------------------------------------------------------------------
RECURRENT RESPIRATORY PAPILLOMATOSIS (RRP)
Synonym: Juvenile laryngeal papillomatosis
Etiopathology:
There is infection of the respiratory epithelium with Human
Papilloma Virus (HPV) type 06 and 11. Frequently the lesions are
multiple. These occur in clusters. The papillomata may be sessile and Endoscopic view of recurrent
spread over a wide area of mucosa or these may be pedunculated and respiratory papillomatosis
localized.
There is tendency for papillomas to occur at anterior part of the glottis. There may also
be involvement of the mucosa of upper air and food passages. The disease may occur at
mucocutaneous junction of nasal cavities, gingiva, lips, soft palate, tonsillar pillars, larynx,
tracheobronchial tree, oropharyngeal inlet and tracheostome. The lesion has a tendency to
involve points of airway constriction. Histologically papillomas are composed of benign
squamous epithelium.

The natural history of recurrent respiratory papillomatosis:


1. Transmission: In some patients the disease is transmitted at the time of delivery from a
mother infected with genital warts. The disease has not been recorded in siblings which
shows that its transmission rate is very low.
2. Recurrence: The disease is notoriously recurrent. The recurrence often occurs over the
apparently normal mucosa adjacent to original lesion.
3. Remission: Remission in this disease is highly uncertain. Duration of remission may
vary from 02 years to life long. It can occur at any age at any time. It has been observed
that remission is greatest if the disease presents between the age of 06 and 10 years.
Clinical feature:
Majority of patients with recurrent respiratory papillomatosis present before the age of four
years. There is complaint of hoarseness of voice, abnormal cry or acute respiratory
obstruction in the form of stridor. On examination there are multiple small warty lesions on
larynx with a tendency to occur at anterior part of the glottis. These usually occur in clusters.
Treatment:
The aim of treatment is to;
CONCEPT BOOK OF DISEASES OF EAR, NOSE & THROAT BY FAHIM AWAN
433 Part E

1. Preserve the airway


2. Preserve the voice
3. Avoid tracheostomy
1. Surgery: The only satisfactory treatment for the condition is endoscopic removal that
is done with microdebrider or with carbon dioxide laser.
2. Adjuvant medical treatment: Local treatment includes mitomycin, cidofovir or alpha
interferon.
3. Photodynamic therapy is effective in limiting the disease.
SELF ASSESSMENT
Q. 1. Recurrent respiratory papillomatosis is also called
a. juvenile laryngeal papillomatosis d. both a & b

b. solitary papilloma e. both b & c


c. viral papilloma
Q. 2. Recurrent respiratory papillomatosis is caused by
a. adenovirus d. HIV

b. cytomegalovirus e. Human Papilloma Virus (HPV)


c. E.B. virus
Q. 3. Recurrent respiratory papillomatosis in children is the result of
a. bacterial infection d. nutritional deficiency

b. chronic laryngitis e. viral infection


c. excessive shouting
Q. 4. There is tendency for recurrent respiratory papillomatosis to occur in
a. anterior part of glottis d. supraglottis

b. posterior part of glottis e. tracheobronchial tree


c. subglottis
Q. 5. All are true about recurrent respiratory papillomatosis except
a. frequently multiple d. occurs in clusters
b. involves mucosa of upper air and e. transmitted at the time of delivery
food passages from mother with genital wart
c. involves points of dilated airway
Q. 6. Usual clinical features of recurrent respiratory papillomatosis
a. abnormal cry d. stridor

b. hoarseness of voice e. all of the above

c. respiratory obstruction
CONCEPT BOOK OF DISEASES OF EAR, NOSE & THROAT BY FAHIM AWAN
434 Part E
Q. 7. All are true about recurrent respiratory papillomatosis except
a. can undergo malignant change with chial tree
repeated removal d. notoriously recurrent
b. can undergo spontaneous remission e. viral in origin
c. implanted elsewhere in tracheobron-
Q. 8. Aim of treatment in recurrent respiratory papillomatosis is
a. avoidance of tracheostomy d. all of the above

b. to preserve the airway e. none of the above


c. to preserve the voice
Q. 9. A 03 years old male child presents with recurrent respiratory papillomatosis. He has
got hoarse voice and slight airway distress for last 01 week. Papillomas are involving the
glottis. The best treatment is
a. antibiotics d. steroids

b. interferon therapy e. tracheostomy and observation


c. microdebrider excision
Q. 10. All are included in the treatment of recurrent respiratory papillomatosis except
a. cidofovir d. laser surgery

b. cryosurgery e. mitomycin
c. interferon
Q. 11. The best laser for recurrent respiratory papillomatosis is
a. argon laser d. KTP-532 laser

b. CO2 laser e. Nd: YAG laser


c. helium-neon laser
Q. 12. In recurrent respiratory papillomatosis, which method of treatment prevents
recurrence?
a. CO2 laser d. Microdebrider excision

b. Interferon e. None of the above


c. Local injection of cidofovir
Q. 13. A 04 years old child has hoarseness of voice with occasional slight stridor for last 02
months. Laryngeal examination shows multiple warty lesions involving true and false
cords. What is treatment of this disease?
a. Antibiotics d. Tracheostomy

b. Laser surgery e. Wait and see

c. Radiotherapy
Scenario: A mother comes to out patient clinic with her 02 years old boy, born through
CONCEPT BOOK OF DISEASES OF EAR, NOSE & THROAT BY FAHIM AWAN
435 Part E

normal vaginal delivery, suffering from progressively increasing stridor and abnormal cry
for last 06 months. There is no fever. Endoscopic examination shows multiple small warty
lesions on anterior part of the glottis.
1. What is the most likely diagnosis?
2. What are objectives of treatment?
3. What is the treatment plan?
------------------------------------------- ANSWERS------------------------------------
------------- -----------------
Q.1. a 2. e 3. e 4. a 5. c 6. e 7. a
8. d 9. c 10. b 11. b 12. e 13. b
Scenario: 1. Juvenile laryngeal papillomatosis 2. Preserve the airway, voice and avoid
tracheostomy 3. Endoscopic removal with microdebrider or with carbon dioxide laser,
adjuvant medical treatment with mitomycin, cidofovir or alpha interferon.
--------------------------------------------------------------------------------------------------------------------------
DYSPHONIA PLICA VENTRICULARIS
Synonyms: False cord voice, Ventricular phonation, Diplophonia, Ventricular
dysphonia
Definition:
It is defined as phonation with false cords instead of true cords.
Etiopathology:
It may be;
1. Functional or psychogenic; It occurs in normal larynx. It is Dysphonia plica
associated with psychogenic disorders or in extreme vocal cords ventricularis
strains.
2. Organic; It occurs in certain pathologies of larynx. It occurs after surgery of vocal
cords, vocal cord paralysis, myasthenia gravis or neoplasms.
3. Idiopathic.
Clinical feature:
Voice is rough, low pitched and unpleasant. It is almost like a duet.
Indirect laryngoscopy or stroboscopy shows false cords meeting in midline on phonation
inhibiting the view of true cords.
Treatment:
1. Underlying cause is addressed.
2. Speech therapy.
3. Psychotherapy is helpful in functional cases.
4. Botulinum toxin injection into false cords may be considered in resistant cases.
CONCEPT BOOK OF DISEASES OF EAR, NOSE & THROAT BY FAHIM AWAN
436 Part E

SELF ASSESSMENT
Q. 1. Dysphonia plica ventricularis is also known as
a. diplophonia d. ventricular phonation

b. false cord voice e. all of the above


c. ventricular dysphonia
Q. 2. Cause of dysphonia plica ventricularis is
a. extreme vocal cords strain d. pathologies of larynx

b. functional e. all of the above


c. idiopathic
Q. 3. Voice produced by dysphonia plica venticularis is characterized by
a. false cords producing voice d. vocal cords are in midline
b. nasal twang in the voice e. vocal cords are in paramedian posi-tion
c. vocal cords are in cadaveric position
Q. 4. All are true about dysphonia plica ventricularis except
a. it is phonation with false cords d. treatment is with microlaryngeal
b. on indirect laryngoscopy, false cords surgery
meet in the midline on phonation e. voice is rough, low pitch and
c. on indirect laryngoscopy, true cords unpleasant almost like a duet
are not visible on phonation
Q. 5. All are true about treatment of dysphonia plica ventricularis except
a. addresses the underlying cause d. psychotherapy

b. botulinum toxin e. speech therapy

c. microlaryngeal surgery
Scenario: A 40 years old patient presented in ENT OPD soon after discharge from
psychiatry department (for the treatment of anxiety neurosis) with complaint of unpleasant
voice production for last 1 year. On examination false cords are seen meeting in midline on
phonation.
1. What is the most probable clinical diagnosis?
2. What investigations will you carry out?
3. How will you manage this patient?
------------------------------------------------------ ANSWERS--------------------------------------------------
-- ---
Q.1. e 2. e 3. a 4. d 5.c
Scenario: 1. Dysphonia plica ventricularis 2. Flexible laryngoscopy 3. Treatment of
underlying psychiatric illness, speech therapy, botulinum toxins.
--------------------------------------------------------------------------------------------------------------------------
PUBERPHONIA
CONCEPT BOOK OF DISEASES OF EAR, NOSE & THROAT BY FAHIM AWAN
437 Part E

Synonyms: Pubophonia, Incomplete vocal mutation, Functional falsetto voice, Mu-


tational falsetto voice
Definition:
Failure of the voice to change from higher to lower pitch at puberty is called puberphonia.
Etiopathology:
Normally voice of children has a higher pitch. In male when the larynx grows at puberty,
vocal cords increase in length and voice changes to that of lower pitch. In male voice changes
at the start of adolescence and this continues over a period of six months. When the duration
of these changes lasts for more than one year, it is termed as puberphonia. It is seen in boys
who are emotionally immature. Psychologically they shun to accept male identification.
Spasm of cricothyroid muscle is responsible for puberphonia.

Clinical features:
1. Pitch breaks at puberty may be very severe and frequent.
2. There is attempt to control this by “holding on” to the childhood voice. The production
of adulthood voice may be inhibited.
3. Indirect laryngoscopy shows that the glottis is oval or elliptical. Sometimes vocal
process partially closes to produce a triangle posteriorly known as mutational triangle.

Treatment:
1. Training in relaxing the vocal cord i.e., manipulating the position of larynx by pressing
Adam’s apple backward and downward (Gutzmann’s pressure) while the patient
phonates. This maneuver relaxes the tense vocal cords and helps to lower the pitch.
2. Psychological counselling is very helpful.
SELF ASSESSMENT
Q. 1. Puberphonia is also known as

a. functional falsetto voice d. pubophonia

b. incomplete vocal mutation e. all of the above

c. mutational falsetto voice


Q. 2. Puberphonia is commonly found in

a. adolescent female d. elderly female

b. adolescent male e. old male


c. children
Q. 3. Causative factors of puberphonia include
a. difficulty with acceptance of adult- hood
CONCEPT BOOK OF DISEASES OF EAR, NOSE & THROAT BY FAHIM AWAN
438 Part E

b. difficulty with male identification d. spasm of crico thyroid muscle

c. psychological e. all of the above


Q. 4. All are true about puberphonia except
a. failure of voice to change from higher to pitch
lower pitch d. treatment is psychological counselling
b. failure of voice to change from lower to e. treatment is training in relaxing the
higher pitch vocal cords
c. Gutzmann pressure helps to lower the
Scenario: A 16 years old boy having six sisters and no brother is brought by mother.
Mother complains that the boy has not developed male type of voice. He is very shy.
Endoscopic examination shows mutational triangle of the glottis.
1. What is the most likely diagnosis?
2. What is the treatment plan?
ANSWERS-----------------------
-------------------------------------------------------- ------------------------------
Q.1. e 2. b 3. e 4.b
Scenario: 1. Puberphonia 2. Training in relaxing the vocal cord, psychological counselling.
--------------------------------------------------------------------------------------------------------------------------
FUNCTIONAL APHONIA
Synonyms: Hysterical aphonia, Functional paralysis of adductors
Etiopathology:
It is functional paralysis of (bilateral) adductors during phonation. It most often occurs in
emotionally unstable individuals, particularly young girls.
Clinical features:
1. Aphonia is usually sudden and complete. Often the voice is reduced to whisper. The
complaint of aphonia is intermittent. The onset and recovery are usually sudden and
associated with periods of mental stress. There is no other laryngeal symptom.
2. Ability to have an explosive coughing is diagnostic of functional aphonia.
On indirect laryngoscopy, vocal cords fail to adduct on phonation but these adduct on
explosive coughing. This indicates normal adductor function.
Treatment:
1. Strong reassurance must be given.
2. Psychotherapy should be instituted. Help of psychiatrist is usually needed.
3. Speech therapy can be of great help.
SELF ASSESSMENT
CONCEPT BOOK OF DISEASES OF EAR, NOSE & THROAT BY FAHIM AWAN
439 Part E
Q. 1. Functional aphonia is also known as
a. functional paralysis of adductors d. both a & b

b. hysterical aphonia e. both a & c


c. puberphonia
Q. 2. All are true about functional aphonia except
a. intermittent in nature d. occurs in emotionally unstable indi-
b. more common in young boys viduals
c. more common in young girls e. sudden in onset
Q. 3. All are features of functional aphonia except
a. abducted position of vocal cords d. failure of vocal cords to approximate on
b. associated with periods of mental phonation
stress e. failure of vocal cords to approximate on
c. failure of vocal cords to approximate on deep respiration
coughing
Q. 4. Functional aphonia is diagnosed in case of
a. bilateral abductor palsy d. unilateral adductor palsy

b. bilateral adductor palsy e. none of the above


c. unilateral abductor palsy
Q. 5. Treatment of functional aphonia is
a. psychotherapy d. all of the above

d. reassurance e. none of the above

c. speech therapy
Scenario: An 18 years old girl presented in ENT OPD with complaint of sudden loss of voice
for last 01 week following a quarrel at home. Coughing is normal. On indirect laryngeal
examination vocal cords fail to adduct on phonation.
1. What is the most probable clinical diagnosis?
2. How will you manage this patient?
ANSWERS---------------------------------------
-------------------------------------------------------- --------------
Q.1. d 2. b 3. c 4. b 5. d
Scenario: 1. Functional aphonia 2. Strong reassurance, speech therapy, psychotherapy
--------------------------------------------------------------------------------------------------------------------------
LARYNGOCELE
Definition:
It is defined as an air-filled sac due to dilatation of laryngeal saccule and its extension
above the level of upper border of thyroid cartilage.
CONCEPT BOOK OF DISEASES OF EAR, NOSE & THROAT BY FAHIM AWAN
440 Part E

Etiopathology:
Laryngocele is thought to arise from prolonged increase in transglottic pressure such as glass
blowers, weight lifters and trumpet players. It may be classified as;
1. External laryngocele: In it distended saccule herniates through the thyrohyoid
membrane and presents as swelling in the neck. It constitutes 30% of the laryngocele.
2. Internal laryngocele: It is confined within
larynx and presents as a distension of false vocal
cords and aryepiglottic folds. It constitutes 20% of
laryngocele.
3. Combined/mixed laryngocele: In it both the internal and external components are seen.
It constitutes 50% of laryngocele.
Clinical features:
1. Hoarseness of voice is the most common presentation.
2. In case of internal laryngocele, there may also be complaint of
airway obstruction and stridor. On indirect laryngoscopy there is
distension of false cords and aryepiglottic folds.
3. In case of external laryngocele, there may also be reducible
swelling in neck over thyrohyoid membrane which increases in
size on coughing and performing Valsalva maneuver.

A case of Rt.
external
Laryngoce
Investigations:
1. Plain X-ray neck (AP view) with or without Valsalva maneuver is usually diagnostic
showing an air-filled sac.
2. CT scan/ MRI is carried out to find out extent of laryngocele.
3. Direct laryngoscopy must be undertaken in cases of unilateral laryngocele to rule out
carcinoma of laryngeal ventricle. Because carcinoma can act as valve allowing air into
but not out of saccule thus, allowing formation of laryngocele.
Treatment:
1. No treatment is required if laryngocele is discovered incidentally and is limited within
the larynx. Patient is kept under annual review.
2. Excision of external laryngocele is done through external neck incision.
3. Marsupialization of internal laryngocele is done through laryngoscopy.
SELF ASSESSMENT
Q. 1. Laryngocele arises from

a. aryepiglottic fold b. laryngeal saccule

CONCEPT BOOK OF DISEASES OF EAR, NOSE & THROAT BY FAHIM AWAN


441 Part F Esophagus

c. paraglottic space
e. vallecula
d. preepiglottic space
Q. 2. The laryngocele may be
a. combined d. all of the above

b. external e. none of the above


c. internal
Q. 3. All are true about laryngocele except
a. bilateral laryngocele is usually asso- d. it is due to prolonged increase in
ciated with carcinoma of larynx transglottic pressure
b. external laryngocele presents as e. internal laryngocele is confined within
swelling in the neck the larynx
c. it is dilatation of laryngeal sacule
Q. 4. All are usual clinical features of laryngocele except
a. airway obstruction d. reducible swelling on thyrohyoid
b. dysphagia membrane
c. hoarseness e. stridor
Q. 5. All are true about laryngocele except
a. IDL shows distention of false cords d. swelling in neck that increases in size on
and a r y e p i g l o t t i c f o l d s performing Valsalva maneuver
b. palpation of swelling shows gurgling e. swelling is compressible
c. swelling in neck that increases in size on
coughing
Q. 6. Which statement is true regarding laryngocele?
a. CT/MRI determines extent of laryngo- c. plain X-ray neck shows an air-filled
cele sac
b. in case of unilateral laryngocele car- d. swelling in neck which increases in size
cinoma of laryngeal ventricles should be on performing Valsalva maneauver
ruled out by direct laryngoscopy e. All of the above
Q. 7. All are true about management of laryngocele except
a. endoscopy must be done in unilateral is needed
laryngocele to rule out carcinoma of d. internal laryngocele should be excised by
larynx external neck incision
b. external laryngocele should be ex- e. internal laryngocele should be marsu-
cised by external neck incision pilized by laryngoscopye.
c. if discovered incidentally, no treatment
Scenario: A 40 years old male presents with progressive hoarseness of voice for last 06
CONCEPT BOOK OF DISEASES OF EAR, NOSE & THROAT BY FAHIM AWAN
442 Part F Esophagus

months. Examination shows distension of left false cord and aryepiglottic fold. There is also
reducible swelling over left thyrohyoid membrane.
1. What is the most probable clinical diagnosis?
2. Which investigation will help to reach the diagnosis?
3. What is the treatment of this patient?
ANSWERS--------------------------------------
-------------------------------------------------------- ---------------
Q.1. b 2. d 3. a 4.b
5. b 6. e 7. d
Scenario: 1. Laryngocele 2. Plain X-ray neck A.P view, CT/MRI, direct laryngoscopy
3. Excision of external laryngocele, marsupialization of internal laryngocele.
-----------------------------------------------------------------------------------------------------------------
----------------

PART F
ESOPHAGUS
Chapter 14
DISEASES OF ESOPHAGUS
Plummer Vinson syndrome
Synonyms: Paterson-Brown-Kelly syndrome, Sidropenic dysphagia

Definition:
It is defined as dysphagia associated with anemia, koilonychia (spoon shape nails), angular
chelitis, smooth tongue, achlorhydria and splenomegaly.
Etiopathology:
There is hyperkeratinization with web or fibrous stricture formation in the postcricoid
region or upper esophagus. Dysphagia is due to web or fibrous stricture. The anemia is usually
microcytic but may be macrocytic. There is also vitamin B6 (pyridoxine) and B12 deficiency. It
usually affects middle-aged females. It is a premalignant condition with 10% risk of
developing carcinoma.
CONCEPT BOOK OF DISEASES OF EAR, NOSE & THROAT BY FAHIM AWAN
443 Part F Esophagus

Clinical features:
1. Patient presents with progressive dysphagia due to postcricoid web.
Dysphagia may be lasting for several years.
2. Absolute dysphagia usually indicates malignant change into
postcricoid carcinoma.
On examination;
1. There is pallor, koilonychia, angular chelitis, smooth tongue and
splenomegaly. Plummer Vinson
syndrome with
2. In cases of severe dysphagia indirect laryngoscopy may show pooling of koilonychia
saliva in the pyriform sinuses.
Loss of laryngeal crepitus usually indicates postcricoid carcinoma.
Investigations:
1. Barium swallow usually shows web in the postcricoid
region/upper esophagus.
2. Flexible/rigid esophagoscopy confirms web in the
postcricoid region/upper esophagus.
3. Hematological investigations usually show;
a. Hemoglobin, MCH (mean corpuscular haemoglobin)
and serum iron concentration is decreased. X-ray barium swallow showing web

b. Iron binding capacity is increased.


Treatment:
1. Iron supplements, vitamin B6 and vitamin B12 are given.
2. Repeated dilatation with bougies or laser excision of pharyngeal/esophageal web is
carried out.
SELF ASSESSMENT
Q. 1. Plummer Vinson syndrome is also called
a. Paterson Brown Kelly syndrome d. both a & b

b. Sidropenic dysphagia e. both a & c


c. Weber syndrome
Q. 2. All are true about Plummer Vinson syndrome except
a. dysphagia is due to hyperkeratiniza-tion tion with web formation in postcricoid
with web formation in postcricoid region region
b. dysphagia is due to hypokeratiniza- c. dysphagia is persistent in nature

CONCEPT BOOK OF DISEASES OF EAR, NOSE & THROAT BY FAHIM AWAN


444 Part F Esophagus

d. dysphagia is progressive in nature e. dysphagia usually lasts for several


years
Q. 3. All are true about Plummer Vinson syndrome except

a. angular chelitis d. shrunken spleen

b. koilonychia e. smooth tongue


c. pallor
Q. 4. All are true about Plummer Vinson syndrome except
a. achlorhydria d. splenomegaly

b. hepatomegaly e. upper esophageal web


c. pharyngeal web
Q. 5. What is true about Plummer Vinson syndrome?
a. Hb concentration is decreased d. serum iron is decreased

b. iron binding capacity is increased e. All of the above


c. MCH is decreased
Q. 6. All are true about investigation of Plummer Vinson syndrome except
a. barium swallow shows web in postcri- coid region
coid region d. esophagoscopy shows web in lower
b. barium swallow shows web in upper esophagus
esophagus e. esophagoscopy shows web in upper
c. esophagoscopy shows web in postcri- esophagus
Q. 7. Treatment of Plummer Vinson syndrome includes all except
a. esophagectomy d. repeated dilatation of web with bou-

gies
b. iron supplements

c. laser excision of web e. Vitamin B6 and B12

Scenario A: A 40 years old female suffering from progressive dysphagia for solids for last 08
years. Her clinical examination revealed koilonychia, pallor and angular chelitis.
1. What is the most probable clinical diagnosis?
2. What is the cause of dysphagia?
3. How will you manage this patient?
B: A 50 years old lady presented with progressive dysphagia for last 05 years. She is having
pallor smooth tongue and splenomegaly. Her Hb was 7 g/dl.
1. What is the most probable clinical diagnosis?
2. Which investigation will help to reach the diagnosis?
3. What is the treatment of this patient?
--------------------------------------------------------ANSWERS-----------------------------------------------------

CONCEPT BOOK OF DISEASES OF EAR, NOSE & THROAT BY FAHIM AWAN


Part F
445 Esophagus
Q.1. d 2. b 3. d 4. b 5. e 6. d 7. a
Scenario A: 1. Plummer Vinson syndrome 2. Postcricoid web 3. Iron supplements, vitamin
B6 and vitamin B12 are given, repeated dilatation with bougies or laser excision of
pharyngeal/esophageal web is carried out.
B: 1. Plummer Vinson syndrome 2. Barium swallow, flexible rigid endoscopy, hematological
investigations (hemoglobin, mean corpuscular haemoglobin and serum iron concentration is
decreased, iron binding capacity is increased) 3. Iron supplements, vitamin B6 and vitamin
B12 are given, repeated dilatation with bougies or laser excision of pharyngeal/esophageal
web is carried out.
--------------------------------------------------------------------------------------------------------------------------
FOREIGN BODY OF PHARNYX AND ESOPHAGUS
Etiopathology:
Usually there is a clear history of ingestion of foreign body. Vast majority of swallowed
foreign bodies pass through the digestive tract and excreted often undetected. A small number
of objects get impaction in the pharynx or esophagus.
Different types of foreign bodies which are usually encountered in the food passage are
coin, piece of meat, bone chip, denture, safety pin, metal nails, dry batteries or plastic objects.
Coins and large rounded foreign bodies are usually impacted at cricopharynx. Once these
pass through the cricopharynx, these may negotiate whole of GIT without causing any
problem. These foreign bodies may take a maximum of 06 weeks for their excretion in
faeces.
Food bolus especially meat may get impaction in elderly because the bolus is not
adequately chewed often in patients with ill fitting dentures.
Sharp foreign bodies like fish bone or common pin may be impacted in the vallecula,
tonsils or pyriform fossae.
Irregular foreign bodies may get impaction anywhere along GIT (mouth to anus).

Clinical features:
1. History of ingestion of foreign body is usually present.

2. Dysphagia, discomfort or pain in throat, substernal region or epigastrium.

3. A large foreign body in the upper esophagus can cause airway obstruction, by causing
compression of trachea.
On examination of oral cavity, oropharynx or hypopharynx;
1. Foreign body may be visible.
2. Pooling of saliva in the pyriform sinuses is usually seen on indirect laryngoscopy.

CONCEPT BOOK OF DISEASES OF EAR, NOSE & THROAT BY FAHIM AWAN


446 Part F Esophagus

3. There may be tenderness on palpation of neck.


Investigations:
1. X-ray of neck and chest PA often show site of impaction of
radiopaque foreign body. If there is doubt, lateral view will help to
determine whether the object is in the pharynx or the airway.
2. Radiolucent foreign body may show an air bubble on X-ray. Foreign body
esophagus
Treatment:
Foreign bodies like fish bone in the oropharynx may be removed under local anesthesia.

1. Endoscopic removal. Whenever possible try to get a template of the foreign body. It
helps in making the strategy and choosing proper instruments for safe removal of foreign
body. It also helps in anticipating the possible complications of the foreign body and its
removal. Once the diagnosis of pharnygeal/esophageal foreign body has been made,
arrangements should be made for safe and early removal as edema and mucosal swelling
will make removal of foreign body more difficult. Foreign bodies like alkaline batteries
are particularly dangerous because these may induce tissue necrosis.

For esophageal foreign bodies, flexible or rigid esophagoscope may be used. But for
irregular or pointing foreign bodies, rigid esophagoscope is preferable as complication of
such foreign bodies are reduced by their removal from within the rigid endoscope. Large
foreign bodies are removed as a trailing foreign body (both the endoscope and the foreign
body are removed simultaneously).

2. Cervical or transthoracic esophagotomy may be required for impacted foreign body of


esophagus.
ESOPHAGOSCOPY
Indications:
A. Diagnostic:
1. Removal of tissue for biopsy
2. Unexplained dysphagia
B. Therapeutic: Rigid esophagoscopes

1. Removal of foreign bodies


2. Removal of benign tumors
3. Dilatation of benign and malignant strictures
4. Treatment of achlasia cardia
5. Esophageal varices

CONCEPT BOOK OF DISEASES OF EAR, NOSE & THROAT BY FAHIM AWAN


447 Part F Esophagus

Contraindications:
1. Aneurysm of arch of aorta
2. Deformity of vertebral spine
3. Systemic disease like hypertension, diabetes, liver and
kidney diseases
4. Recent severe haematemesis
Procedure:
After general anesthesia the patient is made to lie supine
on the table with head projecting beyond the edge of the table
and supported by an assistant. The esophagus should be treated as if the walls were made
of wet blotting paper. Patient should be given a good muscle relaxant and the lumen kept in
view at all times. Initially head is flexed.
The esophagoscope is passed identifying the landmarks of uvula, epiglottis, endotracheal
tube and arytenoids. Head of the patient is gradually extended. The esophagoscope is
gradually advanced applying suction whenever required. When the esophagoscope has passed
through upper one third of the esophagus, the assistant lowers the head to the level of chest.
Cardiac end of the stomach is identified by the star shaped appearance of the mucosa.
During esophagoscopy any tumor, stricture or foreign body identified and dealt
accordingly. If removal of foreign body has been traumatic, a nasogastric tube should be
passed.
Complications:
1. Esophageal tear
2. Rupture of esophagus resulting in surgical emphysema or mediastinitis
3. Tracheoesophageal fistula
4. Perforation of aorta
5. Stricture of esophagus
SELF ASSESSMENT
Q. 1. Large smooth ingested foreign bodies are usually impacted at
a. cardiac end of stomach d. mid esophagus

b. cricopharynx e. upper esophagus


c. lower esophagus
Q. 2. All are usual features of foreign body esophagus except
a. discomfort d. odynophagia

b. dysphagia e. pooling of saliva in pyriform sinuses

c. fever
CONCEPT BOOK OF DISEASES OF EAR, NOSE & THROAT BY FAHIM AWAN
448 Part F Esophagus
Q. 3. Small rounded ingested foreign body is usually excreted into faeces upto maximum
period of
a. 02 weeks d. 08 weeks

b. 04 weeks e. 10 weeks
c. 06 weeks
Q. 4. The most definitive test for establishing the identification of radiolucent foreign body
in upper aerodigestive tract is
a. endoscopy d. P.A and decubitus X-ray

b. lateral decubitus X-ray e. P.A chest X-ray


c. lateral chest X-ray
Q. 5. All are usual indications of esophagoscopy except
a. dilatation of benign and malignant c. removal of foreign bodies
strictures d. removal of tissue for biopsy
b. removal of benign tumor e. treatment of hiatus hernia
Q. 6. All are usual contraindications of esophagoscopy except
a. aneurysm of arch of aorta d. removal of benign tumor
b. deformity of vertebral spine e. systemic disease like hypertension,
c. recent severe haematemesis diabetes, liver and kidney diseases
Q. 7. All are possible complications of esophagoscopy except
a. esophageal tear gical emphysema or mediastinitis

b. GERD d. stricture of esophagus

c. rupture of esophagus resulting in sur- e. tracheoesophageal fistula


Scenario A: Two years old child brought to medical emergency department with the history
of ingestion of five rupees coin 24 hours back. Parents complain of refusal to take feed.
1. How will you manage the case?
2. What are possible complications of the procedure carried out on this patient?
B: A 60 years old person presented in accident and emergency department with complaint of
odynophagia and absolute dysphagia for last 02 hours. He is attributing his complaint to
dislodging his artificial denture during sleep.
1. What procedure will you adopt to help this patient?
2. What are other indications for this procedure?
3. What would be the management plan?
---------------------------------------------- ANSWERS-----------------------------------
---------- ------------------
Q.1. b 2. c 3. c 4. a 5. e 6. d 7. b
Scenario A: 1. Detailed history, physical examination, investigations and endoscopic
removal

CONCEPT BOOK OF DISEASES OF EAR, NOSE & THROAT BY FAHIM AWAN


449 Part F Esophagus

2. Esophageal tear, rupture of esophagus, tracheoesophageal fistula, perforation of aorta,


stricture of esophagus.
B: 1. Rigid esophagoscopy. 2. Diagnostic, therapeutic 3. Endoscopic removal of foreign
body.

--------------------------------------------------------------------------------------------------------------------------

ACHALASIA CARDIA
Synonyms: Hypochalasia cardia, Dyschlasia cardia, Esophagectasia, Mega esophagus,
Cardiospasm
Etiopathology:
It is characterized by failure of
relaxation of the lower esophageal
sphincter during swallowing thus,
causing dysphagia. It is due to
degeneration of ganglionic cells in
Auerbach’s myenteric plexus.
It is more common between 30 and 60
years of age. It is more common in
women than men.
Clinical features:
1. Patient presents with long standing
history of progressive dysphagia.
Dysphagia is more for liquids than
for solids (opposite to that seen in
malignancy and stricture). Dysphagia is Diagramatic view of achalasia cardia
progressively more severe with latter parts of
meal, as the esophagus becomes filled with meal.
2. There is regurgitation of swallowed food which can lead to aspiration, choking
and coughing at night.
Investigations:
1. Barium swallow shows a dilated esophagus with a narrowed ‘rat
tail’ or ‘bird’s beak’ appearance at lower end.
2. Esophageal manometry shows;
a. Failure of the lower esophageal sphincter to relax. Contrast study show-
ing achalasia cardia
b. Low pressure in the body of esophagus.
c. High pressure at the lower esophageal sphincter.
Treatment:
1. Calcium channel blockers
2. Botox injections may be given into the lower esophageal sphincter.
3. Sequential dilatation of the lower esophageal sphincter with intraluminal balloon or
hydrostatic bag.
4. Modified Heller’s myotomy of narrowest lower part of esophagus is the treatment of
CONCEPT BOOK OF DISEASES OF EAR, NOSE & THROAT BY FAHIM AWAN
450 Part F Esophagus
choice in resistant cases.
SELF ASSESSMENT
Q. 1. Achalasia cardia is also called

a. dyschlasia cardia d. all of the above

b. esophagectasia e. none of the above

c. hypochalasia cardia
Q. 2. Achalasia cardia is also called

a. cardiospasm d. both a & b

b. hiatus hernia e. both a & c


c. mega esophagus
Q. 3. Findings of achalasia cardia include all except
a. dilated esophagus sphincter
b. failure of the lower esophageal d. low pressure at the lower esophageal
sphincter to relax sphincter
c. high pressure at the lower esophageal e. rat tail/bird’s beak appearance
Q. 4. All are included in the treatment of achalasia cardia except
a. botox injection d. modified Heller’s operation

b. calcium channel blockers e. sequential dilatation of lower part of

esophagus
c. Dohlman’s operation
Scenario: A 50 years old female presents with long standing history of progressive
dysphagia. Dysphagia is more for liquids than for solids. There is regurgitation of swallowed
food, choking and coughing at night. Barium swallow shows a dilated esophagus with a
narrowed ‘rat tail’ or ‘bird’s beak’ appearance at lower end.
1. What is the most probable clinical diagnosis?
2. What points you would like to confirm on manometry?
3. What would be the management plan?
-------------------------------------------------------- ANSWERS-----------------------------------------------------
Q.1. d 2. e 3. d 4.c
Scenario: 1. Achalasia cardia 2. Failure of the lower esophageal sphincter to relax, low
pressure in the body of esophagus, high pressure at the lower esophageal sphincter
3. Calcium channel blockers, botox injection, sequential dilatation, modified Heller’s
myotomy.
--------------------------------------------------------------------------------------------------------------------------
HIATUS HERNIA
Definition:
It is defined as displacement of stomach up through hiatus (esophageal opening) of the
CONCEPT BOOK OF DISEASES OF EAR, NOSE & THROAT BY FAHIM AWAN
451 Part F Esophagus

diaphragm into lower mediastinum.

Etiopathology:
There are basically following types of hiatus hernia;
1. Sliding hiatus hernia: In this gastroesophageal
junction ascends into chest and stomach is in line
with the esophagus. The consequence is
gastroesophageal reflux.
2. Paraesophageal hernia: In this
gasteroesophageal junction retains its normal
anatomical position below the diaphragm.
However, fundus of stomach ascends into chest
around side of esophagus through large hiatus.
Angle between stomach and the esophagus is
Diagramatic view of sliding hiatus hernia
maintained. These patients have no reflux.
Clinical features:
Most patients are over the age of 40 years.
In sliding hiatus hernia, there is reflux i.e., heart burn on bending and lying,
retrosternal discomfort, nausea and vomiting. Occasionally dysphagia may occur.
In paraesophageal hernia, there is anemia, dyspnea on exertion, pseudoangina and
abdominal discomfort.
Investigations:
1. Radiology
a. X-ray chest lateral view shows gastric air shadow behind heart.
b. Fluoroscopic studies of esophagus using contrast confirm the Diagramatic
diagnosis. representation of
hiatus hernia
2. Esophagoscopy is done to determine the level of gastroesophageal junction and to find
degree of esophagitis.
3. pH studies
4. Manometry
Treatment:
When there is no symptom, no treatment is required. When symptoms are severe, treatment is
instituted in the following order;
A. Conservative:
1. Weight reduction
2. Avoidance of tobacco and alcohol
3. Elevation of head end of bed at night about 03 cm
4. Drugs that reduce acidity
a. Antacid

CONCEPT BOOK OF DISEASES OF EAR, NOSE & THROAT BY FAHIM AWAN


452 Part F Esophagus

b. H2 receptor antagonist
c. Proton-pump inhibitor
d. Antispasmodic
B. Surgical: Surgical treatment is considered in severe cases or where medical treatmet has
failed. In it, replacement of the stomach in the abdomen and repair of the diaphragmatic
opening is usually performed. Different surgical procedures are;
1. Belsey-Mark IV operation
2. Nissen fundoplication
3. Hill’s anti reflux procedure
SELF ASSESSMENT
Q. 1. All are true about hiatus hernia except
a. in paraesophageal hernia portion of geal junction ascends into chest
stomach protrudes up around side of d. in sliding hiatus hernia there is anemia
esophagus and psuedoangina
b. in paraesophageal hernia there is e. in sliding hiatus hernia there is heart
anemia and psuedoangina burn and retrosternal discomfort
c. in sliding hiatus hernia gastroesopha-
Q. 2. All are usual clinical features of paraesophageal hernia except
a. abdominal discomfort d. heart burn

b. anemia e. pseudo angina


c. dyspnea
Q. 3. All are usual investigations of hiatus hernia except
a. CT scan d. pH studies
b. esophagoscopy is done to determine the e. X-ray chest showing gastric air shadow
level of gastroesophageal junction behind heart
c. fluroscopy
Q. 4. Conservative treatment of hiatus hernia includes all except
a. avoidance of tobacco and alcohol d. sleeping with head and chest raised.

b. H1 receptor antagonist e. weight reduction


c. proton pump inhibitors
Q. 5. Surgical treatment of hiatus hernia is
a. Belsey-Mark 1V d. all of the above

b. Hill’s anti reflux procedure e. none of the above

c. Nissen fundoplication
Scenario: A 50 years old patient presents with long-standing history of reflux on bending
CONCEPT BOOK OF DISEASES OF EAR, NOSE & THROAT BY FAHIM AWAN
453 Part F Esophagus

and lying, retrosternal discomfort, nausea and vomiting. X-ray chest lateral view shows
gastric air shadow behind heart.
1. What is the most probable clinical diagnosis?
2. What would be the management plan?
-------------------------------------------------------- ANSWERS-----------------------------------------------------
Q.1. d 2. d 3. a 4. b 5. d
Scenario: 1. Hiatus hernia. 2. A; Conservative. Weight reduction, avoidance of tobacco and
alcohol, elevation of head end of bed at night about 3 cm, drugs that reduce acidity.
B; Surgical. Belsey-Mark IV operation, Nissen fundoplication, Hill’s anti reflux procedure.
---------------------------------------------------------------------------------------------------------------------------
CORROSIVE STRICTURE OF ESOPHAGUS
Etiopathology:
Corrosive may be acid or alkali. Swallowing of corrosive may be suicidal or accidental.
Severity of injury depends on nature of corrosive, quantity, concentration and duration of its
contact with the tissues.
Alkalies are more destructive and penetrate into the deeper layers of esophagus.
Swallowed corrosive causes more damage in the mouth and lower third of esophagus
(because peristalsis is slow in this part). Late result of ingestion of corrosive is stricture
formation.

Clinical features:
These depend on the stage at which patient presents.
1. Pain and odynophagia are the most common symptoms in the initial stages.
2. Coughing and choking usually occur due to spill over.
3. In the later stages there is relative or absolute dysphagia.

Complications:
1. Immediate complications are laryngeal edema and airway obstruction.
2. Late complication is formation of esophageal stricture.
Investigations:
1. Barium swallow with fluoroscopy should be carried out to find
out site and length of the stricture.
2. Early esophagoscopy may lead to perforation of esophagus. So, it
Barium swallow
should be delayed for at least two weeks. showing cor-
rosive stricture
Treatment:
1. Immediate treatment: It is directed towards neutralization of corrosive,
maintenance

CONCEPT BOOK OF DISEASES OF EAR, NOSE & THROAT BY FAHIM AWAN


454 Part F Esophagus
of airway, treatment of shock, acid base imbalance, pain, parenteral feeding and intravenous
antibiotics. Incidence of stricture formation is reduced by corticosteroids. Steroids are
started within 48-96 hours for 4-6 weeks. A nasogastric tube is passed to feed the patient and
to maintain the esophageal lumen.
2. Late treatment: Repeated serial dilatation with bougies (gum elastic bougies,
Maloney mercury bougies) or hydrostatic bags is usually satisfactory to treat stricture.
If stricture is tight, fibrotic and elongated then resection of the affected segment and
reconstruction using colon and jejunum is required.
SELF ASSESSMENT
Q. 1. All are true about corrosive stricture of esophagus except
a. barium swallow with fluoroscopy is tation
investigation of choice d. more damage to lower 3rd of esopha-
b. immediate problem is laryngeal ede-ma gus
e. more damage to mouth
c. late complication is esophageal dila-
Q. 2. All are included in the immediate treatment of corrosive ingestion except
a. cytotoxic drugs d. parenteral antibiotics and corticoste-

roids
b. maintenance of airway

c. neutralization of corrosive e. treatment of pain and shock

Q. 3. All are included in the late management of corrosive stricture of esophagus except
a. laryngopharyngectomy d. repeated serial dilatation with
b. repeated serial dilatation with gum Maloney mercury bougies
elastic bougies e. resection of tight and elongated stric-ture
c. repeated serial dilatation with hydro-
static bag
Scenario: A female presented in ENT OPD with complaint of absolute dysphagia for last 04
days. There is history of corrosive intake 02 months back.
1. What is the most probable clinical diagnosis?
2. What investigations will you carry out?
3. How will you manage this patient?
-------------------------------------------------------- ANSWERS-----------------------------------------------------
Q.1. c 2. a 3. a
Scenario: 1. Corrosive stricture of esophagus 2. Barium swallow, esophagoscopy
3. Repeated and serial dilatation, resection and anastomosis if dilatation fails.
--------------------------------------------------------------------------------------------------------------------------
-
CONCEPT BOOK OF DISEASES OF EAR, NOSE & THROAT BY FAHIM AWAN
455 Part F Esophagus

PERFORATION/RUPTURE OF ESOPHAGUS
Etiopathology:
There are following causes of esophageal perforation;
1. Instrumental such as esophagoscopy and bouginage is the
most common cause of esophageal perforation.
2. Foreign body penetration such as by needle, nail or bone Perforation with air in soft
chip. tissues

3. Post-emetic rupture of all layers of esophagus is called


Boerhaave syndrome.
4. Cut throat or gun shot injury.
5. Stress associated with neurological disease, after burns or operations even away from
esophagus.
6. Nd: YAG laser such as in cutting out of a passage through passable tumors of
hypopharynx or esophagus.
Perforation may occur in thoracic or abdominal esophagus. Perforation of esophagus carries
high morbidity and mortality.
Clinical features:
1. Severe pain in the neck or interscapular region.
2. Dysphagia
3. Fever are the most common symptoms.
On examination there is;
1. Tenderness in the neck
2. Surgical emphysema i.e., air in the soft tissues
3. Sharp rise in body temperature

Investigations:
A. Plain X-ray chest and neck usually shows
1. Air in the soft tissues
2. Air in the mediastinal spaces
3. Air under diaphragm
4. Widening of mediastinum
5. Pneumothorax
6. Pleural effusion

CONCEPT BOOK OF DISEASES OF EAR, NOSE & THROAT BY FAHIM AWAN


456 Part F Esophagus

B. Contrast studies of esophagus are required to determine site of perforation and


underlying pathology.
Treatment:
Diagnosis and treatment of esophageal perforation is started as early as possible.
1. N.P.O. (Nothing Per Oral i.e., do not allow any fluid or food by mouth).
2. Central venous line is passed for parentral feeding.
3. Broad-spectrum antibiotics are started immediately.
4. Further management depends on site of perforation, clinical state and underlying
pathology.
5. In instrumental perforation of cervical esophagus, simple conservative measures are
adopted.
6. In other types of perforations, if diagnosis is made within six hours, excision of
esophagus containing perforation is followed by reconstruction.
7. If suppuration occurs, surgical drainage is done.
8. Gastrostomy and jejunostomy is done for feeding purposes.
9. In cases of perforation of thoracic esophagus, active management is required with repair
of perforation and drainage of pleural cavities.
10. In cases of late diagnosis, major reconstructive surgery is required which means
excision of damaged esophagus, pleural drainage, cervical esophagostomy and closure.
SELF ASSESSMENT
Q. 1. The most common cause of perforation of esophagus is
a. cut throat or gun shot d. post-emetic rupture of all layers of
b. esophagoscopy and bouginage esophagus
c. foreign body penetration such as by e. stress associated with neurological
needle, nail or bone chip disease, after burns or operations even
away from esophagus
Q. 2. All are usual symptoms of perforation of esophagus except
a. decrease in body temperature d. severe pain in interscapular region

b. dysphagia e. severe pain in neck


c. increase in body temperature
Q. 3. The usual sign of rupture of esophagus is
a. pneumothorax d. tenderness in the neck

b. sharp rise in body temperature e. all of the above

c. surgical emphysema

CONCEPT BOOK OF DISEASES OF EAR, NOSE & THROAT BY FAHIM AWAN


457 Part F Esophagus
Q. 4. Plain X-ray gives all the findings in rupture of esophagus except
a. air in the mediastinum d. narrowing of mediastinum

b. air in the soft tissues e. pleural effusion


c. air under diaphragm
Q. 5. All are treatments of rupture of esophagus except
a. broad-spectrum antibiotic surgically immediately

b. CVP line is passed d. NPO

c. instrumental perforation is managed e. suppuration is drained


Scenario: A 50 years old patient presented with history of ingestion of meat bolus containing
bone chip two hours back. Foreign body was removed after rigid esophagoscopy. Just after
recovery from anesthesia patient complained of severe pain in the neck/interscapular region,
dysphagia and fever. On examination there is tenderness in the neck, surgical emphysema,
and sharp rise in body temperature.
1. What is the most probable clinical diagnosis?
2. What would be your management plan?
-------------------------------------------------------- ANSWERS-----------------------------------------------------
Q.1. b 2. a 3. e 4. d 5. c
Scenario: 1. Perforation of esophagus 2. N.P.O., central venous line, broad-spectrum
antibiotics, if suppuration occurs, surgical drainage, major reconstructive surgery.
--------------------------------------------------------------------------------------------------------------------------
-
GASTROESOPHAGEAL REFLUX DISEASE (GERD)
Synonym: Laryngopharyngeal reflux (LPR)

Definition:
Reflux of gastric contents into the esophagus is known as gastroesophageal reflux disease
(GERD).

Etiopathology:
It occurs due to;
1. Laxity of lower esophageal sphincter
2. Pregnancy
3. Hiatus hernia
4. Excessive use of alcohol
5. Drugs which relax smooth muscles (calcium channel blockers and anticholinergics)
CONCEPT BOOK OF DISEASES OF EAR, NOSE & THROAT BY FAHIM AWAN
458 Part F Esophagus

Clinical features:
1. Substernal pain, heart burn and regurgitation are the most common symptoms.
2. There may be complaint of hoarseness of voice and irritation in throat.
Complications:
1. Esophagus; e.g., esophagitis, benign esophageal stricture, esophageal erosions and
hemorrhage
2. Lungs; e.g., pneumonia, bronchiectasis

3. Larynx; e.g., posterior laryngitis, contact ulcer/granuloma, paroxysmal


laryngospasm
4. Miscellaneous; e.g., globus hystericus
Treatment:
It consists of;
1. Removal of underlying cause.
2. Avoidance of smoking, alcohol, tea, coffee, chocolates and carbonated water (cola
drinks).
3. Avoiding food at least 3-4 hours before bedtime.
4. Elevation of about 03 cm head end of bed at night.
5. Drugs.
a. Antacids
b. H2 receptor antagonists e.g., cimetidine and ranitidine
c. Proton pump inhibitors e.g., omeprazole
d. Drugs which increase lower esophageal sphincter tone e.g., metoclopramide
SELF ASSESSMENT
Q. 1. Gastroesophageal reflux disease is also called

a. achalasia cardia d. hiatus hernia

b. dyschlasia cardia e. laryngopharyngeal reflux (LPR)


c. esophagectasia
Q. 2. All are usual symptoms of gastroesophageal reflux disease except
a. heart burn d. regurgitation

b. hemoptysis e. substernal pain


c. hoarseness
Q. 3. All are usual complications of GERD except
CONCEPT BOOK OF DISEASES OF EAR, NOSE & THROAT BY FAHIM AWAN
459 Part G Neck

a. carcinoma of larynx d. pneumonia

b. esophagitis e. posterior laryngitis


c. globus hystericus
Q. 4. All are true about treatment of GERD except
a. avoiding coffee, chocolates and car- c. avoiding smoking, alcohol and tea
bonated water (cola drinks) d.eElevation of about 03 cm foot end of
b. avoiding food at least 3-4 hours be- bed at night
fore bedtime e. removal of underlying cause
Q. 5. All are true about treatment of GERD except
a. antacids d. H2 receptor antagonists e.g., cimetidine
b. drugs which increase lower esopha- and ranitidine
geal sphincter tone e.g., metoclopramide e. proton pump inhibitors e.g., omepra-
c. H1 receptor antagonists e.g., citrizine zole
Scenario: A 35 years old obese patient presented with complaint of constant irritation in
throat for last 06 months. Patient also complains of occasional hoarseness of voice. There is
also history of heartburn and retrosternal pain after meal.
1. What is the most probable clinical diagnosis?
2. How will you manage this patient?
-------------------------------------------------------- ANSWERS-----------------------------------------------------
Q.1. e 2. b 3. a 4. d 5. c

Scenario: 1. Gastroesophageal reflux disease 2. Life style modifications, medications.


---------------------------------------------------------------------------------------------------------------------------
CONCEPT BOOK OF DISEASES OF EAR, NOSE & THROAT BY FAHIM AWAN
460 Part G Neck

PART G
NECK
Chapter 15
CLASSIFICATION OF NECK
MASSES
Most of the neck masses fall into the following categories:
1. Thyroid masses: This is the most common group
of neck swelling. Thyroglossal cyst is a common cause of
neck swelling.
2. Lymph nodes: These may be enlarged because of;
a. Infection in draining sites.
b. Infection of the lymphoreticular system e.g.,
infectious mononucleosis, toxoplasmosis and
cytomegalovirus. Neck masses

c. Malignancies of lymphoreticular system e.g., Hodgkin’s diseases, non-Hodgkin’s


lymphoma and Burkit’s lymphoma.
d. Granulomatous diseases e.g., tuberculosis and sarcoidosis of lymph nodes.
3. Skin and associated structures: e.g., sebaceous cyst, lipoma,
neurofibroma, melanoma etc.
4. Salivary gland diseases: e.g., sialectasis, sialadenitis, sialolithiasis, benign and ma
lignant tumors of salivary glands.
5. Primary lesions of neck structures: e.g., pharyngeal pouch, laryngocele, benign and
malignant tumors of oropharynx, hypopharynx, larynx and parapharyngeal space (caro-
tid body tumor, neurogenic tumor, glomus tumor).

Investigations: In cases of neck lump after detailed history and examination, generally,
following investigations are carried out;
1. Fine Needle Aspiration Cytology (FNAC).

CONCEPT BOOK OF DISEASES OF EAR, NOSE & THROAT BY FAHIM AWAN


461 Part G Neck

2. Ultrasonography/CT/MRI
If FNAC is inconclusive then Core Biopsy (CB) is carried out. If both FNAC and CB are
inconclusive then Open Biopsy (OB) is performed.

SELF ASSESSMENT
Q. 1. The most common group of neck swellings seen in hospital are from
a. lymph node d. skin

b. metastasis e. thyroid
c. salivary gland
Q. 2. The most common skin lump in neck is
a. dermoid cyst d. melanoma

b. epidermoid cyst e. sebaceous cyst


c. lipoma
Q. 3. A patient presented with a 3.5 cm hard, enlarge lymph node in the subman-dibular
region. Examination of the head and neck did not reveal any lesion. The next investigation
to be done is
a. Chest X-ray c. panendoscopy

b. fine needle aspiration cytology d. supravital staining of oral mucosa

(FNAC)
e. ultrasonography
Q. 4. A 50 years old man presents with a mass of lymph nodes in the upper cervical region.
He is a smoker and also drinks two to three times per week. Complete phys-ical
examination of the upper aerodigestive tract does not reveal any primary tumor. Our next
diagnostic step should be
a. CT scan neck d. incisional biopsy of node

b. excision of cervical node e. observation of appearance of any

primary
c. fine needle aspiration cytology
(FNAC)
Q. 5. The mainstay initial investigation in cervical lymphadenopathy is
a. biopsy d. ultrasonography

b. blood test e. X-ray

c. FNAC
-------------------------------------------------------- ANSWERS-----------------------------------------------------
Q.1. e 2. e 3. b 4. c 5. c
---------------------------------------------------------------------------------------------------------------------------

CONCEPT BOOK OF DISEASES OF EAR, NOSE & THROAT BY FAHIM AWAN


462 Part G Neck

THYROID TUMORS
1. Follicular adenoma: It is the most common benign
tumor
of thyroid gland. It is usually solitary encapsulated mass varying in
consistency from firm to soft. Adenoma may be non functional
(cold), normally functioning (warm), or hyperactive (toxic).
It is very important to rule out follicular carcinoma histologically
by looking for the absence of venous or capsular invasion. Thyroid carcinoma

Diagnosis is generally, made retrospectively when a solitary


nodule is removed by lobectomy.

2. Papillary carcinoma: It is most common malignant tumor of thyroid gland.


It comprises 60% of all malignant thyroid tumors. It grows slowly. Histologically neucleoli
show Orphan annie-eye appearance.
It is more common under the age of 40 years. Incidence of lymph node metastasis is 40%.
In lungs, it gives diffuse milliary infiltrates.
3. Follicular carcinoma: It comprises 18% of all malignant thyroid tumors. It
grows rapidly. It occurs around the age of 50 years. It is typically encapsulated and may be
mistaken for a follicular adenoma. Gross invasion of capsular veins is a bad prognostic sign.
Incidence of lymph node metastasis is only 4%. In lungs it gives large rounded opacities. In
thyroid tumors, hematogenous spread is more common in follicular carcinoma.
Hurthle cell carcinoma is a variant of follicular carcinoma.
4. Medullary carcinoma: It comprises 05-10% of all malignant thyroid tumors. It
arises from parafollicular or C cells of the thyroid gland. So, it is often neuroectodermal in
origin. There are following two categories of medullary carcinoma;
a. Sporadic: It occurs between 3rd and 7th decade of life with a female predominance.
b. Familial: It may appear as early as 2nd decade of life. It affects both genders
equally. Other tumors are seen in about 10% of patients with familial medullary
carcinoma in the form of MEN (Multiple Endocrine Neoplasia) 2A and MEN 2B.
MEN 1=Pituitary adenoma, parathyroid hyperplasia, pancreatic tumor.
MEN 2A=Parathyroid hyperplasia, medullary thyroid carcinoma,
pheochromocytoma.
MEN 2B=Pheochromocytoma, medullary thyroid carcinoma, mucosal neuromas,
Marfanoid body habitus.
Medullary carcinoma is characterized by an elevation of serum calcitonin level. Infusion of
calcium or ingestion of whisky are potent stimuli for release of calcitonin. This fact is made
use of when cases of occult tumor are suspected or when screening the siblings of a person
known to suffer from the disease.
These tumors secrete prostaglandins and histaminases. Its blood borne metastasis is
seen in the liver, lungs and bones. It is non encapsulated. Incidence of lymph node metastasis
is 75%.
5. Anaplastic carcinoma: It forms 5% of all thyroid malignant neoplasms.
Average age of presentation is 60 years. It is more prevalent in women. It is characterized
CONCEPT BOOK OF DISEASES OF EAR, NOSE & THROAT BY FAHIM AWAN
463 Part G Neck

by rapid growth, pain and early invasion of surrounding structures. It is hard and fixed to the
surrounding structures. Patients rarely survive for longer than one year after the diagnosis is
made.
Treatment of thyroid carcinomas:
Generally, if the tumor is;
a. Intrathyroidal then total thyroidectomy is performed and thyroxine is given life long.
b. Extrathyroidal then total thyroidectomy is followed by external beam radiotherapy
and thyroxine is given life long.

6. Lymphoma: The prognosis of lymphoma is comparatively favorable. Clinical


course of the disease is extremely variable.
Treatment is the same as that for lymphoma elsewhere in the body.
SELF ASSESSMENT
Q. 1. The most common tumor of thyroid is

a. adenoma d. medullary carcinoma

b. anaplastic carcinoma e. papillary carcinoma


c. follicular carcinoma
Q. 2. The most common malignant tumor of thyroid is
a. anaplastic carcinoma d. medullary carcinoma

b. follicular carcinoma e. papillary carcinoma


c. lymphoma
Q. 3. Regarding thyroid tumors, greatest likelihood of nodal spread is in
a. anaplastic carcinoma d. medullary carcinoma

b. follicular carcinoma e. papillary carcinoma


c. hurthle cell carcinoma
Q. 4. Follicular adenoma is differentiated from follicular carcinoma by
a. absence of capsular and venous inva- c. fixity

sion
d. site

b. consistency e. size

Q. 5. Hurthle cell carcinoma is a variant of


a. anaplastic carcinoma d. medullary carcinoma

b. follicular carcinoma e. papillary carcinoma


c. hurthle cell carcinoma
Q. 6. In thyroid tumors, hematogenous spread is more common in

CONCEPT BOOK OF DISEASES OF EAR, NOSE & THROAT BY FAHIM AWAN


464 Part G Neck

a. follicular carcinoma d. papillary carcinoma

b. lymphoma e. thyroid adenoma


c. medullary carcinoma
Q. 7. All of the following statements are correct about medullary carcinoma except
a. arises from parafollicular or C cells d. other tumors are seen in about 10% of
b. familial type may appear as early as 2nd patients with familial medullary carci-
decade of life noma in the form of MEN 1
c. neuroectodermal in origin e. sporadic type occurs between 3rd and
7th decade of life
Q. 8. All of the following statements are correct about medullary carcinoma except
a. characterized by an elevation of se- d. secretes prostaglandins
rum calcitonin level e. 10% cases are associated with MEN 2A
b. secretes histaminase and MEN 2B.
c. secretes leukotrines
Q. 9. Investigations necessary before excision of lingual thyroid include all except
a. biopsy d. serum TSH

b. indirect laryngoscopy e. thyroid scan


c. serum T3 and T4
Q. 10. Superior vena cava syndrome may be caused by
a. bronchogenic carcinoma d. thymoma

b. mediastinal neoplasm e. all of the above


c. retrosternal goiter
Q. 11. All are true of lingual thyroid except
a. always associated with presence of d. presents in adulthood when demand for
cervical thyroid thyroxine is increased
b. can shrink with thyroxine e. presents in the base of tongue
c. can be excised surgically
Scenario: A 40 years old female presented with a solitary thyroid nodule for last 06
months. Nodule is gradually increasing in size. Investigations show increased calcitonin
level.
1. What is the most probable clinical diagnosis?
2. Name different categories of this tumor?
3. How will you treat this patient?
-------------------------------------------------------- ANSWERS-----------------------------------------------------
Q.1. a 2. e 3. e 4. a 5. b 6. a
CONCEPT BOOK OF DISEASES OF EAR, NOSE & THROAT BY FAHIM AWAN
465 Part G Neck
7. d 8. c 9. a 10. e 11. a
Scenario: 1. Medullary carcinoma of thyroid 2. Sporadic and familial 3. Total thyroidectomy
with neck dissection, postoperative suppressor therapy with thyroxine and follow up with
serum calcitonin level.
---------------------------------------------------------------------------------------------------------------------------
THYROGLOSSAL CYST
Embryology:
The thyroid primordium develops in the floor of the primitive
pharynx at the site of foramen caecum. It lies in the midline at
the junction of anterior two thirds with posterior one third of
tongue.
The descending thyroid gland migrates through the tongue
tissue passing anterior to the hyoid bone to lie in the anterior
neck. The line of descent from the foramen caecum is marked by
the thyroglossal duct. If involution of the duct is incomplete, a
cyst may lie at any point along the tract.
Etiopathology:
It occurs as a result of remnant of part of descent of the thyroid gland. Ninety percent of the
cysts lie in the midline and 10% are to one side of the midline. Of those 95% are on the left
side and 05% are on the right. It most commonly presents in first decade of life. Gender
distribution is equal.
It may be intralingual, suprahyoid, thyrohyoid or suprasternal. The most common
location is thyrohyoid. It may contain the only functioning thyroid tissue. Rarely carcinoma
may develop in the cyst.

Clinical features:
1. Painless lump in the midline just below hyoid bone lasting for
several years is the most common presentation.
A case of
2. If the cyst is infected it becomes painful and there may be odynophagia. thyroglossal cyst
3. It moves on swallowing and usually on protrusion of the tongue as
well.
Thyroglossal cyst may change into thyroglossal sinus due to spontaneous or surgical
drainage/excision of thyroglossal cyst.

Differential diagnosis:
1. Dermoid cyst
2. Lipoma
3. Pyramidal lobe hypertrophy
4. Subhyoid bursitis
CONCEPT BOOK OF DISEASES OF EAR, NOSE & THROAT BY FAHIM AWAN
466 Part G Neck

Investigations:
Diagnosis is usually made on clinical grounds.
1. Thyroid radionucleotide scan is carried out to confirm the presence and location of
normal thyroid gland.
2. FNAC and USG of lump is required in doubtful cases.
3. Sinogram/fistulogram is done in cases of thyroglossal sinus/fistula to find out length and
pathway of the tract.

Treatment:
1. If thyroglossal cyst is asymptomatic, no treatment is required and it may be left as such.
2. If there is infection with pain or swelling, initially wide bore needle aspiration is
carried out along with systemic antibiotics and analgesics.
3. Sistrunk’s operation is the best treatment. In this operation excision of the cyst along
with the tract, body of hyoid bone medial to the lesser cornu and the core of base of
tongue upto the region of foramen caecum is performed.
SELF ASSESSMENT
Q. 1. The most common congenital neck mass below hyoid bone in the first decade of life is
a. branchial cyst d. lymphangioma

b. cystic hygroma e. thyroglossal cyst


c. dermoid cyst
Q. 2. Which statement is correct about thyroglossal cyst?
a. originates at the junction of anterior 1/3 2/3 with posterior 1/3 of tongue
with posterior 2/3 of tongue d. usually presents above the hyoid bone
b. originates at the junction of anterior e. usually presents below the cricoid
1/2 with posterior 1/2 of tongue cartilage
c. originates at the junction of anterior
Q. 3. The most common location of thyroglossal cyst is
a. intralingual d. thyrohyoid

b. suprahyoid e. None of the above


c. suprasternal
Q. 4. All are usual clinical features of thyroglossal cyst except
a. dyspnea c. odynophagia

b. moves on protrusion of tongue d. painless lump in midline neck

CONCEPT BOOK OF DISEASES OF EAR, NOSE & THROAT BY FAHIM AWAN


467 Part G Neck
e. presents in the first decade of life
Q. 5. All are true of thyroglossal cyst except
a. increases in size during infection d. presents as a midline swelling
b. originates from 2nd brachial cleft e. requires excision of mid-portion of
c. presents just below the hyoid bone hyoid bone to prevent recurrence
Q. 6. Differential diagnosis of thyroglossal cyst is
a. dermoid cyst d. subhyoid bursitis

b. lipoma e. all of the above


c. pyramidal lobe hypertrophy
Q. 7. The most important investigation of thyroglossal cyst is
a. CT neck d. thyroid radionucleotide scan

b. MRI e. X-ray neck

c. open biopsy
Q. 8. Treatment of thyroglossal cyst is

a. Belsey Mark IV d. Sistrunk’s operation

b. Heller’s myotomy e. Young’s operation

c. Nissen fundoplication
Scenario: A 06 years old boy presents with a mass in the midline of neck just below the
hyoid bone noted 06 months back. It moves with protrusion of tongue.
1. What is the most probable clinical diagnosis?
2. How will you investigate this case?
3. How will you manage this case?
ANSWERS------------------------------------------------
-------------------------------------------------------- -----
Q.1. e 2. c 3. d 4. a 5. b
6. e 7. d 8. d
Scenario: 1. Thyroglossal duct cyst 2. Thyroid radioneucleoside scan, FNAC and USG of
lump 3. Sistrunk’s operation.
---------------------------------------------------------------------------------------------------------------------------
BRANCHIAL CYST
Synonym: Lateral cervical cyst
This is usually a result of anomaly of second brachial arch. This is located deep to
sternomastoid at the junction of upper one third
and lower two third. This is common in second decade of life but can occur at any age. These
may be bilateral. Cyst presents as a smooth, rounded, fluctuant, non tender and

CONCEPT BOOK OF DISEASES OF EAR, NOSE & THROAT BY FAHIM AWAN


468 Part G Neck

non- transilluminant swelling. Swelling may increase in size during upper respiratory tract
infection. A branchial cyst may be associated with a sinus or a fistula.

Branchial sinus: Synonym: Branchial pit


This opens at anterior border of sternomastoid on a line joining tragus and sternoclavicular
joint. Branchial sinus is usually noted at birth. 60% occur on left side and 40% on right side.
Branchial fistula:
These have two openings. External opening lies in the neck at junction of upper one third and
lower two third of anterior border of sternomastoid muscle. The internal opening usually lies
at posterior tonsillar pillar.
Collaural fistula: It is least common of first branchial cleft anomalies.
Investigations:
1. Sinogram/fistulogram is required to find out exact extent and pathway of tract.
2. FNAC

Treatment:
If branchial cyst/sinus/fistula is asymptomatic, no treatment is required.
1. If there is infection with pain or swelling, initially wide bore needle aspiration is
carried out along with systemic antibiotics and analgesics.
2. When the infection has settled, excision of the cyst/sinus/fistula is performed. Methylene
blue can be injected into sinus/fistula to detect the pathway of tract.
Step ladder incision is made in neck for exposure of the tract of cyst/sinus/fistula. Care
is taken not to leave behind any portion of the tract otherwise, it may lead to recurrence.
SELF ASSESSMENT
Q. 1. Branchial cyst originates from
a. the junction of anterior 1/2 with pos- 2/3 of sternomastoid muscle
terior 1/2 of tongue d. the junction of upper 1/2 with lower 1/2
b. the junction of anterior 1/3 with pos- of sternomastoid muscle
terior 2/3 of tongue e. the junction of upper 2/3 with lower 1/3
c. the junction of upper 1/3 with lower of sternomastoid muscle
Q. 2. All are true about branchial cyst except
a. anomaly of second branchial arch fistula
b. common presentation is in the second d. may increase in size during upper
decade of life respiratory tract infection
c. may be associated with a sinus or e. presents in the midline
Q. 3. All are true about branchial sinus/fistula except
CONCEPT BOOK OF DISEASES OF EAR, NOSE & THROAT BY FAHIM AWAN
469 Part H General Topics

a. 40 % are on right side mastoid muscle


b. 60 % are on left side d. internal opening usually lies at poste-rior
c. external opening lies in the neck at tonsillar pillar.
junction of upper one third and lower e. sinuses are usually noted in second
two third of anterior border of sterno- decade of life
Q. 4. The most important investigation of branchial fistula is
a. CT neck d. thyroid radionucleotide scan

b. fistulogram e. X-ray neck


c. MRI
Q. 5. All are treatments of infected branchial cyst except
a. analgesics d. excision

b. antibiotics e. wide bore aspiration

c. anti-inflammatory drugs
Q. 6. Treatment of branchial fistula is

a. Belsey Mark IV d. Sistrunk’s operation

b. excision with step ladder incision e. Young’s operation

c. Nissen fundoplication
-------------------------------------------------------- ANSWERS-----------------------------------------------------
Q.1. c 2. e 3. e 4. b 5. d 6. b
---------------------------------------------------------------------------------------------------------------------------
CONCEPT BOOK OF DISEASES OF EAR, NOSE & THROAT BY FAHIM AWAN
470 Part H General Topics

PART H
GENERAL TOPICS
Chapter 16
ACQUIRED IMMUNODEFICIENCY SYNDROME (AIDS)
Etiopathology:
The agent responsible for Acquired Immune Deficiency Syndrome is retrovirus. HIV-1
enveloped retrovirus having a genomic strand with RNA. Viral DNA is integrated into host
cell DNA, called the provirus which is reverse transcribed into viral RNA. This RNA is
utilized for RNA genome of virus or acts as messenger RNA for translation to viral proteins.
Retrovirus do not cause cell lysis, so the infection becomes permanently encoded.
HIV can be isolated from body fluids like blood, semen, saliva, vaginal secretions, urine,
tears, CSF and breast milk.
Transmission:
It can occur from:
1. Sexual intercourse
2. I/V drug abuse and infected needles
3. Blood transfusion and organ transplantation
4. Mother to child transmission, both in utero and milk
Clinical features:
Fourty per cent of patients present with otolaryngological symptoms. Some of which are
in the mouth.
1. Kaposi sarcoma: Raised or flat red patches which may ulcerate on mucous membrane.
Histological examination shows sarcomatous change. Thirty percent of AIDS patients
have Kaposi sarcoma and 25 % present with it.
2. Hairy leukoplakia: White patches on the tongue which regress and recur. Histology it
shows leukoplakia with marked keratinization.
3. Candidiasis: Appearance in a young male must be viewed with suspicion unless there
are obvious predisposing factors.
4. Persistent cervical lymphadenopathy: It may be associated with generalized lymphade-
CONCEPT BOOK OF DISEASES OF EAR, NOSE & THROAT BY FAHIM AWAN
471 Part H General Topics

nopathy, pyrexia, weight loss and lypmhopenia.


5. Nonspecific pharyngitis, tonsillitis or rhinosinusitis may also be the presenting symp-
toms.
Investigations:
Diagnostic tests are based on identification of viral antigens and antibodies. Antibodies are
found after 02-12 weeks of infection.
1. ELISA test: It is a very sensitive test for HIV.
2. Western blot test: It is specific for HIV antibodies. It is a confirmatory test.
3. CD4 count: Decreasing count indicates immunosuppression and corresponding risk of
development of opportunistic infection.
4. p24 antigen assay: It detects p24 core protein of HIV. The test is positive even before
seroconversion.
5. PCR test: It determines HIV-RNA.
Management: Avoidance of exposure by adopting universal precautions is the best
management.
1. Treatment of related infections.
2. Symptomatic treatment of pain and control of bleeding.
3. Relief of aerodigestive tract obstruction.
4. Kaposi sarcoma treated by radiotherapy/chemotherapy.
5. Preventing transmission to hospital staff:
a. Mandatory HIV testing of medical staff.
b. Immediate administration of AZT (azidothymidine) administration on contact.
c. Staff with open wounds to stay away.
d. Patient categorizing into high/low risk.
e. Minimum staff should wash up for surgery.
f. Avoid direct contact with blood/secretions.
g. Cautious use of sharp instruments/needles.
h. Use of heavy double impermeable gloves.
i. Special plastic impermeable gown/mask.
j. Thorough washing after surgery/degloving.
k. Proper hospital waste management protocols.
There are four major classes of antiretroviral drugs which are used in HIV:
1. Nucleoside reverse transcriptase inhibitors e.g., azidothymidine (AZT) with the name of

CONCEPT BOOK OF DISEASES OF EAR, NOSE & THROAT BY FAHIM AWAN


472 Part H General Topics

zidovudine, didanosine. These prevent conversion of DNA to RNA.


2. Non-nucleoside reverse transcriptase inhibitors e.g., delavirdine, nevirapine. These
prevent conversion of RNA to DNA.
3. Protease inhibitors: e.g., saquinavir, ritonavir. These prevent cleavage of viral proteins
into their functional forms by binding to viral protease enzyme.
4. Fusion inhibitors: e.g., enfuviritide. These interfere with entry of virus into target cells.
SELF ASSESSMENT
Scenario: A 35 years old male, IV drug abuser presented to OPD with multiple body
abscesses and contracture formations for last 03 months. On examination of oral cavity, oral
hygiene is very poor. There are also multiple ulcers on mucous membrane and white patches
on the tongue. There is generalized wasting of muscles and no hepatosplenomegaly. His labs
are, Hb: 8.6 g/dl, TLC: 3000 cmm.
1. What is your clinical diagnosis?
2. What is your treatment plan?
--------------------------------------------------------ANSWERS-----------------------------------------------------
Scenario: 1. AIDS 2. Symptomatic treatment.
------------------------------------------------------------------------------------------------------------------------
LYMPHOMA
This group of diseases very commonly presents in the head and neck region. The most
common sites involved are lymph nodes in the posterior triangle or around the carotid
bulb.
Nasopharynx, tonsils, thyroid or salivary gland may also be affected. The presence of
lymphoma in the neck is often only one clinical manifestation of a more widespread systemic
disorder. Such a finding may represent either primary or secondary disease.
The affected lymph nodes are often firm, rubbery and matted together. On cross section these
have a smooth and whitish appearance.

Classification of lymphoma:
Lymphoma is generally, divided into;
1. Hodgkin’s disease
2. Non-Hodgkin’s lymphoma
Etiopathology:
In most cases exact etiology is unknown.
There is a high Epstein-Barr virus titre in Burkitt’s lymphoma. It is B-cell lymphoma
of lymphoblastic type with evidence of chronic immunosuppression due to medicines.
T-cell lymphoma is associated with HTLV-I infection.
CONCEPT BOOK OF DISEASES OF EAR, NOSE & THROAT BY FAHIM AWAN
473 Part H General Topics

All lymphomas are malignant.

Histopathology:
Hodgkin’s disease:
On histology Reed-Sternberg (mirror image) cells are hallmark of Hodgkin’s disease.
Histologically the disease is classified into four subtypes according to the Rye classification
which was modified from Lukes Butler system;
1. Lymphocytic predominant
2. Nodular sclerosis
3. Mixed cellularity
4. Lymphocytic depleted
These subtypes also give an indication of prognosis. Lymphocytic predominant and nodular
sclerosing types have a better prognosis than the other two categories.
However, clinical staging is more important than the histological subtypes in determining
the prognosis of Hodgkin’s disease.
Non-Hodgkin’s lymphoma:
Kiel’s classification (modified from Lennert) of non-Hodgkin’s lymphoma is usually used. It
is divided into low-grade and high-grade lymphomas.
Low-grade lymphomas usually contain mature cells, such as centrocytic lymphoma,
lymphocytic lymphoma, immunocytoma, plasmocytoma etc.
High-grade lymphomas contain premature cells such as centroblastic lymphoma,
lymphoblastic lymphoma, immunoblastic lymphoma etc.
Non-Hodgkin’s lymphoma is classified on the basis of predominant cell type involved
and subdivided according to whether the neoplastic cells are arranged in nodules or in a
diffuse pattern. A nodular pattern is associated with a better prognosis.
Features which favor low-grade rather than high-grade are;
a. A nodular rather than a diffuse histological pattern
b. A lymphocytic rather than a histiocytic morphology.
c. Well differentiated rather than poorly differentiated cells.
Histological grading is more important than the clinical staging in determining the
prognosis of non-Hodgkin’s lymphoma.

CONCEPT BOOK OF DISEASES OF EAR, NOSE & THROAT BY FAHIM AWAN


474 Part H General Topics

Differences between Hodgkin’s disease and non-Hodgkin’s lymphoma


Sr. Features Hodgkin’s disease Non-Hodgkin’s lymphoma
No.
1 Prognosis Excellent Poor
2 Histology Reed-Sternberg (mirror image) No Reed-Sternberg (mirror
cells image) cells
3 Type Nodal disease Extra nodal disease
4 Localization Localized lymph nodes Non localized involvement of
structures
5 Involvement Involvement of contiguous Involvement of non-contiguous
lymph nodes lymph nodes
6 Prognostic Clinical staging is more Histological grading is more
factor important important
7 Complications Rare, results from pressures More due to dissemination of
on adjacent structures e.g., disease to GIT, nasopharynx,
obstructive jaundice or spinal thyroid, bones and soft tissues.
cord compression.
8 Classification Rye classification Kiel’s classification
for histological
grading
9 Classification Ann-Arbor classification Ann-Arbor classification
for clinical
staging
Clinical/anatomical staging:
Usually Ann-Arbor classification is used both for Hodgkin’s disease and non-Hodgkin’s
lymphoma. In this clinical as well as pathological criteria of disease are considered.
Ann-Arbor classification of lymphoma:
Stage I:
Involvement of a single lymph node region (I) or of a single extralymphatic organ or site (I
E).
Stage II:
Involvement of two or more lymph node regions (number to be stated) on the same side of
the diaphragm (II) or localized involvement of an extralymphatic organ or site and in one or
more lymph node regions on the same side of the diaphragm (II E).
Stage III:
Involvement of lymph node regions on both sides of the diaphragm, which may also be
CONCEPT BOOK OF DISEASES OF EAR, NOSE & THROAT BY FAHIM AWAN
475 Part H General Topics

accompanied by localized involvement of extralymphatic organ or site (III E), by


involvement of spleen (III S), or both (III E+S).
Stage IV:
Diffuse or disseminated involvement of one or more extralymphatic organ tissues with or
without associated lymph node enlargement. The reason for classifying the patient as stage
IV is identified further by specifying the site.
The staging is then further subdivided into cases without systemic symptoms (substage
A), and those cases with weight loss greater than 10% during the 6 months preceding
presentation or with unexplained fever or sweating (substage B).
Clinical features:
Hodgkin’s disease:
It is basically a nodal disease. There is localized lymph node involvement. There is
involvement of contiguous lymph nodes.
Patient usually presents with a painless lymph node enlargement (lump) in either the neck,
axilla or groin.
Very occasionally enlarged lymph nodes will give rise to complications resulting from
pressure such as obstructive jaundice or spinal cord compression.
Prognosis in cases of Hodgkin’s disease is excellent. That is why most of the
pathologists do not include it in lymphoma.
Non-Hodgkin’s lymphoma:
It is usually an extranodal disease. There is non-localized involvement of the structures.
There is involvement of non-contiguous lymph nodes.
Presentation is similar to Hodgkin’s disease, but pattern of the disease is disseminated.
The extranodal structures are involved much more frequently. Such sites include
gastrointestinal tract, nasopharynx, bones, soft tissues and thyroid gland. True localized
disease is, therefore, much less common than Hodgkin’s disease. More than 50% of patients
have extranodal involvement at presentation.
Prognosis in cases of non-Hodgkin’s lymphoma is poor as compared to Hodgkin’s
disease.
Investigations:
Diagnosis of non-Hodgkin’s lymphoma is usually made from biopsy of lymph node but
occasionally an extranodal site or bone marrow examination reveals the nature of disease.
Following investigations are done in cases of lymphoma.
1. Biopsy of lymph nodes is the most important investigation to find out type of lymphoma
and its histological grading.
2. Immunohistochemistry is done to find out exact histological type of lymphoma.
CONCEPT BOOK OF DISEASES OF EAR, NOSE & THROAT BY FAHIM AWAN
476 Part H General Topics

3. Radiology:
a. Whole body scan to find out area of active disease.
b. X-ray chest PA view to demonstrate hilar lymph node involvement, mediastinal
disease and parenchymal lung disease.
c. Ultrasound, CT/MRI of chest, abdomen and pelvis to detect involvement of lymph
nodes, spleen or other extralymphatic structures.
4. Hematology: CBC, ESR, liver function tests, renal function tests, serum uric acid (as
hyperuricemia may result from early stages of treatment).
Treatment:
1. Hodgkin’s disease:
For stage I and II, radiotherapy is the treatment of choice.
For stage III and IV, chemotherapy is the treatment of choice.
For disease above the diaphragm, ‘mantle technique’ of radiotherapy is used. It covers
the cervical, clavicular, axillary, mediastinal and hilar lymph nodes.
For diseases below the diaphragm, ‘inverted Y technique’ of radiotherapy is used. It
covers the para-aortic nodes from the level of diaphragm to the bifurcation of the aorta,
together with pelvic and inguinal nodes.
Some clinicians advocate chemotherapy along with radiotherapy (mantle + inverted Y
technique) in stage III. In stage IV disease, combination chemotherapy is used in the form of
following regimens. The regimens include;
a. MOPP i.e., Nitrogen mustard, Oncovin, Procarbazine and Prednisolone
b. MVPP i.e., Nitrogen mustard, Vinblastine, Procarbazine and Prednisolone
2. Non-Hodgkin’s lymphoma:
This disease is usually more widespread at the time of presentation and, therefore, unsuitable
for radiotherapy.
For stage I and II, radiotherapy is the treatment of choice especially in low-grade malignancies.
For stage III and IV, chemotherapy is the treatment of choice. The regimen depends on
histological classification;
a. For low-grade tumors, alkylating agents are given.
b. For high-grade tumors, aggressive combination chemotherapy is instituted in the
form of following regimens;
COPD i.e., Cyclophosphamide, Oncovin, Prednisolone and Doxorubicin
CHOP i.e., Cyclophosphamide, Hydroxydaunorubicin, Oncovin, Prednisolone.
SELF ASSESSMENT
CONCEPT BOOK OF DISEASES OF EAR, NOSE & THROAT BY FAHIM AWAN
477 Part H General Topics
Q. 1. Extranodal involvement is more commonly a manifestation of
a. Cytomegalovirus d. Non-Hodgkin’s lymphoma

b. Hodgkin’s disease e. all of the above


c. Infectious mononucleosis
Q. 2. All of the following are histologic types of Hodgkin’s disease except
a. lymphocyte predominant d. nodular sclerosis

b. lymphocyte depletion e. all of the above


c. mixed cellularity
Q. 3. Which one of the following histological type of Hodgkin’s disease has most favorable
prognosis?
a. Lymphocyte depletion d. Nodular sclerosis

b. Lymphocyte predominant e. None of the above


c. Mixed cellularity
Q. 4. Which of the following statements is correct about Hodgkin’s disease?
a. Clinical staging is more important than than the clinical staging in determining
the histological grading in determining the the prognosis
prognosis d. Prognosis is poor as compared to
b. Hematological investigations are most non-Hodgkin’s
important for diagnosis e. None of the above
c. Histological grading is more important
Q. 5. Which of the following statements is incorrect about lymphoma?
a. Biopsy is the most important part of for disease below the diaphragm
investigation d. Mantle technique is used
b. Immunohistochemistry is done to find for disease above the diaphragm
out clinical stage of disease
e. Whole body scan to find out areas of
c. Inverted Y technique is used active disease
Q. 6. Which of the following statements is incorrect about lymphoma?
a. Stage I is treated with radiotherapy d. Stage IV is treated with radiotherapy

b. Stage II is treated with radiotherapy e. Stage IV is treated with surgery


c. Stage III is treated with chemotherapy
Q. 7. All are included in the treatment of lymphoma except
a. cyclophosphamide d. oncovin

b. doxorubicin e. prednisolone

c. heparin
Scenario: A 22 years old boy presented to ENT OPD with the complain of progressively
increasing swelling in lower neck for last 04 months. He has mild fever but no history of
CONCEPT BOOK OF DISEASES OF EAR, NOSE & THROAT BY FAHIM AWAN
478 Part H General Topics

weight loss or vomiting. On examination, there is 03 cm rounded swelling in left


supraclavicular region attached to muscle. Bilateral axillary lymph nodes are also enlarged.
His abdomen is soft with no hepatosplenomegaly.
1. What is the most probable clinical diagnosis?
2. What investigations will you carry out?
3. What are two different types of this pathology?
--------------------------------------------------------ANSWERS-----------------------------------------------------
Q.1. d 2. e 3. b 4. a 5. b
6. d 7. c
Scenario: 1. Lymphoma 2. Biopsy of lymph node, immunohistochemistry, whole body scan
3. Hodgkin’s and non-Hodgkin’.
---------------------------------------------------------------------------------------------------------------------------
LYMPHOMA
LEVEL/REGION OF LYMPH NODE INVOLVEMENT
There are a number of lymph node levels/regions within the neck. These lymph nodes
represent the first echelon. Site of metastasis from head and neck primary tumor are described
below;
Level I: Submental and submandibular group:
It consists of submental and submandibular group of lymph nodes.
Level II: Upper jugular group:
It consists of lymph nodes located around upper one third of internal jugular vein,
extending from the skull base down to the level of hyoid bone.
Level III: Middle jugular group:
It consists of lymph nodes located around middle one third of internal jugular vein
extending from superiorly (bottom of level II), down to the cricothyroid membrane.
Level IV: Lower jugular group:
It consists of lymph nodes located around lower one third of internal jugular vein
extending from cricothyroid membrane down to the level of clavicle.
Level V: Posterior triangle group:
It consists of lymph nodes located around lower half of the spinal accessory nerve and the
transverse cervical artery.
Supraclavicular nodes are also included in this group. Posterior boundary is anterior border
of trapezius. Anterior boundary is anterior border of sternomastoid muscle.

CONCEPT BOOK OF DISEASES OF EAR, NOSE & THROAT BY FAHIM AWAN


479 Part H General Topics

Level VI: Anterior group (visceral compartment):


It consists of lymph nodes around the midline visceral structures of the neck. It extends
form hyoid bone superiorly to suprasternal notch inferiorly. It contains parathyroid,
paratracheal, prelaryngeal and precricoid lymph nodes.
Level VII: Superior mediastinal group:
It consists of lymph nodes in the superior mediastinum.
Classification of regional lymph nodes (N)
Node Description
Nx Regional lymph nodes cannot be assessed
N0 No regional lymph node metastasis
N1 Metastasis in a single ipsilateral lymph node 03 cm or less in greatest dimension
(without extranodal extension)
N2a Metastasis in a single ipsilateral lymph node, more than 03 cm but not more than
06 cm in greatest dimension (without extranodal extension)
N2b Metastasis in multiple ipsilateral lymph nodes, none more than 06 cm in greatest
dimension (without extranodal extension)
N2c Metastasis in bilateral or contralateral lymph nodes, none more than 06 cm in
greatest dimension (without extranodal extension)
N3a Metastasis in a lymph node more than 6 cm in greatest dimension (without
extranodal extension)
N3b Metastasis in a lymph node more than 6 cm in greatest dimension with
extranodal extension
Classification of distant metastasis (M)
Mx Distant metastasis cannot be assessed
M0 No distant metastasis
M1 Distant metastasis present

Staging:
Definition:
It is the grouping together of TNM features which may share a level of prognosis or certain
treatment.
TNM/Clinical staging
Stage 0 Tis, N0, M0
Stage I T1, N0, M0

CONCEPT BOOK OF DISEASES OF EAR, NOSE & THROAT BY FAHIM AWAN


480 Part H General Topics

Stage II T2, N0, M0


Stage III T3, N0, M0/T1, T2 or T3 with N1, M0
Stage IV A T4, N0, M0 or T4, N1, M0
Stage IV B Any T, N2, M0 or Any T, N3, M0
Stage IV C Any T any N, M1
TREATMENT OF NECK NODES
Generally, treatment of malignant lymph nodes is as follows;
N1:
1. When primary is small, the treatment of choice is radiotherapy.
2. If in a neck dissection specimen, there is extranodal spread of disease or histological
evidence of involvement of multiple levels of lymph nodes, then neck and primary site
should be treated by postoperative radiotherapy.
3. If radiotherapy for primary site is to be given before neck dissection, then regional nodes
should be included in radiotherapy.
N2 and N3:
These are best managed by a combination of surgery (radical neck dissection) and
postoperative radiotherapy.
SELF ASSESSMENT
Q. 1. The lymph nodes of the level I are found in
a. around lower one third of internal c. around upper one third of internal
jugular vein jugular vein

b. around middle one third of internal d. submental and submandibular region

jugular vein
e. superior mediastinum
Q. 2. The lymph nodes of the level II are found in
a. around lower one third of internal c. around upper one third of internal
jugular vein jugular vein

b. around middle one third of internal d. submental and submandibular region

jugular vein
e. superior mediastinum
Q. 3. The lymph nodes of the level III are found in
a. around lower one third of internal c. around upper one third of internal
jugular vein jugular vein

b. around middle one third of internal d. submental and submandibular region


jugular vein
e. superior mediastinum

CONCEPT BOOK OF DISEASES OF EAR, NOSE & THROAT BY FAHIM AWAN


481 Part H General Topics
Q. 4. The lymph nodes of the level IV are found in
a. around lower one third of internal c. around upper one third of internal
jugular vein jugular vein

b. around middle one third of internal d. submental and submandibular region

jugular vein
e. superior mediastinum
Q. 5. The lymph nodes of the level V are found in

a. anterior triangle d. posterior triangle

b. carotid triangle e. superior mediastinum


c. occipital triangle
Q. 6. The lymph nodes of the level VI are found in
a. around lower one third of internal c. around upper one third of internal
jugular vein jugular vein
b. around middle one third of internal d. midline visceral structures of neck
jugular vein e. superior mediastinum
Q. 7. The lymph nodes of the level VII are found in
a. around lower one third of internal c. around upper one third of internal
jugular vein jugular vein
b. around middle one third of internal d. midline visceral structures of neck
jugular vein e. superior mediastinum
Q. 8. Midline nodes are considered
a. bilateral d. all of the above

b. contralateral e. none of the above


c. ipsilateral
ANSWERS--------------------------------------------
-------------------------------------------------------- ---------
Q.1. d 2. c 3. b 4. a 5. d
6. d 7. e 8. c
---------------------------------------------------------------------------------------------------------------------------
NECK DISSECTION
Definition:
It is an operation to remove lymph nodes and surrounding fibrofatty tissues from the neck. It
is done to eradicate metastasis to the cervical lymph nodes from malignancies of the head and
neck region.

1. Radical neck dissection:


In a classical radical neck dissection following important structures are removed;
a. Eleventh cranial nerve
CONCEPT BOOK OF DISEASES OF EAR, NOSE & THROAT BY FAHIM AWAN
482 Part H General Topics

b. Internal jugular vein


c. Sternomastoid muscle
There are different types of radical neck dissections;
2. Modified neck dissection:
It is of the following types:
Type I: Only one structure i.e., eleventh cranial nerve is conserved, whereas internal
jugular vein and sternomastoid muscle are sacrificed.
Type II: Two structures i.e., eleventh cranial nerve and internal jugular vein are
conserved, whereas sternomastoid muscle is sacrificed.
Type III: All three structures are conserved.
Type 3 modified neck dissection is also known as:
a. Comprehensive neck dissection
b. Functional neck dissection
3. Selective neck dissection:
It is of the following types:
a. Supraomohyoid neck dissection (level 1 to 2 lymph nodes are removed).
b. Lateral neck dissection (level 2 to 4 lymph nodes are removed).
c. Posterolateral neck dissection (level 2 to 5, sub-occipital and postauricular lymph
nodes are removed).
d. Anterior compartment neck dissection (level 4 lymph nodes are removed).

Complications of neck dissection:


1. Hemorrhage due to damage of julgar vein, subclavian vein, pharyngeal vein,
superior and inferior thyroid pedicles.
2. Chyle leak/fistula due to damage to thoracic duct if dissection is carried on
left side.
3. Nerve lesions such as phrenic nerve, sympathetic trunk, brachial plexus,
vagus, accessory, facial, hypoglossal and lingual nerve.
4. Facial edema
5. Frozen shoulder due to loss of innervation of trapezius muscle.
6. Infection and wound breakdown
7. Rupture of carotid arteries
8. Recurrence in skin or gland
9. Cerebral oedema may come after bilateral neck dissection or if large
dominant internal juglar vein has been removed.
10. Visual loss which is temporary initially but may become permanent in cases
of bilateral radical neck dissection.

SELF ASSESSMENT
Q. 1. In classical/radical neck dissection all of the following structures are removed except
a. all lymph nodes extending from the hyoid c. spinal accessory nerve
and mandible above to the clavicle below d. sternomastoid
e. vagus nerve
b. internal jugular nerve
Q. 2. In type I modified neck dissection the structure conserved is
a. all lymph nodes extending from the hyoid region
and mandible above to the clavicle below c. internal jugular nerve
d. spinal accessory nerve
b. all lymph nodes in the submandibular e. sternomastoid
Q. 3. In type II modified neck dissection, the structure conserved is
a. all lymph nodes extending from the below

hyoid and mandible above to the clavicle


b. internal jugular nerve
CONCEPT BOOK OF DISEASES OF EAR, NOSE & THROAT BY FAHIM AWAN
483 Part H General Topics

c. spinal accessory nerve e. both b & c


d. sternomastoid
Q. 4. In type III modified neck dissection, the structure conserved is
a. internal jugular nerve d. all of the above

b. spinal accessory nerve e. none of the above


c. sternomastoid
Q. 5. Type III modified neck dissection is also known as
a. comprehensive neck dissection d. all of the above

b. functional neck dissection e. none of the above

c. selective neck dissection

-------------------------------------------------------- ANSWERS-----------------------------------------------------
Q.1. e 2. d 3. e 4. d 5. d
--------------------------------------------------------------------------------------------------------------------------
-
OCCULT PRIMARY
Patient presenting with a metastatic cervical lymph node from an unknown primary source is
said to have an occult primary.
The following primary sites should always be under suspicion as a source of metastatic
lymph nodes in the cervical region.
1. Nasopharynx
2. Tonsils
3. Base of tongue
4. Thyroid
5. Hypopharynx
Search for primary disease:
When a patient presents with a cervical mass which is considered to be a metastatic node, the
search for primary tumor should follow an established sequence of investigations. The order
of investigations may have to be tailored to the individual. After a careful history and a
general physical examination, nasal cavity, oral cavity, oropharynx, hypopharynx, larynx and
nasopharynx should be carefully investigated. Examination is further facilitated by palpation
in areas where this can be carried out bimanually. Palpation is an important part of
examination. It should be performed during examination of head and neck where a finger
may reach likening its value to that of having the added facility of a third eye.
Investigations:
1. Sinus and chest X-rays, contrast study of the upper and lower gastrointestinal tract.
CONCEPT BOOK OF DISEASES OF EAR, NOSE & THROAT BY FAHIM AWAN
484 Part H General Topics

2. Panendoscopy (oropharyngoscopy, hypopharyngoscopy, laryngoscopy, tracheobron-


choscopy, esophagoscopy and nasopharyngoscopy) under general anaesthesia.
3. Random biopsies from nasopharynx, tonsils and base of tongue should be taken if no
lesion detected.
4. FNAC, CT/MRI, radiolabelled antibodies.
When no evidence of a primary tumor has been found, even after a full diagnostic
evaluation, the patient falls into the category of occult primary tumor.

Treatment:
If the lesion appears resectable, surgery should be performed in the form of radical neck
dissection as a treatment of choice. However, for patients with multiple enlarged unilateral
nodes, radical neck dissection is combined with postoperative radiotherapy.
Second primary tumor:
Cancer of the head and neck tends to be multiple in both time and space. The incidence of a
second primary tumor is about 10%. The interval between the diagnosis of the first cancer
may be long but mostly present within 03 years. The most common site for a second primary
is another site in the head and neck, particularly in the mouth. The oropharynx and the lungs
are also other common sites for second primary tumor.
RADIOTHERAPY
Definition:
It is defined as treatment with ionizing radiations.
Now radiotherapy is being extensively used in
the treatment of E.N.T, head and neck
malignancies.
Indications of radiotherapy:
For the treatment of epithelial malignancies in
head and neck region, a number of options are
available. These options are surgery, radiotherapy,
chemotherapy or combination of above.

Radiotherapy is indicated in following cases: Radiotherapy


1. When cure is likely with preservation of function/organ.
2. When surgery is contraindicated because of some systemic illness
3. When surgery is refused by patient.
Contraindications of radiotherapy:
1. Previous radiotherapy
2. Cartilage invasion
3. Active perichondritis

CONCEPT BOOK OF DISEASES OF EAR, NOSE & THROAT BY FAHIM AWAN


485 Part H General Topics

Complications of radiotherapy:
A. Early:
1. Radiation sickness
2. Mucositis
3. Moniliasis
4. Dryness of mucous membrane
5. Xerostomia
6. Hematopoietic suppression
7. Skin reactions (erythema)
B. Late:
1. Permanent xerostomia
2. Decaying of teeth
3. Osteoradionecrosis
4. Transverse myelitis
5. Trismus
6. Endocrinal defects and malignancy
Clinical radiotherapy:
1. Curative radiotherapy: Also called radical radiotherapy and is given with intent to cure
the disease. Although time consuming and unpleasant but it has two main distinct advantages.
Firstly, it preserves function. Secondly it can achieve control particularly in small lesions
where the results are comparable to surgery and provides increased disease-free interval.
2. Palliative radiotherapy: Although basically given with palliative intent, when given in
radical doses, it has the remarkable ability to achieve disease control. Palliative situations
include too large a primary tumor, positive distant metastasis or the general condition of the
patient does not permit radical surgery or radical radiotherapy.
3. Preoperative radiotherapy: Preoperative radiotherapy has the ability to decrease the
size of primary tumor which is then amenable to surgery. This therapy causes fibrosis,
thereby blocking lymphatic spread at the time of surgery without much interference with
tissue oxygenation. In addition, it achieves an excellent control of microscopic disease in
neck nodes. It however, decreases tissue vitality, delays healing and tissue fibrosis. It leads to
difficult tumor margin identification at the time of surgery. It may increase the incidence of
fistula, flap necrosis and carotid artery rupture post-operatively.
4. Postoperative radiotherapy: In contrast, post operative radiotherapy is able to achieve
better control due to removal of tumor mass, identification of residual disease and exact areas
to be covered. Tumor margins are easily identified at surgery with good healing. There are
lesser chances of fistula formation, flap necrosis or other complications. It has the
CONCEPT BOOK OF DISEASES OF EAR, NOSE & THROAT BY FAHIM AWAN
486 Part H General Topics

distinct ability to achieve control of microscopic disease not encompassed in surgical fields.
However, reduced blood supply and the oxygenation may make it ineffective when used
postoperatively.
Mechanism of action:
Basic aim is to achieve maximum radiation dose to the tumor for the purpose of cure and
minimum exposure to adjacent structures.
It damages DNA of the cells. Healthy cells undergo repair, whereas malignant cells do
not have this capacity and are, therefore, killed.
Head and neck squamous cell carcinoma are less sensitive to radiotherapy as compared to
lymphoma, thereby require more doses. Close approximation of vital structure like brain stem
in head and neck require precision and controlling the exposure.
Radiotherapy patterns:
1. External beam radiotherapy/teletherapy: Photon or electron beams are projected by
kilo voltage or linear accelerator machine onto the tumor. These energies containing photon
beams have poor penetration power, about 01 cm, and are useful for superficial tumors.
Linear accelerators produce high energy million-volt beams which have enhanced penetration
power and gradual dose fall off with added advantage of well-defined edge and equal
absorption in different tissues. Electrons are used for superficial lesions or as dose boosters.
Beneficial effects of the neutron therapy are not universally accepted.
2. Interstitial radiotherapy/brachytherapy: It works on the principle of placing a
radioactive isotope material in close contact with the tumor. Thus, giving maximum dose to
the tumor locally and sparing the adjacent regions. The radioactive material can be implanted
in the form of needles, wires or seeds of radium which are removable or radon implants
which are permanent.
Intracavity implant is another type in which the radioactive isotope is placed in the
body cavity near the tumor e.g., carcinoma maxillary sinus or nasopharynx. The isotope
can also be moulded to fit the site of implant.
Types of radiation:
1. Photon beam therapy: Photon is a bundle of electromagnetic radiations. It includes
high energy X-ray and gamma rays from radioactive elements. It is the most commonly used
type of radiotherapy.
2. Electron beam therapy: Electron beam is emitted by linear accelerator. These increase
the radiation dose to the target due to sharp rise and fall of the dose. This type of radiotherapy
is the second most common in practice.
3. Radioactive isotopes: In these radioactive particles like neutrons or protons are given
intravenously or orally. These particles build up in malignant tissues generating high local
dose. An example is radioactive iodine for thyroid malignancies.
Radiotherapy sources:
CONCEPT BOOK OF DISEASES OF EAR, NOSE & THROAT BY FAHIM AWAN
487 Part H General Topics

1. Kilovolt machine: It is basically an X-ray machine. It is now obsolete from clinical


radiotherapy protocols except for small superficial lesions. It can produce energy of upto 300
KV. The energy level is measured in electron volts (ev) thereby 1000 ev equalled 1KV.
2. Linear accelerator: Also called supervoltage or megavoltage machines which are
electric powered producing radiations up to 20 million volts (MV). It can produce 04 MV
beam for superficial lesions and 20 MV beam for deep homogenous tissue penetration.
3. Betatron or microtron: These are newer generation megavoltage electric powered
machines which are safe to operate. These can produce radiation beam of both photons and
electrons by simply changing the metal target. These can achieve 30 MV radiation dose.
4. Cobalt accelerator: This accelerator utilizes cobalt 60 radioactive source. It is electric
powered, safe to operate and can generate gamma rays ranging from 1-1.5 MV. Although
used commonly, it has the inherent disadvantage of natural decay requiring replacements.
5. Radioactive isotopes: Radioactive isotopes are economical and can be placed near the
tumor to deliver maximum local dose. Different implant materials include radium needles,
cesium pellets, iridum, gold grains, iodine seeds and cobalt needles.
Therapy ratio:
Ratio in curative dose and dose which causes damage is less. Therefore, tumor requiring cure
needs more doses at the expense of complications.
Measurements:
Radiation dose is measured in RAD (Radiation Absorbed Dose) which is equal to 100 ergs.
Now dose is internationally measured in Gray (Gy) which is equal to 100 rads or one centi
Gray (cGy) is equal to one rad.
SELF ASSESSMENT
Q. 1. Teletherapy is otherwise known as
a. external beam radiotherapy d. all of the above

b. interstitial radiotherapy e. none of the above


c. intracavitary brachytherapy
Q. 2. Brachytherapy is otherwise known as
a. external beam radiotherapy d. all of the above

b. interstitial radiotherapy e. none of the above

c. teletherapy
Q. 3. One rad is equivalent to

a. 0.001 Gy d. 1.0 Gy

b. 0.01 Gy e. 10.0 Gy
c. 0.1 Gy
Q. 4. All are common complications of radiotherapy of nasopharyngeal carcinoma
CONCEPT BOOK OF DISEASES OF EAR, NOSE & THROAT BY FAHIM AWAN
488 Part H General Topics

except

a. aphthous ulceration d. trismus

b. moniliasis e. xerostomia

c. transverse myelitis
-------------------------------------------------------- ANSWERS-----------------------------------------------------
Q.1. a 2. b 3. b 4. a
--------------------------------------------------------------------------------------------------------------------------
-
CHEMOTHERAPY
Mechanism of action:
The tumor cells have a cell cycle and an intervening resting phase. Chemotherapy has anti
cellular properties against normal and cancer cells. Few drugs act throughout the cycle and a
few act on resting phase.
Drugs are tested as phase 1, phase 2, phase 3 clinical trials.
1. In phase 1 trial, dose is gradually increased to assess toxic levels.
2. In phase 2 trial, the toxicity along with clinical response is assessed.
3. In phase 3 trial, the main point of interest is in survival figures.
Aims of chemotherapy:
1. Treatment of advanced, recurrent or disseminated disease.
2. Relief of symptoms.
3. Control the micro/occult metastasis after the surgery/radiotherapy.
Uses of chemotherapy:
Chemotherapy can be used alone for palliation or in combination with other treatment options
like radiotherapy and surgery. Usual head and neck malignancies are squamous cell
carcinomas which limit the use of chemotherapy due to low sensitivity. Hodgkin’s and non-
Hodgkin lymphomas in head and neck region are however, sensitive and show very good
response to chemotherapy.
If used before primary therapy, it decreases tumor load and reduces occult metastasis at
the time of surgery or radiotherapy. However, it does not seem to improve the overall
survival rates.
Drugs commonly used in chemotherapy:
Single drug or combination of following different drugs is usually used.
1. Antifolates e.g., methotrexate
2. Anti-metabolites e.g., 5-flourouracil
3. Alkaloids e.g., vincristine

CONCEPT BOOK OF DISEASES OF EAR, NOSE & THROAT BY FAHIM AWAN


489 Part H General Topics

4. Alkylating agents e.g., cisplatin


Toxic effects of chemotherapy:
1. Marrow suppression with decrease in blood count
2. Thrombocytopenia
3. Renal toxicity
4. Oral ulceration
5. Alopecia
6. Transient skin rash
7. Cholestatic jaundice
8. Severe diarrhea
Scheduling of chemotherapy:
It may be given before, during or after definitive treatment.
1. Neoadjuvant chemotherapy: If chemotherapy is given before radiotherapy it is
known as Neoadjuvant chemotherapy. It is also known as induction chemotherapy, up-
front chemotherapy or anterior chemotherapy.
2. Synchronous chemotherapy: If chemotherapy is given during radiotherapy it is
known as synchronous chemotherapy. It is also known as concomitant chemotherapy.
3. Adjuvant chemotherapy: If chemotherapy is given after radiotherapy it is known as
adjuvant chemotherapy. It is also known as subsequent or posterior chemotherapy.

SELF ASSESSMENT
Q. 1. Neoadjuvant chemotherapy is given

a. after radiotherapy d. both before and during radiotherapy

b. before radiotherapy e. during radiotherapy


c. both before and after radiotherapy
Q. 2. Synchronous chemotherapy is given
a. after radiotherapy d. both before and during radiotherapy

b. before radiotherapy e. during radiotherapy

c. both before and after radiotherapy


Q. 3. Adjuvant chemotherapy is given

a. after radiotherapy d. both before and after radiotherapy

b. before radiotherapy e. both before and during radiotherapy


c. during radiotherapy

CONCEPT BOOK OF DISEASES OF EAR, NOSE & THROAT BY FAHIM AWAN


490 Part H General Topics
-------------------------------------------------------- ANSWERS-----------------------------------------------------
Q.1. b 2. e 3. a
--------------------------------------------------------------------------------------------------------------------------
-

CRYOSURGERY
Rapid cooling of tissues to bring it below freezing level and then gradual defrosting or
warming is the basic principle of cryosurgery by which tissues are destroyed.
Cryoprobe:
It is an extension of the system through which a compressed gas when passed, get expanded
resulting in sub zero temperatures. It can achieve -30 to -70 degrees Celsius temperature.
Special thermometer attachment gives the overall temperature around a probe.
Agents used:
1. Nitrogen in liquid form
2. Nitrous oxide snow
3. Carbon dioxide snow
Mechanism of action:
It works on the principle of cellular dehydration and denaturation of proteins along with
arresting vascular flow and cellular respiration. Cryoprobe is applied to the tissue for 05-10
minutes under anesthesia. It is allowed to defrost slowly. Procedure may need repetitions.
Clinical applications:
1. Carcinoma near cartilage
2. Nasal turbinate reduction
3. Leukoplakia
4. Oral and cheek tumors
5. Recurrent tumors
6. Angiofibroma
7. Glomus tumor
Special features:
1. Major anesthesia avoidable
2. Minimal blood loss
3. Manageable as O.P.D procedure
4. Minimal postoperative scaring
5. Makes tissue useless for histology

CONCEPT BOOK OF DISEASES OF EAR, NOSE & THROAT BY FAHIM AWAN


491 Part H General Topics

6. Margin of tumors cannot be seen


LASER
It is an acronym for Light Amplification by Stimulated Emission of
Radiation.
Basic characteristic of laser beam is that it is an intense beam of pure
monochromatic light which does not diverge and the light waves are of the
same length, travel in the same direction and in phase, rising and falling
together. This beam can be focused by a concave mirror to a fine point
producing very high energy levels. Difference between ordinary light beam
and laser beam is that the ordinary light is just like different people travelling
on a road in different directions with different speed and to different Carbon dioxide
destinations. Laser beam is just like an army unit in which all the men are laser machine
travelling in the same direction stepping together, with same speed and
towards same destination.
Advantages of laser surgery:
1. Bloodless dissection
2. Dissection is extremely precise
3. Minimal damage to adjacent normal tissues
4. Minimal instrumentation
5. Immediate tissue destruction
Types of lasers and their usage:
1. CO2 laser:
It has got a wavelength of 600 nm which is invisible. It requires an aiming beam of helium
neon laser. It is the most commonly used laser in ENT surgery. CO2 laser is extremely useful
for treatment of vocal nodule, laryngeal polyp, laryngeal web, subglottic stenosis, bilateral
vocal cord paralysis and recurrent respiratory papillomatosis.
2. Nd: YAG laser:
It has got a wavelength of 1060 nm. It is used in the treatment of esophageal, tracheal and
endobronchial tumors.
3. KTP-532 laser:
It has got a wavelength of 532 nm. It is used for microsurgery of ear.
4. Argon laser:
It has got a wavelength of 488-514 nm. It is used for stapedectomy, welding spots of
tympanoplasty grafts, lysis of middle ear adhesions, hemangioma and telangiectasia.
Safety precautions:
1. All the staff in the operating room should be trained in the safety measures while using

CONCEPT BOOK OF DISEASES OF EAR, NOSE & THROAT BY FAHIM AWAN


492 Part H General Topics

laser.
2. All the exposed areas of the patient should be protected by the saline soaked gauze.
3. Only non-inflamed gases like halothane and enflurane should be used. While using CO2
laser, endotracheal tube should be wrapped with reflective metallic wire. Cuff of the en-
dotracheal tube is inflated with color saline.
4. Protection of eyes: All the personnel in the operation theatre should wear protective
glasses as laser beam may cause burn to the cornea and retina.
SELF ASSESSMENT
Scenario: A 40 years old male came to ENT OPD with the complaint of hoarseness of voice
for 20 days and he is a teacher by profession. On indirect laryngoscopic examination, there
are bilateral vocal nodules.
1. What are different treatment options?
2. What is best treatment of vocal nodules in modern era?
--------------------------------------------------------ANSWERS-----------------------------------------------------
Scenario: 1. Voice rest, speech therapy, microlaryngeal surgery, laser surgery 2. Excision
with CO2 laser.
--------------------------------------------------------------------------------------------------------------------------
-
FUNCTIONAL ENDOSCOPIC SINUS SURGERY (FESS)
Indications:
It was originally designed for the treatment of rhinosinusitis and nasal
polypi but now its use has been extended e.g.,
1. Allergic fungal sinusitis
2. Repair of cerebrospinal fluid leak FESS under progress

3. Dacryocystorhinostomy
4. Choanal atresia
5. Hypophysectomy
6. Management of epistaxis
7. Management of some benign and malignant tumors
Contraindications:
An acute infection leading to;
1. Meningitis
2. Epidural abscess
3. Cavernous sinus thrombosis

CONCEPT BOOK OF DISEASES OF EAR, NOSE & THROAT BY FAHIM AWAN


493 Part H General Topics

4. Visual loss
Complications:
1. CSF rhinorrhoea
2. Visual loss
3. Frontoethmoidal mucocele
4. Epistaxis
SELF ASSESSMENT
Scenario: A 35 years old male came to ENT OPD with the complaint of progressive bilateral
nasal obstruction, post nasal discharge and loss of sense of smell for last one year. On
examination, there are whitish grape like pedunculated glistening masses (most probably
ethmoidal polyp).
1. What is most modern surgical intervention?
2. What are other indications of this surgical procedure?
--------------------------------------------------------ANSWERS-----------------------------------------------------
Scenario: 1. Functional endoscopic sinus surgery 2. Repair of cerebrospinal fluid leak,
dacryocystorhinostomy, choanal atresia, hypophysectomy, management of epistaxis,
management of some benign and malignant tumors.
--------------------------------------------------------------------------------------------------------------------------
-
DIFFERENCES BETWEEN CT AND MRI

CT MRI
1. Patient is exposed to ionizing radiations Patient is not exposed to ionizing radiations
2. No potential for direct three plane Potential for direct three plane imaging
imaging
3. Chances of artifacts Absence of artifacts
4. Tissue characteristic is not so good Tissue characteristics is much better
5. Contrast enhancement is required for Superior density resolution without
density resolution contrast enhancement
6. Radioiodinated compound (conray) is Paramagnetic agent i.e., Gd-DTPA
used as a contrast (Gadolinium-Diethylene Triamine Penta
Acetic acid) is used as a contrast

Flap:
It is defined as a mass of tissue for grafting (usually including skin) only partly removed from
one part of the body so that it retains its own blood supply during its transfer to another site.
CONCEPT BOOK OF DISEASES OF EAR, NOSE & THROAT BY FAHIM AWAN
494 Part H General Topics

It is also defined as a piece of tissue which retains its own blood supply.
Graft:
Any tissue or organ for implantation or transplantation.
SYNDROMES IN E.N.T
1. Alport syndrome:
Glomerulonephritis, corneal dystrophy and progressive sensorineural hearing loss.
2. Apert syndrome:
Syndactyly and conductive hearing loss.
3. Branchio-oto-renal syndrome:
Branchial fistula, malformed pinna and conductive hearing loss.
4. Boerhaave syndrome:
Spontaneous rupture of esophagus due to severe vomiting.
5. CHARGE syndrome:
C-Coloboma of eye, H-Heart defects, A-Atresia of choana, R-Retarded growth, G-
Genital hypoplasia, E-Ear anomalies.
6. Collet Sicard syndrome:
Paralysis of 9th, 10th, 11th, and 12th cranial nerves.
7. Costen syndrome:
Abnormality of temporomandibular joint due to defective bite, otalgia, blocked ear,
tinnitus, vertigo.
8. Down syndrome:
Mental retardation, slanting eyes, short stature, ear canal stenosis and conductive hearing
loss.
9. Eagle syndrome also known as syndrome of elongated styloid process:
Discomfort in throat, pain in ear, upper neck and dysphagia. Cranial nerve 5th, 7th, 9th
and 10th nerve may be irritated.
10. Frey’s syndrome also known as gustatory sweating:
Sweating and flushing of skin over parotid area during mastication.
11. Grisel syndrome:
Atlanto-axial dislocation. It follows adenoid surgery/nasopharyngeal infection. Painful
neck torticollis.
12. Gradenigo’s syndrome:

CONCEPT BOOK OF DISEASES OF EAR, NOSE & THROAT BY FAHIM AWAN


495 Part H General Topics

Persistent ear discharge, 5th and 6th cranial nerve palsy (deep seated orbital pain).
13. Horner’s syndrome:
Ptosis, meiosis, anhydrosis, enophthalmos and loss of ciliospinal reflex.
14. Hugling Jackson syndrome:
Paralysisof 10th, 11th and 12th cranial
nerves.
15. Jugular foramen syndrome:
Paralysis of 9th, 10th, 11th cranial nerves. It is seen in carcinoma of nasopharynx, glomus
jugulare, large acoustic neuroma and thrombophlebitis of jugular bulb.
16. Kartagener’s syndrome:
Bronchiectasis, sinusitis and situs inversus.
17. Lermoyez syndrome:
Variant of Meniere’s disease. Initially hearing loss and tinnitus occur but after an episode
of vertigo follows relieving tinnitus and hearing loss.
18. Lemierre’s syndrome:
Thrombosis of internal jugular vein.
19. Pierre Robin syndrome:
Micrognathia, glossoptosis, cleft palate and sensorineural hearing loss.
20. Pendred syndrome:
Goiter and sensorineural hearing loss.
21. Ramsay Hunt syndrome:
Herpes zoster oticus with involvement of 5th ,7th ,8th cranial nerves.
22. Schmidt syndrome:
Paralysis of 10th and 11th cranial nerves.
23. Sjogren’s syndrome:
Xerostomia and xerophthalmia.
24. Sleep apnea syndrome:
Thirty apneic spells (each spell lasting at least 10 seconds) during a period of 7 hours sleep.
25. Treacher Collins syndrome:
Coloboma lower eyelid and mandibular & malar hypoplasia, malformed pinna,
malformed malleus and incus and conductive hearing loss.
26. Usher syndrome:
Rhinitis pigmentosa, night blindness, sensorineural hearing loss.

CONCEPT BOOK OF DISEASES OF EAR, NOSE & THROAT BY FAHIM AWAN


496 Part H General Topics

27. Vernet’s syndrome:


Paralysis of 9th, 10th and 11th
cranial nerves.
28. Villaret’s syndrome:
Paralysis of 9th, 10th, 11th, 12th cranial nerves plus Horner’s syndrome.
29. Von der Hoeve syndrome:
Osteogenesis imperfecta, blue sclera and mixed hearing loss.
30. Wallenberg syndrome:
Loss of pain and temperature sensation on same side of face and contralateral side of
limbs.
SELF ASSESSMENT
Q. 1. The cranial nerves which may be irritated in Eagle’s syndrome are
a. 5th and 7th cranial nerves d. 9th, 10th and 11th cranial nerves

b. 5th, 7th, 9th and 10th cranial nerves e. 11th and 12th cranial nerves
c. 9th and 10th cranial nerves
Q. 2. Horner’s syndrome may be associated with
a. Eagle syndrome d. Vernet’s syndrome

b. Schmidt syndrome e. Villaret’s syndrome


c. Tapia syndrome
Q. 3. Horner’s syndrome may be associated with all the following syndrome except
a. Pancoast syndrome c. Vernet’s syndrome

b. Villaret’s syndrome d. Wallenberg’s syndrome


Q. 4. Weber syndrome is characterized by all the following features except
a. contralateral hemiplegia c. ipsilateral ocular palsy

b. contralateral hypoglossal palsy d. ipsilateral facial palsy


Q. 5. The nerves involved in Villaret’s syndrome are
a. IX and X d. IX, X, XI, XII and Horner’s syndrome

b. IX, X and XI e. none of the above

c. IX, X, XI and XII


Q. 6. Frey’s syndrome is relieved by section of

a. chorda tympani nerve d. tympanic branch of CN IX

b. greater auricular nerve e. vidian nerve


c. greater petrosal nerve
Q. 7. Posterior retropharyngeal syndrome is otherwise known as
a. Vernet’s syndrome b. Villaret’s syndrome
CONCEPT BOOK OF DISEASES OF EAR, NOSE & THROAT BY FAHIM AWAN
497 Part I Miscellaneous MCQs

c. Vail syndrome e. none of the above


d. Waardenburg’s syndrome
Q. 8. In which syndrome sensorineural hearing loss is associated with abnormality of
thyroxine synthesis?
a. Alport syndrome d. Pendred syndrome

b. Klipple-Feil syndrome e. Usher syndrome

c. Pancoast syndrome
Q. 9. Sensorineural hearing loss associated with night blindness is seen in

a. Alport syndrome d. Van der Hoeve syndrome

b. Pendred syndrome e. Waardenburg’s syndrome

c. Usher syndrome
-------------------------------------------------------- ANSWERS-----------------------------------------------------
Q.1. b 2. e 3. c 4. d 5. d
6. d 7. b 8. d 9. c
--------------------------------------------------------------------------------------------------------------------------
-
CONCEPT BOOK OF DISEASES OF EAR, NOSE & THROAT BY FAHIM AWAN
498 Part I Miscellaneous MCQs

PART I
MISCELLANEOUS MCQs
EAR
Q. 1. Following structures develop from 1st brachial arch except
a. incus d. tensor palati

b. malleus e. tensor tympani


c. stapes
Q. 2. All of the following structures develop from 2nd brachial arch except
a. anterior belly of digastric d. posterior belly of digastric

b. buccinator muscle e. stapes

c. frontalis muscle
Q. 3. Pinna develops from

a. four mesodermal tubercles d. six mesodermal tubercles

b. six ectodermal tubercles e. two ectodermal and four mesodermal

tubercles
c. six endodermal tubercles
Q. 4. Tympanic membrane is derived from
a. ectoderm only d. all of the above

b. endoderm only e. none of the above

c. mesoderm only
Q. 5. The cartilage of pinna is

a. elastic d. all of the above

b. fibrous e. none of the above


c. hyaline
Q. 6. The thickness of tympanic membrane is
a. 0.075 to 0.1 mm d. 0.5 to 01 cm

b. 0.1 to 0.5 mm e. 01 cm to 02 cm
c. 0.5 to 01 mm
Q. 7. Stapes footplate covers

a. epitympanic recess d. round window

b. oval window e. sinus tympani

c. pyramid

CONCEPT BOOK OF DISEASES OF EAR, NOSE & THROAT BY FAHIM AWAN


499 Part I Miscellaneous MCQs

Q. 8. Area of stapes footplate is

a. 1.5 sq mm d. 3.2 sq mm

b. 2.2 sq mm e. 04 sq mm
c. 3.0 sq mm
Q. 9. Use of Seigel’s speculum during examination of the ear provides all except
a. assessment of movement of the tym- c. as applicator for the powdered anti-
panic membrane septic to ear
b. as applicator for the powdered antibi-otic d. magnification
to ear e. removal of foreign body from the ear
Q. 10. Which focal length in the objective piece of microscope is commonly used for ear
surgery?
a. 150 mm d. 450 mm

b. 250 mm e. 550 mm
c. 350 mm
Q. 11. All are functions of the middle ear except
a. act as air cushion to protect inner ear c. decreases the force of sound energy
b. converts mechanical energy to electrical d. increases the force of sound energy
impulse e. all of the above
Q. 12. The length of external auditory meatus in an adult is
a. 15 mm d. 27 mm

b. 19 mm e. 35 mm
c. 24 mm
Q. 13. The length of internal auditory meatus is
a. 0.5 cm d. 04 cm

b. 01 cm e. 06 cm
c. 02 cm
Q. 14. Plate of bone separating the middle cranial fossa from middle ear is called
a. epitympanum(attic) d. septum musculotuberosum

b. Korner’s septum e. tegmen tympani


c. sinus plate
Q. 15. All are true of the attic region of middle ear except
a. communicates with antrum through b. contains head of malleus and body of
the aditus incus
c. lies medial to pars flacida e. lies under the tegmen tympani
d. lies medial to pars tensa
Q. 16. The pinna attains 90-95% of adult size by

CONCEPT BOOK OF DISEASES OF EAR, NOSE & THROAT BY FAHIM AWAN


500 Part I Miscellaneous MCQs

a. birth d. 11-12 years

b. 05-06 years e. 15-16 years


c. 09-10 years
Q .17. Tympanic membrane functionally is part of
a. external ear d. all of the above

b. internal ear e. none of the above


c. middle ear
Q .18. The most common malignant tumor involving pinna is
a. basal cell carcinoma d. squamous cell carcinoma

b. lymphoma e. tumor of sweat glands


c. melanoma
Q. 19. Costen’s syndrome is characterized by all except
a. dizziness d. tinnitus

b. otalgia e. vertigo

c. temporomandibular joint dislocation


------------------------------------------- ANSWERS--------------------------------------
------------- ---------------
Q.1. c 2. a 3. b 4. d 5. a 6. a 7. b
8. d 9. e 10. b 11. d 12. c 13. b 14. e
15. d 16. b 17. c 18. d 19. c
--------------------------------------------------------------------------------------------------------------------------
-

NOSE
Q. 1. Nasal vestibule is lined by
a. columnar ciliated epithelium d. transitional cell epithelium

b. cuboidal epithelium e. none of the above


c. skin
Q. 2. The space between bulla ethmoidalis and uncinate process is called
a. agger nasi d. limen nasi

b. frontonasal recess e. olfactory cleft

c. hiatus semilunaris
Q. 3. Torus tubaris refers to
a. bony swelling in the hard palate palatopharyngeus
b. elevation raised by cartilage of eustav- d. mucosal fold raised by salpingopha-
chian tube ryngeus muscle near the eustachian tube
c. mucosal ridge raised by fibers of e. none of the above

CONCEPT BOOK OF DISEASES OF EAR, NOSE & THROAT BY FAHIM AWAN


501 Part I Miscellaneous MCQs
Q. 4. Torus platinus refers to
a. bony swelling in the hard palate atopharyngeus
b. elevation raised by cartilage of eusta- d. mucosal fold raised by salpingopha-
chian tube ryngeus muscle near the eustachian tube
c. mucosal ridge raised by fibers of pal- e. none of the above
Q. 5. Pharyngeal recess is otherwise known as
a. facial recess d. pyriform fossa

b. frontal recess e. sphenoethmoidal recess


c. fossa of Rosenmuller
Q. 6. All of the following statements are true about adenoids except
a. these are situated in the roof and pos- um
terior wall of nasopharynx d. these are part of inner Weldeyer’s
b. these are present at birth and disap- ring
pear by puberty e. these do not have a capsule on exter-nal
c. these are lined by squamous epitheli- surface
Q. 7. Choana are
a. openings between nasal cavity and c. openings between oral cavity and
nasopharynx nasopharynx
b. openings between nasal cavity and d. all of the above
oropharynx e. none of the above
Q. 8. Nasolacrimal duct opens into
a. anterior end of middle meatus d. posterior end of middle meatus

b. anterior end of inferior meatus e. posterior end of superior meatus


c. posterior end of inferior meatus
Q. 9. Which sinus is not present at birth?
a. anterior ethmoid d. posterior ethmoid

b. frontal e. sphenoid
c. maxillary
Q. 10. Trephining for acute frontal sinusitis is done through
a. anterior wall d. roof

b. floor e. none of the above

c. posterior wall
Q. 11. All are true about features of congenital syphilis except
a. atrophic rhinitis may occur c. perforation in nasal septum
b. mucocutaneous lesions d. saddle nose deformity

CONCEPT BOOK OF DISEASES OF EAR, NOSE & THROAT BY FAHIM AWAN


502 Part I Miscellaneous MCQs
e. snuffles in newborn
Q. 12. In paranasal sinuses, osteoma commonly involves
a. ethmoid sinus d. sigmoid sinus

b. frontal sinus e. sphenoid sinus


c. maxillary sinus
Q. 13. The most common tumor involving the skin of the nose is
a. adenocarcinoma d. squamous cell carcinoma

b. basal cell carcinoma e. none of the above


c. melanoma
Q. 14. All are true about sphenopalatine ganglion except
a. anesthetized by injection of local canal
anaesthetic just above and behind the c. located in infratemporal fossa
posterior end of middle turbinate d. located in pterygopalatine fossa
b. anesthetized by injecting local anaes- e. supplies innervation to nose, palate
thetic into greater palatine foramen and and lacrimal gland
Q. 15. Which of the following ganglion is associated with lacrimation?
a. Ciliary ganglion d. Submandibular ganglion

b. Gasserian ganglion e. Sphenopalatine ganglion


c. Otic ganglion
Q. 16. The most common malignant tumor of nasopharynx in children is
a. carcinoma d. neuroblastoma

b. lymphoma e. rhabdomyosarcoma
c. malignant teratoma
Q. 17. A female infant is born with a tumor of the nasopharynx, the likely nature of tumor
is
a. chondroma d. rhabdomyosarcoma

b. craniopharyngioma e. teratoma
c. neurofibroma
Q. 18. Which of the following statements is not correct?
a. Bilateral choanal atresia in a newborn pathic
precipitates respiratory distress d. Lower border of alar cartilage forms
b. Bilateral nasal packing increases pul- the nasal valve
monary resistance e. Posterior epistaxis can present as a
c. the most common cause of epistaxis is case of hematemesis
idio-
Q. 19. The Stenson’s duct of the parotid gland passes through buccinator to open in the
mucosa against the
CONCEPT BOOK OF DISEASES OF EAR, NOSE & THROAT BY FAHIM AWAN
503 Part I Miscellaneous MCQs

a. 1st upper molar tooth d. 3rd upper molar tooth

b. 2nd lower molar tooth e. upper canine tooth


c. 2nd upper molar tooth
Q. 20. A 50 years old fair skinned pathan lady presented in ENT OPD with com-plaint of
brownish colored mole on right side of cheek which has increased in size during the last 02
years. On examination the swelling is non tender and there is no lymphadenopathy. What
is the most likely diagnosis?
a. Benign nevus d. Squamous cell carcinoma.

b. Basal cell carcinoma e. Tattoo

c. Malignant melanoma
-------------------------------------------------
------- ANSWERS-----------------------------------------------------
Q.1. c 2. c 3. b 4. a 5. c 6. c 7. a
8. b 9. b 10. b 11. b 12. b 13. b 14. c
15. e 16. e 17. b 18. d 19. c 20. c
--------------------------------------------------------------------------------------------------------------------------
-

THROAT

Q. 1. The duct of submandibular salivary gland is also known as


a. duct of Bartholin d. duct of Wharton

b. duct of Rivinus e. none of the above

c. duct of Stensen
Q. 2. The duct of parotid gland is also known as

a. duct of Bartholin d. duct of Wharton

b. duct of Rivinus e. none of the above


c. duct of Stensen
Q. 3. Usual clinical features of Quinke’s disease include all except
a. edema of soft palate d. muffled voice

b. edema of uvula e. sudden onset


c. high fever
Q. 4. The most common type of esophageal cancer is
a. adenocarcinoma d. squamous cell carcinoma

b. adenoid cystic carcinoma e. transitional cell carcinoma


c. mucoepidermoid carcinoma
Q. 5. Glossopharyngeal neuralgia is characterized by all except
CONCEPT BOOK OF DISEASES OF EAR, NOSE & THROAT BY FAHIM AWAN
504 Parrt J OSPE

a. paroxysmal attacks of pain d. swallowing, talking and even laughing


b. pain radiates in the area of tongue, precipitate the pain
tonsil and ear e. usually a bilateral condition
c. responds to carbamazepine
Q. 6. The following statement best describes the trigeminal neuralgia
a. also called lower half headache d. prolonged and deep bilateral facial
b. effectively treated with NSAIDS pain
c. pain in oropharynx, throat and ipsilat- e. unilateral facial pain with specific
eral ear trigger zones
Q. 7. During superficial parotidectomy, the most reliable landmark to identify main trunk
of facial nerve is
a. cartilage of external meatus d. temporomandibular joint

b. mastoid tip e. tympanomastoid suture


c. styloid process
Q. 8. The delphian lymph node is found on the
a. cricothyroid membrane d. quardrangular membrane

b. cricovocal membrane e. thyrohyoid membrane

c. cricotracheal membrane
Q. 9. The longest cranial nerve in the body is
a. 4th cranial nerve d. 10th cranial nerve

b. 6th cranial nerve e. 12th cranial nerve


c. 8th cranial nerve
Q. 10. The largest cranial nerve in the body is
a. 3rd cranial nerve d. 9th cranial nerve

b. 5th cranial nerve e. 11th cranial nerve


c. 7th cranial nerve
Q. 11. The smallest cranial nerve in the body is
a. 3rd cranial nerve d. 6th cranial nerve

b. 4th cranial nerve e. 7th cranial nerve


c. 5th cranial nerve
------------------------------------------------
-------- ANSWERS-----------------------------------------------------
Q.1. d 2. c 3. c 4. d 5. e 6. e
7. e 8. e 9. d 10. b 11. b
--------------------------------------------------------------------------------------------------------------------------
-

CONCEPT BOOK OF DISEASES OF EAR, NOSE & THROAT BY FAHIM AWAN


505 Parrt J OSPE

PART J
OSPE

EAR
Q.1. Carefully examine the given photograph and answer the following questions:
1. What test is being performed? 02
2. What will be the finding of this test if the patient has conductive
hearing loss in his left ear? 02
Key:
1. Tuning fork test (Weber test)
2. Weber will be lateralized to left ear
Q.2. Carefully examine the given picture and answer the following
questions:
1. What is the diagnosis? 01
2. What is the cause in this case? 01
3. How will you treat is? 02
Key:
1. Keloid
2. Ear piercing
3. Excision, with local steroid injection. In refractory cases it may need excision with low
dose radiation within 24 hours of surgery.

Q.3. Carefully examine the given photograph and answer the following questions:
1. What is your diagnosis? 02
2. How will you manage the case? 02
Key:
1. Otomycosis
2. Management:
• Suction clearance
• Topical antifungal medicine (2% salicylic acid in rectified spirit)
• Prevent water from entering in the ear
• Avoid poking the ears

CONCEPT BOOK OF DISEASES OF EAR, NOSE & THROAT BY


FAHIM AWAN
506 Parrt J OSPE

Q.4. Carefully examine the patient and answer the following questions:
1. Which cranial nerve is paralyzed? 01
2. Which side is affected? 01
3. Give at least three common causes of such paralysis. 02
Key:
1. Facial nerve
2. Left side
3. Congenital, middle ear and mastoid surgery, acute/chronic suppurative otitis media,
tumors of IAC/middle ear/parotid, idiopathic Bell’s palsy.
Q.5. Carefully examine the given photograph and answer the following questions:
1. What is shown in this picture? 01
2. What procedures are accomplished through this? 01
3. What are the advantages of this procedure? 02
Key:
1. Postauricular approach to the mastoid
2. Different mastoidectomy operation are accomplished through this
approach
3. Mastoid antrum is approached directly

Q.6. Carefully examine the given photograph and answer the following questions:
1. What is shown in this picture?
01
2. What procedures are accomplished through this?
01
3. What are complications of this procedure?
02
Key:
1. Endaural approach to the mastoid process
2. Different mastoidectomy operations are carried out through
this approach
3. Damage to facial nerve, damage to labyrinth
Q.7. Carefully examine the given photograph and answer the following questions:
1. What is this graph? 01
2. What is this showing? 01
3. What is threshold? 01
4. Suppose this graph is obtained from a child of five
years, what do you suspect in this case? 01
Key:

CONCEPT BOOK OF DISEASES OF EAR, NOSE & THROAT BY


FAHIM AWAN
507 Parrt J OSPE
1. This is pure tone audiogram
2. It shows bilateral conductive hearing loss
3. This is the minimum intensity of sound in dB which is perceivable
4. The child is suffering from otitis media
Q.8. Carefully examine the given photograph and answer the following questions:
1. What are these two graphs A and B?01
2. What does the graph A and B show? 01
3. What are different types of such
graphs and what is their clinical
importance? 02
Key:
1. These graphs A and B are tympanograms
2. Graph A shows three different types of abnormal tympanograms and graph B shows
normal tympanogram
3. There are mainly three types of tympanograms, type A type B and type C
Type A tympanogram denotes normal middle ear
Type B tympanogram is flat showing fluid in middle cavity
Type C tympanogram is characterized by negative pressure in the middle cavity (blockage
of eustachian tube)
Q.9. Carefully examine the given photograph and answer the following questions:
1. What is this gadget and where it is used? 01
2. How does it work? 01
3. Where it is placed while we make use of it? 01
4. What is air bone gap? 01
Key:
1. A clinical bone conduction vibrator and it is used in accessing the bone conduction
2. A bone vibrator is transducer
3. It is place behind the pinna, on the mastoid process while accessing the bone conduction
4. It is a difference in dB between the air conduction and bone conduction
Q.10. A boxer developed a swelling of the pinna following a hit on his right ear for 3
days, swelling is obliterating all the contours of pinna. On examination pinna is
tender, red and swollen.

CONCEPT BOOK OF DISEASES OF EAR, NOSE & THROAT BY


FAHIM AWAN
508 Parrt J OSPE
1. What is the diagnosis? 01
2. Mention two other conditions leading to what has been done? 01
3. How will you treat this condition? 01
4. What complication can occur if you treat this condition inappropriately?
01
Key:
1. Auricular hematoma/perichondritis
2. Insect bite on pinna, spontaneous
3. Aspiration with wide bore needle (if small), incision and drainage, pressure application
4. Deformity of pinna
Q.11. Carefully observe the given photograph and answer following questions

A B C D

1. What is A and its uses? 01


2. What is B and its uses? 01
3. What is C and its uses? 01
4. What is D and its uses? 01

KEY
1. Tuning fork to assess hearing
2. Jobson Horne probe to remove wax
3. Agneo’s myringotome to make incision in tympanic membrane
4. Mollison’s self retaining mastoid retractor for ear surgeries e.g., cortical
mastoiddectomy, radical mastoidectomy and modified radical mastoidectomy
Q.12. Carefully observe the given radiograph and answer the following questions
1. Name the radiograph and its view? 01
509 Parrt J OSPE
2. Describe the findings? 01
3. What will be your diagnosis? 02

Key
1. X-ray mastoid both ears lateral oblique view
2. Sclerosis of mastoid air cells
3. Chronic suppurative otitis media
Q.13. Carefully observe the given photograph and answer the following questions
1. What is the diagnosis, and which side is affected? 01
2. What are two types, according to site of lesion? 01
3. What are the medical and surgical treatment options in
idiopathic type? 02

Key
1. Facial nerve palsy, left side
2. a) Upper motor neuron pasly
b) Lower motor neuron palsy
3. Medical treatment: steroids, antiviral therapy, physiotherapy, facial exercises,
protection of the eye
Surgical treatment: facial nerve decompression

CONCEPT BOOK OF DISEASES OF EAR, NOSE & THROAT BY FAHIM AWAN


510 Parrt J OSPE

NOSE
Q.1. Carefully examine the given photograph and answer the following questions:
1. Identify the instrument? 01
2. Name of the procedure where it is used? 01
3. What are indications of its use? 01
4. Name at least three usual complications it can produce? 01
Key:
1. Tilley’s Lichtwitz trocar and cannula
2. Antral washout
3. Acute and subacute sinusitis
4. Complications:
• Trauma to cheek
• Trauma to orbit
• Bleeding
• Air embolism
Q.2. Carefully examine the given photograph and answer the following questions:
1. What is the diagnosis? 01
2. Enumerate its two causes? 01
3. What is the treatment? 01
4. What complications can arise if untreated? 01
Key:
1. Septal hematoma
2. Fracture nose and septal surgery
3. Incision & drainage plus antibiotics
4. Septal abscess, cavernous sinus thrombosis, septal perforation and saddle nose
Q.3. Carefully examine the given photograph and answer the following questions:
1. What investigation is this? 01
2. What are the findings? 02
3. Give at least two differential diagnoses? 01
Key:
1. Coronal CT scan of nose and paranasal sinuses
2. Findings:
• Air fluid level in left maxillary sinus
• Haziness of left nasal cavity
CONCEPT BOOK OF DISEASES OF EAR, NOSE & THROAT BY
FAHIM AWAN
511 Parrt J OSPE
• Haziness of left ethmoid sinuses
• Haziness of left frontal sinus

3. Differential diagnosis:
a. Acute sinusitis
b. Chorionic sinusitis
Q.4. Carefully examine the given X-ray and answer the following questions:
1. What is this X-ray? 01
2. What is the positive finding in this radiograph? 01
3. Mention two complications associated with this pathology? 01
4. How will you manage? 01
Key:
1. This is X-ray PNS, occipitomental view
2. Radiopaque, rounded shadow seen in one nasal cavity
3. Rhinitis, sinusitis, nose bleeding
4. Removal of rhinolith
Q.5. Carefully examine the given photograph and answer the following questions:
1. What is the name of the instrument? 01
2. What is its function and in which operation kit it is used? 01
3. Mention three complications of the operation? 02
Key:
1. Ballenger swival knife
2. It is used to cut the septal cartilage in one block. It is used in SMR operation
3. Septal hematoma, septal perforation, saddle nose
Q.6. Carefully examine the given photograph and answer the following questions:
1. What are the names of these instruments? 01
2. In which operation these are used? 01
3. What are the usual complications associated with the conditions
where these instruments are used? 02
Key:
1. Asch’s forceps and Walsham’s forceps
2. For correction of fractured nasal bones
3. Massive nasal bleeding, septal haematoma, septal ascess and CSF rhinorrhoea

CONCEPT BOOK OF DISEASES OF EAR, NOSE & THROAT BY


FAHIM AWAN
512 Parrt J OSPE
Q.7. Carefully examine the given photograph and answer the following questions:
1. What is this X-ray? 01
2. What is the positive finding in this radiograph? 01
3. How will you manage? 01
4. What are possible complications of the operation? 01
Key:
1. X-ray soft tissue neck lateral view of a child
2. There is soft tissue shadow in the region of nasopharynx, causing
significant obstruction to the airway. Hypertrophied adenoid
3. Patient is best managed by operation that is adenoidectomy
4. Complications:
• Bleeding
• Trauma to teeth, gums and posterior pharyngeal wall
Q.8. Carefully observe the given photograph and answer the following question:
1. What are positive findings? 02
2. What is the likely diagnosis? 01
3. Name the radiological investigation required in the case. 01
Key:
1. Frog shape nose, mass in left nasal cavity, left proptosis, fullness in
the left cheek
2. Nasopharyngeal angiofibroma
3. X-ray PNS lateral view, CT scan, angiography

Q.9. Carefully observe the given photograph and answer following questions
1. Name the radiograph and its view? 01
2. Describe finding in this radiograph? 01
3. Name types of fractures of nasal bones? 02

Key
1. Radiograph nasal bones lateral view
2. Loss of continuity and loss of alignment of nasal bones
3. There are three classes/types of fracture nasal bones
a) Class 1 fracture b) Class 11 fracture c) Class 111 fracture

CONCEPT BOOK OF DISEASES OF EAR, NOSE & THROAT BY


FAHIM AWAN
513 Parrt J OSPE
Q.10. Carefully observe the given photograph and answer following questions
1. Identify the instrument? 01
2. What are the uses of this instrument? 01
3. What complications can occur during use of this
instrument? 02
Key:
1. Luc’s forceps
2. Polypectomy, septoplasty, turbinectomy and tissue biopsy from nose/throat
3. Hemorrhage, septal perforation, CSF leakage
Q.11. Carefully observe the given photograph and answer following questions
1. Identify the instrument? 01
2. What are the uses of this instrument? 01
3. What complications can occcur during the use of this
instrument? 02
Key
1. Cottle nasal speculum
2. Examination of nose, during septoplasty and other nasal surgery
3. Hemorrhage, trauma to surrounding structures

Q.12. Carefully observe the given photograph and answer following questions
1. What are the clinical findings in this 10-years-old boy? 01
2. What is the most common cause? 01
3. How will you treat this patient? 01
4. Where pathology is suspected? 01
Key:
1. Solitary globular whitish mass in left nasal cavity, left proptosis
2. Left antrochoanal polyp
3. FESS, Intranasal polypectomy
4. Left maxillary sinus

Q.13. Carefully observe the given photograph and answer


following questions
1. What are the clinical findings? 01

CONCEPT BOOK OF DISEASES OF EAR, NOSE & THROAT BY


FAHIM AWAN
514 Parrt J OSPE
2. What are the common causes? 01
3. How will you treat this patient? 02
Key:
1. Deviated nasal pyramid
2. Road traffic accident, assault, personal injury, sports injury
3. Reduction of nasal bone fracture/ rhinoplasty
Q.14. Carefully observe the given photograph and answer following questions

1. What is the most likely diagnosis? 01


2. What is differential diagnosis? 02
3. How will you treat this? 01
Key
1. Basal cell carcinoma
2. Squamous cell carcinoma, granuloma
3. Excision with healthy margins and repair
THROAT
Q.1. Please look at the photograph and answer the following questions:
1. Enumerate the clinical findings? 02
2. What is the most probable clinical diagnosis? 01
3. What microorganisms produce this condition? 01
Key:
1. Clinical findings:
• Protrusion of tongue
• Raised floor of mouth
• Submandibular swelling
2. Ludwig’s angina
3. Organism:
• Streptococcus viridians
• Escherichia coli
Q.2. Carefully examine the given photograph and answer the following questions:
1. What is your diagnosis? 01
2. How will you confirm the diagnosis? 01
3. What is the treatment? 02
Key:

CONCEPT BOOK OF DISEASES OF EAR, NOSE & THROAT BY FAHIM AWAN


515 Parrt J OSPE
1. Bilateral vocal nodules
2. Direct laryngoscopy
3. Voice rest, speech therapy and excision of vocal nodules under general anesthesia
Q.3. Carefully examine the given photograph and answer the following questions:
A 50 years old male presents with persistent hoarseness of voice for last 03 months. He is a
smoker for past 20 years
1. What is your diagnosis? 01
2. How will you confirm your diagnosis? 01
3. What are the treatment options? 02
Key:
1. Malignancy of larynx
2. Direct laryngoscopy and biopsy
3. Radiotherapy and/or surgery, depending on the TNM stage

Q.4. Carefully examine the given photograph and answer the following questions:

A 20 years old male presents with two days history of severe odynophagia, high-grade
fever and trismus:
1. What is your diagnosis? 01
2. What is the treatment of this condition? 01
3. What complications can occur if not treated? 02
Key:
1. Peritonsillar abscess (quinsy)
2. Incisions and drainage, I.V. antibiotics
3. Complications:
• Airway obstruction
• Parapharyngeal abscess
• Retropharyngeal abscess
• Internal jugular vein thrombosis
Q.5. Carefully examine the given photograph and answer the following questions:
1. What is the diagnosis? 01
2. How will you confirm whether it is in the esophagus or trachea? 01
3. How will you manage it? 01
4. Mention one important complication of its removal. 01
CONCEPT BOOK OF DISEASES OF EAR, NOSE & THROAT BY FAHIM AWAN
516 Parrt J OSPE
Key:
1. Foreign body
2. Needs lateral view to decide
3. Rigid endoscopy and removal of foreign body under general anesthesia
4. Damage to teeth, esophageal tear, esophageal perforation
Q.6. Carefully examine the given photograph and answer the following questions:
1. What is this picture? 01
2. What are radiological findings? 01
3. What is pathology? 01
4. How will you deal with this pathology? 01
Key:
1. CT scan axial view showing upper part of neck and floor of mouth
2. There is radiopaque shadow in the left submandibular region
3. Stone in submandibular gland.
4. Excision of submandibular gland.
Q.7. Carefully examine the given photograph and answer the following questions:
1. What is radiograph A and radiograph B? 01
2. What is importance of radiograph A and radiograph B. 01
3. What are positive radiological findings? 01
4. How will you manage this case? 01
Key:
1. Radiograph A is AP view and radiograph B is a lateral view of chest and neck
2. Radiograph A is done to detect the pathology and radiograph B is done to determine the
position of the pathology
3. There are radiopaque shadows in both radiographs coinciding with coin
4. Endoscopic removal of coin under general anesthesia

Q.8. Carefully examine the given photograph and answer the following questions:
1. What does this diagram show? 01
2. It is used for what purpose? 01
3. Enumerate three disabilities of such a patient who had this tube
inserted. 02
Key:
1. Tracheostomy tube

CONCEPT BOOK OF DISEASES OF EAR, NOSE & THROAT BY FAHIM AWAN


517 Parrt J OSPE
2. It is used in upper airway obstruction.
3. Cannot speak, cannot lift heavy weight, cannot swim.
Q.9. Carefully examine the given photograph and answer the following questions:
1. What is the name of the instrument and where it is used? 01
2. What are usual complications associated with the use of this
instrument? 01
3. What are the usual complications of the operation where this
instrument is used? 02
Key:
1. Boyle Davis mouth gag. It is used to open the mouth and doing tonsillectomy
2. Dental trauma, trauma to posterior pharyngeal wall and dislocation of
temporomandibular joint
3. Bleeding, infection and damage to anterior faucial pillars and nearby structures
Q.10. Carefully examine the given photograph and answer
the following questions:
1. What is the name of this instrument? 02
2. Where is it used? 01
3. What is function of the instrument? 01
Key:
1. Eve’s tonsillar snare
2. Lower pole of tonsil during tonsillectomy
3. To secure hemostasis at lower pole of tonsil

Q.11. Carefully observe the given photograph and answer following questions
1. Identify the instrument? 02
2. Name the procedure in which this is used? 01
3. What is the significance of these markings? 01
Key:
1. Rigid esophagoscope
2. Esophagoscopy

3. It helps to know the site/location of distal end during esophagoscopy


Q.12. Carefully observe the given photograph and answer
following questions
1. What is the most likely diagnosis?
CONCEPT BOOK OF DISEASES OF EAR, NOSE & THROAT BY
FAHIM AWAN
518 Parrt J OSPE
01
2. How is the most common cause? 01
3. How will you treat this? 02
Key
1. Ludwig’s angina
2. Dental infection
3. Parentral antibiotics, anti-inflammatory, analgesics, incision drainage, observation
for airway obstruction, dental treatment
Q.13. Carefully observe the given photograph and answer
following questions
1. Describe the clinical picture? 01
2. What is the most likely diagnosis? 01
3. What organisms are responsible? 01
4. How will you treat this? 01
Key
1. Red, congested, hypertrophied tonsils
2. Acute parenchymatous tonsillitis
3. Beta hemolytic Streptococcus, H-influenza
4. Antibiotics, analgesics, antiseptic gargles
Q.14. A 20 years old boy presents with swelling in this region for last 10 years
1. What is the most likely diagnosis? 01
2. What is the differential diagnosis? 02
3. How will you treat this patient? 01
Key
1. Infected thyroglossal cyst
2. Submental lymph node, dermoid cyst, lipoma
3. Sistrunk operation
Q.15. Carefully observe the given photograph and answer
following questions
1. Identify the picture? 01
2. What is the site of insertion? 02
3. What is the disadvantage of this tube? 01
Key

CONCEPT BOOK OF DISEASES OF EAR, NOSE & THROAT BY FAHIM AWAN


519 Parrt J OSPE
1. Metallic tracheostomy tube
2. Below tracheal ring number 02
3. More traumatic, does not have a cuff, cannot be used for ventilation purpose
Q.16. Carefully observe the given photograph and answer following questions
1. Identify the picture? 1/2
2. Give two main indication of it? 1/2
3. What are the main types? 01
4. Two main routes of its insertion? 01
5. Once introduced blindly where else it can enter? 01
Key
1. Endotracheal tube
2. Induction of general anesthesia, restoration of airway
3. Cuffed, non cuffed
4. Oral and nasal route
5. In the esophagus

CONCEPT BOOK OF DISEASES OF EAR, NOSE & THROAT BY FAHIM AWAN


520 Parrt J OSPE

CONCEPT BOOK OF DISEASES OF EAR, NOSE & THROAT BY FAHIM AWAN


521 Parrt J OSPE

CONCEPT BOOK OF DISEASES OF EAR, NOSE & THROAT BY FAHIM AWAN


522 Parrt J OSPE

CONCEPT BOOK OF DISEASES OF EAR, NOSE & THROAT BY FAHIM AWAN


523 Parrt J OSPE

CONCEPT BOOK OF DISEASES OF EAR, NOSE & THROAT BY FAHIM AWAN


524 Parrt J OSPE

CONCEPT BOOK OF DISEASES OF EAR, NOSE & THROAT BY FAHIM AWAN


525 Parrt J OSPE

CONCEPT BOOK OF DISEASES OF EAR, NOSE & THROAT BY FAHIM AWAN


526 Parrt J OSPE

CONCEPT BOOK OF DISEASES OF EAR, NOSE & THROAT BY FAHIM AWAN


527 Parrt J OSPE

CONCEPT BOOK OF DISEASES OF EAR, NOSE & THROAT BY FAHIM AWAN


528 Parrt J OSPE

CONCEPT BOOK OF DISEASES OF EAR, NOSE & THROAT BY FAHIM AWAN


529 Parrt J OSPE

CONCEPT BOOK OF DISEASES OF EAR, NOSE & THROAT BY FAHIM AWAN


530 Parrt J OSPE

CONCEPT BOOK OF DISEASES OF EAR, NOSE & THROAT BY FAHIM AWAN


531 Parrt J OSPE

CONCEPT BOOK OF DISEASES OF EAR, NOSE & THROAT BY FAHIM AWAN


532 Parrt J OSPE

CONCEPT BOOK OF DISEASES OF EAR, NOSE & THROAT BY FAHIM AWAN


533 Parrt J OSPE

CONCEPT BOOK OF DISEASES OF EAR, NOSE & THROAT BY FAHIM AWAN


534 Parrt J OSPE

CONCEPT BOOK OF DISEASES OF EAR, NOSE & THROAT BY FAHIM AWAN


535 Parrt J OSPE

CONCEPT BOOK OF DISEASES OF EAR, NOSE & THROAT BY FAHIM AWAN


536 Parrt J OSPE

CONCEPT BOOK OF DISEASES OF EAR, NOSE & THROAT BY FAHIM AWAN


537 Parrt J OSPE

CONCEPT BOOK OF DISEASES OF EAR, NOSE & THROAT BY FAHIM AWAN


538 Parrt J OSPE

CONCEPT BOOK OF DISEASES OF EAR, NOSE & THROAT BY FAHIM AWAN


539 Parrt J OSPE

CONCEPT BOOK OF DISEASES OF EAR, NOSE & THROAT BY FAHIM AWAN


540 Parrt J OSPE

CONCEPT BOOK OF DISEASES OF EAR, NOSE & THROAT BY FAHIM AWAN


541 Parrt J OSPE

CONCEPT BOOK OF DISEASES OF EAR, NOSE & THROAT BY FAHIM AWAN


542 Parrt J OSPE

CONCEPT BOOK OF DISEASES OF EAR, NOSE & THROAT BY FAHIM AWAN


543 Parrt J OSPE

CONCEPT BOOK OF DISEASES OF EAR, NOSE & THROAT BY FAHIM AWAN


544 Parrt J OSPE

CONCEPT BOOK OF DISEASES OF EAR, NOSE & THROAT BY FAHIM AWAN

You might also like